Текст
                    СБОРНИК ЗАДАЧ
ПО ЭЛЕМЕНТАРНОЙ
ФИЗИКЕ

СБОРНИК ЗАДАЧ ПО ЭЛЕМЕНТАРНОЙ ФИЗИКЕ Пособие для самообразования Издание четвертое, переработанное ИЗДАТЕЛЬСТВО «НАУКА» П<77II ГЛАВНАЯ РЕДАКЦИЯ ||Ц7/|| ФИЗИКО-МАТЕМАТИЧЕСКОЙ ЛИТЕРАТУРЫ Москва 1974
630.1 С 23 УДК 530.10(075.4) Авторы» Б. Б. БУХОВЦЕВ, В. Д. КРИВЧЕНКОВ, Г. Я. МЯКИШЕВ, И. М. САРАЕВА Сборник задач по элементарной физике. Главная ре- дакция физико-математической литературы издательства «Наука», 1974 г. Настоящий сборник задач по физике составлен в со- ответствии с материалом, изложенным в «Элементарном учебнике физики» под редакцией академика Г. С. Ландс- берга. Большинство задач значительно превосходит по трудности задачи, предлагаемые обычно учащимся сред- них школ. Все они снабжены подробными решениями. Часть задач сборника составляют переработанные задачи школьных олимпиад последних лет, которые проводи- лись на физическом факультете Московского универси- тета. Задачник может быть рекомендован для самообра- зования учащимся старших классов средних общеобра- зовательных школ, техникумов и специальных средних школ. Ряд задач может быть полезен для студентов первых курсов высших учебных заведений. © Главная редакция физико-математической литературы издательства «Наука», 1974 г., с изменениями. 20401-131 _ 053(02)-74 ,02'74
ОГЛАВЛЕНИЕ Предисловие к четвертому изданию 5 Из предисловия к первому изданию 5 Задачи Ответы и решения Глава I. Механика ................................... 7 163 § 1. Кинематика равномерного прямолинейного движения.................................... 7 163 § 2. Кинематика неравномерного и равноперемен- ного прямолинейного движения............... 10 170 § 3. Кинематика криволинейного движения . . 13 174 § 4. Динамика прямолинейного движения .... 20 188 § 5. Закон сохранения количества движения . . 29 199 § 6. Статика............................... 31 203 § 7. Работа и энергия...................... 36 214 § 8. Динамика криволинейного движения .... 43 226 § 9. Закон всемирного тяготения..... 52 j 241 § 10. Гидро- и аэростатика..................... 54 246 § 11. Гидро- и аэродинамика................. . 61 254 Глава II. Теплота. Молекулярная физика.............. 67 259 § 12. Тепловое расширение твердых и жидких тел 67 259 § 13. Закон сохранения энергии. Теплопроводность 68 261 § 14. Свойства газов............................ 70 263 § 15. Свойства жидкостей........................ 75 271 § 16. Взаимные превращения жидких и твердых тел 78 275 § 17. Упругость и прочность..................... 79 276 § 18. Свойства паров............................ 82 278 Глава III. Электричество и магнетизм................ 85 280 § 19. Электростатика............................ 85 280 § 20. Постоянный электрический ток.............. 95 301 § 21. Электрический ток в газах и вакууме . ... 105 319 § 22. Магнитное поле тока. Действие магнитного поля на ток и движущиеся заряды.............. 109 325
§ 23. Электромагнитная индукция. Переменный ток П4 331 § 24. Электрические машины................... 122 343 Глава IV. Колебания и волны...................... 126 348 §25. Механические колебания................. 126 348 § 26. Электрические колебания................ 131 358 § 27. Волны.................................. 133 360 Глава V. Геометрическая оптика................... 136 363 § 28. Фотометрия............................. 136 363 § 29. Основные законы оптики................. 137 365 § 30. Линзы и сферические зеркала............ 142 375 §31. Оптические системы и оптические приборы . 146 382 Глава VI. Физическая оптика...................... 153 400 § 32. Интерференция света.................... 153 400 § 33. Дифракция света........................ 158 407 §34. Дисперсия света и цвета тел.............161 411
ПРЕДИСЛОВИЕ К ЧЕТВЕРТОМУ ИЗДАНИЮ Настоящее издание значительно переработа- но по-сравнению с тремя предыдущими. Исклю- чено большое количество простых задач и до- бавлены задачи повышенной трудности. Боль- шая часть новых задач являются оригинальны- ми. В издание включен также ряд задач школь- ных олимпиад последних лет, проводимых на физическом факультете Московского универ- ситета. Авторы ИЗ ПРЕДИСЛОВИЯ К ПЕРВОМУ ИЗДАНИЮ Настоящий сборник задач по физике состав- .лен на основе материала, изложенного в «Эле- ментарном учебнике физики» под редакцией ака- демика Г. С. Ландсберга. Поэтому содержание, уровень задач и характер расположения мате- риала в основном соответствуют указанному учебнику. В сборник не включен раздел «Атом- ная физика», так как упражнения по этому воп- росу в учебнике Ландсберга достаточно полна иллюстрируют изложенный материал. Отдель- ные задачи на данную тему включены в другие главы сборника. 6
При составлении сборника авторы уделяли большое внимание задачам повышенной труд- ности, требующим глубокого понимания основ- ных физических законов и уменья использовать их при самых разнообразных условиях. Все трудные задачи снабжены подробными решения- ми. Кроме того, в целях развития навыков и куль- туры решения физических задач некоторые бо- лее простые задачи также снабжены реше- ниями. Часть задач сборника составляют перерабо- танные задачи школьных олимпиад, которые про- водились на физическом факультете Москов- ского университета. В переработанном виде включено некоторое количество задач из сбор- ников для высшей школы. Большинство задач являются оригинальными. Авторы
ЗАДАЧИ Глава I. МЕХАНИКА § 1. Кинематика равномерного прямолинейного движения С/ 1. Идущая вверх по реке моторная лодка встретила сплавляемые по течению реки плоты. Через час после встре- чи лодочный мотор заглох. Ремонт мотора продолжался 30 мин. В течение этого времени лодка свободно плыла вниз по течению. После ремонта лодка поплыла вниз по течению с прежней относительно воды скоростью и нагнала плоты на расстоянии 5=7,5 км от места их первой встречи. Оп- ределить скорость течения реки, считая ее постоянной. 2. Из Москвы в Пушкино с интервалом t—10 мин вы- шли два электропоезда со скоростями о=30 км/ч. С какой скоростью и двигался поезд, идущий в Москву, если он повстречал эти электропоезда через т=4 мин один после другого? —± з. Завод, на котором работает инженер, находится за городом. Каждый раз к приходу поезда на станцию приез- жает заводская машина, которая доставляет инженера на место работы. Однажды инженер приехал на станцию на час раньше обычного и, не дожидаясь машины, пошел на завод пешком. По дороге он встретил автомашину и при- ехал на завод на 10 минут раньше обычного. Сколько време- ни шел инженер до встречи с заводской автомашиной? (Решить задачу графически.) +^~4. Трое туристов, обладающих одним велосипедом, должны прибыть на базу в кратчайший срок (время оцени- вается по последнему прибывшему). Велосипед может взять лишь двоих, поэтому третьему туристу приходится сначала идти пешком. Велосипедист довозит второго туриста до не- которой точки дороги, откуда тот продолжает движение пеш- ком, и возвращается за третьим. Найти среднюю скорость туристов, если скорость пешехода их=4 км/ч, а велосипе- диста оа=20 км/ч. 7
О 5. Почтовая связь между речными пристанями М и К осуществляется двумя катерами. В условленное время ка- теры отплывают от своих пристаней, встречаются, обмени- ваются почтой и возвращаются обратно. Если катеры от- плывают от своих пристаней одновременно, то катер, вы- ходящий из 7И, тратит на путь в оба конца 3 часа, а катер из* К — 1,5 часа. Скорости обоих катеров относительно во- ды одинаковы. Определить графически, на сколько позже должен отплыть катер из М после отплытия катера из /(, чтобы оба катера находились в пути одно и то же время, г 6. Используя условия предыдущей задачи, опреде- лить скорость катеров относительно воды, скорость течения реки и место встречи катеров в случае, если они отплыва- ют от своих пристаней одновременно. Расстояние между при- ^станями равно 30 км. 7. От пристани С к пристани Т по реке плывет со скоростью V!=3 км/ч относительно воды весельная лодка. От пристани Т к пристани С одновременно с лодкой отходит катер, скорость которого относительно воды р2=10км/ч. За время движения лодки между пристанями катер успе- вает пройти это расстояние четыре раза и прибывает к Т одновременно с лодкой. Определить направление течения. 8. Человек высотой h проходит в стороне от фонаря, ^ висящего на высоте Н над землей. Найти величину и на- правление скорости перемещения по земле тени от головы человека, если скорость человека равна v. к 9. Две свечи, высоты которых в начальный момент бы- '~"ли одинаковы и равны Л, находятся на расстоянии а друг от друга. Расстояние между каждой свечой и ближайшей Рис. 1. Рис. 2. к ней стеной также равно а (рис. 1). С какой скоростью дви- жутся тени от свечей по стенам, если одна свеча сгорает за время /1, а другая — за 4? 8
10. Человек находится на берегу озера в точке Л. Ему необходимо в кратчайшее время попасть в точку В, на- ходящуюся на озере (рис. 2). Расстояние от точки В до бе- рега BC=d, а расстояние ЛС=5. Скорость движения чело- века в воде vl9 а по берегу Каким путем должен двигаться человек: плыть ли из точки А по прямой АВ или пробежать по берегу некоторое расстояние и после этого плыть по направлению к точке В? 11. По шоссе со скоростью иг= 16 м/с движется авто- бус, а человек находится на расстоянии а=60 м от шоссе.и 6=400 м от автобуса. В каком направлении‘должен бежать человек, чтобы выйти к какой-либо точке шоссе одновре- менно с автобусом или раньше него? Человек может бежать со скоростью v2=4 м/с. 12. При какой наименьшей скорости человек (см. за- дачу 11) может встретить автобус? В каком направлении должен при этом бежать человек? 13. В данный момент автобус (см. задачу 11) находится в точке А и движется по прямому шоссе АЕ, Найти геомет- рическое' место тех точек, где может находиться в настоящий момент человек, чтобы успеть сесть в автобус. 14. Человек на лодке должен попасть из точки А в точку 5, находящуюся на противоположном берегу реки (рис.З). Расстояние ВС=а. Ширина реки ЛС=6. С какой наименьшей скоростью и относительно воды должна плыть лодка, чтобы приплыть в В? Скорость течения реки и0. С а В Г.... .....*" /| А Рис. 3. 15. Из пункта Л, расположенного на берегу реки, необходимо попасть в пункт В, двигаясь по прямой АВ (рис. 4). Ширина реки ЛС=1 км, расстояние ВС=2 км, максимальная скорость лодки относительно воды и=5 км/ч, а скорость течения реки v=2 км/ч. Можно ли проплыть рас- стояние АВ за 30 минут? 9
Рис. 5. ' 16. По реке из точки А в точку В, находящуюся на противоположном берегу, вдоль прямой АВ, образующей с линией берега угол а, плывет катер (рис. 5). Под прямым углом к берегу дует ветер со скоростью и. Флаг на мачте катера об- разует угол Р с направле- нием движения катера. Оп- ределить скорость катера относительно берега. Мож- но ли поданным, приведен- ным в условии задачи, определить скорость течения реки? 17. По пересекающимся под углом а дорогам движутся две автомашины с постоянными скоростями Vi и va. Опре- делить величину и направление скорости одного автомо- биля относительно другого. Через какое время после встречи на перекрестке расстояние между машинами будет равно S? 18. Автомашины (см. задачу 17) не встретились на пе- рекрестке, причем вторая машина проехала перекресток на промежуток времени т позже первой. Каково было наи- меньшее расстояние между автомашинами? 19. Две пересекающиеся прямые движутся поступатель- но в разные стороны со скоростями Vi и vt, перпендикуляр- ными соответствующим прямым. Угол между прямыми равен а. Определить скорость точки пересечения прямых. § 2. Кинематика неравномерного и равнопеременного прямолинейного движения 20. .Определить среднюю скорость и среднее ускорение точки за 5 и 10 секунд, если движение ее задано графиком ю
21. Человек, стоящий на крутом берегу озера, тянет лодку. Скорость, с которой постоянна и равна v. Какую за веревку находящуюся на воде человек выбирает веревку, скорость будет иметь лодка в момент, когда угол между веревкой и поверхностью воды равен а? 22. Точечный источник света S находится на рас- стоянии / от вертикального экрана АВ. От источника к экрану по прямой движется поступательно, с постоянной скоростью v непрозрачный предмет высоты Рис. 7. h. Определить мгновенную скорость перемещения верхнего края тени предмета по экрану (рис. 7). 23. Координата точки, движущейся прямолинейно вдоль оси х, меняется со временем по закону х= 11 -1^35^4- 4-4U» (х измеряется в см, а / — в секундах). Опреде- лить скорость и ускорение точки. 24. Демонстрационная тележка двигалась вдоль длин- ной линейки с постоянным ускорением. В момент, когда секундомер показывал tr=l с, тележка находилась про- тив отметки Xi=70 см, в момент /2= 9 с — против отметки х2=80 см и при /3=15 с — против отметки х8=230 см. С каким ускорением дви- галась тележка? 25. На рис. 8 и 9 изображены: график скорости тела и график изменения координаты тела (парабола) в зави- симости от времени. Начало отсчета времени на обоих 11
графиках совпадает. Одинаковые ли движения изобра- жены на этих графиках? 26. Пункты А и В расположены на расстоянии /=4 км друг от друга. Из пункта А по направлению к пункту В выехал автомобиль, который двигался все время равномер- но. Одновременно навстречу первому из пункта В с началь- ной скоростью Vo=32 м/с выехал автомобиль, движущийся с постоянным ускорением а=0,2 м/с4, направленным все время так же, как скорость первого автомобиля. Известно, что в пути автомобили два раза обгоняли друг друга. В ка- ких пределах лежит скорость первого автомобиля? 27. С высоты И на упругую горизонтальную подстав- ку свободно падает шарик. Построить график изменения ко- ординаты и скорости шарика в зависимости от времени, считая, что временем соударения можно пренебречь. Удар абсолютно упругий. .28. На упругую плиту свободно падают стальные ша- рики. Первый падает с высоты Лх=44 см, второй — спустя время т после первого, с высоты Л»= 11 см. Через некоторое время скорости шариков совпадают по величине и напра- влению. Определить время т и промежуток времени, в тече- ние которого скорости обоих шариков равны. Шарики между собой не соударяются. 29. За какое время тело, свободно падающее без на- чальной скорости, проходит п-й сантиметр своего пути? 30. С высокой башни друг за другом бросают два тела с одинаковыми по величине скоростями v0. Первое тело бро- сают вертикально вверх; спустя время т бросают второе — вертикально вниз. Определить скорость тел друг относитель- но друга и расстояние между ними в момент времени />т. 31. Три точки А, В и С в начальный момент времени расположены на одной горизонтальной прямой, на равных расстояниях друг от друга. Точка А начала двигаться вер- тикально вверх с постоянной скоростью v, а точка С — без начальной скорости, вертикально вниз с постоянным ус- корением а. Как должна двигаться по вертикали точка В, чтобы все три точки находились все время на одной прямой? Точки начинают двигаться одновременно. 32. Лифт движется с ускорением а. Пассажир, находя- щийся в лифте, роняет книгу. Чему равно ускорение книги относительно пола лифта, если лифт движется вверх? Если лифт движется вниз? 33. Два автомобиля выехали навстречу друг другу из городов А и В с одинаковыми по величине скоростями и 12
одинаковыми по величине ускорениями, равными а. Ускоре- ние автомобиля, выехавшего из А, было все время направле- но в Л, а выехавшего из В — направлено в В. На сколько позже выехал один из этих автомобилей, если третий авто- мобиль, двигавшийся все время с постоянной скоростью vit присутствовал при обеих встречах первых двух автомо- билей? Рис. 10. 34. В лифте, движущемся с ускорением а, с высоты Н над полом человек роняет шарик. Через время т после на- чала падения шарика ускорение лифта меняет знак, а че- рез время 2т становится рав- ным нулю. После этого ша- рик касается пола. На какую высоту от пола лифта под- скочит шарик после удара? Удар считать абсолютно упругим. 35. На клин, плоскость которого составляет угол а с горизонтом, положили тело А (рис. 10). Какое ускорение необходимо сообщить клину в горизонтальном направлении, чтобы тело А свободно падало вертикально вниз? § 3. Кинематика криволинейного движения 36. Маленький тяжелый шарик брошен горизонтально с начальной скоростью v0- Найти нормальное и тангенциаль- ное ускорения шарика через время т после начала движения. 37. Найти величину ускорения тела А, _____________________ соскальзывающего без начальной скорости s .__________________по винтовому желобу с шагом h и радиу- ' сом /?, в конце n-го витка (рис. 11). Тре- нием пренебречь. —• 38. Скорость течения реки возрастает . пропорционально расстоянию от берега, достигая своего максимального значения < ----Ц- v0 на середине реки. У берегов скорость х. л I течения равна нулю. Лодка движется по реке таким образом, что ее скорость и от- носительно воды постоянна и перпендику- Рис. 11. лярна течению. Найти расстояние, на ко- торое будет снесена течением лодка при переправе, если ширина реки с. Определить также траек- торию лодки. 13
39. Два рельса скреплены под прямым углом друг к другу. По этим рельсам движутся две тележки, скреплен- ные между собой шарнирно стержнем длины I. Тележка А (рис. 12) начинает движение из точки пересечения рельсов и движется равномерно вверх со скоростью v. Определить закон движения и скорость тележки В. 40. Тело брошено с началь- ной скоростью под углом а к горизонту. Сколько времени длится полет? На каком рассто- янии от места бросания упадет тело? При каком значении угла а дальность полета тела будет максимальной? На какой высоте тело бу- дет через промежуток времени т после начала движения? Какую скорость будет иметь тело по величине и направлению в этот момент времени? Считать т большим, чем время подъема тела до максимальной высо- ты. Сопротивлением воздуха пренебречь. 41. Найти траекторию движения тела, брошенного под углом к горизонту (см. задачу 40). 42. Необходимо с земли перебросить мяч через вер- тикальную стенку высоты Н, находящуюся на расстоянии S Рис. 13. (рис. 13). При какой наименьшей на- чальной скорости это возможно? Под каким углом а к горизонту должна быть в этом случае направлена ско- рость? 43. Испытание осколочной гра- наты производится в центре дна ци- линдрической ямы глубины Н. Обра- зующиеся при взрыве осколки, ско- рость которых не превышает v0, не должны попадать на поверхность земли. Каким должен быть минимальный диаметр D ямы? 44. Тело брошено в воду с крутого обрыва высотой И. Начальная скорость тела составляет угол а с горизонтом и равна ц0. На каком расстоянии от берега упадет тело? Че- рез какое время после начала движения тело окажется на высоте h над водой? Какова скорость тела в момент падения в воду? 14
45. Под каким углом к горизонту необходимо бросить камень с обрывистого берега реки, чтобы он упал в воду возможно дальше от берега? Высота обрыва /to=*2O м, на- чальная скорость камня v0=14 м/с. 46. С какой наименьшей скоростью нужно бросить тело с вершины башни высотой Л, чтобы оно упало на рас- стоянии S от подножья башни? 47. За телом, брошенным под углом а к горизон- ту, наблюдают в оптическую трубу, установленную в точке бросания. При каких углах а в движении тела будут моменты, когда его скорость будет перпендикулярна оси трубы? 48. Пикирующий бомбардировщик сбрасывает бомбу с высоты Н, находясь на расстоянии L от цели. Скорость бомбардировщика V. Под каким углом к горизонту он дол- жен пикировать? 49. Шарик свободно падает с высоты h на наклонную плоскость, составляющую угол а с горизонтом (рис. 14). Найти отношение расстояний между точками, в которых подпрыгивающий шарик касается наклонной плоскости. Соударения шарика с плоскостью рассматривать как аб- солютно упругие. 50. Из точки х—у=0 (рис. 15) одновременно выбро- шены два тела с одинаковой начальной скоростью v0 под разными углами ах и а2 к горизонту. Чему равна скорость движения тел друг относительно друга? Чему равно рас- стояние между телами по прошествии времени т? (Тела дви- жутся поступательно.) 51. С башни по всевозможным направлениям с одина- ковой начальной скоростью и0 брошены камни. Оказалось, что камень, подлетевший к земле по наиболее пологой у са- мой земли траектории, имеет при подлете вектор скорости, 15
составляющий угол <р с горизонтом. Определить высоту башни. 52. Со стола высотой h сбрасывают упругий шарик, сообщая ему некоторую горизонтальную скорость. В мо- мент, когда шарик испытывает' одно из бесчисленных уп- ругих соударений с полом, с того же стола горизонтально сбрасывают другой шарик, сообщая ему такую скорость, чтобы он столкнулся с первым шариком. На какой высоте произошла встреча? 53. Снаряд вылетает из. пушки с начальной скоростью ов. Определить «зону безопасности», т. е. геометрическое место всех тех точек пространства, куда снаряд не может по- пасть. 54. Гусеница трактора состоит из п звеньев. Длина каждого звена а. Радиусы колес, на которые надета гусени- ца, R. Трактор движется со скоростью v. Предполагается, что гусеница не провисает. 1) Какое число звеньев гусеницы движется в данный мо- мент поступательно, какое число звеньев покоится (относи- тельно земли) и какое число звеньев участвует во вращатель- ном движении? 2) Трактор прошел путь S^>na. Сколько времени каждое звено гусеницы двигалось поступательно, покоилось и участ- вовало во вращательном движении? 55. Для того чтобы повернуть трактор, движущийся со скоростью и0=18 км/ч, тракторист притормаживает одну из гусениц так, что ось ее ведущего колеса начинает дви- гаться вперед со скоростью Vi=14 км/ч. Расстояние между гусеницами d=l,5 м. Дугу какого радиуса опишет центр трактора? 56. Можно наблюдать в горах следующее явление: звез- да «на глазах» быстро скрывается за отдаленной вершиной. (То же явление можно, конечно, наблюдать на равнине, ес- ли имеется достаточно удаленное высокое сооружение.) С какой скоростью нужно бежать, чтобы все время видеть звезду на одном и том же угловом расстоянии от горы? Рас- стояние от наблюдателя до вершины 10 км. Наблюдение ве- дется на полюсе. 57. Сплошной диск катится без скольжения по гори- зонтальному участку пути с постоянной скоростью v (рис. 16). 1) Доказать, что линейная скорость вращения относи- тельно центра О любой точки диска, лежащей на его ободе, равна скорости поступательного движения диска. 16
2) Определить величину и направление скорости то- чек Л, В, С и D, лежащих на ободе диска, относительно не- подвижного наблюдателя. 3) Какие точки диска имеют, относительно неподвижно- го наблюдателя, ту же по абсолютной величине скорость, что и центр диска? Л Рис. 16. Рис. 17. 68. Палочка АВ длины I движется в плоскости чертежа (рис. 17) так, что в данный момент времени скорость ее кон- ца А направлена под углом а, а скорость конца В — под углом 0 к палочке. Величина скорости конца А равна v. Определить величину скорости конца В. Найти положение неподвижной оси, перпендикулярной плоскости чертежа, относительно которой в рассматриваемый момент времени палочка вращается (т. е. найти положение мгновенной оси вращения палочки). 59. Начертить распределение скоростей вдоль палочки АВ в условиях предыдущей задачи. 60. Толпа муравьев волочит кусочек коры в форме рав- ностороннего треугольника. Известно, что в некоторый мо- мент времени скорость вершины В равна v и найравлена вдоль АВ, а скорость вершины С направлена вдоль СВ. Найти величину скорости вершины С в тот же момент. 61. Легковая машина едет по горизонтальному шоссе за грузовиком. Между двойными шинами задних колес грузо- вика застрял камень. На каком расстоянии от грузовика должна ехать легковая машина, чтобы камень, вырвавший- ся из колес грузовика, не попал в нее? Машины движутся со скоростью 50 км/ч. 62. На экране демонстрируется движущаяся повоз- ка. Радиусы передних колес повозки г=0,35 м, задних — /?=1,5 г. Передние колеса имеют Afi=6 спиц. Съемочная ка- мера перемещает пленку со скоростью 24 кадра в секунду. Считая, что колеса повозки катятся без скольжения, опреде- 17
лить, с какой минимальной скоростью должна даигаться повозка, чтобы передние колеса ее казались на экране не- вращающимися? Какое наименьшее число спиц Na должны иметь при этом задние колеса, чтобы они также казались не- вращающимися? 63. При каких скоростях движения повозки, движущей- ся слева направо (см. условие задачи 62), кинозрителям бу- дет казаться, что 1) спицы колес вращаются против часо- вой стрелки? 2) спицы передних колес и задних колес вра- щаются в противоположные сто- роны? Число спиц передних и зад- них колес одинаково и равно 6. 64. Катушка, состоящая из цилиндрической части и двух одинаковых сплошных дисков, катится своей цилиндрической частью без скольжения по шеро- ховатому бруску, расположен- ному горизонтально, с постоян- ной скоростью» (рис. 18). Ради- Рис. 18. ус цилиндрической части г, дисков R. Какую скорость име- ют точки А и В, находящиеся на ободе одного из дисков? 65. Какие точки дисков из предыдущей задачи имеют мгновенную скорость, равную по величине скорости оси катушки? 66. Начертить траектории точен А, В и С катушки (рис. 19), которая катится по бруску без скольжения своей цилиндрической частью. 67. Шарикоподшипник поддерживает конец оси вала, вращающегося с угловой скоростью <о. Диаметр оси вала d (рис. 20), диаметр обоймы шарикоподшипника D. Найти ли- нейную скорость движения центра одного из шариков, если 18
обойма неподвижна и если обойма вращается с угловой ско- ростью й. Считать, что в обоих случаях шарики катятся по валу и обойме без скольжения. 68. По плоскости без скольжения катится конус. Ось конуса вращается со скоростью <л вокруг вертикали, про- ходящей через его вершину. Высота конуса h, угол между осью и образующей а. Чему равна угловая скорость вра- щения конуса вокруг своей оси? Определить линейную ско- рость произвольной точки диаметра основания конуса, лежащего в вертикальной плоскости. 69. На рис. 21 схематически изображена дифферен- циальная передача автомобиля, которая применяется для Рис. 21. того, чтобы ведущие колеса автомобиля не скользили при прохождении ими криволинейного участка пути. (При этом они должны вращаться с различными скоростями.) Двигатель вращает колесо В, с которым жестко скрепле- на ось А. Вокруг нее может свободно вращаться пара кони- ческих шестерен Е. Эта пара шестерен связана зубьями с другой парой конических шестерен, по которым она катит- ся. Ось ведущих колес автомобиля (обычно задних) разреза- на пополам, и на концах оси сидят шестерни С и D. Эти половинки оси могут вращаться с различными угловыми скоростями, оставаясь связанными дифференциальной пере- дачей. Найти соотношение между угловыми скоростями й, ®, ®г и ®2 дифференциальной передачи, если радиусы шес- терен Е равны г, а радиусы шестерен D и С равны н. 70. Четыре черепахи находятся в углах квадрата со стороной а. Одновременно они начинают двигаться с пос- тоянной по величине скоростью v, причем первая черепаха 19
все время держит курс на вторую, вторая на третью, третья на четвертую, четвертая на первую. Встретятся ли черепа- хи, и если встретятся, то через какое время? 71. От прямолинейного участка берега отошли одно- временно два корабля Л и В, находившиеся первоначально на расстоянии п=3 км друг от друга. Корабль А двигался по прямой, перпендикулярной к берегу. Корабль В держал непрерывно курс на корабль А, имея в каждый момент оди- наковую с ним скорость. Очевидно, что через достаточно большое время второй корабль будет следовать за первым, находясь от него на некотором расстоянии. Найти это рас- стояние. 72. На песке покоятся две стальные плиты М и N вы- соты Н (рис. 22). Расстояние между плитами /=20 см. По плите М равномерно движется ша- рик, скорость которого точно не определена. Известно только, что она лежит в интервале от 200 см/с до 267 см/с. 1) При какой высоте Н нельзя предсказать направление скорости шарика по горизонтали в момент падения его на песок? (До падения на песок шарик успевает удариться о плиту N по крайней мере раз.) 2) При какой наименьшей вы- соте плит нельзя будет предсказать, на каком участке от- резка I упадет шарик? Временем соударения шарика с плитой пренебречь. Удар считать абсолютно упругим. § 4. Динамика прямолинейного движения 73. Однородный брусок, масса которого Af, движется ускоренно под действием силы F по гладкой поверхности. Найти величину Т силы, с которой часть бруска А длины х действует на часть бруска В. Длина бруска Z. (Рис. 23.) F Рис. 23. Рис. 24. 20
74. Однородный брусок движется ускоренно под дей- ствием силы F. Масса бруска М. Определить силы, дейст- вующие на заштрихованную часть бруска. Линейные раз- меры указаны на рис. 24. Трением пренебречь. 75. На однородный стержень длины L действуют две силы Fi и Fit приложенные к его концам и направленные в противоположные стороны (рис. 25). С какой силой F растянут стержень в сечении, находящемся на расстоянии I от одного из его концов? 76. Брусок массы т лежит на полу лифта. Лифт дви- жется с ускорением а, направленным вниз. Определить силу, с которой брусок действует на пол лифта. При каком ускорении лифта исчезают деформации в бруске? С какой силой брусок будет действовать на пол лифта, если лифт нач- нет двигаться с ускорением а, направленным вверх? 77. Мальчик массы М бежит вверх по неподвижной дос- ке массы т, находящейся на наклонной плоскости с углом при основании а. Трение между доской и плоскостью от- сутствует. Какой путь прошел мальчик к моменту, когда его скорость, равная вначале и0, уменьшилась в 2 раза, оставаясь той же по направлению? 78. Однородный брусок висит на нити. Нить разре- зают. У каких частиц бруска будет большее ускорение в начальный момент времени: у верхних или у нижних? 79. Однородный брусок лежит на горизонтальной под- ставке. Подставку внезапно убирают. У каких частиц брус- ка будет большее ускорение в начальный момент времени: у верхних или у нижних? 80. Человек с. поднятыми руками стоит на платформе медицинских весов. Как изменяются показания весов при ускоренном движении рук вниз? 81. На одной чашке весов находится бутылка. В бу- тылке — муха. Пока муха спит, весы уравновешены. На- рушится ли равновесие весов, когда проснувшаяся муха оторвется от стенки бутылки и полетит сначала в горизон- тальном направлении, а затем вверх с ускорением а? 21
82. На концах нити, перекинутой через блок с непод- вижной осью, висят на высоте Я=2 м от пола два груза, массы которых /П1=100 г и mt=200 г (рис. 26). В начальный момент грузы покоятся. Определить натяжение нити при движении грузов и вре- мя, за которое груз мас- сы т3 достигнет пола. Массу блока и нити не учитывать. 83. К оси подвижно- го блока прикреплен груз весом Р (рис. 27). С какой силой F нужно тянуть конец нити, пе- рекинутой через второй блок, чтобы груз Р двигался вверх с уско- рением а? Чтобы груз покоился? Массой бло- Рис. 26. Рис, 27. ков и нити пренебречь. 84. Определить ускорения грузов с массами т3, т3, а также натяжение нитей в системе, изображенной на рис. 28, если m1=ms+m8. Массы нитей и блоков нич- тожно малы по сравнению с массами грузов. 85. Через два неподвижных блока перекинута нить, к концам которой подвешены чашки с гирями весом по Р=» =30 Н на каждой. Нить между блоками разрезали и присое- динили к динамометру (рис. 29). Что покажет динамометр? 22
Какой груз Pt нужно добавить на одну из чашек для того, чтобы показание динамометра не изменилось после того, как с другой чашки была снята гиря весом Ра=10 Н? Массами чашек, блоков, нити и динамометра пренебречь. 86. На нити, перекинутой через блок, подвешены гру- зы с массами mi и /п2. Блок в заторможенном состоянии (грузы не движутся) уравновешен на рычажных весах, как показано на рис. 30. На сколько придется изменить вес гирь на правой чашке, чтобы при освобождении блока и последующем движении грузов равновесие сохранилось? № Рис. 31. 87. Система состоит из двух блоков с неподвижными осями и одного подвижного (рис. 31). Через блоки переки- нута нить, на концах которой подве- шены грузы с массами и т3, а к оси подвижного блока подвешен груз с массой т2. Участки нити, не лежа- щие на блоках, вертикальны. Опреде- лить ускорение каждого из грузов, если массами блоков и нитей, а также тре- нием можно пренебречь. 88. Определить ускорения грузов в системе, изображенной на рис. 32. Мас- сами блоков, нити и трением пренебречь. В какую сторону будут вращаться блоки при движении грузов? 89. Определить натяжения нитей, на которых висят грузы в системе, изоб- раженной на рис. 33. Массой нитей и блоков пренебречь. Трение отсутствует. Массы грузов ти mt, та, mit ть, mt, Рис. 32. m,, ms известны," 23
90. Определить ускорение груза массы в систе- ме, изображенной на рис. 34. Массой нитей и блоков Рис. 34. | Inyfaa пренебречь. Трение отсутствует. Массы грузов mlt тг, т3, т4 указаны на чертеже. 91. Тележка массы Л!=500 г скреплена нитью с грузом массы т=200 г. В начальный момент тележка имела ско- рость v0—7 м/с и двигалась влево по горизонтальной плос- кости (рис. 35). Определить величину и направление скоро- сти тележки, место, где она будет находиться, и путь, прой- денный ею, через /=5 с. Рис. 35. Рис. 36. 92. Для механической системы, изображенной на рис. 36, определить ускорения тел, массы которых m1, та и тя. Трение между соприкасающимися поверхностями отсутст- вует. Массой блока А и нити пренебречь. 24
93. Между двумя неподвижными муфтами может без тре- ния перемещаться вниз и вверх стержень, масса которого т. Стержень нижним концом касается гладкой поверх- ности клина массы М. Клин лежит на гладком горизонтальном столе (рис. 37). Определить уско- рения клина и стержня. 94. На стержень, дли- на которого 21, надета бу- синка массы т. Бусинка может перемещаться по рис. 37. стержню без трения. В на- чальный момент бусинка находилась на середине стержня. Стержень поступательно передвигался в горизонтальной плоскости с ускорением а в направлении, составляющем угол а со стержнем (рис. 38). Определить ускорение бусинки относительно стержня, силу реакции со стороны стержня на бусинку и время, через которое бусинка покинет стержень. 95. Невесомая нить, перекинутая че- рез блок с неподвижной осью, пропущена через щель (рис. 39); при движении нити на нее со стороны щели действует постоянная сила трения F. На концах нити подвешены грузы, массы которых и т,. Определить ускорение грузов. 96. К концам пружи- м — ны прикреплены два Рис 40 бруска, массы которых равны М и т (М>т). Под действием двух равных сил F, приложенных к брускам, как показано на рис. 40, пружи- на сжата. Бруски покоятся на столе. Что произойдет, 25
если силы F перестанут действовать? Коэффициент тре- ния брусков о стол равен k. 97. На задней стенке вагона на нити, перекинутой че- рез гвоздь, висит картина. Как будет двигаться картина относительно вагона, если нить оборвется, в случае, когда 1) скорость вагона возрастает? 2) скорость вагона убыва- ет? В обоих случаях абсолютная величина ускорения ваго- на равна а. Коэффициент трения картины о стену вагона равен Ь. 98. Два груза с массами и т2 связаны между собой нитью, перекинутой через блок. Плоскости, на которых ле- жат грузы, составляют с горизонталью углы аир (рис.41). Правый груз находится ниже левого на величину Л. Через время т после начала движения оба груза оказались на одной высоте. Коэффициент трения между грузами'и плоскостями равен k. Определить отношение масс грузов. 99. Из точки О по желобам, лежащим в одной вертикаль- ной плоскости и составляющим разнообразные углы с вер- тикалью, одновременно начинают соскальзывать песчинки. Найти геометрическое место точек, в которых окажутся пес- чинки через время t, если коэффициент трения каждой пес- чинки о желоб равен k. 100. Ледяная гора составляет с горизонтом угол а=30°; по ней пускают снизу вверх камень, который в течение tt= =2 с проходит расстояние /= 16 м, после чего соскальзывает вниз. Какой промежуток времени t2 длится соскальзывание камня вниз? Каков коэффициент трения между горой и камнем? 101. Тележка массы М движется без трения по горизон- тальным рельсам со скоростью о0- На передний край те- лежки кладется тело массы т. Его начальная скорость рав- на нулю. При какой длине тележки тело не соскользнет с нее? Размерами тела по сравнению с длиной тележки I пре- небречь. Коэффициент трения между телом и тележкой равен k. 26
102. Брусок массы М лежит на горизонтальной плос- кости. На бруске лежит тело массы т (рис. 42). Коэффициент трения между телом и бруском, а также между бруском и плоскостью, равен, k. Исследовать движение при различ- ных значениях силы F. • Рис. 42. 103. Брусок массы М лежит на гладкой горизонталь- ной плоскости, по которой он может двигаться без тре- ния. На бруске лежит тело массы т (рис. 42). Коэффициент трения между телом и бруском равен k. При каком зна- чении силы F, приложенной к бруску в горизонтальном направлении, тело начнет скользить по бруску? Через сколько времени тело упадет с бруска? Длина бруска I. 104. Стол весом /\= 150 Н может без трения перемещать- ся по горизонтальному полу. На столе лежит груз весом Р»= 100 Н. К грузу прикреплена веревка, перекинутая через два блока, закрепленных на столе (рис. 43). Коэффициент трения между грузом и столом Л=0,6. С каким ускорением будет двигаться стол, если к свободному концу веревки при- ложить постоянную силу, равную 80 Н? Рассмотреть два случая: 1) сила направлена’Горизонтально; 2) сила направ- лена вертикально вверх. 105. На стержень, длина которого 21, надета бусинка массы т. Коэффициент трения между бусинкой и стержнем равен k. В начальный момент бусинка находилась на середи- 27
не стержня. Стержень поступательно передвигается в го- ризонтальной плоскости с ускорением а в направлении, составляющем угол а со стержнем (рис. 38). Определить ус- корение бусинки относительно стержня, силу реакции со стороны стержня на бусинку и время, через которое бусин- ка покинет стержень. Силу тяжести не учитывать. 106. Старинная пушка, не имеющая противооткатного устройства, стоит на горизонтальной площадке. Пушка стреляет под углом а к горизонту снарядом, масса которого т, а начальная скорость о0. Какую скорость ог будет иметь пушка непосредственно после выстрела, если масса пушки М, а ускорение снаряда в канале ствола значительно боль- ше ускорения свободного падения? Коэффициент трения между пушкой и площадкой равен k. 107. Ракета имеет запас топлива т=8 тонн. Масса ракеты (включая топливо) Л4=15 тонн. Топливо сгорает за 40 с. Расход топлива и сила тяги F=200 ООО Н посто- янны. . 1) Ракета установлена горизонтально на тележке. Оп- ределить ускорение, которое имеет ракета в момент запуска. Найти зависимость ускорения от времени движения раке- ты и изобразить эту зависимость графически. По графику оценить величину скорости, которую приобретает ракета через 20 с после начала движения. Трение не учитывать. 2) Ракета запускается вертикально вверх. Измерения по- казали, что через 20 с ускорение ракеты было равно 0,8 g. Рассчитать силу сопротивления воздуха, которая действо- вала на ракету в этот момент. Ускорение g считать посто- янным. 3) Для измерения ускорения ракеты на ней помещен прибор, представляющий собой пружину, закрепленную в вертикальной трубке. В покое пружина растянута при- крепленным на ее конце грузом на /0=1 см. Определить за- висимость растяжения пружины от ускорения ракеты. На- чертить шкалу прибора. 108. Два кубика сложены вплотную и лежат на абсо- лютно гладкой горизонтальной поверхности. Ребро каж- дого кубика равно I, а масса равна М. В кубик попада- ет пуля массы т, летящая вдоль .линии центров кубиков. Считая горизонтальную силу сопротивления, возникающую при движении пули, постоянной и равной F, определить, в каких пределах должна находиться начальная скорость пули, чтобы последняя прошла через первый кубик и за- стряла во втором 28
§ 5. Закон сохранения количества движения 109. Обладает ли количеством движения однородный диск, вращающийся вокруг своей оси? Ось диска непо- движна. ПО. Две лодки идут параллельными курсами навстре- чу друг другу с одинаковыми скоростями. Когда лодки встречаются, с одной лодки на другую перебрасывают мешок, а затем со второй лодки йа первую перебрасывают такой же мешок. В другой раз мешки перекидывают из лодки в лод- ку одновременно. В каком случае скорость лодок после пе- ребрасывания грузов будет больше? 111. Лягушка массы т сидит на конце доски массы М и длины L. Доска плавает на поверхности пруда. Лягушка прыгает под углом а к горизонту вдоль доски. Какой долж- на быть при этом начальная скорость лягушки ц0, чтобы пос- ле прыжка лягушка оказалась на другом конце доски? 112. Клин с углом при основании а лежит на гладком горизонтальном столе. По наклонной поверхности клина ползет вверх жук с постоянной относительно клина скоро- стью и. Определить скорость клина. Предполагается, что жук начал ползти, когда клин покоился. Масса клина равна 7И, масса жука равна т. 113. Клин с углом при основании а может без трения перемещаться по гладкой горизонтальной поверхности (рис. 44). При каком соотношении масс тх и /п2 грузов, ^/П Рис. 44. связанных нитью, перекинутой через блок, клин будет неподвижен и при каком соотношении масс клин начнет перемещаться вправо или влево? Коэффициент трения меж- ду грузом массы /п2 и клином равен k. 114. Вдоль гладкой наклонной плоскости, составляю- щей угол а с горизонталью, начал соскальзывать с нулевой начальной скоростью ящик с песком массы М. После того, как ящик прошел путьЗ, в него попал камень массы т, под- летевший по горизонтальному направлению. Какой была 29
скорость камня и, если ящик с песком после попадания в не- го камня на мгновенье остановился? Скорости камня и ящи- ка лежат в одной плоскости. 115. Ракета, запущенная вертикально вверх, взрывает- ся в высшей точке своего подъема. При взрыве образуются три осколка. Доказать, что векторы начальных скоростей всех трех осколков лежат в одной плоскости. 116. На поверхности озера находится лодка. Она пер- пендикулярна линии берега и обращена к нему носом. Рас- стояние между носом лодки и берегом равно 0,75 м. В на- чальный момент лодка была неподвижна. Человек, находя- щийся в лодке, переходит с носа лодки на корму. Причалит ли лодка к берегу, если ее длина 2 м? Масса лодки Л1= 140 кг, масса человека /п=60 кг. 117. С концов неподвижной платформы длиной /=9,2 м бегут навстречу друг другу взрослый и ребенок. Определить, на сколько откатится платформа, когда взрослый добежит с одного конца платформы до другого. Известно, что взрос- лый бежит в два раза быстрее, чем ребенок. Масса платфор- мы /«1=600 кг, масса взрослого ms=60 кг, ребенка т»=30 кг. 118. На абсолютно гладкой горизонтальной плоскости лежит обруч. На обруче находится жук. Какую траекторию будут описывать жук и центр обруча, если жук начнет дви- гаться вдоль обруча? Масса обруча М, радиус R, масса жука т. 119. В начальный момент ракета массы М имела ско- рость о0- В конце каждой секунды из ракеты выбрасывается порция газа массы т. Скорость порции газа отличается от скорости ракеты до сгорания данной массы газа на постоян- ную величину и, т. е. скорость истечения газа постоянна. Пренебрегая действием силы тяжести, определить скорость ракеты через п секунд. 120. Будет ли увеличиваться скорость ракеты, если ско- рость истечения газов относительно ракеты меньше скорости самой ракеты, т. е. вытекающие из сопла ракеты газы ле- тят вслед за ракетой? 121. Пушка массы М, которая может двигаться только по горизонтали, стреляет под углом а к горизонту снарядом массы т, вылетающим со скоростью и0. Понимая под va начальную скорость снаряда относительно земли либо от- носительно пушки, а под углом а — угод наклона вектора начальной скорости v0 или угод наклона ствола пушки, найти скорость о отката пушки для всех четырех вариантов. 30
§6. Статика — 122. На неподвижный цилиндр намотана нить, длина ко* торой l=RQ, где R — радиус цилиндра, 0 — угол в радиа- нах между радиусами, проведенными в начальную и конеч- ную точки касания нити с цилиндром. Один конец нити тянут с силой Г». Коэффициент трения между нитью и по- верхностью цилиндра равен k. Определить силу натяжения второго конца нити, если известно, что это максимальная сила, при которой еще отсутствует проскальзывание. £ 123. На концах нити, перекинутой через два блока, ви- сят два одинаковых груза (рис. 45). На какое расстояние опустится третий груз той же массы, если его прикрепить к середине нити? Расстояние между осями блоков равно 21. Трение в осях блоков су- ществует, но очень мало. ТГ Рис. 45. Рис. 46. “ 124. Равнобедренный клин с острым углом а забит в щель. При каких значениях угла а клин не будет вытолкнут из щели, если коэффициент трения между клином и мате- риалом щели равен k? 125. Каково соотношение между грузами Р и Q, если ‘ известно, что система, изображенная на рис. 46, находится в равновесии? Длина стержней AD, ВС, CH, DT и длина плеча 001 в два раза больше длины стержней АЕ, ЕВ, TS, SH и длины плеча КО соответственно. Весом стержней и рычага пренебречь. 126. Для того чтобы сдвинуть прямоугольный ящик Длины I и высоты h, к его верхнему ребру перпендикулярно грани прикладывают горизонтальную силу F. Какую вели- чину должен иметь коэффициент k трения между ящиком и полом, чтобы ящик сдвинулся не опрокидываясь? 31
127. Однородная балка, вес которой Р, лежит на полу. Коэффициент трения балки о пол равен k. Что легче: повер- нуть балку в горизонтальной плоскости относительно ее центра или перемещать балку поступательно? В обоих слу- чаях балку двигают два человека. 128. Мостовой кран, вес которого Р=2-10‘ Н, имеет пролет £=26 м (рис. 47). Трос, на котором подвешен груз, находится на расстоянии Z=10 м от одного из рельсов. Определить,, силы давления крана на рельсы, если он подни- мает груз весом Ро= 10* Н с ускорением а=9,8 м/с*. 129. Рычаг изогнут так, что стороны его АВ, ВС и CD равны между собой и образуют друг с другом прямые уг- лы (рис. 48). Ось рычага—в точке В. Перпендикулярно пле- чу рычага АВ в точке А приложена сила Р. Определить ми- нимальное значение силы, которую нужно приложить в точке D, чтобы рычаг на- ходился в равновесии. Be- сом рычага пренебречь. ri j_ АВ Рис. 49. Рис. 4в. 130. Между двумя одинаковыми ящиками, стоящими на полу, вставлена палка, немного не доходящая до пола (рис. 49). К верхнему концу палки приложена горизонталь- ная сила. Какой из ящиков сдвинется раньше? 131. Тяжелый однородный шар подвешен на нити, ко- нец которой закреплен на вертикальной стене. Точка при- крепления нити к шару находится на одной вертикали с 32
центром шара. Какую величину должен иметь коэффициент трения между шаром и стеной, чтобы шар находился в рав- новесии? 132. Кирпич лежит на наклонной плоскости, прилегая к ней всем основанием (рис. 50). Какая половина кирпича, правая или левая, оказывает большее давление на наклон- ную плоскость? 133. Для подъема тяже- лого цилиндрического катка радиуса А? на прямоугольную ступеньку пришлось прило- жить к его оси горизонталь- но направленную силу, равную весу катка. Определить максимальную высоту ступеньки. 134. На двух наклонных плоскостях, образующих с горизонтом углы а!=30° и а2=60°, лежит шар весом Р. Определить силы давления шара на каждую из плоскостей, если известно, что трение между шаром и одной из плоскостей отсутствует. I 135. На передней стенке ящика шкафа имеются две сим- ' метрично расположенные ручки. Расстояние между руч- ками Z, длина ящика а. Коэффициент трения между ящиком и шкафом равен k. Всегда ли можно выдвинуть ящик из шкафа, прикладывая только к одной ручке силу, перпен- дикулярную стенке ящика? ДГ136. На шероховатом, горизонтально расположенном 1 бревне уравновешена однородная доска (рис. 51). После того, как утяжелили один из ее концов, обнаружилось, что равно- весие достигается, когда доска со- ставляет угол а с горизонтом. Каков коэффициент трения между доской и бревном? 137. Верхний конец лестницы опирается на гладкую вертикаль- ную стену, а нижний конец лестницы стоит на шерохо- ватом полу. Коэффициент трения между лестницей и полом равен k. Определить, при каком значении угла а между лестницей и стеной лестница будет находиться в равновесии. 138. Решить предыдущую задачу в предположении, что стена не гладкая и коэффициент трения между лестни- цей и стеной также равен kt 2 Б. Б. Буховцев я др. 4 Рис. 51. 33
139. Однородная тонкая палочка А В длиной I лежит на горизонтальной поверхности стола. К концу В палочки при- креплена нить длиной 21 (рис. 52). Как будет двигаться па- лочка, если другой конец нити С медленно поднимать вдоль неподвижной вертикальной прямой DO, проходящей че- рез конец А палочки? Весом нити пренебречь. 140. При каком значении коэффициента трения человек, бегущий по прямой твердой дорожке, не может поскольз- нуться? Максимальный угол между вертикалью и линией, соединяющей центр тяжести бегуна с точкой опоры, ра- вен а. Рис. 52. Рис. 53. Рис. 54. 141. К гладкой вертикальной стене дома прислонена лестница. Угол между лестницей и горизонтальной поверх- ностью земли а=60а. Длина лестницы /. Центр тяжести ее находится посередине. Как направлена сила, действующая на лестницу со стороны земли? 142. Лестница, центр тяжести которой находится посе- редине, опирается на абсолютно гладкие пол и стену (рис. 53). Каким должно было бы быть натяжение веревки, привязанной к се- редине лестницы, чтобы удержать ее от падения? 143. По лестнице, при- слоненной к гладкой верти- кальной стене, поднимает- ся человек. Лестница начи- нает скользить лишь тогда, когда человек поднялся на определенную- высоту. По- чему? 144. Картина подвеше- на к вертикальной стене с помощью шнура АС длины /, образующего со стеной угол а. Высота картины BC=d (рис. 54). Нижняя часть кар- тины не закреплена. При каком значении коэффициента трения между картиной и стеной картина будет в равно- весии? 34
145. Четыре однородных стержня шарнирно скреплены друг с другом в точках В, С и D (рис. 55). Два крайних стержня АВ и DE могут свободно вращаться относительно неподвижных точек А и Е, расположенных на горизонталь- ной прямой. Длины стержней попарно равны, AB—ED и ВС—CD. Массы стержней одинаковы. Показать, что при равновесии углы а и 0 связаны соотношением tga = 3tg₽. 146. Чему равен коэффициент трения между полом и ящиком весом в 100 Н, если наименьшая си ла* необходимая для того, чтобы сдвинуть ящик с места, составляет 60 Н? 147. На цилиндр массы т намотана невесомая нерастя- жимая нить (рис. 56). С какой наименьшей силой Fm[n и под каким углом ах к горизонту нужно тянуть эту нить, чтобы цилиндр, вращаясь, оставался на месте? Коэффициент трения между цилиндром и полом равен k. Рис. 55. 148. На рис. 57 изображена упрощенная схема паровой машины и кривошипно-шатунного механизма паровоза. Рис. 57, а и б соответствуют моментам, когда пар находится в левой и правой частях цилиндра соответственно. Рас- считать силу тяги для этих случаев в момент, когда точка А лежит на одной вертикали с осью ведущего колеса. Давле- ние пара в цилиндре р, площадь поршня S, радиус ведущего 2* 35
колеса R, а расстояние ОЛ=г. Массой кривошипно-шатун- ного механизма, поршня и ведущего колеса пренебречь. 149. Кирпичи укладывают друг на друга без связующе- го вещества так, что часть каждого последующего кирпича выступает над нижележащим (рис. 58). На какое максималь- ное расстояние правый край верхнего кирпича может вы- ступать над правым краем самого ^_| __ нижнего кирпича, служащего осно- 1 ванием всей кладки? Длина каждого —---------—' кирпича I. ---------- 150. Найти центр тяжести тонкой Рис. 58. однородной проволоки, согнутой в виде полуокружности радиуса г. 151. Определить положение центра тяжести однород- ного тонкого полукруга радиуса г. 152. Определить положение центра тяжести тонкой од- нородной проволоки, изогнутой по дуге радиуса г (рис. 59). Рис. 61. - 153. Определить положение центра тяжести тонкой од- нородной пластинки, вырезанной в виде сектора радиуса г, имеющего центральный угол а (рис. 60). 154. Определить положение центра тяжести тонкой од- нородной пластинки, представляющей собой прямоуголь- ник со сторонами г и 2г, из которого вырезан полукруг ра- диуса г (рис. 61). § 7. Работа и энергия 155. Какая работа будет совершена, если силой 30 Н поднять груз весом 10 Н на высоту 5 м? 156. Опыт Герике с «магдебургскими полушариями» состоял в том, что две медные полусферы плотно соединя- 36
лись основаниями и из получившегося полого шара выка- чивался воздух. Атмосферное давление настолько плотно прижимало полушария друг к другу, что их могли разъеди- нить только с помощью нескольких лошадей. Определить, сколько лошадей нужно для отрыва полушарий, если каж- дая лошадь тянет с силой F? Радиус полусферы /?, атмосфер- ное давление равно р. 157. Как объяснить тот факт, что при падении камня на Землю изменение количества движения Земли равно изме- нению количества движения камня, а изменение кинети- ческой энергии Земли не учитывается? 158. Сваю массой в 100 кг забивают в грунт копром, мас- са которого 400 кг. Копер свободно падает с высоты 5 м, и при каждом его ударе свая опускается на глубину 5 см. Определить силу сопротивления грунта, считая ее посто- янной. 159. Ящик с песком, имеющий массу М, подвешен на тросе длины L. Длина троса значительно больше линейных размеров ящика. Пуля, масса которой т, летит в горизон- тальном направлении, попадает в ящик и застревает в нем. Трос после попадания пули отклоняется на угол а от вер- тикали. Определить скорость пули. 160. Две тележки с заклиненными колесами расталки- ваются взрывом заряда Q, помещенного между ними (рис.62). Рис. 62. Тележка массой 100 г проходит дуть 18 м и останавливается. Какой путь пройдет вторая тележка массой 300 г? Коэффи- циент трения между землей и тележками равен k, 161. Решить задачу 101, применяя закон сохранения ко- личества движения и рассматривая изменение кинетической энергии тележки и тела. 162. Ракета взлетает вертикально вверх, выбрасывая раскаленные газы последовательно двумя равными пор- циями. Скорость истечения газов относительно ракеты по- стоянна и равна и. Каким должен быть промежуток време- ни между сгоранием порций, чтобы ракета достигла макси- мальной высоты? Сгорание топлива происходит мгновенно. Сопротивлением воздуха пренебречь. 37
163. Горючее ракеты сгорает равными порциями мас- сы т. Сгорание происходит мгновенно. Будет ли скорость истечения газов относительно ракеты постоянна, если при сгорании каждой порции механическая энергия системы меняется на одинаковую величину? 164. Тело поднимают на вершину горы один раз по пути ADC и другой раз — по АВС (рис. 63). Доказать, что при медленном подъеме совер- -j-£ шенная работа будет одной и Л 1 \\ той же, если коэффициент тре- | \\ ния на обоих склонах одинаков. } \ \ 165. Какую силу следует | I у? приложить к рукоятке винто- п । .А \ вого домкрата, чтобы удержать I \. в равновесии поднятый домкра- ! том груз Р? Шаг винта равен h, v длина рукоятки Р. Трение от- ур сутствует. г* *4 166. Найти максимальный Рис. 63. коэффициент полезного действия винтового домкрата, у которого силы трения не позволяют грузу опускаться. 167. К корзине воздушного шара массы М привязана веревочная лестница длины I, на конце которой стоит чело- век массы т. Вся система находится в воздухе в положении равновесия. Определить, какую работу должен совершить человек, чтобы подняться в корзину. Какова будет скорость воздушного шара, если человек перемещается по лестнице со скоростью v относительно нее? 168. Как должна измениться мощность мотора насоса, чтобы он стал перегонять через узкое отверстие вдвое боль- шее количество воды в единицу времени? 169. Прямоугольная яма, площадь основания которой S и глубина Н, наполовину заполнена водой. Насос выка- чивает воду и подает ее на поверхность земли через цилинд- рическую трубу радиуса R. 1) Какую работу совершил насос, если он выкачал всю воду за время т? 2) Какую работу совершил насос за то же время, если на дне ямы лежит каменная плита прямоугольной формы, площадь основания которой Si и высота й? (Глубина воды в яме по-прежнему равна Н)2.) 170. Какую работу нужно совершить, чтобы за время т подняться по движущемуся вниз эскалатору метро? Вы- 33
сота подъема h, скорость эскалатора постоянна н равна v, угол наклона эскалатора к горизонту а. 171. Берем пружину за среднюю точку О и оттягиваем на расстояние х (рис. 64), а затем отпускаем. Пружина быст- ро становится растянутой равномерно, причем переход к этому состоянию сопровождается некоторой потерей энер- гии. Оценить ее, считая жесткость k пружины очень боль- шой. (После того, как пружина станет растянутой равно- мерно, возникнут колебания груза т, сопровождающиеся дополнительными потерями энергии.) Рис. 65. Рис. 64. 172. В вагоне равномерно движущегося поезда стоит человек, растягивающий пружину с силой F (рис. 65). Поезд прошел путь L. Какую работу совершит человек в системе координат, связанной с Землей? 173. В вагоне равномерно движущегося поезда человек растянул пружину, прикрепленную к передней стенке ва- гона, на длину I. За это время поезд прошел путь L. Какую работу совершил человек в системе координат, связанной с Землей? Чему равна эта работа в системе, связанной с поез- дом? Растягивая пружину, человек идет против движения поезда. 174. Происходит соударение двух абсолютно упругих шаров, массы которых равны mi и т2. Начальные скорости шаров Ох и оа. Найти скорости шаров после удара. Удар считать центральным: скорости шаров направлены вдоль линии, соединяющей их центры. Проанализировать два случая: 1) скорость второго шара до удара равна нулю; 2) массы шаров равны. 175. Происходит соударение двух абсолютно упругих шаров, массы которых равны т* и т». Начальные скорости шаров и оа. Удар центральный. Определить максимальную энергию упругой деформации. 39
176. На абсолютно гладкой горизонтальной плоскости покоятся два упругих бруска равной массы т, скрепленных пружиной длины / (рис. 66). Коэффициент упругости пружи- ны равен k. На один из брусков, например на левый, на- летает со скоростью v третий брусок, имеющий также т т Рис. 66. массу т. Показать, что связанные пружиной бруски всегда будут двигаться в одну сторону. Определить скорости брус- ков в момент, когда пружина максимально растянута. 177. Две пластинки, массы которых равны /и, скреплены пружиной с коэффициентом жесткости k (рис. 67). Верхнюю пластинку опустили настолько, что деформация пружины стала равной х, и затем отпустили. Определить, на какую высоту поднимется после этого центр масс системы. Рис. 67. Рис. 68. 178. Шарик, движущийся со скоростью v, налетает на стенку, которая движется навстречу шарику со скоростью и (рис. 68). Происходит упругий удар. Определить ско- рость шарика после удара. За счет чего изменяется кинети- ческая энергия шарика? Массу стенки считать бесконечно большой. 179. С высоты й=73,5 м сбрасывают два одинаковых по массе камня, связанных веревкой, длина которой /=39,2 м. Первый камень начинает падать на т=2 с раньше второго. Через какое время после начала падения камни упадут на землю? Падение происходит без начальной скорости. Рас- смотреть два случая: 1) веревка абсолютно упругая; 2) веревка абсолютно неупругая. 180. Несколько одинаковых упругих шаров так под- вешены рядом на нитях равной длины (рис. 69), что расстоя- 40
ния между соседними шарами очень малы. Как будут вес- ти себя шары, если отклонить крайний шар и отпустить, отклонить одновременно два шара, отклонить три шара и т. д.? 181. На плоскости ле- жат в ряд (с небольшими промежутками) шарики одинакового размера (рис. 70). Один из средних шари- ков сделан из стали, ос- тальные — из слоновой ко- сти (масса стального ша- рика больше). На шарики справа вдоль линии центров налетает костяной шарик (той же массы). Как будут двигаться шарики после удара? Рис. 70. 182. На концах очень длинной нити подвешены равные грузы массы т (рис. 71). Нить перекинута через два малых неподвижных блока, расположенных на расстоянии 21 друг от друга. Найти скорости грузов по истечении доста- точно большого промежутка времени, если к середине нити прикреплен груз массы 2т. 183. Груз массой тг=536 г, удерживаемый первоначаль- но у потолка посередине между точками А и В, начинает 41
опускаться вниз (рис. 72). При каком значении угла ANB его скорость по абсолютной величине станет равной ско- рости груза массы /п8=1000 г? Как будут двигаться грузы в дальнейшем? 184. На двух катках разных радиусов лежит тяжелая доска, образующая угол а с горизонтом. Определить, как будет двигаться доска. Проскальзывание отсутствует. Мас- сой катков пренебречь. 185. Однородная цепочка длины 21 и массы М лежит на абсолютно гладкой доске. Небольшая часть цепочки про- пущена в отверстие, сделанное в доске (рис. 73). В началь- ный момент времени лежащий на доске конец цепочки Рис. 73. горизонтальным рельсам. придерживают, а затем отпускают, и цепочка начинает со- скальзывать с доски под действием силы тяжести свешиваю- щегося конца. Определить скорость движения цепочки в тот момент, когда длина свешивающейся части будет рав- на х(х</). Определить для того же момента времени уско- рение цепочки и реакцию края доски. 186. Тележка массы М может двигаться без трения по На тележке укреплен математи- ческий маятник (шарик мас- сы т, подвешенный на нити длиной Z) (рис. 74). В началь- ный момент времени тележка и маятник покоились, и нить отклонили на угол а от верти- кали. Какова скорость тележ- ки в тот момент, когда нить маятника будет составлять с вертикалью угол 0(0<а)? 187. Клин, масса которого М, находится на абсолютно гладкой горизонтальной плоскости. На клине лежит бру- сок массы т. Брусок под действием силы тяжести может скользить по клину без трения. Считая, что в начальный мо- мент времени система находилась в покое, определить ско- рость клина в тот момент времени, когда брусок опустит- ся по вертикали на высоту h. Рис. 74. 42
188. Стержень, закреплен- ный между двумя муфтами, может свободно перемещаться в вертикальном направлении (рис. 75). Нижний конец стер- жня опирается на гладкий клин, лежащий на гори- зонтальной плоскости. Масса стержня т, клина М. Трение отсутствует. В начальный мо- мент стержень и клин поко- ились. Определить: скорость v клина в момент, когда стер- жень опустится на высоту Л; сительно движущегося клина Рис. 75. скорость потн стержня отно- ; ускорение а стержня. § 8. Динамика криволинейного движения 189. Определить натяжение троса баллистического маятника (см. задачу 159) в первый момент после попада- ния в него пули. 190. На гибкой нерастяжимой нити, весом которой мож- но пренебречь, укреплены, как показано на рис. 76, че- тыре одинаковых груза. Вся система вращается с угловой скоростью со вокруг вертикальной оси, проходящей через т I т I л? I м +-----» « -е 5 % % Рис. 76. Рис. 77. точку О. Грузы движутся по гладкой горизонтальной по- верхности. Определить натяжение нити в различных участках. 191. На концах невесомого стержня длины / закрепле- ны массы /П1 и /и2. Скорости масс /nx и т2 лежат в одной плос- кости и равны соответственно и vz (рис. 77). Определить, с какой скоростью движется центр тяжести системы и 43
с какой угловой скоростью вращается стержень относи- тельно оси, проходящей через центр тяжести. 192. В центре платформы, свободно вращающейся во- круг вертикальной оси, стоит пушка. Ось вращения про- ходит через ее казенную часть. В горизонтальном направле- нии вдоль радиуса платформы сделан выстрел. Изменится ли при этом скорость вращения платформы? 193. Небольшое тело начинает скользить без начальной скорости с наклонной плоскости высоты Н (рис. 78). Счи- тая, что трение отсутствует и удар тела о горизонтальную Рис. 78. плоскость АВ абсолютно упругий, определить характер движения тела после того, как оно покинет наклонную плоскость. Ответить на тот же вопрос, если удар абсолютно неупругий. 194. Чему равен минимальный радиус дуги, которую мо- жет описать мотоциклист, если его скорость и=21 м/с, а коэффициент трения резины о землю Л=0,3? Под каким уг- лом а к горизонту должен быть при _/& этом наклонен мотоцикл, если счи- тать его массу сосредоточенной в центре масс? g 195. На легкий стержень насажен / массивный шар (рис. 79). В каком слу- / чае стержень упадет быстрее: если его / поставить вертикально на конец А или / на конец В? Стоящий на земле конец Л/ стержня не проскальзывает. 196. На конце легкого стержня, Рис. 79. поставленного вертикально на пол, закреплен массивный шар. Стержень начинает падать без начальной скорости. При каком зна- чении угла а между стержнем и вертикалью конец стержня перестанет давить на пол? При каком значении коэффициен- та трения конец стержня не проскользнет до этого момента? 44
197. На каком расстоянии от основания стержня упа- дет шар (см. задачу 196), если коэффициент трения k >/5/2? 198. Проволока изогнута по дуге окружности радиуса R (рис. 80). На проволоку надета бусинка, которая может без трения перемещаться вдоль проволоки. В начальный момент времени бусинка находилась в точке О. Какую го- ризонтальную скорость надо сообщить бусинке, чтобы,прой- дя часть пути в воздухе, она в точке В попала вновь на про- волоку? 199. Небольшое тело соскальзывает по наклонной по- верхности, переходящей в мертвую петлю, с минимальной высоты, при которой оно не отрывается от поверхности петли (рис. 81). Какой симметричный вырез, характеризуе- мый углом а<90°, можно сделать в петле, чтобы тело, про- делав часть пути в воздухе, попало в точку В петли? Как будет двигаться тело, если угол а будет больше или меньше найденного? Трением и сопротивлением воздуха пренеб- речь. Рис. 82. 200. К концам нити, переброшенной через два гвоздя, прикреплены двигающиеся по окружностям грузы (рис. 82). 45
Слева висят два груза массы т каждый, справа — один груз массы 2m. Будет ли система в равновесии? 201. На очень тонкой нити подвешен шарик. Нить при- водят в горизонтальное положение и отпускают. В каких точках траектории ускорение шарика направлено верти- кально вниз, в каких вертикально вверх и в каких горизон- тально? В начальный момент нить не растянута. 202. Невесомый стержень может вращаться в вертикаль- ной плоскости относительно точки О. На стержне укрепле- ны массы mt и та на расстояниях г\ и га от О (рис. 83). Стержень отпущен без начальной скорости из положения, составляющего угол а с вертикалью. Определить линейные скорости масс mt и та в тот момент, когда стержень занимает вертикальное положение. 203. На оси центробежной машины укреплен отвес, к которому привязан маленький шарик на нити длиной /=12,5 см. Найти угол а отклонения нити от вертикали, ес- ли машина делает 1 оборот в секунду, 2 оборота в секунду. 204. Невесомый стержень, изогнутый, как показано на рис. 84, вращается с угловой скоростью <л относительно оси 00'. К концу стержня прикреплен груз массы т. Опре- делить силу, с которой стержень действует на груз. 205. Невесомый стержень АОО’, изогнутый, как пока- зано на рис. 85, вращается с угловой скоростью со относи- тельно оси ОО'. На стержень надета бусинка массы т. Оп- ределить, на каком расстоянии / от точки О бусинка будет находиться в равновесии, если коэффициент трения между бусинкой и стержнем равен k. 46
206. К вертикальному стержню, вращающемуся с уг- ловой скоростью со, прикреплена нить длиной I, на конце которбй находится груз массы т. К грузу в свою очередь прикреплена другая нить такой же длины, несущая на своем конце второй груз массы т. Показать, что угол между первой нитью и вертикалью будет меньше угла между вертикалью и второй нитью. Весом нити пренебречь. 207. На невесомом стержне укреплены два груза с массами т и М. Стержень шарнирно связан с вертикальной осью 00' (рис. 86). Ось 00' вращается с угловой скоростью о. Определить угол <р, образуемый стержнем и вертикалью. 208. Горизонтальная прямая штанга вращается с по- стоянной угловой скоростью вокруг вертикальной оси. По штанге может без трения скользить тело. Первоначаль- но тело удерживается в положении равновесия пружиной (рис. 87). Что произойдет с телом, если сообщить ему на- .47
чальную скорость вдоль штанги? В нерастянутом состоя- нии длиной пружины можно пренебречь. 209. Металлическая цепочка длины /=62,8 см, кон- цы которой соединены, насажена на деревянный диск (рис. 88). Диск вращается, делая п=60 оборотов в секун- ду. Определить натяжение цепочки Т, если ее масса т=40 г. 210. Порезиновойтрубке, свернутой в виде кольца, цир- кулирует вода со скоростью» (рис. 89). Радиус кольца R, ди- аметр трубки d<^.R. С какой силой растянута резиновая трубка? 211. Однородный стержень, длина которого /, а масса пг, вращается с угловой скоростью со в горизонтальной плос- кости вокруг оси, проходящей через его конец. Найти на- тяжение стержня на расстоянии х от оси вращения. 212. Шарик массы пг, укрепленный на невесомом стержне, вращается с постоянной скоростью» в горизонтальной плос- кости (рис. 90). Его кинетическая энергия в системе коорди- нат, неподвижной относительно оси вращения, постоянна и равна то*1<2. По отношению к системе отсчета, движущей- ся в горизонтальной плоскости прямолинейно со скоростью » относительно оси, кинетическая энергия меняется с те- чением времени от нуля до 4т»2/2. Какая причина вызывает это изменение энергии? 213. Тонкий однородный обруч катится по горизонталь- ной поверхности с постоянной скоростью ». Как и под дей- ствием каких сил меняется полная энергия небольшого участка АВ, находящегося в данный момент в наивысшей точке обруча? 48
214. Тяжелая катушка, с намотанной на нее нитью, лежит на горизонтальной шероховатой поверхности, по которой может катиться без скольжения. Если нить тянуть в горизонтальном направлении влево, то катушка будет катить- ся также влево. Если же ме- нять направление нити (рис. 91), то при некотором значении угла а между направлением нити и вертикалью катушка покатится вправо. Определить этот угол. Что будет с катушкой при дан- ном значении угла? Радиус вне- шней части катушки Р, внут- ренней г. 215. Найти кинетическую энергию обруча массы М и радиуса /?, если он движется равномерно со скоростью и и вращается с угловой скоростью со вокруг оси, проходя- щей через центр. 216. Определить кинетическую энергию гусеницы трак- тора, движущегося со скоростью v. Расстояние между ося- ми колес, на которые надета гусеница, равно /; радиус ко- лес г. Единица длины гусеницы весит Р. 217. Как узнать, какой из двух одинаковых по вели- чине цилиндров полый, если они сделаны из неизвест- ных материалов разной плотности? Массы цилиндров равны. 218. На катушку радиуса R в один ряд намотан гиб- кий кабель (рис. 92). Вес единицы длины кабеля р. Вес всего кабеля Р. Катушка без проскальзывания дви- жется по инерции вдоль горизонтальной плоскости. Кабель разматывается и ложится на плоскость. Вначале, когда весь кабель был намотан на катушку, скорость центра катушки была равна v. Пренебрегая радиусом поперечного сечения кабеля (по сравнению с R) и массой самой катушки, оце- гхахххххххххххххххххххг] Рис. 92. I Л нить скорость центра катушки в тот момент времени, когда на плоскости будет лежать часть кабеля длиной х. За счет действия какой силы изменяется количество движе- ния кабеля? 49
219. К шкиву радиуса г, вращающемуся вокруг непо- движной оси, приложена сила трения f (рис. 93). Опреде- лить изменение угловой скорости шкива со временем, если в начальный момент времени угловая скорость равна <оо. Масса шкива равна т; массой спиц можно пренебречь. 220. Вращающийся с угловой скоростью ,<в0 обруч радиуса г по- ставили на горизонтальную шеро- ховатую плоскость. Определить скорость v центра обруча после то- го, как прекратилось его проскаль- зывание. В начальный момент ско- рость центра обруча равна нулю. 221. Обручу радиуса г, постав- ленному на горизонтальную шеро- ховатую плоскость, сообщили в го- ризонтальном направлении поступательную скорость v0. Определить угловую скорость со вращения обруча после того, как проскальзывание обруча прекратилось. 222. Вращающийся с угловой скоростью <оо обруч ради- уса г поставлен на горизонтальную шероховатую плоскость. Обручу сообщена поступательная скорость v0 (рис. 94). Считая, что сила трения скольжения равна f, опре- делить характер движения обруча. Рис. 95. Рис. 94. жжетжж 223. Цилиндрическая трубка радиуса г скреплена с по- мощью спиц с двумя обручами радиуса R. Масса обоих об- ручей равна М. Массой трубки и спиц, по сравнению с мас- сой обручей М, можно пренебречь. На трубку намотана нить, перекинутая через невесомый блок. К концу нити прикреплен груз массы т (рис. 95). Найти ускорение груза, натяжение нити и силу трения, действующую между обру- 50
чами и плоскостью. (Считать, что обручи не проскальзывают.) Определить, при каком значении коэффициента трения об- ручи будут проскальзывать. 224. На наклонной плоскости лежит катушка. На ка- тушку намотана нить, свободный конец которой с прикреп- ленным к нему грузом массы т перекинут через невесомый блок (рис. 96). Предполагает- ся, что масса катушки М рав- номерно распределена по ок- ружности радиуса 7?. Трение отсутствует. Определить, при каком значении угла наклона а центр тяжести катушки бу- дет оставаться в покое. 225. Доска массы М поло- Рис. 96. жена на два одинаковых ци- линдрических катка радиуса R. Катки лежат на горизон- тальной плоскости. В начальный момент времени система находилась в покое. Затем к доске приложили в горизон- тальном направлении силу Q. Найти ускорение доски и величину сил трения, действующих между катками и дос- кой, а также между катками и горизонтальной плоскостью. Считать, что проскальзывание отсутст- вует. Катки представляют собой тон- костенные цилиндры массы т каждый. 226. Двухступенчатый блок состоит из двух жестко скрепленных между собой тонких обручей, радиусы кото- рых R и г, а массы соответственно Mi и Л12. На каждую из ступеней блока намо- таны нити, к концам которых прикреп- лены грузы массой mi и пг2 (рис. 97). Найти ускорения грузов mi и т2, на- тяжения нитей и сцлу, с которой систе- ма действует на ось блока. 227. Однородный тонкостенный ци- линдр радиуса R и массы М без сколь- жения скатывается под действием силы тяжести с плоскости, образующей угол а с горизонтом. Используя закон сохранения энергии, определить: 1) ско- рость центра тяжести и угловую скорость вращения ци- линдра спустя время t после начала движения из состояния покоя; 2) ускорение центра тяжести цилиндра. 51
§ 9. Закон всемирного тяготения 228. Почему Земля сообщает всем телам одно и то же ускорение независимо от их массы? 229. Найти величину и размерность в системе СГС гравитационной постоянной, принимая во внимание, что средний радиус Земли Я=6,4-10* см, а масса Земли Л1=6-10г’ г. 230. При каких условиях тела внутри космического ко- рабля будут находиться в так называемом состоянии неве- сомости, т. е. перестанут оказывать давление на стенки ка- бины корабля? 231. Легкий маятник, состоящий из стержня и диска (рис. 98), укреплен на деревянной рамке, которая может свободно падать вдоль направляющих про- 1 волок. Маятник отклонили от положения равновесия на угол а и отпустили. В мо- ----;---h мент, когда маятник проходил крайнее 4-------нижнее положение, рамку перестали удер- {X живать, и она начала свободно падать. Как V будет двигаться маятник относительно рам- ° ки? Трением и сопротивлением воздуха Ч г пренебречь. 232. Планета движется по эллипсу, в фокусе которого расположено Солнце. При- нимая во внимание работу силы тяготения, Рис. 98. указать, в какой точке траектории ско- рость планеты будет максимальной и в ка- кой — минимальной? 233. Искусственный спутник Земли движется на высоте А=670 км по круговой орбите. Найти скорость движения спутника. 234. Как изменяется со временем скорость искусствен- ного спутника Земли при движении его в верхних слоях атмосферы? 235. По круговой орбите на небольшом расстоянии друг от друга в одном направлении движутся два спутника. С первого спутника на второй нужно перебросить контей- нер. В каком случае контейнер быстрее достигнет второго спутника: если его бросить по движению пер- вого спутника или против движения? Скорость контейне- ра относительно спутника и много меньше скорости спут- ника V. 52
236. Оценить массу Солнца М, зная, что средний ра- диус орбиты Земли #=149-10‘ км. 237. Определить минимальное удаление h аг поверхности Земли первого искусственного спутника, запущенного в СССР 4 октября 1957 г., если известны следующие данные: максимальное удаление спутника от поверхности Земли //=900 км; период обращения спутника вокруг Земли Т=96мин; большая полуось лунной орбиты #=384 400 км; период движения Луны вокруг Земли 7’=27,3 суток и ра- диус Земли Яо=6370 км. 238. В воде имеется пузырек воздуха радиуса г и желез- ный шарик того же радиуса. Будут ли они притягиваться друг к другу или отталкиваться? Какова величина силы взаимодействия между ними? Расстояние между центрами шарика и пузырька равно #. - 239. В воде имеется два пузырька воздуха радиуса г. Притягиваются или отталкиваются пузырьки? Какова величина силы взаимодействия? Расстояние между пузырь- ками #. 240. Свинцовый шар радиуса #=50 см имеет внутри сферическую полость радиуса г=5 см, центр которой нахо- дится на расстоянии d=40 см от центра шара (рис. 99). С какой силой будет притягиваться к шару материальная точка массы /п=10 г, находящаяся на расстоянии /=80 см от центра шара, если линия, соединяющая центры шара и полости, составляет угол а=60° с линией, соединяющей центр шара с материальной точкой? 241. Тело, размерами которого можно пренебречь, по- мещено внутрь тонкой однородной сферы. Доказать, что сила притяжения, действующая со стороны сферы на тело, равна нулю при любом положении тела внутри сферы. 53
242. С какой силой притягивается к центру Земли тело массы т, находящееся в глубокой шахте, если расстояние от тела до центра Земли равно г? Плотность Земли считать всюду одинаковой и равной р. § 10. Гидро- и аэростатика 243. В стакане с водой плавает в вертикальном положе- нии брусок. Как изменится уровень воды в стакане, если брусок перейдет в горизонтальное положение? J~244. Сосуд с водой установ- лен на ребре доски (рис. 100). Нарушится ли равновесие, если на поверхность воды положить дощечку и на нее поставить ги- рю так, что дощечка с гирей будет плавать на поверхности воды? '/ 245. В стакане с водой пла- вает кусок льда. Как изменится уровень воды в стакане, когда лед растает? Рассмотреть случаи: 1) лед совершенно однородный; 2) во льду находится вмерзший камень; 3) внутри куска льда имеется пузырек воздуха. 5 246. Сплошное однородное тело» будучи погружено в жидкость с удельным весом dit весит Plt а в жидкости с удельным весом d3 весит Р3. Определить удельный вес d тела. Z. 247. Посередине большого озера прорубили прорубь. Толщина льда оказалась равной 10 м. Какой длины нужна веревка, чтобы зачерпнуть ведро воды? 248. В чаше с водой плавает спичечная коробка, на дне которой лежит небольшой камень. Изменится ли уровень во- ды в чаше, если камень вынуть из коробки и опустить в воду? — 249. Судно проходит шлюз, поднимаясь на более высо- кий уровень в камере шлюза, куда вода накачивается насоса- ми со стороны нижнего уровня (рис. 101). В каком случае насосы совершат большую работу: когда в камере находится большой теплоход или маленькая лодка? £ 250. Из двух пластин одинаковой толщины с плотнос- тями 3,5 г/см3 и 2 г/см3 вырезаны квадрат со стороной а и прямоугольник со сторонами а и 2а, причем квадрат вы- 54
резан из более тяжелого материала. Квадрат и прямоуголь- ник скреплены в виде буквы Г и поставлены на дно пустого сосуда (рис. 102). Что произойдет, если сосуд заполнить водой? радиуса R погружают Рис. 102./ Ь 251. В воде плавает в вертикальном положении труба (рис. 103). Высота выступающей из воды части трубы й=5 см. Внутрь трубы наливается масло плотности <f=0,9 г/см8. Какой длины должна быть труба для того, чтобы ее можно было за- полнить маслом целиком? У" 252. Поршень весом Р=30 Н представляет собой круглый диск радиуса R=4 см с отверстием, в ко- торое вставлена тонкостенная труб- ка радиуса г=1 см. Поршень мо- жет плотно и без трения входить в стакан и сначала лежит на дне стакана. На какую высоту Н под- нимется поршень, если влить в трубку т=700 г воды? 253. Резиновый мяч массы т и под воду на глубину h и отпускают. На какую высоту, счи- тая от поверхности во- ды, подпрыгнет мяч? Сопротивление воды и воздуха при движении не учитывать. 254. На камень, вы- ступающий над водой, верхним своим концом опирается доска длины I. Часть доски длины а на-i ходится выше точки опоры (рис. 104). Какая часть доски находится под водой, если удельный вес древесины d? Рис. 104. 85
255. Человек, который нес автомобильную камеру, ре- шил облегчить свою ношу, использовав выталкивающую силу воздуха (по закону Архимеда). Для этого человек накачал камеру, увеличив тем самым ее объем. Достиг ли он этим своей цели? 256. На точных аналитических весах, находящихся под стеклянным колпаком, взвешивают тело. Изменятся ли показания весов, если выкачать из-под колпака воздух? 257. Какая ошибка допущена при взвешивании тела объемом У=1 л, если при взвешивании в воздухе тело было уравновешено на весах медными гирями массой Л!1=800 г? Плотность меди di—8,8 г/см8, воздуха d0=l,29 г/л. 258. Оценить массу атмосферы Земли. 259. В море на большой глубине затонула незакуйорен- ная бутылка. Увеличится или уменьшится вместимость бу- тылки под влиянием давления воды? 260. Сосуд представляет собой призму (рис. 105). Дно сосуда — прямоугольник с размерами а и Ь. В сосуд до высоты h налита жидкость, плотность которой р. Найти си- лу, с которой действуют боковые стенки на дно сосуда. Весом стенок пренебречь. 261. Сосуд без дна, имеющий форму и размеры, изобра- женные на рис. 106, стоит на столе. Края сосуда плотно прилегают к поверхности стола. Вес сосуда Р. В сосуд на- ливают жидкость. После того, как уровень достигнет высо- ты Л, сосуд под действием жидкости приподнимается. Оп- ределить плотность р налитой жидкости. 262. Конический сосуд без дна стоит на столе. Края сосуда плотно прилегают к поверхности стола. После того, как 56
уровень жидкости в сосуде достигает высоты Л, давление жидкости приподнимает сосуд. Радиус нижнего, большего основания сосуда 7?, угол между образующей конуса и вер- тикалью а, вес сосуда Р. Какова плотность налитой жид- кости? 263. Три сосуда с приставным дном погружены в воду на одинаковую глубину. Дно каждого из сосудов (рис. 107) отпадет, если налить в них по 1 кг воды. Отпадет ли дно, если налить в сосуды по 1 кг масла? налить в сосуды по 1 кг ртути? положить в каж- дый сосуд по гире массой в 1 кг? 264. В двух цилиндриче- ских сообщающихся сосудах налита ртуть, а поверх нее — вода. Уровень воды в обо- Рис. 107. их сосудах одинаков. Будет ли одинаков уровень воды и ртути, если в один из сосудов опустить кусочек дерева, а в другой долить равное ему по весу количество воды? Рассмотреть случаи одинаковых и разных сечений со- судов. 265. В сообщающиеся цилиндрические сосуды с различ- ными площадями поперечных сечений налита ртуть. В ши- рокий сосуд опустили железный кубик объемом 1Л>, вслед- Рис. 108. ствие чего уровень ртути в этом сосуде повысился. Затем в этот же сосуд налили воду до тех пор, пока уровень рту- ти в нем не принял прежнего положения. Найти высоту столба воды й, если площадь поперечного сечения узкого сосуда равна 266. На весьма чувстви- тельных весах уравновешены два ртутных барометра: ча- шечный (с широкой чашкой) и U-образный (рис. 108)1 Баро- метры изготовлены из одного и того же материала, имеют одинаковый диаметр трубок и содержат одинаковое количество ртути. Расстояния между запаянными концами трубок и верхними уровнямй ртути 57
& них также одинаковы. Как изменится равновесие весов при повышении атмосферного давления? 267. Надувной матран заполнен воздухом до некоторого давления, превышающего атмосферное. В каком случае давление воздуха в матраце будет больше: когда человек станет на него или ляжет? 268. Колесо автейлобиля устроено следующим образом: на металлический обод колеса надевается резиновая каме- ра, заключенная в покрышку. Камера затем накачивается воздухом. Давление воздуха в нижней и верхней Частях камеры одинаково. Кроме давления воздуха на обод дейст- вует сила тяжести (рис. 109). Почему же обод не опускается вниз? Что его удерживает в состоянии равновесия? Рис. 109. " 269. Паровой котел состоит из цилиндрической части и двух полусферических днищ (рис. НО). Радиусы цилинд- рической части котла и обоих днищ одинаковы. Толщина стенки цилиндрической части 0,5 см. Все части котла сдела- ны из одного материала. Какой толщины должны быть стенки днищ, чтобы прочность всех частей котла была одинакова? 270. Какую форму должен иметь паровой котел, чтобы при данной толщине стенок прочность котла была наи- большей? 271. Сосуд с водой падает с ускорением a<Zg. Как ме- няется давление р в сосуде с глубиной? 272. Сосуд с плавающим в нем телом падает с ускорением a<Zg. Всплывет ли при этом тело? 273. Цистерна, имеющая вид, изображенный на рис. 111, целикам заполнена водой и движется с ускорением а 58
в горизонтальном направлении. Определить силу, с которой вода действует на крышку цистерны. Рис. 111. а 274. Определить в предыдущей задаче силу, с которой вода действует на дно цистерны. 275. На тележке стоит бак кубической формы, це- ликом заполненный водой (рис. 112). Тележка дви- жется с постоянным уско- рением а. Определить дав- ление на глубине h в точ- ке А, отстоящей от перед- ней стенки на расстояние I, если бак сверху плотно закрыт крышкой. (При равномерном движении крышка не оказывает давления на воду.) 276. Прямоугольный бак без крышки (размеры бака ука- заны на рис. 113) движется с ускорением а. В бак налита вода '777^^77777777777777777777777777. Рис. 112. Рис. 113. на высоту h. Каким должно быть ускорение а, чтобы вода начала выливаться из бака? 277. Сосуд с отверстием в дне укреплен на тележке. Масса сосуда с тележкой Л1; площадь основания сосуда S. С какой силой F надо тянуть тележку, чтобы в сосуде 59
осталось максимальное количество воды? Размеры сосуда указаны на рис. 114. Трение отсутствует. 278. Можно ли с помощью сифона перекачивать воду через стенку высотой 20 м? Рис. 114. 279. Сосуд, изображенный на рис. 115, целиком запол- нен водой. Что произойдет, если вынуть пробку А? Радиус отверстия 0,5 см. 280. Четыре поршневых насоса сделаны из отрезков труб большого и малого диаметров. Насосы подняли воду на одинаковую высоту HA-h (рис. 116). Какой из поршней Рис. 116. надо тянуть с большей силой, чтобы удержать его в равнове- сии? Весом поршней пренебречь. 281. Предложен следующий проект вечного двигателя (рис. 117). Герметический сосуд разделен на две половины герметической же перегородкой, сквозь которую пропуще- на трубка и водяная турбина особого устройства. Турбина 60
имеет камеры с автоматически закрывающимися и откры- вающимися крышками. Давление рг в нижней части сосуда больше, чем давление рг в верхней части, и вода поднимает- ся по трубке, наполняя открытую камеру турбины. После этого камера закрывается и колесо поворачивается. В ниж- ней части сосуда камера автоматически открывается, воз- вращая воду. После этого камера герметически закры- вается и т. д. Почему данная машина не будет работать вечно? санного в задаче 281. Герметические камеры (рис. 118) за- полняются водой в правой части колеса и опускаются вниз. Внизу камеры открываются, и, в отличие от двигателя, опи- санного в задаче 281, стенки камер автоматически задвига- ются внутрь колеса. В верхней части сосуда стенки автома- тически выдвигаются и заполняются водой. В остальном вечный двигатель устроен так же, как и описанный ранее. Почему он тоже не будет работать? 283. Почему при подъеме на стратостате обязательно берут балласт? Ведь лишний вес уменьшает «потолок» стра- тостата. § 11. Гидро- и аэродинамика \ 284. В стенке сосуда с водой просверлены одно над другим два отверстия площадью 5=0,2 см2 каждое. Рас- стояние между отверстиями /7=50 см. В сосуд ежесекундно вливается Q= 140 см3 воды. Найти точку пересечения струй, вытекающих из отверстий. <” 285. На гладкой поверхности стола стоит широкий сосуд с водой. Уровень воды в сосуде Л, вес сосуда вместе с водой Q. В боковой стенке сосуда у дна имеется закрытое проб- кой отверстие (с закругленными краями) площади 5. При 61
Рис. 119. каком значении коэффициента трения между дном и поверх- ностью стола сосуд придет в движение, если вынуть пробку? 286. При истечении струи жидкости из сосуда через от- верстие площадью So сила, действующая на стенку с отвер- стием, на 2pS« меньше силы, действующей на противополож- ную стенку (см. задачу 285). Если в отверстие вставить труб- ку, как показано на рис. 119, то разность сил, действующих на противоположные стенки, станет равной примерно pS0, так как благодаря трубке движения жидкости у стенки не будет. С другой стороны, изменение количества движения жид- кости за единицу времени при истечении из сосуда всегда равно 2pS, где S — площадь поперечного сечения струи. Как согласовать между собой эти факты? f 287. Струя воды, вытекающая из трубки диаметром _,d=lcM со скоростью v=l м/с, ударяется о вертикальную стенку. Определить действующую на стенку силу, считая, что трубка перпендикулярна стенке, и пренебрегая раз- брызгиванием воды. ч 288. По изогнутой под прямым углом трубе поперечно- го сечения S со скоростью v течет газ. Плотность газа р. С какой силой газ действует на трубу? Сжатием газа и тре- нием пренебречь. 3~~289. Найти силу, действующую на лопасть нижнебой- ного колеса (рис. 120), считая, что струя после удара о ло- пасть продолжает движение со скоростью лопасти. Высота напора воды h, радиус колеса 7?, углевая скорость вращения колеса ® и площадь поперечного сечения струи S. 62
по вертикали дав- 3 290. Судно получило большую пробоину в подводной части (рис. 121). В какую сторону оно начнет перемещаться вследствие этого? 291. Из широкого сосуда через узкую трубку вытекает жидкость (рис. 122). Как распределены ление и скорость жидкости в сосуде и в трубке? 5* 292. Сосуд с водой, описанный в предыдущей задаче, подвешен на пру- жинных весах. Нижний конец трубки закрыт пробкой. Как изменятся по- казания весов в первый момент, ког- да вынута пробка и началось вытека- ние жидкости? п 293. На одной из чашек весов сто- ит сосуд с водой (рис. 123). Весы уравновешены. Изменится ли равно- весие, если открыть кран? (Вытекающая вода попадает на ту же чашку весов, на которой стоит сосуд.) 294. На рис. 124 изображен гидравлический таран — самодействующее водоподъемное устройство. Принцип его работы основан на явлении ком возрастании давления в жидкости, текущей по трубе, при ее внезапной остановке, например при закрытии клапана, выпус- кающего из трубы воду. Труба длиной Z=2 м, Рис. 123. гидравлического удара — рез- диаметром d=20 см опущена в ручей, скорость течения воды в котором v=400 см/с. Пусть вначале клапан К2 открыт, а /(1 закрыт. Резкое увеличение давления заставляет кла- пан Ki открыться (клапан К2при этом закрывается), и вода устремляется вверх в сосуд А. Давление падает, клапан 63
Ki закрывается, а Л2 открывается. Вода в трубе приходит в движение, и явление повторяется в прежней последова- тельности. Определить количество воды, поднимаемое тара- ном за час на высоту Л=30 м, если известно, что каждый клапан открывается 30 раз в минуту. 295. Во время бурь, когда скорость ветра достигает значительной величины, ветер срывает крыши построек. Наблюдаются два вида срыва крыш: 1) Если крыша прочнее укреплена в точках Л и В, чем в коньке С, то воздушный поток как бы раскрывает крышу, ломая ее по линии конь- ка (рис. 125, а). 2) Если крыша прочно укреплена в коньке и менее прочно в точках Л и В, то воздушный поток сначала приподнимает крышу вверх, а потом уже относит в сторону (рис. 125, б). Как объяснить эти явления? Рис. 126. 296. Почему легкий целлулоидный шарик, помещенный в струю воздуха или воды, вытекающую с большой скоростью из трубки с узким отверстием, будет свободно парить в этой струе (рис. 126)? 297. Демонстрационный прибор, изображенный на рис. 127, состоит из двух дисков Л и В. В центре диска Л имеет- ся отверстие, соединенное трубкой с баллоном сжатого воз- духа. Диск В висит на трех стерженьках, вдоль которых он может свободно перемещаться по вертикали. Если по труб- ке пропустить струю сжатого воздуха, то нижний диск нач- нет стучать по верхнему. Объяснить причину этого явления. 298. В дне широкого сосуда имеется узкая труба, по ко- торой вода, заполняющая сосуд, может из него вытекать 64
(рис. 128). Между сосудом и трубой помещена сетка. Если легкий шарик погрузить на дно сосуда в момент, когда вода из него вытекает, то шарик не всплывет. Если вытекание воды из трубы прекратить, шарик немедленно всплывает. Почему? (Этот опыт легко проделать в водопроводной рако- вине с мячиком от настольного тенниса.) 299. Насос представляет собой расположенный гори- зонтально цилиндр с поршнем площади S и выходным отвер- стием площади s, расположенным у оси цилиндра. Опреде- лить скорость истечения струи из насоса, если поршень под действием силы F перемещается с постоянной скоростью. Плотность жидкости, р. 300. В задаче 299 при s->S скорость v становится сколь угодно большой даже при малой силе F. Объяснить, с чем связано появление этого парадоксального результата. 301. Древнегреческие водяные часы (клепсидра) пред- ставляют собой сосуд с небольшим отверстием О (рис. 129). Время отсчитывается по уровню воды в сосуде. Какова Рис. 130. должна быть форма сосуда, чтобы шкала времени была рав- номерной? 3 Б. Б. Буховцев и др. 65
302. Цилиндрический сосуд с жидкостью вращается с угловой скоростью со вокруг вертикальной оси (рис. 130)< Определить изменение давления в горизонтальном сечении сосуда в зависимости от расстояния до оси вращения. (Применить метод, изложенный при решении задачи 211.) 303. Найти форму поверхности жидкости в цилиндри- ческом сосуде, вращающемся с угловой скоростью ш вокруг вертикальной оси (т, е. найти высоту уровня жидкости в зависимости от расстояния до оси вращения). 304. Почему после размешивания чая ложкой чаинки собираются в середине стакана?
Глава II. ТЕПЛОТА. МОЛЕКУЛЯРНАЯ ФИЗИКА § 12. Тепловое расширение твердых и жидких тел 305. На деревянное колесо, имеющее диаметр 100 см, требуется надеть железную шину, диаметр которой на 5 мм меньше диаметра колеса. На сколько следует для этого по- высить температуру шины? Коэффициент линейного расши- рения железа ах=12- 10-в град-1. 306. Почему в бетонных конструкциях в качестве ар- матуры используется только железо (железобетон), а дру- гие металлы, например дюралюминий, не применяются? 307. Для того чтобы измерить температуру человече- ского тела, приходится держать термометр под мышкой в те- чение 5—8 мин. В то же время «стряхнуть» его можно прак- тически сразу после измерения температуры. Почему? 308. При температуре /х высота ртутного столба, изме- ренная по латунной шкале, равна Ях. Какую высоту Но будет иметь столб ртути при /0=0 °C? Коэффициенты ли- нейного расширения латуни а и объемного расширения ртути р известны. 309. Как измерить медицинским термометром темпера- туру тела человека, если температура окружающего воз- духа +42 °C? 310. Определить длины железной и медной линеек /о и Го при /=0 °C, если разности их длин при /1=50 °C и /2—450 °C одинаковы по модулю и равны 1=2 см. Коэффи- циент линейного расширения железа ах=12-10-в град-1, меди а2=17-10-“ град-1. 311. Период колебаний маятника зависит от длины, ко- торая меняется с температурой. Каким образом может быть осуществлен подвес маятника, чтобы его длина не менялась с температурой? 312. При температуре /0=0°С стеклянный баллон вме- щает /ио=100 г ртути, При /1=20°С баллон вмещает 3 67
/И!=99,7 г ртути. (В обоих случаях температуру ртути счи- тать равной температуре баллона.) Найти по этим дан- ным коэффициент линейного расширения стекла а, учи- тывая, что коэффициент объемного расширения ртути Pi=18-10-5- град-1. 313. Часы с металлическим маятником спешат на Ti=5 с в сутки при температуре /!=-|-15 °C и отстают на т2= 10 с в сутки при температуре /2=+30 °C. Найти коэффи- циент а теплового расширения' металла маятника, учиты- вая, что период колебаний маятника Т = 2л|/l/g (I —длина маятника, g— ускорение свободного падения). §13. Закон сохранения энергии. Теплопроводность ШЯШЛ 314. На тележку массы М, двигающуюся по инерции со скоростью t>0, опускают сверху кирпич массы /и. Опреде- лить, сколько при этом выделится тепла. 315. Вдоль невесомого резинового шнура длины /0 (рис. 131) соскальзывает железная шайба. Сила трения, дей- ствующая между шнуром и шайбой, по- стоянна и равна f. Коэффициент упру- гости шнура k известен. Найти выделив- шееся при- этом количество тепла Q. 316. Холодильник, потребляющий мощность w, за время т превратил в лед воду, имевшую массу m и температуру t. Какое количество тепла выделит хо- лодильник в комнате за это время при условии, что теплоемкостью холодиль- ника можно пренебречь? 317. Понизится ли температура в ком- нате, если открыть дверцу работающего холодильника? 318. Проще всего отапливать помеще- ния за счет энергии электрического то- ка, используя электронагревательные приборы. Является лй этот способ энер- гетически наиболее выгодным? количества соли растворяют в двух одина- ковых сосудах с водой. В одном случае соль берут в виде одного большого кристалла, а в другом — в виде порошка. В каком случае температура раствора после полного раст- ворения соли будет выше, если до растворения соль,и вода находились в обоих случаях при одинаковых температурах? 68 к Рис. 131. 319. Равные
320. Известно, что если нагревать или охлаждать воду с соблюдением некоторых мер предосторожности, то можно получить ее в жидком состоянии при температурах, мень- ших 0 °C и больших +100 °C. В калориметре, теплоемкость которого ^=1700 Дж/град, находится т^\ кг воды, охлажденной до /х=—10 °C. Туда же помещают т2= 100 г воды, перегретой до 4= +120 °C. Какова установившаяся температура в калориметре? 321. Лампочка накаливания, расходующая М=54 Вт, погружена в прозрачный калориметр, содержащий V = 650 см3 воды. За т=3 мин вода нагревается на /=3,4 °C. Какая часть Q расходуемой лампочкой энергии пропускается ка- лориметром наружу в виде лучистой энергии? 322. Площадь кирпичной стены, выходящей на улицу, S= 12 м* а толщина d=l м. Температура наружного воз- духа То=—15°, а температура воздуха в комнате Т’= + 15°. Чему равно количество тепла, выходящего из комнаты в те- чение 24 часов? Коэффициент теплопроводности кирпича fe=l,3 Дж/(м-с-град). 323. Стенка состоит из двух соприкасающихся пластин, сделанных из различных материалов. Коэффициенты тепло- проводности и толщины пластин равны k19 di и k2i d2 соот- ветственно (рис. 132). Температуры внешних поверхностей стенки равны 7\ и Т,0(7,0>Т,1) и поддерживаются постоянны- ми. Определить температуру Тг на по- верхности раздела пластин. 324. Предполагая в задаче 323, что пластины имеют одинаковую тол- щину d, определить коэффициент теплопроводности стенки. Тг То кг к. у > Т, кг тг и7 Рис. 1 кг 32. То X, кг Рис. 133. 325. Стенка состоит из чередующихся брусков длины d с коэффициентами теплопроводности kr и k2 (рис. 133). 69
Площади поперечного сечения брусков одинаковы. Опре- делить коэффициент теплопроводности стенки. 326. Две стенки А и В одинаковой толщины составлены из разнородных металлов так, как это указано на рис, 134 Л Рис. 134. и 135. В каком случае коэффициент теплопроводности стенки больше? 327. В кастрюле с кипящей водой за секунду выкипает вода массой т. Считая, что. тепло передается воде только через дно кастрюли, и пренебрегая отдачей тепла стенками кастрюли и поверхностью воды окружающему воздуху, определить температуру Т поверхности дна кастрюли, соприкасающейся с нагревателем. Площадь дна кастрюли S, его толщина d и коэффициент теплопроводности k. § 14. Свойства газов 328. В колпачке автоматической ручки обычно делается маленькое отверстие. Если оно засорится, то ручка начи- нает течь: чернила вытекают из-под пера. В чем причина этого явления? 329. Барометр дает неверные показания вследствие при- сутствия небольшого количества воздуха над столбиком ртути. При давлении pOi=755 мм рт. ст. барометр показы- вает рх=748 мм, а при рО2=740 мм он показывает р2=736 мм, Найти длину / трубки барометра (рис. 136). 330. Стеклянная трубка, имеющая длину 1=50 см и по- перечное сечение S=0,5 см2, запаяна с одного конца. Трубку погружают в воду, как показано на рис. 137. Какую силу F нужно приложить, чтобы удержать трубку под водой, если 70
расстояние от поверхности воды до запаянного конца равно h= 10 см, а атмосферное давлениеро=76О мм рт. ст.? Масса трубки т= 15 г. 331 ♦ Через пробку сосуда с водой пропущена открытая с обеих сторон узкая трубка, не доходящая до дна сосуда, (Сосуд Мариотта, изображенный на рис. 138.) Начертить Рис. 136. график зависимости давления р воздуха в сосуде от коли- чества Q вытекшей воды. 332. Поршневой насос при каждом качании захватывает объем v0 воздуха. При откачке этим насосом воздуха из со- суда объема V насос совершил п качаний. Начальное дав- ление внутри сосуда р0 равно атмосферному. Затем другой насос с тем же рабочим объемом v0 начал нагнетать воздух из атмосферы, совершив также п качаний. Какое давление установится в сосуде? 333. Посередине горизонтальной, закрытой с обоих концов трубки длины L находится столбик ртути длины I. Если трубку поставить вертикально, то Столбик ртути пере- местится на расстояние AZ от своего первоначального поло- жения. На каком расстоянии от 'середины трубки будет центр столбика, если открыть один из концов трубки в го- ризонтальном положении? открыть верхний или нижний конец трубки в вертикальном положении? Атмосферное да- вление равно давлению ртутного столба высотой Я. Тем- пература неизменна. 71
334. Учитывая, что по закону Авогадро объем одного моля любого газа при нормальных условиях (температу- ра О °C и давление 1 атм) равен 22,4 л, определить постоян- ную в уравнении состояния идеального газа (уравнении Клапейрона — Менделеева) для количества газа, равного одному молю, и показать, что эта постоянная одинакова для всех газов. 335. Записать уравнение состояния для произвольной массы идеального газа, молярная масса которого р из- вестна. 336. Как изменилось бы давление внутри газа или жид- кости, если бы силы притяжения между молекулами вне- запно исчезли? 337; В сосуде находится 1 литр воды при температуре 27 °C. Чему стало бы равным давление внутри сосуда, если бы силы взаимодействия между молекулами воды внезап- но исчезли? 338. Одинаково ли давление внутри газа и у стенки со- суда, содержащего газ? 339. Одинакова ли концентрация молекул газа внутри сосуда и у его стенки? 340. Определить температуру газа, находящегося в закрытом сосуде, если давление газа увеличивается на 0,4% первоначального давления при нагревании газа на 1 °C. 341. Тонкостенный резин'овый шар весом Р=0,5 Н на- полнен азотом и погружен в озеро на глубину Л—100 м. Найти массу т азота, если шар находится в положении рав- новесия. Будет ли равновесие устойчивым? Атмосферное давление ро=760 мм рт. ст. Температура в глубине озера /=+4 °C. Натяжением резины пренебречь. 342. Два полых стеклянных шарика соединены трубкой, посередине которой находится капелька ртути. Можно ли по положению капельки судить о температуре окружающего воздуха? 343. Закрытый с обеих сторон цилиндр разделен на две равные (по 42 см) части теплонепроницаемым поршнем. В обеих половинах находятся одинаковые массы газа при температуре 27 °C и давлении в 1 атм. Насколько надо на- греть газ в одной части цилиндра, чтобы поршень сместился на 2 см? Найти давление р газа после смещения. 344. Сухой атмосферный воздух состоит из азота (78,09% по объему), кислорода (20,95%), аргона (0,93%) и углекис- лого газа (0,03%). Пренебрегая ничтожными примесями 72
других газов (гелия, неона, криптона, ксенона), определить (в процентах) состав воздуха по массе. 345. Найти среднюю (эффективную) молярную массу сухого атмосферного воздуха, предполагая известным про- центный состав воздуха (см. задачу 344). 346. Плотность пара некоторого соединения углерода с водородом равна 3 г/л при 43°С и 820 мм рт. ст. Какова молекулярная формула этого соединения? 347. В каком случае изменение давления газа будет большим: при сжатии его на определенную величину в теп- лонепроницаемой оболочке или же при изотермическом сжатии? 348. Газ, занимающий объем Vx=l л при давлении pi= 1 атм, расширился изотермически до объема V2=2 л. Затем при этом объеме давление газа было уменьшено в два раза. В дальнейшем газ расширялся при постоянном давлении до объема V4=4 л. Начертить график зависимости р от V и, используя его, установить, при каком из перечис- ленных процессов газ совершил наибольшую работу. Как менялась температура? 349. С некоторым количеством идеального газа совер- шен круговой процесс (цикл) 1—2—3—/, изображенный на графике зависимости объема от температуры (рис. 139). Изобразить- тот же процесс на графике зави- симости давления от объема и ука- зать, на каких стадиях процесса газ получал, а на каких отдавал тепло. 350. Газовая нагревательная колонка потребляет Vo= 1 ,8 м3 ме- тана (СН4) в час. Найти температу- ру t2 воды, подогреваемой этой ко- лонкой, если, вытекающая струя имеет скорость v=0,5 м/с. Диаметр струи D= 1 см, начальная темпера- тура воды и газа /1=11° С, теплотворная способность метана г=55 ООО Дж/г. Газ в трубе находится под давлением р= 1,2 атм. К. п. д. нагревателя г]=60%. 351. В закрытом теплонепроницаемом сосуде находится озон (О3) при температуре /х=527 °C. Через некоторое время озон полностью превращается в кислород (О2). Определить, во сколько раз возрастает при этом давление в сосуде, если на образование одного моля озона из кислорода нужно затратить q= 142 000 Дж. Теплоемкость одного моля кисло- 73
рода при постоянном объеме считать равной Cv=21 Дж/(моль-град). 352. 20 г гелия, заключенных в цилиндре под поршнем, бесконечно медленно переводятся из состояния с объемом Рис. 140. =32 л и давлением pi=4,l атм в состояние с V2=9 л и р2=15,5 атм. Какой наибольшей темпе- ратуры достигает газ при этом процессе, если на графике зави- симости давления газа от объема процесс изображается прямой линией (рис. 140)? 353. Увеличится ли энергия воздуха в комнате, если в ней протопить печь? (Энергию и еди- ницы массы воздуха считать пропорциональной абсолют- ной температуре: и—сТ.) 354. В комнате объемом в 30 м8 температура с 15 °C поднялась до 25 °C. На сколько при этом изменилась масса воздуха в комнате, если атмосферное давление р=1 атм? Молярную массу воздуха (среднюю) принять равной р,= =28,9 г/моль. 355. В наполненном водой открытом сосуде с сеткой вверху находится маленькая, заполненная воздухом и от- крытая снизу непереворачивающаяся пробирка (рис. 141)* Начертить график зависимости глубины погружения про- бирки от температуры воды при условии, что температура перво- начально медленно повышается, а затем начинает медленно пони- жаться. 356. В цилиндре под тяжелым поршнем находится т=20 г угле- кислого газа. Газ нагревается от температуры /г=20 °C до /2= 1080 С. Какую работу он при этом совер- шает? 357. Какое количество тепла должно быть сообщено углекислому газу (см. условие задачи 356), рас- ширяющемуся при постоянном дав- рис i4j лении вследствие нагревания? Мо- лярная теплоемкость углекислого газа (теплоемкость од- ного мол я) при постоянном объеме Сг=28,8 Дж/(моль • град) 4 74
§ 15,( Свойства жидкостей 358. Что «труднее» сжать до трех атмосфер: литр воз- духа или литр воды? 359. Каким образом, используя явления смачивания и несмачивания, можно осуществить минимальный и макси- мальный термометры? 360. Поверхностный слой жидкости часто уподобляют растянутой резиновой пленке. В каком отношении эта ана- логия не соответствует действительности? 361. При удалении с поверхности ткани жирного пят- на рекомендуется смачивать пропитанной бензином ваткой края пятна. Смачивать бензином сразу само пятно не сле- дует. Почему? 362. Почему разрыхление почвы при бороновании спо- собствует сохранению в ней влаги? 363. Для того чтобы мазь лучше впитывалась в смазан- ные лыжные ботинки, их нагревают. Как нужно нагревать ботинки: снаружи или изнутри? 364. Почему с помощью утюга можно выводить пятна жира с костюма? 365. Почему при сушке дров на солнце на конце полена, обращенном в тень, выступают капельки воды? 366. Сосуд, дно которого имеет круглые отверстия диа- метром d=0,1 мм, наполняется водой. Найти максимальную высоту уровня воды h, при которой ся. Вода не смачивает дно сосуда. 367. На прямоугольную про- волочную рамку, расположенную вертикально, натянута мыльная пленка (рис. 142). Какие силы удерживают участок пленки abed в равновесии? 368. Смачиваемый водою кубик массы т=20 г плавает на поверх- ности воды. Ребро кубика имеет длину а—3 см. На каком расстоя- нии от поверхности воды будет на- ходиться нижняя грань кубика? 369. Конец капиллярной трубки радиуса г опущен в воду. Какое количество тепла выделится при поднятии жид- кости по капилляру? 370. Капилляр опущен в сосуд с жидкостью, давлением паров которой можно пренебречь. Плотность жидкости р. она еще не выливает- b d Рис. 142. 75
Сосуд с капилляром находится в вакууме под колоколом воздушного насоса (рис. 143). Найти давление внутри жидкости в капилляре на высоте h от уровня жидкости в сосуде. 371. Для доказательства сущест- вования избыточной потенциальной энергии у молекул поверхностного слоя жидкости обычно рассуждают следующим образом. На молекулу, находящуюся внутри жидкости, дей- ствуют силы притяжения со стороны других молекул, которые в среднем компенсируют друг друга. Если же выделить молекулу на поверхности, то результирующая сил притяжения со стороны других молекул будет на- правлена внутрь жидкости. Поэтому Рис. 143. молекула будет стремиться внутрь жидкости, и нужно затратить опреде- ленную работу, чтобы извлечь ее на поверхность. Следо- вательно, каждая молекула поверхностного слоя обладает избытком потенциальной энергии, равным этой работе. Однако средняя сила, действующая на любую молекулу со стороны всех остальных, обязательно равна нулю при равновесии жидкости. Поэтому и работа по перемещению жидкости из глубины на поверхность должна равняться нулю. Откуда же в таком случае появляется поверхностная энергия? 372. Конец стеклянной капиллярной трубки радиуса г=0,05 см опущен в воду на глубину Л=2 см. Какое давле- ние необходимо, чтобы выдуть пузырек воздуха через ниж- ний конец трубки? 373. Стеклянная капиллярная трубка, внутренний диа- метр которой 0,5 мм, погружена в воду. Верхний конец трубки выступает на 2 см над поверхностью воды. Какую форму будет иметь мениск? 374. В капиллярной трубке, опущенной вертикально в воду на глубину Z, вода поднялась на высоту h (рис. 144). Нижний конец трубки закрывают, вынимают ее из воды и вновь открывают. Определить длину столбика воды, остав- шейся в трубке. 375. В сосуд с водой опущены два капилляра одинако- вого сечения (рис. 145). Вода в прямом капилляре подни- мается на высоту й. На каком уровне установится вода в 76
изогнутом капилляре и какую форму примет в нем мениск? Нижний конец изогнутого капилляра расположен ниже уровня воды в сосуде на расстоянии Н. Рассмотреть пять случаев: 1) H>h\ 2) H=h\ 3) (ХЖй; 4) /7=0; 5) ЖО (конец изогнутого капилляра выше уровня воды в сосуде). 376. На мыльный пузырь радиуса R посажен другой пузырь радиуса г (рис. 146). Какую форму примет мыльная 77
пленка, разделяющая оба пузыря? Какие углы образуются между пленками в местах их соприкосновения? 377. В воде плавает деревянный крест. Каждая лопасть креста покрыта с одной стороны лаком (рис. 147). Вследст- вие различного смачивания дерева и лака вода с двух сторон Рис. 148. каждой лопасти поднимется на разную высоту; краевой угол будет различен, и, следовательно, горизонтальная состав- ляющая силы поверхностного натяжения F будет различ- на с обеих сторон каждой лопасти (рис. 148). Будет ли крест вследствие этого вращаться? 378. Легкие тела, смачиваемые водою (например, две спички), плавая на поверхности воды, притягиваются друг к другу. То же самое наблюдается, если тела не смачиваются (например, спички, покрытые тонким слоем парафина). Если же одно тело смачивается водой, а другое не смачива- ется, то тела будут отталкиваться. Как объяснить эти явления? § 16. Взаимные превращения жидких и твердых тел 379. Вода в стакане замерзает при О °C. Если же эту воду расчленить на маленькие капельки, то вода в них может быть переохлаждена до —40 °C. Так, например, капельки воды, из которых состоят облака, обычно начинают замер- зать при температуре ниже —17 °C. Как объяснить эти факты? 380. Сосуд со 100 г воды при температуре 0 °C был под- вешен посередине комнаты. Через 15 минут температура воды поднялась до 2 °C. Когда же в сосуде находилось рав- ное по массе количество льда, то он растаял за 10 часов. Можно ли по этим данным оценить удельную теплоту плав- ления льда Л? 78
381. Два одинаковых кусочка льда летят навстречу друг другу с равными скоростями и при ударе обращаются в пар. Оценить минимально возможные скорости льдинок перед ударом, если их температура равна —12 °C. 382. В калориметре находится лед. Определить тепло- емкость калориметра, если для нагревания его вместе с со- держимым от 270 до 272 °К требуется Q!=2100 Дж тепла, а от 272 до 274 ° К требуется Q2=69 700 Дж. 383. В калориметре находилось 400 г воды при темпера- туре +5 °C. К ней долили еще 200 г воды при температуре + 10 °C и положили 400 г льда при температуре —60 °C. Какая температура установится в калориметре? 384. В медный сосуд, нагретый до температуры /1=350 °C, положили пг2=600 г льда при температуре /2=—10 °C. В результате в сосуде оказалось т3=550 г льда, смешанно- го с водой. Найти массу сосуда. Удельная теплоемкость меди С!=420 Дж/(кг-град). 385. При помещении в переохлажденную воду неболь- шого кристаллика льда вода немедленно начинает замер- зать. 1) Какое количество льда образуется из Л1=1 кг воды, переохлажденной до температуры t——8 °C? 2) Какую температуру должна была бы иметь переох- лажденная вода для того, чтобы целиком превратиться в лед? Зависимость теплоемкости воды от температуры не учи- тывать. 386. 100 г льда при температуре 0 °C заключены в тепло- непроницаемую оболочку и подвергнуты сжатию до дав- ления р=1200 атм. Найти массу растаявшей части льда, если понижение температуры плавления происходит прямо пропорционально давлению и при увеличении давления на 138 атм температура плавления понижается на 1 °C. § 1,7. Упругость и прочность 387. На стальной стержень радиуса R=100,125 см на- дето медное кольцо, имеющее радиус г=100 см и площадь поперечного сечения 5=4 мм2. С какой силой F будет рас- тянуто кольцо, если модуль упругости меди £=12- 1010Н/м2? Деформацией стержня пренебречь. 388. Какую работу может совершить стальной стер- жень длины I и площади поперечного сечения S при нагре- вании на Д/? 79
389. Между двумя столбами натянута проволока длины 2/. К проволоке, точно посередине, подвешен фонарь массы М. Площадь поперечного сечения проволоки 5, модуль упру- гости Е. Определить угол а провисания проволоки, считая его малым (рис. 149). 390. Между двумя неподвижными абсолютно жесткими стенками вставлен без зазора стальной стержень сечением 5=1 см2. С какой силой F стержень будет действовать на стенки, если его нагреть на Д/=5 °C? Коэффициент линей- ного расширения стали а—1,1 • 10-5 град-1, модуль упруго- сти Е=20-1010 Н/м2. 391. Между массивными стенками расположены два стержня из разных материалов (рис. 150). Сечение стерж- ней S. Их длины It и /2. Стержни нагреваются на Д/ градусов. Найти силу, с которой стержни действуют друг на друга, если коэффициенты теплового расширения стержней и Рис. 150. а2 и модули упругости материала стержней и Е2 известны. Деформацией стенок пренебречь. 392. Однородный брусок массой т= 100 кг висит на трех вертикальных проволоках равной длины, расположенных 80
симметрично (рис. 151). Определить натяжения проволок, если средняя проволока стальная, а две другие медные. Площади поперечного сечения всех проволок одинаковы. Модуль Юнга стали считать в два раза большим модуля Юнга меди. 393. Железобетонная колонна сжимается силой Р. По- лагая, что модуль Юнга бетона Е6 составляет х/10 модуля Юнга железа £ж, а площадь поперечного сечения железа составляет.1^ площади поперечного сечения бетона, най- ти, какая часть нагрузки приходится на бетон. — "чх ч 394. Стальной болт , вставлен в медную трубку, как показано на рис. 152. Найти си- у лы, возникающие в болте и трубке при по- вороте гайки на один оборот, если длина труб- ки /, шаг нарезки болта й, а площади поперечного сечения болта и трубки равны Sc и SM соответственно. 395. Медная пластинка сварена по концам с двумя сталь- ными пластинками так, как изображено на рис. 153. Какие 81
натяжения возникнут в пластинках при повышении темпе- ратуры на /? Площади поперечного сечения всех трех пластинок одинаковы. 396. Определить максимально допустимое значение линейной скорости при вращении тонкогд свинцового коль- ца, если предел прочности свинца Р=2000 Н/см2, а его плотность р= 11,3 г/см3. 397. Железный брусок АВ закреплен неподвижно обои- ми концами. В середине бруска имеется отверстие, в кото- ром укрепляется с помощью двух гаек крюк С (рис, 154), Брусок стянут гайками с силой Fo. Какие по величине силы будут действовать на верхнюю и нижнюю гайки со стороны бруска, если на крюк подвешивать груз, вес которого может изменяться от нуля до P=2F^ Прогибом бруска и весом крюка пренебречь. § 18. Свойства паров 398. В калориметр, содержащий 100 г льда при темпе- ратуре —20 °C, впускают 150 г водяного пара, имеющего температуру +100 °C. Какую температуру примет содер- жимое калориметра, если его теплоемкость 300 Дж/град? 399. Почему в момент выключения газовой горелки из кипящего чайника сразу же вырывается сильная струя пара, хотя до этого пара не было видно? 400. Показать, что плотность водяного пара при темпе- ратурах, близких к комнатным, выраженная в г/м3, приб- лиженно равна давлению водяного пара, выраженному в миллиметрах ртутного столба.
15 °C относительная 401. Давление насыщенных паров воды в герметически закрытом сосуде возрастает с температурой так, как пока- зано на рис. 155. Давление же идеального газа при постоянном объеме прямо пропорционально температуре. Используя таблицу свойств насыщенного водяного па- ра (см. табл. I), выяснить, при- годно ли уравнение состояния идеального газа для вычисления плотности или удельного объема насыщенных паров воды. Объ- яснить полученный результат. 402. В цилиндре под поршнем изотермически сжимают 9 г водя- ного пара при температуре 30 °C. При каком объеме пар начнет кон- денсироваться ? (Воспользоваться табл. I.) 403. В комнате при температуре влажность равнялась 10%. Как изменится относительная влажность, если температура в комнате постепенно уве- личится на 10 °C? 404. На улице целый день моросит холодный осенний дождь. В комнате развешено выстиранное белье. Высохнет ли белье быстрее, если открыть форточку? ТАБЛИЦА I Свойства насыщенного водяного пара /, °C Давление, атм Удельный объем пара, м8/кг /, °C Давление, атм Удельный объем пара, м3/кг 17,2 0,02 68,3 151,1 5 0,3818 45,4 0,1 14,96 158,1 6 0,3214 59,7 0,2 7,80 164,2 ' 7 0,2778 75,4 0,4 4,071 169,6 8 0,2448 85,45 0,6 2,785 174,5 9 0,2189 93,0 0,8 2,127 179,0 10 0,1980 96,2 0,9 1,905 187,1 12 0,1663 99,1 1 1,726 194,1 14 0,1434 100 1,0333 1,674 200,4 16 0,1261 116,3 1,8 0,996 206,2 18 0,1125 119,6 2 0,902 211,4 20 0,1015 132,9 3 0,617 232,8 30 0,0679 142,9 4 0,4708 249,2 40 0,0506 33
405. Два сосуда, соединенные трубками с кранами, на- полнены водой до разных уровней (рис. 156). Воздух из сосудов откачан. Что произой- дет, если соединить сосуды, 1) открыв кран в нижней трубке, 2) открыв кран в верхней трубке? 406. Чему равна относи- тельная влажность воздуха при температуре /х= 10 °C, если конденсация влаги из этого воздуха, нагретого пред- варительно до /а=30 °C, на- чалась при давлении в 10 ат- мосфер? Начальное давление воздуха равно 1 атм. (Вос- пользоваться данными табл. I.) 407. Пористое тело было помещено для просушки под колокол вакуумного насоса. Давление под колоколом дер- жалось на уровне 6,5 мм рт. ст. в течение часа, после чего резко упало. Производительность насоса 60 л/мин. Уста- новившаяся под колоколом насоса температура 1=5 °C. Какое количество воды содержало тело? 408. В теплоизолированном цилиндре под невесомым поршнем находится т=30 г воды при температуре 0 °C. Площадь поршня S=512 см2, внешнее давление р=1 атм. На какую высоту поднимется поршень, если находящийся в цилиндре электрический нагреватель выделит Q= = 24 200 Дж тепла?
Глава III. ЭЛЕКТРИЧЕСТВО И МАГНЕТИЗМ § 19. Электростатика 409. С какой силой взаимодействовали бы точечные за- ряды в один кулон, находясь на расстоянии 1 км друг от Друга? 410. Сравнить силы гравитационного и электрического притяжения между электроном и протоном. . 411. Предположим, что сила, действующая между дву- мя точечными зарядами, зависит от расстояния, как 1/г®, где 1) а>2; 2) а<2. Как будет вести себя точечный заряд, помещенный внутрь равномерно заряженной сферы? В на- чальный момент времени точечный заряд покоился. 412. Два маленьких шарика, заряженные равными, но разноименными зарядами, закреплены в горизонтальной плоскости на некотором расстоянии а друг от друга. Тре- тий заряженный шарик подвешен на нита. Точку подвеса один раз перемещают так, что этот шарик в состоянии равно- весия оказывается точно над первым закрепленным шари- ком, на расстоянии а от него, а другой раз — над вторым. Найти углы отклонения нити от вертикали, если известно, что над первым шариком угол отклонения в два раза боль- ше, чем над вторым. 413. На расстоянии d от большой проводящей пластины находится точечный электрический заряд +q. С какой си-, лой действует на него пластина? -* 414. Два заряда +Q неподвижны и расположены на расстоянии а друг от друга. Вдоль оси симметрии системы этих зарядов может перемещаться третий заряд —q, обла- дающий массой т. Считая расстояние заряда —q от прямой, соединяющей заряды +Q, малым, определить период коле- баний заряда —q. 85
• 415. Тонкое проволочное кольцо радиуса R несет элек- трический заряд q. В центре кольца расположен одно- именный с q заряд Q, причем QJ><7. Определить силу, с ко- торой растянуто кольцо. 416. Тело массы т подвешено на нити длины I (рис. 157). На расстоянии h под ним находится бесконеч- ная металлическая пластина. । , Тело имеет заряд q. Найти пе- риод свободных колебаний £ ।, этого тела. / I • 417. Определить период ма- I лых колебаний полярной моле- Т~®т кулы в однородном электриче- ском поле, напряженность ко- | торого Е=3-104 В/м. ' ' Полярную молекулу схема- Рис. 157. тически представить в виде «гантельки» длины 1(1= 10~8см), на концах которой находятся равные точечные массы т (tn=\Q~u г), несущие заряды +q и—q соответствен- но (<7=15,7-10-2’ Кл). I 418. Три одинаковых положительных заряда q распо- Д ложены в вершинах равностороннего треугольника. Сторо- на треугольника равна а. Найти напряженность поля в вер- шине правильного тетраэдра, построенного на этом тре- угольнике. Ъ 419. Два точечных заряда qY и q2 расположены на рас- стоянии d друг от друга. Найти напряженность электри- ческого поля в точке, находящейся на расстоянии от за- ряда <?1 и г2 от заряда q2. -Рассмотреть случаи разноименных одноименных зарядов. 420. Найти напряженность поля электрического диполя с моментом p=ql в точке, отстоящей от оси диполя на рас- стояние г в двух случаях: 1) точка лежит на прямой, проходящей через ось диполя; 2) точка лежит на прямой, перпендикулярной оси диполя. Примечание. В простейшем случае электриче- ский диполь представляет собой два одинаковых, но раз- ных по знаку заряда (+q и —q). Важной характеристикой диполя является его электрический момент p—ql. Электри- ческим дипольным моментом называется вектор, направлен- ный от отрицательного заряда к положительному и численно равный p=ql, где I — расстояние между зарядами, обра- 0! зующими диполь. 86
О 421. Положительный заряд Q равномерно распределен по тонкому проволочному кольцу радиуса 7?. Найти на- пряженность электрического поля на оси кольца в зависи- мости от расстояния г до центра кольца. ц 422. Тонкое проволочное кольцо радиуса R имеет элект- рический заряд +Q. Как будет двигаться точечное тело массы т9 имеющее заряд —q, если в начальный момент вре- мени оно покоилось в некоторой точке на оси кольца на _расстоянии от его центра? Кольцо неподвижно. s 423. Исходя из соображений размерности, найти (разу- меется, с точностью до числового коэффициента) напряжен- ность электрического поля, создаваемого: 1) бесконечно протяженной пластиной, заряженной с поверхностной плот- ностью о; 2) бесконечно длинной нитью, заряженной с ли- нейной плотностью т. 424. Прямоугольной металлической пластинке со сто- ронами а и b сообщен заряд +q. Толщина пластинки с много меньше а и Ь. Определить напряженность поля, создаваемо- го этой заряженной пластинкой в точках пространства, близких к центру пластинки. 425. Две металлические параллельные пластины, пло-. щадь каждой из которых равна S, несут заряды Расстояние между пластинами много меньше их линейных размеров. Определить напряженность электрического поля в точках Л, В, С (рис. 158). 426. Чему равна напряженность электриче- ского поля на поверхности проводника, если плотность поверхностного заряда о? 427. Все пространство между двумя беско- нечными параллельными пластинами занимает заряд с постоянной объемной плотностью р. Расстояние между пластинами а. Найти зави- Qi и Qa. 41 q2 SBC < • • Рис. 158. симость напряженности электрического поля от расстояния, отсчитанного от середины между пластинами, 428. Внутри шара радиуса R имеется объемный заряд постоянной плотности р. Найти зависимость напряжен- ности электрического поля от расстояния до центра шара. 429. Найти напряженность электрического поля внут- ри и вне /бесконечно длинного цилиндра, заряженного с объемнойгллотностью р. Радиус цилиндра /?. 430. Внутри шара, заряженного с постоянной объемной плотностью р, имеется сферическая полость. Расстояние между центрами шара и полости равно а, Показать, что на- 87
пряженность Е электрического поля внутри полости равна E=pa/3zQ и направлена вдоль прямой, соединиющейищнт- ры сфер. 431. Внутри цилиндра, заряженного с постоянной объем- ной плотностью р, имеется цилиндрическая полость. Рас-1 стояние между осями цилиндра и полости равно а. Пока- зать, что напряженность Е поля внутри полости равна Е=ра/2е0 и направлена параллельно перпендикуляру, соединяющему оси. 432. Молекула находится на расстоянии г от оси беско- нечно длинного металлического цилиндра. Цилиндр заря- жен равномерно так, что заряд, приходящийся на единицу его длины, равен т. Молекула представляет собой «гантель- ку» длины X, на концах которой находятся заряды +q и —q. Определить силу, действующую на молекулу. 433. На некотором расстоянии от оси равномерно заря- женного цилиндра находятся две молекулы равной массы. Одна молекула имеет постоянный электрический момент p=qK (см. задачу 420). Расстояние между зарядами другой молекулы определяется соотношением qE=kK, где £ — средняя напряженность поля, действующего на молекулу, k — постоянный коэффициент. В начальный момент элект- рические моменты молекул одинаковы, а их скорости рав- ны нулю. Какая молекула под действием силы притяжения быстрее достигнет поверхности цилиндра? 434. Прямоугольной металлической пластинке со сто- ронами а и & сообщен заряд +q. Толщина пластинки с много меньше а и Ь. К центру пластинки на расстояние d подно- сится точечный заряд + Q. Расстояние d много меньше сто- рон пластинки. Определить силу, с которой действует пла- стинка на заряд +Q. В каком случае положительно заря- женная пластинка будет притягивать положительный за- ряд? 435. Внутри шара радиуса 7? имеется объемный заряд постоянной плотности р. Найти зависимость потенциала от расстояния до центра шара. 436. На расстоянии d от точечного заряда q расположен центр незаряженного проводящего шара радиуса 7?. Чему равен потенциал шара? 437. На расстоянии R от точечного заряда +q распо- ложен проводящий шар радиуса г, соединенный тонкой длинной проволочкой с землей. Определить величину от- рицательного заряда, индуцированного на шаре. Влиянием проволочки пренебречь. 88
438. В металлической трубе переменного "“X. сечения движется электрон (рис. 159). Как * будет меняться его скорость при приближе- нии к сужению трубы? S 439. Две концентрические незаряженные Рис 159 металлические сферы, радиусы которых и /?3, причем соединены тонкой проволочкой. Проволочка проходит сквозь маленькое отверстие в сфере, расположенной концентрически между первыми двумя. Эта сфера имеет радиус Т?2 и несет заряд +Q, распределенный на ней равномерно. Пренебрегая влиянием соединительной проволочки, определить заряд, индуцированный на внут- ренней металлической сфере. 440. На одной прямой находятся три заряда: положи- тельный +q и два отрицательных —Q. При каком соотно- шении величин зарядов они будут находиться в равновесии? Будет ли равновесие устойчивым? Начертить зависимость потенциальной энергии каждого заряда от его положения на прямой при условии, что два других заряда непод- вижны. 441. Может ли электрический заряд, помещенный в элек- тростатическое поле, находиться в состоянии устойчивого равновесия? 442. Уединенный проводящий шар радиуса R имеет заряд +Q. Какой энергией обладает шар? 443. Две тонкие концентрические металлические сферы радиусов /?х и имеют заряды и Q2 соответ- ственно. Определить энергию такой системы зарядов. 444. Имеется п тонких концентрических металличе- ских сфер, радиусы которых-в порядке возрастания равны G,..., гй. Эти сферы имеют заряды qlt q2i. . .,qn соответ- ственно. Определить энергию такой системы зарядов. 445. Пластины плоского конденсатора емкости С, от- стоящие на расстояние / друг от друга, несут заряды + Q и — Q. Электрон влетел в середину конденсатора со скоростью v0, направленной параллельно пластинам. Чему равна ско- рость электрона на достаточно большом расстоянии от кон- денсатора? Каков характер изменения скорости электрона (по абсолютной величине) при его движении внутри и вне конденсатора? Рассмотреть случаи, когда электрон в на- чальный момент находится: 1) на равном расстоянии от пла- стин конденсатора; 2) на расстоянии Z/4 от положитель- ной пластины; 3) на расстоянии Z/4 от отрицательной пла- стины. 89
446. Два одноименных точечных заряда qr и q2 с массами тпх и т2 движутся навстречу друг другу. В момент, когда расстояние между зарядами равно они имеют скорости Vi и и2. До какого минимального расстояния г2 сблизятся за- ряды? 447. Из бесконечности к металлической пластине дви- жется точечный заряд +q. Определить энергию взаимодей- ствия заряда и пластины, а также скорость заряда в тот мо- мент, когда он будет находиться на расстоянии d от пла- стины. Находясь на бесконечно большом расстоянии от пластины, заряд имел скорость, равную нулю. 448. По тонкому кольцу ра- д диуса R равномерно распределен -----------------------—«-заряд +?. Найти скорость отри- цательного точечного заряда (—q) в момент прохождения че- Рис. 160.-рез центр кольца, если заряд —q первоначально находился в покое в достаточно удаленной от кольца точке А на оси (рис. 160). Масса заряда —q равна т. Кольцо неподвижно. 449. Положительный заряд + Q равномерно распреде- лен по тойкому проволочному кольцу радиуса R. В центре кольца находится точечный заряд —q, масса которого т. Заряду сообщается начальная скорость v вдоль оси кольца. Определить характер движения заряда в зависимости от ве- личины начальной скорости. Кольцо неподвижно. 450. Металлический шар диаметром 2 м расположен в центре большого помещения и заряжен до потенциала 100 000 В. Какое количество тепла выделится, если шар соединить проводником с землей? 451. Два маленьких шарика несут заряды, различные по величине, но одинаковые по знаку. Один из шариков за- креплен. Второй шарик, удаляясь под действием электро- статических сил отталкивания, может совершить механи- ческую работу Ль Если перед началом движения второго шарика оба шарика на некоторое время соединить провод- ником, то второй, удаляясь, сможет совершить механиче- скую работу Л2. Определить количество тепла, выделившее- ся в проводнике при соединении шариков, и выяснить, за счет какой энергии выделяется это тепло и изменяется меха- ническая работа. 452. Сферическая оболочка радиуса R заряжена равно- мерно зарядом Q. Найти растягивающую силу, приходя- щуюся на единицу площади оболочки. 90
453. Какой заряд Q можно сообщить капле радиуса R, если коэффициент поверхностного натяжения равен о? 454. Найти емкость Со батареи одинаковых конденса- торов (рис. 161). 455. Из проволоки сделан куб, в каждое ребро которого проводниками, присоединенными к противоположным вер- шинам А и В куба. 456. Для получения кратковременных высоких напря- жений может быть использован искровой конденсаторный трансформатор Аркадьева. Схема прибора изображена на рис. 163. Группа конденсаторов, соединенная параллельно провод- никами АВ и CD очень большого сопротивления подклю- чена к источнику высокого напряжения. Верхняя пластина каждого конденсатора сое- динена через искровой про- межуток с нижней пласти- ной последующего конден- сатора (промежутки /, 2, 3, 4). Каждый последую- щий промежуток больше предыдущего. В момент, Рис. 163. когда разность потенциалов между обкладками достигнет пробивного напряжения пер- вого промежутка, произойдет разряд. Вслед за этим будут пробиты второй, третий и четвертый промежутки. Какой величины достигнет разность потенциалов при пробое последнего промежутка, если имеется п конденсаторов и приложенное напряжение равно Vo? 91
457. Пластины заряженного плоского конденсатора по- переменно заземляют. Будет ли при этом конденсатор раз- ряжаться? 458. Два плоских конденсатора емкостью С\ и Cs за- ряжены до разности потенциалов Ui и Ut соответственно Показать, что при параллельном соединении этих конденсаторов их общая электростатическая энергия умень- шается. Почему это происходит? 459. Как известно, продолговатые кусочки диэлектрика устанавливаются вдоль силовых линий электрического поля. Но ведь отдельные молекулы неполярного диэлектри- ка, казалось бы, должны только растягиваться вдоль поля, но не поворачиваться. В диэлектрике, состоящем из диполь- ных молекул, среднее число молекул, поворачивающихся при включении поля по часовой стрелке, равно числу моле- кул, поворачивающихся в противоположную сторону. По- чему же весь кусок диэлектрика будет поворачиваться? 460. Диэлектрический шар радиуса 7? поляризован од- нородно, т. е. дипольные электрические моменты всех моле- кул равны и параллельны друг другу. Найти напряжен- ность электрического поля внутри диэлектрика, если в еди- нице объема содержится N молекул, дипольный момент каждой из которых равен 0 Рис. 164. 461. Диэлектрический шар помещен в однородное эле- ктрическое поле, напряжен- ность которого равна Е. Ди- электрическая проницаемость материала щара е. Найти на- пряженность поля внутри ша- ра, а также в точках Л, В, С и D (рис. 164), лежащих вне шара. 462. Найти закон распре- деления поверхностного заря- да на сфере, если известно, что этот заряд создает внутри сферы однородное поле с напряженностью Е. 463. Металлический шар радиуса /?, имеющий заряд + Q, помещен в однородное электрическое поле с напряжен- ностью Е. Найти зависимость поверхностной плотности за- ряда от угла 0, а также напряженность электрического поля в точках А, В, С и D (рис. 164), находящихся вне шара. 464. Бесконечный ци'линдр из материала с диэлектри- ческой проницаемостью е поляризован однородно в направ- 92
лении, перпендикулярном ’оси цилиндра. Радиус цилиндра Дипольный момент молекулы р. Число молекул в единице объема N. Найти напряженность электрического поля внутри цилиндра. 465. Бесконечный цилиндр радиуса /? из материала с диэлектрической проницаемостью е помещен в однородное электрическое поле, напряженность которого Е направлена перпендикулярно оси цилиндра. Определить напряжен- ность поля внутри цилиндра, а также в точках А, В, С и D вне цилиндра (рис. 164). 466. Заряженный металлический цилиндр радиуса R помещен в однородное электрическое поле, напряженность которого Е направлена перпендикулярно оси цилиндра. Заряд, приходящийся на единицу длины цилиндра, равен х. Найти зависимость плотности заряда от угла 0, а также напряженность электрического поля в точках А, В, С и D (рис. 164), находящихся вне цилиндра. 467. Шар, равномерно заряженный зарядом q, поме- щают в однородный изотропный безграничный диэлектрик с диэлектрической проницаемостью е. Определить поляри- зационный заряд на границе диэлектрика с шаром. 468. Пространство между двумя концентрическими сфе- рами радиусов rt и гг заполнено диэлектриком с диэлектри- ческой проницаемостью е. В центре сфер находится точеч- ный заряд +Q. Найти напряженность и потенциал как функцию расстояния от центра сфер, а также величину по- ляризационных зарядов. 469. Пространство между двумя тонкими концентри- . ческими металлическими сферами радиусов гх и г» заполнено _ диэлектриком с диэлектрической проницаемостью е. Заряды внутренней и внешней металлических сфер +Q и —Q соот- ветственно. Найти разность потенциалов, плотность поля- ризационных зарядов и емкость такого сферического кон- денсатора. Рис. 165. 470. Пространство между обкладками плоского конден- сатора заполнено диэлектриком с диэлектрической прони- цаемостью е, как показано на рис. 165. Площадь пластин 93
конденсатора S. Определить емкость конденсатора в обоих случаях. 471. Плоский конденсатор, пластины которого заряжены зарядами +q и —q, на половину высоты пластин погружен в жидкость с диэлектрической проницаемостью 8. Какова плотность поляризационных зарядов диэлектрика, если площадь пластин S? 472. Диэлектрик состоит из молекул, каждую из кото- рых можно представить в виде двух зарядов -\-q и —q, расположенных на расстоянии х друг от друга. При этом расстояние х зависит от напряженности поля £, действую- щего на заряды, следующим образом: kx=qE, где k — по- стоянный коэффициент. Пусть в единице объема диэлектрика содержится п мо- лекул. Определить напряженность поля Е внутри конден- сатора, заполненного таким диэлектриком, если до запол- нения напряженность поля была £^. Определить диэлектри- ческую проницаемость диэлектрика. 473. Конденсатор заполнен диэлектриком, свойства ко- торого описаны в задаче 472. Найти энергию, запасенную в диэлектрике вследствие его поляризации. 474. Две расположенные параллельно металлические пластины заряжены зарядами +qL и—q2, причем q^>qz. Про- странство между пластинами заполнено однородным изо- тропным диэлектриком с диэлектрической проницаемостью е. Определить силу, действующую на единицу площади по- верхности диэлектрика. Площадь каждой пластины равна S. 475. Определить энергию плоского конденсатора, про- странство между пластинами которого заполнено диэлект- риком. Известны заряд конденсатора и разность потенциа- лов между его обкладками. 476. Две прямоугольные пластины длины I и площади S расположены ’параллель- но друг другу на расстоя- нии d. Пластины заряжены до разности потенциалов U, В пространство между пластинами втягивается диэлектрик с диэлектриче- ской проницаемостью 8< Толщина диэлектрика рав- на d, его ширина равна ширине пластин, а длина больше I (рис. 166). Найти зависимость силы, действующей на ди- электрик со стороны поля, от расстояния х. 94
477. Решить задачу 476 в случае, когда разность потен- циалов между пластинами поддерживается постоянной и равной U. 478. Над поверхностью жидкости, налитой в большой сосуд, находятся вертикальные пластины конденсатора, касающиеся поверхности жидкости. Площадь каждой пла- стины конденсатора равна S, расстояние между пластина- ми d, их высота'/. Конденсатор присоединяют к батарее с э.д.с., равной U. Плотность жидкости р, ее диэлектриче- ская проницаемость 8. Найти максимальную высоту, на которую поднимается жидкость в процессе колебаний, а также высоту, на которой установится уровень жидкости. 479. На дне сосуда находится тонкая металлическая пластинка, площадь которой S много меньше площади дна сосуда. В сосуд налита жидкость с диэлектрической прони- цаемостью 8. Глубина жидкости много меньше линейных раз- меров пластинки. Что произойдет с жидкостью, если пла- стинке сообщить заряд +Q? § 20. Постоянный электрический ток 480. На поверхности бесконечно длинного цилиндра рас- пределены заряды таким образом, что правая половина по- верхности цилиндра от сечения 00' заряжена положитель- ным электричеством, а левая — отрицательным (рис. 167). В обоих направлениях плотность зарядов увеличивается прямо пропорционально расстоянию от сечения 00'. Пока- зать, что во всех точках внутри цилиндра напряженность Рис. 168. электрического поля будет везде одинакова и направлена вдоль оси цилиндра, как это указано на рисунке стрелкой. 481. Имеется ли вблизи поверхности проводника, по ко- торому течет постоянный ток, электрическое поле? £ 482. Начертить примерное расположение силовых ли- ний электрического поля вокруг однородного проводника, 95
согнутого в форме дуги (рис. 168). По проводнику течет по- стоянный ток. 5*83. Два проводника с температурными коэффициента- ми сопротивления и а2 имеют при О °C сопротивления /?01 и ^?02- Найти температурный коэффициент цепи, состоя- щей из этих проводников, если проводники соединены по- следовательно и если проводники соединены параллельно. Qy 484. Найти сопротивление цепи, изображенной на рис. 169. Сопротивлением соеди- ।s1 нительных проводов АС С и A. R Я t? R BCD пренебречь. £ 485. Из проволоки длиной L I___________I сопротивлением R необходимо С1 изготовить нагреватель для Рис. 169. включения в сеть с напряжением U. Известно, что по проволоке можно пропускать без риска ее пережечь ток не более /0. Какое наибольшее количество тепла q можно получить в единицу времени при помощи нагревателя? При изготов- лении проволоку можно разрезать на куски и соединять последовательно и параллельно. 486. Найти сопротивление тетраэдра ABCD. изготовлен- ного из шести проволочек сопротивлением R каждая. Под- водящие провода присоединены к вершинам А и В, 487. Найти сопротивление шестиугольника, изображен- ного на рис. 170, если он включен в цепь между точками А и В. Сопротивление каждого проводника схемы равно /?. Рис. 171. Ъ 488. Найти сопротивление проволочного куба при вклю- чении его в цепь между точками Л и В (рис. 171). Сопротив- ление каждого ребра куба равно R. 96
Ц 489. Из одинаковых по сечению S и удельному сопро- тивлению р проволок спаян прямоугольник ADBC с диаго- налью АВ, сделанной из проволоки такого же сечения и материала (рис. 172). Найти сопротивление между точками А и В и сопротивление между точками С и D, если AD — = ВС—а, AC=BD=b. Рис. 172. 490. На рис. 173 изображена схема мостика Уитстона для измерения сопротивлений. Rx — неизвестное сопро- тивление, 7?0 — эталонное сопротивление, G — гальвано- метр, соединенный скользящим контактом D с однородным проводом большого сопротивления АВ (реохорд). Показать, что при отсутствии тока через гальванометр имеет место соотношение /?ж/7?0=/1//2. Сопротивлением соединительных проводов можно пренебречь. * 491. КайЬе сопротивление необходимо включить между точками С и D (рис. 174), чтобы сопротивление всей цепочки (между точками А и В) не зависело от числа элементарных ячеек? 492. В выходных цепях генераторов 'для умень- шения выходного напряжения в желаемое число раз при- меняется устройство, называемое аттенюатором. Аттенюа- тор представляет собой делитель напряжения, собранный по схеме, изображенной на рис. 175. Специальный переклю- чатель дает возможность соединять с выходной клеммой или 4 Б. Б. Буховпев и др. .97
точку с потенциалом (70, который вырабатывается генера- тором, или любую из точек Uu U2,. * .,(7Л, потенциал каж- дой из которых меньше потенциала предыдущей в k раз Рис. 175. (fc>l). Вторая выходная клемма и нижние концы сопро- тивлений заземлены. Найти отношение сопротивлений 7?! : Т?2 : 7?3, если число ячеек аттенюатора может быть любым. 493. Какими приборами нужно располагать, чтобы осу- ществить экспериментально проверку закона Ома, т. е. по- казать, что сила тока прямо пропорциональна разности по- тенциалов? 494. Двум плоским одинаковым конденсаторам, соеди- ненным параллельно, сообщен заряд Q. В момент времени /=0 расстояние между пластинами первого конденсатора начинает равномерно увеличиваться по закону d^do+vt, а расстояние между пластинами второго конденсатора рав- номерно уменьшаться по закону d2=d0—vt. Пренебрегая сопротивлением подводящих проводов, найти силу тока в цепи во время движения пластин конденсаторов. 495. Найти работу, совершаемую электростатическим полем (см. условие задачи 494) при одновременном увеличе- нии расстояния между пластинами первого конденсатора и уменьшении расстояния между пластинами второго конден- сатора на величину а. 496. Во время работы с очень чувствительным гальвано- метром экспериментатор, сидящий на стуле у стола, обнару- жил любопытное явление. (Гальванометр был укреплен на стене, а концы его обмотки подведены к разомкнутому клю- чу, расположенному на столе.) Привстав со стула и коснув- шись стола рукой, экспериментатор наблюдал заметный отброс гальванометра. Если же экспериментатор касался стола, сидя на стуле, то отброса не наблюдалось. Точно так же зайчик гальванометра не смещался и в том случае, когда 98
экспериментатор касался стола, не садясь предварительно на стул. Как можно объяснить эти явления? 497. У очень чувствительного гальванометра при разомк- нутой цепи обнаружен следующий эффект. Если поднести к одному из концов обмотки гальванометра заряженное тело, то гальванометр даст отброс. Если же поднести это тело к другому концу обмотки, то отброс получается в ту же сто- рону. Как объяснить это явление? 498. Как распределен потенциал в элементе Даниэля при незамкнутой внешней цепи? 499, Изобразить графически примерный ход потенциала вдоль замкнутых цепей, изображенных на рис, 176. Опре- делить силу тока для каждой цепи и разность потенциалов ^1~^2 &2 Г;=Г2 а} .6) б) г) Рис. 176. между точками А и В. Сопротивлением соединительных проводов пренебречь. 500. При соединении одного моля цинка с серной кисло- той выделяется около 445-103 Дж тепла, а при выделении моля меДи из медного купороса потребляется примерно 235-103 Дж. Найти по этим данным э. д. с. элемента-Да- ниэля. 501. Два элемента Даниэля с внутренними сопротивле- ниями Г1=0,8 Ом и г2=1,3 Ом и одинаковыми э. д. с. сое- динены параллельно и замкнуты на внешнее сопротивление R. Найти отношение количеств цинка, растворившихся в этих элементах за определенный промежуток времени. 502. Элемент Даниэля составлен из абсолютно чистых материалов. Найти расход цинка и кристаллов медного ку- пороса CuSO4*5H2O, если элемент дает ток 0,1 А в течение 8 часов. 503. В элементе Даниэля медь заменили воском, покры- тым слоем графита. Описать явления, которые будут проис- ходить в таком элементе, если соединить проволокой цинк с графитовым слоем. 4* . 99
504. Как изменится э. д. с. батареи, изображенной на изменятся показания рис. 177, если убрать перегородку между сосудами? Электролитом служит раствор сер- ной кислоты. 505. Однородный угольный стер- жень лежит на дне сосуда с электро- литом. К концам стержня подклю- чен вольтметр, обладающий боль- шим сопротивлением. В середину угольного стержня упирается цин- ковый стержень. Что будет показы- вать вольтметр, если цинковый стержень стоит вертикально? Как вольтметра, если цинковый стер- жень наклонить вправо или влево? 506. Пустотелая проводящая сфера радиуса /?=5 см помещена в электролитическую ванну, наполненную раст- вором медного купороса. В поверхности сферы имеется от- верстие радиуса г=0,5 мм. На сколько увеличится вес сфе- ры, если отложение меди длится /=30 мин при плотности тока в электролите /=0,01 А/см2? 507. Если конденсате/), несущий на себе заряд Q, раз- рядить через электролитическую ванну с подкисленной во- дой, то выделится гремучий газ. По закону Фарадея ко- личество выделяющегося при электролизе вещества зависит только от количества прошедшего электричества. Значит, если разряжать конденсатор через N последовательно со- единенных ванн, то выделится в ЛГ раз больше гремучего газа. W можно сделать сколь угодно большим и получить любое количество газа. Сжигая этот газ, получим любое количество энергии, что явно противоречит закону сохра- нения энергии, так как начальная энергия заряженного конденсатора не бесконечно велика. В чем здесь дело? 508. При взрыве гремучего газа на каждый грамм про- реагировавшего водорода выделяется 145-103 Дж тепла. Используя эти данные, найти, при каком наименьшем зна- чении э. д. с. батареи может происходить электролиз воды. 509. При электролизе положительные и отрицательные ионы непрерывно нейтрализуются на соответствующих электродах. Какие причины поддерживают концентрацию ионов в электролитах на постоянном уровне? В каких участках электролита происходит пополнение убыли ионов? 510. Полная плотность тока в электролитах определяет- ся как сумма двух токов — тока положительных ионов и 100
тока отрицательных ионов: j = e(n+v+ +n_v_), где е — заряд иона, п и v — концентрации и скорости по- ложительных и отрицательных ионов. Почему количество вещества, выделившегося, к примеру, на катоде, считается пропорциональным полному току, а не току en+v + ? 511. Температура горячих спаев термобатареи tY—127 °C, холодных /2=27 °C, э. д. с. батареи <£=4 В. Для под- держания постоянной температуры нагретых спаев к ним в единицу времени -подводится две калории тепла. К батарее подключена электролитическая ванна с раст- вором медного купороса. Какое наибольшее (теоретически) количество меди может отлагаться на катоде в единицу вре- мени? 512. Два металлических шарика радиусов гх= 1 см и г2=2 см, находящиеся на расстоянии R—100 см друг от друга, присоединены к батарее с электродвижущей силой <£=3000 В/ Найти силу взаимодействия шариков. Взаимо- действием соединительных проводов пренебречь. 513. Пластины плоского конденсатора присоединены к батарее, э. д. с. которой равна <£. Подсчитать механиче- скую работу, совершаемую электрическим полем при пере- мещении пластин, если вначале расстояние между -пласти- нами равно di, а в конце d2, причем d2<db Выделением тепла в батарее и подводящих про- водах пренебречь. 514. Определить напряже- ния (71 и U2 на конденсаторах (рис. 178), если 12* 103 В, <£2=13-103 В, Ci=3 мкФ, С2 = 7 мкФ. Проводимостью диэлектриков пренебречь. 515. Одна из пластин кон- ------1|--- денсатора, присоединенного к - 1 v батарее с электродвижущей рИс. 178. Рис. 179. силой <£, заземлена (рис. 179). Изменятся ли потенциалы пластин конденсатора относи- тельно земли, если заземление убрать? 516. Через аккумулятор с внутренним сопротивлением г и э. д. с. £ течет ток силой /. Чему равна разность потен- циалов на клеммах аккумулятора? 517. Почему гальванический элемент с э. д. с. в несколь- ко вольт может дать значительныиток, а электростатическая 101
машина с э. д. с. в десятки тысяч вольт дает ток ничтожной силы? 518. В каком случае два последовательно соединенных гальванических элемента, замкнутых на внешнее сопро- тивление, дадут меньший ток, чем один из этих элементов, включенный на то же сопротивление? 519* Для определения места повреждения изоляции между проводами двухпроводной телефонной линии длиной £=5,6 км к одному ее концу присоединили батарею с э. д. с. <£=24 В. При этом оказалось, что если провода у другого конца линии разомкнуты, ток через батарею равен Л= 1,5 А, а если замкнуты накоротко, то ток через батарею равен lt=2 А. Ток короткого замыкания батареи /8=96 А. Со- противление каждого провода линии г=7 Ом. Найти со- противление изоляции 7? в месте повреждения. 520. Гальванические элементы с э. д. с. ^)1=2Ви^>2= = 1,5 В соединены по схеме, указанной на рис. 180, а. Вольт- метр, нуль которого находится посередине шкалы, показы- вает напряжение £/д=1 В, причем его стрелка отклоняется в ту же сторону, что и при разомкнутом ключе ТС Что бу- . дет показывать вольтметр, если соединить приборы по схеме рис. 180, б? Током, ответвляющимся в вольтметр, можно пренебречь. 521. Решить задачу 520 при условии, что при замкнутом ключе К (рис. 180, а) стрелка вольтметра отклоняется в сто- рону, противоположную той, что при разомкнутом ключе. Рис. 181. 522. Два элемента с э. д. с.<£1=2 Ви^2=1 В соединены по схеме, указанной на рис. 181. Сопротивление 7?=0,5 Ом. Внутренние сопротивления элементов одинаковы и равны 1 Ом каждое. Определить силы токов, протекающих через элементы и сопротивление R. Сопротивление подводящих проводов не учитывать. 102
523. При каком значении сопротивления R в схеме за- дачи 522 ток через гальванический элемент с э. д. с. не пойдет? При какйх 7? ток через этот элемент будет направ- лен против э. д. с. элемента? 524. Можно ли с помощью 24 аккумуляторов, каждый из которых имеет э. д. с. ^а=2 В и внутреннее сопротивление г=0,3 Ом, соединяя их в отдельные одинаковые группы, получить во внешней цепи сопротивлением 0,2 Ом ток /=21 А? 525. Электрическую плитку, рассчитанную на напряже- ние 220 В, требуется переделать, не меняя и не укорачивая спирали, на ПО В так, чтобы ее мощность осталась прежней. Что нужно для этого сделать? 526. Почему при включении в сеть нагревательного прибора большой мощности (например, электроутюга) на- кал лампочек в квартире сразу же заметно падает, а через небольшой промежуток времени возрастает, достигая при- мерно прежней величины? 527. Электрический чайник имеет две обмотки. При включении одной из них он закипает через время при включении другой — через время /2. Через сколько времени закипит чайник, если обе обмотки одновременно включить последовательно, параллельно? 528. Электрический кипятильник имеет три обмотки. Если соединить две обмотки параллельно, подключив к ним третью последовательно, то при различных комбина- циях обмоток вода в баке закипает соответственно за 20, 40, 16 минут. За сколько времени закипит вода, если все обмотки соединить 1) последовательно? 2) параллельно? 529. При передаче электроэнергии на большие расстоя- ния е помощью трансформатора повышают напряжение так, чтобы при той же мощности сила тока стала меньше. По за- кону Джоуля — Ленца количество выделенного в прово- дах тепла Q~I2Rf, и, следовательно, потери на выделение тепла будут малы при малых токах. Но ведь, с другой сто- роны, Q = т. е. количество выделенного тепла растет с ростом напряжения. Разъяснить, почему же увеличение напряжения ведет к экономии электроэнергии при передаче ее на большие расстояния. 530. Аккумулятор с э. д. с. <£=10 В и внутренним со- противлением г=1 Ом замкнут на внешнее сопротивление R и выделяет на нем мощность W=9 Вт. Определить 103
разность потенциалов U на клеммах аккумулятора. В чем причина неоднозначности результата? 531. Две плитки, включенные параллельно в городскую сеть, потребляют общую мощность N. Если включить их последовательно, их мощность будет больше мощности лю- бой другой пары последовательно соединенных плиток, по- требляющих при параллельном включении мощность N> Какие мощности потребляют эти плитки при включении в ту же сеть по отдельности? 532. Какую максимальную полезную мощность (мощ- ность, выделяемую на внешнем сопротивлении) может вы- делить аккумулятор с э. д. с. <£=10 В и внутренним сопро- тивлением г= \ Ом? Каково при этом сопротивление внеш- ней цепи? 533. Определить коэффициент полезного действия г| аккумулятора в задачах 530 и 532. Как зависит коэффициент полезного действия от внешнего сопротивления при неиз- менном внутреннем сопротивлении? Как при этом меняется полезная мощность? Может ли г| равняться единице? 534. Зарядка аккумулятора с начальной электродви- жущей силой осуществляется зарядной станцией, напря- жение в сети которой равно U. Внутреннее сопротивление аккумулятора г. Определить полезную мощность, расхо- дуемую на зарядку аккумулятора, и мощность, расходуемую на выделение тепла в аккумуляторе. 535. Превышает ли полезная мощность, расходуемая при зарядке аккумулятора, выделяемое в нем тепло? 536. По проводнику течет ток Z= 10 А. Площадь попе- речного сечения проводника S=5 см2, а число свобод- ных электронов в 1 см3 проводника м= 1023. Определить направленную скорость элект- ронов и, считая ее одинаковой для всех электронов. 537. Металлический прямоугольный парал- лелепипед с размерами сторон d, fe, с b^>c) движется с ускорением а в направлении меньшей стороны (рис. 182). Найти напряжен- ность электрического поля, возникающего вследствие ускоренного движения металличе- ского бруска, а также плотность электрических зарядов на боковых поверхностях бруска, перпендикулярных направ- лению ускорения. 538. Сплошной металлический цилиндр радиуса R вращается с постоянной угловой скоростью со. Найти 104
зависимость напряженности поля от расстояния до оси цилиндра и разность потенциалов между поверхностью цилиндра и осью. 539. Имеется металлический диск радиусом 7? (рис. 183), вращающийся с угловой скоро- стью со. Диск включен в элек- трическую цепь при помощи скользящих контактов, касаю- щихся оси диска и его окруж- ности. Сопротивление диска ни- чтожно мало по сравнению с со- противлением нагрузки 7?0. Оп- ределить количество тепла, выделяющееся за единицу вре- мени. Объяснить с точки зрения электронной теории ме- таллов, что тормозит диск. §21. Электрический ток в газах и вакууме 540. Две частицы с массами тг и т2 испытывают цент- ральное неупругое соударение. Энергия, необходимая для ионизации второй частицы, равна So- Какой минимальной энергией Si min Должна обладать первая частица до соуда- рения, чтобы эта ионизация произошла? Вторая частица до соударения покоилась. 541. На рис. 184 изображен счетчик Гейгера—Мюл- лера элементарных частиц. Между корпусом трубки А и тонкой проволочкой создается вы- Д сокое напряжение, лишь немного .... ||Н-| меньшее «критического», необходи- мого для зажигания разряда. При А попадании в счетчик быстрой* заря- да ^[1 женной частицы происходит иони- зация молекул газа и начинается I---------- разряд, Прохождение по цепи то- Рис. 184. ка сопровождается падением напря- жения на большом сопротивлении R. Это падение напряжения регистрируется после усиле- ния соответствующими устройствами. Для того чтобы счет- чик отвечал своему назначению, необходимо быстрое гашение вызванного частицей разряда. Вследствие какой причины происходит гашение разряда в схеме рис. 184? 542. К источнику высокого напряжения через сопро- тивление /? = 103 Ом подключен конденсатор С=10“11 Ф 105
с расстоянием между пластинами d~ 3 мм (рис. 185). Воз- дух в пространстве между пластинами конденсатора иони- зуется рентгеновскими лучами так, что в 1 см3 обра- зуется п=104 пар ионов в секунду. Заряд -----II 1 каждого иона равен заряду электрона, /7-----------I Найти падение напряжения на сопротивле- П нии /?, считая, что все ионы достигают пластин конденсатора, не успевая реком- Рис. 185. бинировать. 543. Ионизатор создает в единицу време- ни в единице объема газа Дл0 ионов каждо- го знака. В данный момент времени в еди- нице объема газа имеется п0 положительных и столько же отрицательных ионов. Газ находится между двумя плоскими параллельными электродами, площадь ко- торых равна S и расстояние между которыми равно /. Сила тока между электродами /. Считая, что число реком- бинировавших в единице объема в единицу времени ионов равно Апо=у/?2, где у — постоянный коэффициент реком- бинации, определить, при каком условии концентрация ионов между электродами не будет изменяться со временем. Заряд одного иона равен q. 544. Предположим, что в предыдущей задаче ионизо- ванный газ находится между электродами в электрическом поле, напряженность которого равна Е. Показать, что при условии IlqSl<^yn2 для несамостоятельной проводимости справедлив закон Ома. При этом считать, что скорости направленного движения положительных и отрицательных ионов равны и+=Ь+Е и и_=Ь_Е соответственно, где Ь + и Ь_ — постоянные коэффициенты (называемые подвиж- ностью газовых ионов). 545. Показать, что плотность тока ионов в задаче 543 при условии yn^^IIqSl не зависит от разности потен- циалов между электродами. Объяснить, почему плотность тока тем больше, чем больше расстояние между электро- дами. 546. Начертить график распределения напряжения в тлеющем разряде. 547. Описать поведение различных частей тлеющего разряда: 1) при передвижении анода в направлении като- да; 2) при передвижении катода в направлении анода. 548. Что произойдет с горящей электрической дугой, если сильно охладить отрицательный уголь? Что будет при охлаждении положительного угля? 106
549. К электростатической машине подключены соеди- ненные параллельно лейденская банка и разрядник. Ток электростатической машины I—10~5 А. Емкость лейденской банки С=10~8 Ф. Чтобы произошел искровой разряд, ма- шина должна работать время /=30 с. Длительность разряда т=10“6 с. Определить величину тока в разряде /р и напря- жение зажигания искрового разряда U3. Емкостью раз- рядника пренебречь. 550. Разрядник электростатической машины, диски ко- торой вращаются с постоянной скоростью, присоединен к обкладкам лейденской банки. Между шариками разрядника через равные промежутки времени т0 проскакивают искры. Через какие промежутки времени т будут проскакивать иск- ры, если к разряднику присоединить две лейденские банки, соединенные между собой один раз параллельно, а дру- гой — последовательно? Емкость каждой банки такая же, как в первом случае. 551. Какую энергию в эргах приобретает электрон, пройдя в вакууме разность потенциалов 1 В?(В атомной фи- зике эта энергия принимается за единицу «электрон-вольт».) 552. Совпадает ли траектория движения заряженной частицы в электростатическом поле с силовой линией? 553. Между нитью накала, испускающей электроны, и проводящим кольцом создана разность потенциалов U (рис. 186). Электроны движутся ускоренно вдоль оси коль- ца. При этом их кинетическая энергия увеличивается, в то время как батарея, создающая разность потенциалов U, не совершает работы, так как ток в цепи не идет. (Предпо- лагается; что электроны не попадают на кольцо.) Как это согласовать с законом сохранения энергии? 554. Триод прямого накала включен в цепь, изображен- ную на рис. 187. Э.д.с. анодной батареи <^х=80 В, батареи накала В и сеточной батареи ^3=2 В. С какими энергиями будут электроны достигать анода лампы? Как 107
изменится энергия электронов, достигающих анода, если будет изменяться по величине или даже переменит знак? Анодный ток считать малым по сравнению с током накала. 555. Анодный ток некоторой двухэлектродной лампы в определенном интервале напряжений может быть связан с разностью потенциалов Ua между электродами уравне- нием Ia=AUa+BUl. Найти анодный ток, если такая лампа включена последовательное сопротивлением 7?а=2-104 Ом в цепь батареи с э.д.с. ^=120 В. Для данной лампы А = =0,15 мА/B, В=0,005 мА/B2. Внутренним сопротивлением батареи пренебречь. 556. Две электронные лампы соединены параллельно и включены в цепь батареи с э.д.с. <£=300 В последова- тельно с сопротивлением /?=4-103 Ом (рис. 188). Зависи- мость анодного тока i от анодного напряжения Ua для каждой из ламп может быть приближенно представлена в виде i==AUa+BUl, где для одной лампы А!=0,07 мА/В, 2^=0,005 мА/B2, а для* другой лампы А2=0,03 мА/В, В2=0,01 мА/B2. Определить анодные токи ламп. Внутрен- ним сопротивлением батареи можно пренебречь. Рис. 188. 557. Электронная лампа включена в цепь батареи с э.д.с. ^=250 В последовательно с сопротивлением /?= = 104 Ом (рис. 189). Сетка лампы соединена с отрицательным полюсом батареи (^\=3 В), а катод — с положительным ее полюсом. Падение напряжения на сопротивлении R при этом достигает £7Т=95 В. Если же в цепи сетки стоит бата- рея с <£2=6 В, то разность потенциалов на сопротивлении R будет t/2=60 В. Какова будет разность потенциалов меж- ду анодом и катодом лампы, если замкнуть накоротко сетку и катод? (В рассматриваемой области изменений потенциала сетки считать сеточную характеристику лампы прямой ли- нией.) 108
558. Три одинаковых диода, анодные характеристики которых могут быть приближенно представлены отрезками прямых: /а = 0 при t/a^0, /а = kUa при ил > О, где А:==0,12 мА/B, включены в цепь, как показано на рис. 190. Начертить график зависимости тока / в цепи Рис. 190. от напряжения V, если <£1=2 В, <£2=5 В, <£3=7 В, а V может меняться от —10 В до + 10 В. 559. Вычислить чувствительность электронно-лучевой трубки к напряжению, т. е. величину отклонения пятна на экране, вызванного разностью потенциалов в 1 В на управляющих пластинах. Длина управляющих пластин /, расстояние между ними d, расстояние от конца пластин до экрана L, и ускоряющая разность потенциалов (70. § 22. Магнитное поле тока. Действие магнитного поля на ток и движущиеся заряды 560. Исходя из соображений размерности, определить напряженность магнитного поля на расстоянии г от 1) бес- конечно длинной прямой нити, по которой течет ток /; 2) бесконечной плоскости, по которой течет поверхностный ток плотности /. 561. По бесконечной прямолинейной тонкостенной трубе течет ток /. Определить индукцию магнитного поля в про- извольной точке внутри трубы. 562. Учитывая, что индукция магнитного поля внутри длинного цилиндрического проводника равна B=k-2njr, где / — плотность тока, г — расстояние от оси проводника, k — коэффициент, зависящий от выбора системы единиц, определить индукцию магнитного поля в произвольной 109
точке внутри длинной цилиндрической полости, вырезан- ной параллельно оси проводника. По проводнику течет ток плотности /. Расстояние между осями проводника и полости равно d. 563. Начертить распределение линий индукции магнит- ного поля в полости проводника, описанного в задаче 562. • 564. По контуру в виде круга радиуса R течет ток. Определить индукцию магнитного поля в центре круга, если сила тока равна I. Примечание. При определении индукции маг- нитного поля можно воспользоваться законом Био—Сава- ра—Лапласа. Этот закон утверждает, что элемент контура А/, по которому течет ток /, создает в произвольной точке А пространства магнитное поле, индукция которого равна . п , / Д/ sin а = k7—, где г — расстояние от элемента AZ до точки Л, а — угол, который составляет радиус-вектор г с элементом AZ, k — коэффициент, зависящий от выбора системы единиц. Нап- равление АВ определяется правилом буравчика: направле- ние вращения головки буравчика соответствует направле- нию тока 1 в элементе контура А/. Вектор АВ перпендику- лярен к плоскости, содержащей элемент А/ и радиус-век- тор г. * 565. По контуру в виде кольца радиуса R течет ток Л Определить индукцию магнитного поля в произвольной точке, лежащей на перпендикуляре, восставленном к плос- кости кольца из его центра. • 566. По бесконечно длинному проводнику ЛВС, изогну- тому под прямым углом, течет ток / (рис. 191). Во сколько Z7 Рис. 191. v Рис. 192. раз изменится напряженность магнитного поля в точке М, если к точке В присоединить бесконечно длинный пря- но
мой провод BD так, чтобы ток / разветвлялся в точке В на две равные части, а ток в проводнике АВ оставался бы прежним? ♦ 567. По проводнику, расположенному в одной плоскос- ти, как изображено на рис. 192, течет ток. Найти индукцию магнитного поля в произвольной точке линии АВ, являю- щейся осью симметрии проводника. 568. Под длинной горизонтальной шиной на двух оди- наковых пружинах (коэффициент упругости каждой равен k) подвешен провод длиной /. Когда по шине и проводу токи не текут, расстояние между ними Л. Найти расстоя- ние между шиной и проводом, если по Шине течет ток /, а по проводу i. Провод не может выйти из вертикальной плоскости. * 569. Определить силу, с которой действует бесконечно длинный прямой провод на прямоугольный контур, рас- положенный в плоскости провода. Извест- но, что по проводу течет ток /, а по контуру — /г. Стороны контура AD и ВС имеют длину а и расположены парал- лельно проводу. Расстояние от AD до про- вода х. Длина сторон AB=DC=h. Направ- ления токов указаны на рис. 193 стрелками. * 570. Медный провод сечением S, согну- тый в виде трех сторон квадрата, может вращаться вокруг горизонтальной оси (рис. 194). Провод находится в однородном магнитном поле, направленном вертикаль- но. Когда по проводу течет ток /, провод отклоняется на угол а от вертикали. Определить индукцию поля. Плот- ность меди равна р. Рис. 194. Рис. 195. 571. В центре длинного соленоида, на каждый сантиметр длины которого приходится п витков, находится короткая катушка, состоящая из N витков и имеющая сечение S. Ось этой катушки перпендикулярна оси длинного солено- ида и направлена вертикально. Внутренняя катушка укреп- 111
лена на одном конце коромысла весов, которые в отсутствие тока находятся в равновесии. Когда через обе катушки пропускают один и тот же ток /, для уравновешивания ве- сов на правое плечо коромысла (рис. 195) приходится доба- вить груз Р. Длина правого плеча коромысла равна L. Определить силу тока /. Примечание. Индукция магнитного поля вблизи центра длинного соленоида равна В=ц0^Л где п — число витков на единицу длины соленоида, а / —сила тока, те- кущего по соленоиду. 572. По проволочному кольцу радиуса R, подвешенному на двух гибких проводниках, течет ток /. Кольцо помеще- но в однородное магнитное поле с индукцией В. Линии индукции горизонтальны. С какой силой растянуто кольцо? 573. Проволочное кольцо радиуса R находится в неод- нородном магнитном поле, линии индукции которого сос- тавляют в точках пересечения с кольцом угол а относительно нормали к плоскости кольца (рис. 196). Индукция магнит- ного поля, действующего на кольцо, равна В. По кольцу течет ток /. С какой силой магнитное поле действует на кольцо? Рис. 197. 574. Прямоугольный контур A BCD. стороны которого имеют длину а и Ь. находится в однородном магнитном поле индукции В и может вращаться вокруг оси 00' (рис. 197). По контуру течет постоянный ток /. Определить работу, совершенную магнитным полем при повороте контура на 180°, если вначале плоскость контура была перпендику- лярна магнитному полю и расположена так, как показано на рис. 197. 112
575. Как будет двигаться в однородном магнитном поле электрон, если в начальный момент его скорость составляет угол а с линиями индукции поля? 576. По металлической ленте ширины АВ=а течет ток /. Лента помещена в магнитное поле, индукция которого перпендикулярна ленте (рис. 198). Определить разность потенциалов между точками А и В ленты. 577. Незаряженный металлический брусок представля- ет собой прямоугольный параллелепипед со сторонами а. Ь, с (а^>с, Ь^с). Брусок дви- жется в магнитном поле в на- правлении стороны а со скоро- стью V, Индукция магнитного поля В перпендикулярна осно- 0 Рис. 198. ванию бруска со сторонами а, с (рис. 199). Определить напряженность электрического поля в бруске и плотность электрических зарядов на боковых поверхностях парал- лелепипеда, образованных сторонами я, Ь. 578. Незаряженный металлический цилиндр радиуса г вращается в магнитном поле с угловой скоростью <о вокруг своей оси. Индукция магнитного поля направлена вдоль оси цилиндра. Каково должно быть значение индукции магнитного поля, чтобы в цилиндре не возникло электро- статическое поле? 579. Найти напряженность электростатического поля в цилиндре (см. задачу 578), если индукция магнитного поля равна В. 580. Пучок однозарядных ионов попадает в область прост- ранства, где имеется однородное электрическое поле с на- пряженностью £=100 Н/Кл и однородное магнитное поле с индукцией В=0,02 Н/(А-м). Электрическое и магнитное поля направлены под прямым углом друг к другу и оба перпендикулярны к пучку. Ионы проходят эти скрещен- ные электрическое и магнитное поля без отклонения и про- никают через щель в область однородного магнитного поля с индукцией В'=0,09 Н/(А-м), направленной перпендику- 113
лярно движению ионов. Если ионы представляют собой смесь с массами, равными 20 и 22 атомным единицам массы, то на каком расстоянии друг от друга эти ионы окажутся, пройдя половину окружности? § 23. Электромагнитная индукция. Переменный ток 581. Реактивный самолет с размахом крыльев 20 м летит прямо на север со скоростью 960 км/ч и на такой высоте, где вертикальная составляющая индукции магнитного поля Земли равна 6-10“5 Т. Чему равна разность потенциалов между концами крыльев? На каком крыле более высокий потенциал? 582. По двум направляющим металлическим рейкам дви- жется без трения с постоянной скоростью проводник длины Z, электрическое сопротивление которого равно г. Рейки соединены неподвижным проводником, сопротивление кото- рого равно R. Все проводники расположены в одной плос- кости и находятся в постоянном магнитном поле, индукция которого равна В и направлена перпендикулярно плоскости проводников (рис. 200). Электрическое сопротивление реек мало по сравнениюс г и 7?. Определить разность потенциа- лов электрического поля между точками Ли В. Объяснить причину возникновения электрического тока. Д В Рис. 200. Рис. 20L 0Z? Z 583. По двум вертикальным рейкам АВ и CD, соединен- ным сопротивлением J?, может без трения скользить про- водник, длина которого I и масса т. Система находится в однородном магнитном поле, индукция которого В перпен- дикулярна плоскости рисунка (рис. 201). Как будет дви- гаться подвижный проводник в поле силы тяжести, если пренебречь сопротивлением самого проводника и реек? 114
584. По двум металлическим параллельным рейкам, рас- положенным в горизонтальной плоскости и замкнутым на конденсатор емкости С, может без трения двигаться провод- ник массой иг и длины I. Вся система находится в однородном магнитном поле, индукция которого В направлена вверх. К середине проводника перпендикулярно к нему и параллель- но рейкам приложена сила F (рис. 202). Определить ускоре- ние подвижного проводника, если сопротивление реек, под- водящих проводов и подвижного проводника равно нулю. В какие виды энергии превращается работа силы F? Счи- тать, что в начальный момент скорость проводника равна нулю. 585. Прямоугольная рамка расположена в плоскости бесконечного прямолинейного проводника с током, причем стороны AD и ВС параллельны провод- нику (рис. 203). В середине стороны DC включен прибор, измеряющий заряд, протекающий по контуру (на рисунке не показан). Рамку можно перевести в А В Рис. 203. /г /7 = ------*- Рис. 202. новое положение, изображенное на рис. 203 пунктиром, двумя способами: 1) перемещая ее параллельно самой себе; 2) вращая на 180° вокруг стороны ВС. В каком случае заряд, протекший через прибор, будет больше?- 586. Катушка из п витков, площадь каждого из которых S, присоединена к баллистическому гальванометру. (Бал- листический гальванометр измеряет количество прошедшего через него электричества.) Сопротивление всей цепи R. Вначале катушка находится между полюсами магнита в об- ласти, где магнитное поле однородно и индукция его В перпендикулярна площади витков. ЗатехМ катушку пере- местили в пространство, где магнитное поле отсутствует. Чему равно количество электричества, прошедшего через гальванометр? 587. Прямоугольный контур ABCD перемещается пос- тупательно в магнитном поле тока /, текущего по длинному прямому проводу ОО'. Стороны AD и ВС параллельны про- воду. Определить величину и направление тока, индуциро- 115
ванного в контуре, если контур перемещается с постоянной скоростью и. AD—BC=a) AB~DC=b. Сопротивление кон- тура R. (Рис. 204.) 588. Проволочное кольцо радиуса г находится в одно- родном магнитном поле, индукция которого перпендику- лярна плоскости кольца и меняется с течением времени по закону B=kt. Определить напряженность электрического поля в витке. О' Рис. 204. 589. Кольцо прямоугольного сечения (рис. 205) сделано из материала с удельным сопротивлением р. Кольцо нахо- дится в однородном магнитном поле. Индукция магнитного поля направлена по оси кольца и увеличивается прямо про- порционально времени: В—kt. Найти силу тока, индуци- руемого в кольце. 590. Одна половина проволочного кольца радиуса г имеет сопротивление Ru а другая Т?2. Кольцо находится в однородном магнитном поле, индукция которого перпен- дикулярна плоскости кольца и меняется с течением вре- мени по закону B—BQ-\-kt (Во — постоянная величина). Определить напряженность электростатического поля в кольце. 591. Определить силу тока в про- водниках цепи, изображенной на рис. 206, если индукция однород- 0 G F ного магнитного поля перпендику- Рис. 206. лярна плоскости чертежа и изменя- ется во времени по закону B=kt. Сопротивление единицы длины проводников равно г. 592. В однородном круглом проволочном кольце со- здан постоянный индукционный ток /. Переменное маг- 116 а а
нитное поле, создающее этот ток, перпендикулярно плоско- сти кольца, сосредоточено вблизи его оси симметрии и имеет ось симметрии, проходящую через центр кольца (рис. 207). Чему равна разность потенциалов между точками Л и В? Что будет показывать электрометр, присоединенный к этим точкам? 593. Переменное магнитное поле создает в кольцевом проводнике ADBKA постоянную электродвижущую силу (см. задачу 592). Сопротивления проводников ЛОВ, АКВ и АС В (рис. 208) равны соответственно Т?2 и /?3. Какую силу тока будет показывать амперметр С? Магнит- ное поле сосредоточено у оси кольцевого проводника. 594. Сопротивление проводника АС В (см. задачу 593) 7?з=0. Найти силы токов /ь /2 и /3 и разность потенциалов UA-UB. 595. По двум металлическим параллельным рейкам, зам- кнутым на сопротивление 7?, перемещается проводник дли- ны /. Скорость перемещения проводника равна v. Вся сис- тема находится в однородном которого направлена перпен- дикулярно плоскости, в кото- рой лежат рейки, и изменя- ется со временем по закону В=В0+^/. В начальный мо- мент площадь abed равна So. Определить силу тока в цепи. (Рис. 209.) 596. В однородном магнит- ном поле находится проволоч- ное кольцо, способное вращаться вокруг диаметра, пер- пендикулярного линиям магнитной индукции. Индукция поля начинает возрастать. Найти возможные положения магнитном поле, индукция Рис. 209. 117
равновесия кольца и указать положение устойчивого равновесия. Что изменится, если индукция будет убывать? 597. На цилиндр из немагнитного материала намотано N витков проволоки (соленоид). Радиус цилиндра г, его длина I (г<^.Г). Сопротивление проволоки R. Каково должно быть напряжение на концах проволоки, чтобы ток возра- стал прямо пропорционально времени, т. е. чтобы выпол- нялось равенство I=kt? 598. Соленоид (см. задачу 597) присоединен к батарее, э.д.с. которой равна В момент времени /=0 замыкают ключ. Чему будет равна сила тока в цепи соленоида, если пренебречь сопротивлением R соленоида, батареи и подводящих проводов? 599. Подсчитать работу батареи (см. задачу 598) за время т. В какой вид энергии превращается эта работа? 600. Кольцо из сверхпроводника помещено в однород- ное магнитное поле, индукция которого нарастает от нуля до Во. Плоскость кольца перпендикулярна линиям индук- ции магнитного поля. Определить силу индукционного тока, возникающего в кольце. Радиус кольца г, индуктивность L. 601. В однородном магнитном поле с индукцией В нахо- дится сверхпроводящее кольцо радиуса г. Линии магнитной индукции перпендикулярны плоскости кольца. Тока в коль- це нет. Найти магнитный поток, пронизывающий кольцо, после того, как магнитное поле будет выключено. у/////, 602. Перед полюсом электро- магнита на длинной нити под- о л вешено кольцо из сверхпровод- I Г ника (рис. 210). Что произойдет 1-1 1 с кольцом, если по обмотке Й \ Ж электромагнита пропустить пе- V Ш ременный ток? 603. Из провода длины I из- Рис. 210. готовили соленоид длины /0. Ди- аметр соленоида d<^/0- Опреде- лить индуктивность соленоида. 604. Через соленоид длиной 1 м, имеющий 2000 витков диаметром 10 см, течет ток 1 А. Соленоид равномерно рас- тягивается со скоростью 40 см/с, причем приложенная к со- леноиду разность потенциалов непрерывно меняется так, что ток остается постоянным. Каково будет изменение раз- ности потенциалов к тому моменту, когда соленоид растя- нется вдвое? Изменением диайетра соленоида при растяже- нии пренебречь. 118
605. Магнитное поле внутри разомкнутого соленоида однородно, и его индукция меняется с течением времени по закону В=В0 cos (at. Определить напряжение U на концах соленоида, возникающее при этом. Соленоид имеет W вит- ков, и его радиус равен г. 606. Последовательно с соленоидом, имеющим N витков радиуса г, соединено омическое сопротивление /?. Магнит- ное поле внутри соленоида однородно, и его индукция ме- няется со временем по закону В=В0 cos (at. Определить напряжение UАв между точками А и В и ток I в цепи. (Рис. 211.) Рис. 211. Рис. 212. 607. Последовательно с соленоидом включены конден- сатор емкости С и омическое сопротивление R. Соленоид имеет N витков радиуса г. Магнитное поле внутри соле- ноида однородно, и его индукция изменяется со временем по закону B=BQcos(at. Определить напряжение UAB меж- ду точками А и В и напряжение UBC между В и. С, а также ток I в цепи. (Рис. 212.) 608. 1) От середины катушки с железным сердечником (обмотка — толстый медный провод с большим числом витков) сделан отвод С (рис. 213). Между точками В и С создается постоянное напряжение иг. Найти напряжение U2 между точками А и В. 2) Между точками В и С прило- жено переменное напряжение (на- пример, от городской сети) с амплитудой Ult Найти амплитуду U2 переменного напряжения между точками А и В. 609. Обмотка лабораторного регулировочного автотран- сформатора (ЛАТР) намотана на железном сердечнике, име- ющем форму прямоугольного тороида (рис. 214), Для за- 119
щиты от вихревых токов (токов Фуко) сердечник набирают из тонких железных пластин, изолированных друг от друга ' слоем лака. Это можно сделать различны- ми способами: 1) набирая сердечник из тонких колец, положенных стопкой одно на другое; 2) свертывая в рулон длинную ленту, имеющую ширину Л; 3) собирая сердечник из прямоугольных пластин раз- мером /ХЛ, расположив их вдоль радиу- сов цилиндров. Какой способ лучше? £ 610. По катушке, не имеющей омиче- ского сопротивления, протекает перемен- Рис. 214. ный синусоидальный ток. Индуктивность катушки L. Начертить график изменения со временем произведения тока на напряжение (мгновен- ной мощности). Объяснить характер кривой. Чему равна средняя за период мощность, потребляемая катушкой? 611. Найти эффективную величину переменного тока, изменяющегося по закону: Т / = /0 при 0 < t < -у; т т 1 = 0 при у < t < у; / = -/. при L<t<^T- 1 = 0 при 4 Т < t < Т; 1 = 1. при T<t<^T и т. д. (Рис. 215.) 612. Почему наличие очень высокого напряжения во вторичной обмотке повышающего трансформатора не при- водит к большим потерям энергии на выделение тёпла в самой обмотке? 120
Рис. 216. 613. Зачем в сетях переменного тока, содержащих боль- шое число электроприборов значительной индуктивности (например, дросселей), параллельно этим приборам под- ключают конденсаторы? 614. Для определения мощности, выделяемой перемен- ным током в катушке с коэффициентом самоиндукции L и омическим сопротивлением иногда применяют метод трех вольтметров, за- ключающийся в следующем. Включают известное сопротивление R и три вольт- метра так, как указано на рис. 216. Из- мерив с помощью этих вольтметров эф- фективные напряжения: — на катуш- ке, U2 — на сопротивлении R и U — между клеммами катушки и сопротивле- ния, определяют искомую мощность W. Какова она? 6F5. Поверх длинного соленоида, имеющего Afi витков, длину I и площадь сечения S, вплотную намотан по всей длине второй соленоид,, имеющий N2 витков и такое же сечение S. Определить коэффициент взаимной индукции со- леноидов. (Коэффициент взаимной индукции £1а двух кон- туров численно равен потоку магнитной индукции, прони- зывающему второй контур в случае, когда через первый контур протекает ток 1—\ А.) 616. Поверх длинного соленоида вплотную намотана катушка. Ток в соленоиде нарастает'прямо пррпорционально времени. Каков характер зависимости тока от времени в катушке? 617. Два сверхпроводящих кольца радиуса г Находятся на расстоянии d друг от друга, причем d^>r. Центры колец лежат на прямой 00', перпендикулярной плоскостям обоих Рис. 217. колец. Кольца могут перемещаться только вдоль этой пря- мой. В начальный момент по кольцам текут в одном нап- равлении токи одинаковой величины /0. Какие токи уста- новятся в кольцах после того, как они сблизятся вплот- ную? (Рис. 217.) 121
618. Описать характер движения колец задачи 617, если в начальный момент времени текущие по ним токи имели разную величину. Рассматривать только силы маг- нитного взаимодействия. 619. Показать, что при пренебрежении током холосто- го хода трансформатора и омическим сопротивлением его обмоток имеет место соотношение где Л и /2 — токи в обмотках, а Л\. и W2 — числа витков в них. Обмотки рассматривать как катушки с одинаковым по- перечным сечением. 620. Поверх длинного соленоида, имеющего длину /, сечение S и число витков ЛГх, вплотную по всей длине на- мотан второй соленоид, имею- щий число витков ЛГа и такое же сечение S, как и первый. По первому соленоиду течет ток /ъ по второму — ток /а. Найти энергию магнитного поля этой системы. 62Т*. На какие пробивные . напряжения должны быть рассчитаны конденсатор’ С и диод JT, если выпрямитель (рис. 218) может работать как под нагрузкой', таки без нее? $24. Электрические машины 622. Сопротивление в цепи нагрузки генератора пере- менного тока увеличилось. Как должна измениться мощ- ность двигателя, вращающего генератор, чтобы частота пе- ременного тока осталась прежней? 62S. К зажимам генератора синусоидальной э.д.с. пос- тоянной амплитуды подключают конденсаторы Сх и С2. Первый раз конденсаторы соединены между собой парал- лельно, второй — последовательно. Во сколько раз должна измениться частота генератора, чтобы ток через него был одинаковым в обоих случаях? Внутренним сопротивлением генератора пренебречь. 624. Сила, действующая на движущуюся заряженную частицу со стороны магнитного поля (сила Лоренца), всегда перпендикулярна скорости; следовательно, эта сила не со- вершает работы. Почему же в таком случае работает электро- мотор? Ведь сила, действующая на проводник с током, возникает в результате действия поля на отдельные заря- женные частицы, движение которых образует ток, 122
625. Может ли сериесный мотор постоянного тока, включенный в сеть с напряжением (7=120 В, развить мощ- ность №=200 Вт, если сопротивление его обмоток ??= =20 Ом? 626. Сериесный мотор, питающийся ст источника по- стоянного напряжения, работает в режиме, обеспечиваю- щем получение от него максимальной механической -мощ- ности №. Какое количество тепла за единицу времени выделится в моторе, если остановить (заклинить) его вал? 627. Какими параметрами сети определялась бы мощ- ность сериесного электромотора постоянного тока, вклю- ченного в эту сеть, если бы его обмотка была сделана из сверхпроводника? 628. Определить коэффициент полезного действия сери- есного и шунтового моторов при условии, что развиваемая ими мощность максимальна. Напряжение на зажимах I/; сопротивления обмоток ротора и статора /?2 одинаковы у обоих моторов и предполагаются известными. 629. Почему сериесный мотор, включенный в цепь на холостом ходу, «разносит», т. е. его якорь набирает ско- рость, угрожающую механической прочности двигателя? 630. Ротор модели мотора постоянного тока состоит из одного витка, имеющего форму прямоугольника. Индук- ция магнитного поля В, создаваемая постоянным магни- том (слева — север, справа —юг), направлена по радиусу, так как зазор между полюсными наконечниками и железным цилиндром А очень мал (рис. 219). К витку, площадь кото- рого S и сопротивление 7?, приложена разность потенциа- лов (7. Определить мощность мотора как функцию угловой скорости со. При какой угловой скорости «в мощность будет максимальной? Чему будет равна при этом сила тока? 123
631. Используя условие предыдущей задачи, опреде- лить зависимость вращающего момента М от угловой ско- рости. 632. Определить характер зависимости мощнссги мо- дели мотора постоянного тока (см. задачу 630) от индукции магнитного поля В при заданном числе оборотов. При ка- ком значении В мощность максимальна? 633. Определить индукцию магнитного поля в модели мотора постоянного тока (см. задачу 630), при которой вра- щающий момент М максимален. Число оборотов якоря задано. 634. Шунтовой электромотор постоянного тока при нап- ряжении на зажимах £7=120 В развивает механическую мощность W= 160 Вт. Число оборотов в секунду якоря мо- тора п=10 об/с. Определить максимально возможное число оборотов мотора при данном напряжении. Сопротивление якоря 7?=20 Ом. 635. Шунтовой мотор постоянного тока при напряжении на зажимах (7=120 В имеет угловую скорость вращения якоря со= 100 рад/с. Сопротивление обмотки якоря мотора /?=20 Ом. Какую электродвижущую силу разовьет этот мотор, используемый как генератор, если его вращать с той же угловой скоростью? Напряжение н*а обмотках статора поддерживается постоянным и равным 120 В. Механический момент на валу двигателя при указанной скорости М — = 1,6 Н-м. 636. Как изменится скорость вращения шунтового мо- тора при увеличении силы тока в обмотках статора, если напряжение на якоре U и приложенный к оси якоря механический момент М ос- таются постоянными? Рис. 220. 637. Доказать, что если значения индукции магнитных полей, создаваемых тремя парами электромагнитов,равны по амплитуде и смещены по фазе на 2л/3 (рис. 220), то резуль- 124
тирующее магнитное поле можно изобразить вектором, вра- щающимся с постоянной угловой скоростью со вокруг точ- ки О. Каждая пара электромагнитов создает магнитные поля, направленные по соответствующим диаметрам коль- ца: Bi, В2, В3. Электромагниты питаются переменным током частоты со. 638. Магнитное поле индукции В вращается в плоскости чертежа с угловой скоростью со. В этом поле находится рамка, стороны которой равны а и Ь, Омическое сопротив- ление рамки 7?. Нормаль к плоскости рамки вращается в плоскости чертежа с угловой скоростью Q. Найти силу тока, индуцированного в рамке. (Рис. 221.) 639. Найти момент сил, приложенных к рамке, описан- ной в задаче 638.
Глава IV. КОЛЕБАНИЯ И ВОЛНЫ § 25. Механические колебания 640. На неподвижном круглом цилиндре радиуса R ле- жит доска, как показано на рис. 222. Толщина доски равна /г. Найти условие, при котором после отклонения на малый угол от горизонтали доска будет совершать колеба- ния около положения равно- весия. Проскальзывания нет. 641. Определить с точно- стью до безразмерного коэф- фициента период колебаний тела с массой т, прикреплен- ного к пружине с коэффи- циентом жесткости k. 642. Доказать, что период колебаний математического маятника увеличивается с увеличением максимального угла отклонения от положения равновесия. 643. Исходя из соображений размерностей, определить период колебаний математического маятника. 644. Два бруска, массы которых равны Шг и ги2, связаны пружиной жесткости k. Пружина сжата при помощи двух нитей, как показано на рис. 223. Нити пережигают. Опре- делить период колебаний брусков. ГП^ 7/77777?^^^^ Рис. 223. 645. Два груза с массами и с коэффициентом жесткости k. В Рис. 224. т2 соединены пружиной начальный момент пру- 126
I I m Рис. 225. жина сжата на величину х так, что первый груз прижат вплотную к стенке (рис. 224), а второй груз удерживается упором. Как будут двигаться грузы, если упор убрать? 646. Как изменится период вертикальных колебаний груза, висящего на двух одинаковых пружинах, если их последовательное соединение • заменить параллельным? 647. Два математических маятника длины I каждый свя- заны невесомой пружиной так, как указано на рис. 225. Коэффициент упругости пру- жины равен k. При равнове- сии маятники занимают верти-т кальное положение и пружи- на не деформирована. Опреде- лить частоты малых колебаний двух связанных маятников в случаях, когда маятники от- клонены в одной плоскости на равные углы в одну сторону (колебания в фазе) и в разные стороны (колебания в проти- вофазе). 648. Груз на длинной нити может совершать колебания в вертикальной плоскости, отклоняясь на угол а от вер- тикали (математический маятник). Этот же груз может вра- щаться по окружности так, что описывает конус (коничес- кий маятник). В каком случае натяжение нити, отклонен- ной на угол а от вертикали, будет больше? 649. Часы с маятником наповерхности земли идут точно. В каком случае эти часы больше отстанут за сутки: если их поднять на высоту 200 м или же опустить в шахту на глубину 200 м? 650. На концах неве- сомого стержня длиной d =1 м укреплены два маленьких шарика с мас- С JJ сами т=\ г. Стержень подвешен на шарнире так, что может вращать- ся без трения около вер- тикальной оси, проходя- щей через его середину. На одной прямой со стержнем укреплены два больших шара с массами Л4=20 кг. Расстоя- ние между центрами большого и малого шаров L—16 см d Рис. 226. 127
(рис. 226). Вычислить период малых колебаний описан- ного крутильного маятника. 651. Чему равен период колебаний математического ма- ятника, находящегося в вагоне, движущемся горизонтально с ускорением а? 652. Определить период колебаний маятника в лифте, движущемся вертикально с ускорением а, направленным вверх. 653. Решить предыдущую задачу в случае, когда уско- рение а направлено вниз. 654. Кубик совершает малые колебания в вертикальной плоскости, двигаясь без трения по внутренней поверхности сферической чаши. Определить период колебаний кубика, если внутренний радиус чаши R, а ребро кубика много меньше R. 655. Как изменится период колебаний кубика в чаше (см. условие задачи 654), если на чашу кроме силы тяжести будет действовать сила F, направленная вертикально вверх? Масса чаши М много больше массы т кубика. 656. Как изменится период колебаний кубика в чаше (см. задачу 654), если чаша стоит на гладкой горизонталь- ной поверхности, по которой она может перемещаться без трения? 657. Обруч массы т и ра- диуса г может катиться без про- скальзывания по внутренней по- верхности цилиндра радиуса R (рис. 227). Определить период колебаний обруча, считая угол Ф малым. 658. Найти период колебаний маятника, изображенного на рис. 228. Стержень, на котором помещены массы /Hi и тг, считать невесомым. 128
659. Определить период колебаний маятника, состоя- щего из тонкого однородного полукольца радиуса г, под- вешенного на невесомых нитях ОА и ОВ, как показано на рис. 229. 660. На рис. 230 изображена механическая система, состоящая из груза массы т, пружины А с коэффициентом упругости k и блока массы М. Груз посредством нити, пере- кинутой через блок, связан с пружиной. Найти период ко- лебаний груза, если блок представляет собой тонкостенный цилиндр. 661. С какой частотой будет колебаться палка массы /и=2 кг и площади поперечного сечения S=5 см2, пла- вающая на поверхности воды в вертикальном положении? (Принять во внимание, что период колебаний груза на пру- жине дается выражением Т = tn/k, где k — коэффи- циент упругости пружины.) 662. В сообщающиеся сосуды цилиндрической формы налита ртуть. Найти период колебаний ртути, если пло- щадь поперечного сечения каждого сосуда 5=0,3 см2, а масса ртути т=484 г. Плотность ртути р=13,6 г/см3. 663. Представим себе шахту, пронизывающую земной шар по одному из его диаметров. За какое время тело, бро- шенное в эту шахту, достигнет центра Земли? Сопротивле- ние движению отсутствует. 664. Закрепленная на концах струна растянута с силой f. К середине струны прикреплен точечный груз массы т 5 Б. Б. Буховцев и др. (29
(рис. 231). Определить период малых колебаний прикреп- ленного груза. (Массой струны пренебречь. Силу тяжести не учитывать.) Рис. 231. 665. Два одинаковых груза массы т, скрепленные пру- жинами, как показано на рис. 232, лежат на абсолютно гладком горизонтальном столе. Пружины растянуты li Ла l _____________________________I 0 ^7777777777777777777777777777777777777777777/ с силой F. Грузы смеща- ют в направлении, пер- пендикулярном длине пру- жин, на одинаковое ма- рис< 232. лое расстояние х в одну сторону от положения рав- новесия (рис. 233, а). Определить период колебаний грузов. 666. Грузы массы т, скрепленные пружинами, как показано на рис* 232, лежат на абсолютно гладком го- ризонтальном столе. Пружины растянуты с силой F. Грузы смещают на одинаковое малое расстояние х в направлении, 130
перпендикулярном длине пружин, в разные стороны от по- ложения равновесия и отпускают (рис. 233, б). Определить период колебаний грузов. 667. Для того чтобы удержать в равновесии открытую дверь в вестибюле метро (дверь открывается в обе стороны и возвращается в положение равновесия пружинами), нуж- но приложить к ручке двери силу 50 Н. Можно ли открыть дверь силой 1 Н, приложенной к той же ручке? Трением в петлях двери можно пренебречь. 668. С невесомым блоком радиуса г жестко скреплен не- весомый стержень длины I. На конце стержня находится тело массы т (рис. 234). На блок намотана нить, к свобод- ному концу которой прикреплен груз массы М. При каком условии движе- ние системы будет носить колеба- тельный характер, если в начальный момент"* угол а между стержнем и вертикалью равен нулю? 669. Определить отношение частот колебаний для трех молекул: водо- рода, дейтерия и трития. Примечание. Равновесному положению двух протонов в молекуле отвечает определенное расстояние между ними. Если эти два протона несколько сблизить или удалить от равновесного положения, то возникает сила, возвращаю- щая их в это положение. Она пропорциональна величи- не отклонения. 670. Найти частоты продольных колебаний бесконеч- ной линейной цепочки одинаковых атомов. В положении равновесия расстояние между атомами равно а. Масса каж- дого атома равна т. Коэффициент жесткости связи меж- ду атомами k. § 26. Электрические колебания 671. Для чего в телефонной трубке нужен постоянный магнит? Почему напряженность магнитного поля этого маг- нита должна быть больше максимальной напряженности магнитного поля, создаваемого током, проходящим по об- мотке катушки телефона? 672. Найти частоту собственных колебаний в контуре, состоящем из соленоида длины /=15 см, площади попереч- 5* 131
ного сечения Sx= 1 см2 и плоского конденсатора с площадью пластин S2=6 см2 и расстоянием между ними d=0,l см. Число витков соленоида М=1000. 673. Электрический контур состоит из конденсатора пос- тоянной емкости и катушки, в которую может вдвигаться сердечник. Один сердечник спрессован из порошка маг- нитного соединения железа (феррита) и является изоля- тором. Другой сердечник изготовлен из меди. Как изме- нится частота собственных колебаний контура, если в ка- тушку вдвинуть 1) медный сердечник? 2) сердечник из феррита? 674. Что произойдет, если заряженный конденсатор сое- динить сверхпроводником с таким же незаряженным кон- денсатором? 675. На вертикально отклоняющие пластины осцилло- графа подано напряжение Vi=Vio cosco/, а на горизонтально отклоняющие — напряжение V,2=V20cos (<oZ—<р). Найти тра- екторию электронного луча на экране осциллографа при разности фаз между напряжениями на пластинах <р1=л/2 и ф2=Я. 676. На рис. 235 изображена схема, состоящая из бата- реи £, неоновой лампы Af, конденсатора С и сопротивления 7?. Характеристика неоновой лампы (зависимость тока в лампе от напряжения) име- ет вид, изображенный на рис. 236. При малых на- пряжениях ток через лам- пу не идет. Когда потен- циал на лампе достигает величины V3 (потенциал зажига- ния), лампа вспыхивает, ток скачком достигает конечной величины /3 и в дальнейшем растет пропорционально V. При уменьшении напряжения убывание тока происходит медленнее, чем происходило возрастание. Лампа гаснет при потенциале гашения Vr. Начертить примерную зависи- 132
мость изменения напряжения на конденсаторе от времени при замыкании ключа /С 677. Как будет изменяться период релаксационных колебаний в схеме с неоновой лампой (см. задачу 676) при изменении емкости конденсатора С и сопротивле- ния /?? 678. Входящий в колебательный контур плоский кон- денсатор таков, что его пластины могут перемещаться друг относительно друга. Каким образом посредством переме- щения пластин осуществить параметрическую раскачку контура? § 27. Волны 679. Исходя из соображений размерностей, определить скорость распространения волн на поверхности жидкости с учетом только силы тяжести (длинные гравитационные волны). Предполагается, что глубина жидкости в сосуде и амплитуда колебаний частиц в волне (X — длина волны). 680. Исходя из соображений размерностей, определить скорость распространения волн на поверхности жидкости с учетом только сил капиллярности (волны малой длины). Предполагается, что глубина жидкости в сосуде и амплитуда колебаний частиц в волне (X — длина вол- ны). Плотность жидкости р. 681. На рис. 237 изображено поперечное сечение бес- конечно большого сосуда с жидкостью. Слева из среды Рис. 237. с глубиной hi под углом к границе раздела движется плоская волна, длина которой Под каким углом к границе раздела будет распространяться эта волна в среде, где глубина жидкости Л2? Известно, что скорость распрост- ранения длинных гравитационных волн в бесконечно боль- шом сосуде равна c = k\^'ghi где k — постоянный ко- эффициент пропорциональности, а Л — глубина сосуда. 133
682. Исходя из соображений размерностей, определить с точностью до безразмерного коэффициента скорость рас- пространения продольных волн в упругой среде плотности р, модуль Юнга которой равен Е. 683. Тонкую струну заменили струной из того же ма- териала, но имеющей вдвое больший диаметр. Во сколько раз нужно изменить натяжение струны, чтобы частота ко- лебаний струны не изменилась? 684. Найти собственные частоты колебаний стальной струны длины Z=50 см, диаметра d=l мм, если натяжение струны Т=0,1 Н. Плотность стали р=7,8 г/см3. 685. Найти собственные частоты колебаний воздушного столба в закрытой с обоих концов трубе, имеющей длину 7=3,4 м. 686. Над цилиндрическим сосудом высотой 1 м звучит камертон, имеющий собственную частоту колебаний v=340 Гц. В сосуд медленно наливают воду. При каких положениях уровня воды в сосуде звучание камертона значительно усиливается? 687. Какую форму имеет фронт ударной волны, возни- кающей в воздухе при полете пули со скоростью, превы- шающей скорость звука? 688. Реактивный самолет пролетел со скоростью 500 м/с на расстоянии 6 км от человека. На каком расстоянии от человека был самолет, когда человек услышал его звук? 689. Известно, что если источник звука и человек нахо- дятся примерно на одной высоте, то в направлении ветра звук слы- шен лучше, чем в противополож- ном. Как объяснить это явление? 690. Почему устойчивый прием телевизионной передачи возможен только в пределах прямой види- мости? 691. Радиолокатор работает в импульсном режиме. Частота пов- торения импульсов /=1700 Гц, длительность импульса т=0,8 мкс. Найти максимальную и минималь- ную дальность обнаружения цели данным радиолокатором. 692. Антенна телевизора (пункт С на рис. 238) наряду с волной, идущей непосредственно от передающей стан- 134
ции (пункт Л), принимает волну, отраженную от железной крыши здания (пункт В). Вследствие этого изображение двоится. На сколько сантиметров сдвинуты изображения друг относительно друга, если антенна и крыша здания расположены на расстояниях, указанных на рис. 238? Ширина экрана телевизора /=50 см. (Учесть, что изобра- жение в телевизоре разлагается на 625 строк и в секунду передается 25 кадров.) 693.. Вибратор, имеющий длину /=0,5 м, погружен в сосуд с керосином (8=2). Чему равна в пустоте (по выходе из сосуда) длина электромагнитной волны, излученной дан- ным вибратором?
Глава V. ГЕОМЕТРИЧЕСКАЯ ОПТИКА § 28. Фотометрия 694. Круглый зал диаметром D=30 м освещается лам- пой, укрепленной в центре потолка. Найти высоту h зала, если известно, что наименьшая освещенность стены зала в два раза больше наименьшей освещенности пола. 695. На высоте Н=2 м над серединой круглого стола диаметром D=3m висит лампа- в /1=100 кандел. Ее заме- нили лампой в /2=25 кандел, изменив расстояние до стола так, что освещенность середины стола осталась прежней. Как изменится освещенность края стола? 696. В вершинах равнобедренного прямоугольного трет угольника расположены источники света Sx и S2 равной А силы (рис. 239). Как следует расположить маленькую пла- стинку Л, чтобы освещен- / \ ность ее была максималь- / \ на? Стороны треугольника у' Л51=Л5а—а. $2—._________________697. При определении си- 1 лы света некоторого источника Рис. 239. попытка использовать фото- метр не увенчалась успехом, так как сила света была очень велика и уравнять освещен- ность полей фотометра с помощью эталонного источника не удалось даже при положении исследуемого источника на самом краю оптической скамьи. Тогда воспользовались третьим источником, сила света которого была меньше, чем у исследуемого. Эталонный источник давал ту же освещен- ность полей фотометра, что и третий, находясь на расстоя- нии Г1= 10 см от фотометра, в то время как третий находился на расстоянии г2=50см. Затем эталонный источник заменили 136
исследуемым и получили равенство освещенностей при рас- стояниях от фотометра г3=40 см (исследуемый источник) и г4=10 см (вспомогательный источник). Найти, во сколько раз сила света исследуемого источ- ника больше, чем эталонного. 698. Лампа, имеющая силу света / = 100 кандел, зак- реплена на потолке комнаты. Определить суммарный све- товой поток, падающий на все стены и пол комнаты. 699. На оси полого цилиндра радиуса помещена рас- каленная проволочка, длина которой значительно больше высоты цилиндра. Во сколько раз изменится освещенность внутренней поверхности цилиндра, если его радиус станет равным /?2 700. На какой высоте следует поместить лампу над цент- ром круглого стола, чтобы на краях стола получить наи- большую освещенность? 701. Почему сквозь папиросную бумагу можно прочесть текст только в том случае, если бумагу непосредственно на- ложить на страницу книги? § 29. Основные законы оптики 702. Почему тень ног на земле резко очерчена, а тень головы более расплывчата? При каких условиях тень всюду будет одинаково отчетлива? 703. Как нужно держать карандаш над столом, чтобы получить резко очерченную тень, если источником света служит закреп- ленная у потолка лампа дневного света, име- ющая форму длинной трубки? 704. Осенью, когда деревья потеряли всю листву, часто можно видеть тени от двух па- раллельных ветвей. Нижняя ветвь дает рез- кую темную тень, верхняя — более широкую и светлую. Если две такие тени случайно на- лагаются друг на друга, мы видим яркую светлую полосу посредине более темной тени, так что эта тень выглядит как бы двойной (рис. 240). Как объяснить это явление? (Мин- jaapT, «Свет и цвет в природе».) 705. Лучи Солнца, проходя сквозь маленькое отверстие в листве на вершине высокого дерева, дают на земле пятно в форме эллипса. Большая и малая оси эллипса равны соответственно а=12 см и 6 = 10 см. Какова высота л/7|\\ в с л Рис. 240. 137
И дерева? Угловые размеры солнечного диска р= 1/108 рад. . 706. Какой наименьшей высоты должно быть плоское зеркало, укрепленное вертикально на стене, чтобы человек мог видеть свое отражение в нем во весь рост, не изменяя положения головы? На каком расстоянии от пола должен быть нижний край зеркала? 707. Солнечные лучи, отражаясь от горизонтально ле- жащего зеркала, падают на вертикальный экран. На Рис. 241. зеркале стоит продолговатый предмет (рис. 241). Описать характер тени на экране. 708. При каких условиях форма солнечного «зайчика» от небольшого зеркала не будет зависеть от формы зеркала? 709. Как отличить на фотографии реальный пейзаж от его отражения в спокойной воде? 710. Найти графически, при каких положениях глаза \ наблюдатель может видеть в зеркале конечных размеров изображение отрезка прямой, расположенного относительно W/////^//7//a зеркала так, как указано на Рис. 242. РИС712142'п 711. Плоское зеркало рас- положено параллельно стене на расстоянии I от нее. Свет от укрепленного на стене то- чечного источника падает на зеркало и, отражаясь, дает на стене «зайчик». С какой скоростью будет двигаться «зайчик» по стене, если приближать к ней зеркало со ско- ростью у? Как будут меняться размеры «зайчика»? 712. Используя условие задачи 711, определить, будет ли меняться при движении зеркала освещенность стены в том 138
месте, где находится «зайчик». Считать размеры зеркала много меньшими расстояния от зеркала до источника света. 713. Плоское зеркало вращается с постоянной угловой скоростью. Число оборотов в секунду n=0,5. С какой ско- ростью будет перемещаться «зайчик» по сферическому эк- рану радиуса 10 м, если зеркало находится в центре кри- визны экрана? 714. Опыты А. А. Белопольского по эксперименталь- ному исследованию оптического явления Допплера состояли в наблюдении света, многократно щихся зеркал (рис. 243). Зеркала ющихся в разные стороны ди- сках. 1) Зная угловую скорость со вращения дисков, найти угловую скорость Q поворота луча, испытавшего п после- довательных отражений от зеркал. 2) Найти линейную ско- рость п-го изображения в Момент, когда зеркала п'араллельны друг другу и их отражающие участки движутся со скоростью v в разные стороны. 715. “ щаются 716. отраженного от движу- размещались на враща- Рис. 243. М \л/ Рис. 244. Решить задачу 714 при условии, что диски вра- в одну сторону. Узкий пучок света S падает на двугранный угол а=60°, образованный оди- наковыми плоскими зерка- лами ОМ и ON, закреплен- ными на оси О (рис. 244). После отражения от зеркал свет фокусируется линзой Л и попадает в неподвижный приемник П. Зеркала вра- щаются с постоянной угло- вой скоростью. Какая часть световой энергии пучка за время, много большее периода вра- щения, достигнет приемни- ка расстоянии а от оси, равном ка,если пучок проходит половине длины зеркала СМИ? 139
717. Можно ли вместо обычного экрана для показа кино использовать плоское зеркало? 718. Два плоских зеркала АО и ОВ образуют двугран- ный угол (р=2л/п, где п — целое число. Точечный источник света S находится между зеркалами на равном расстоянии от каждого из них. Найти число изображений источника в зеркалах. 719. Два плоских зеркала АО и ОВ образуют произволь- ный двугранный угол у=2л/а, где а — любое число, боль- шее 2. Точечный источник света S находится между зер- калами на равном расстоянии от каждого из них. Найти число изображений источника в зеркалах. 720. В каком направлении нужно пустить луч света из точки А (рис. 245), находящейся внутри зеркального ящика, чтобы он попал в точку В, отразившись по одному разу от всех четырех стенок? Точки А и В находятся в одной плоскости, перпендикулярной стенкам ящика (т. е. в плоскости рисунка). 721. Почему, если с самолета, летящего над морем, смот- реть вниз, кажется, что вода гораздо темнее непосредствен- но внизу, чем на горизонте? 722. На какое расстояние сместится луч, проходящий через плоскопараллельную пластинку, если толщина ее d, показатель преломления п, а угол падения луча /? Может ли смещение луча быть больше толщины пластинки? 723. При каких значениях показателя преломления прямоугольной призмы возможен ход луча, изображенный на рис. 246? Сечение призмы — равнобедренный треуголь- ник; луч падает на грань АВ нормально. 724. В воду опущен прямоугольный стеклянный клин. Показатель преломления стекла п1=1,5. При каких зна- 140
с f чениях угла а (рис. 247) луч света, падающий нормально на грань АВ, целиком достигнет грани АС? Рис. 247. Рис. 248. 725. В ясные солнечные дни на загородных асфальтиро- ванных шоссе водители автомашин часто наблюдают такую картину: некоторые,участки асфальта, находящиеся впереди автомашины на расстоянии около 80—100 м, кажутся по- крытыми лужами. Когда водитель подъезжает ближе к это- му месту, лужи исчезают и снова появляются впереди на других местах, примерно на том же расстоянии. Как объяс- няется это явление? 726. Толстая пластина сделана из прозрачного мате- риала, показатель преломления которого изменяется от значения п± на верхней грани до значения п2 на нижней грани. Луч входит в пластину под углом а. Под каким углом луч выйдет из пластины? 727. Кубический сосуд с непрозрачными стенками рас- положен так, что глаз наблюдателя не видит его дна, но полностью видит стенку CD (рис. 248). Какое количество воды нужно налить в сосуд, чтобы наблюдатель смог уви- деть предмет F, находящийся на расстоянии 6=10 см от угла D? Ребро сосуда а=40 см. 728. Сечение стеклянной призмы имеет форму равно- стороннего треугольника. Луч падает на одну из граней пер- пендикулярно ей. Найти угол <р между лучом падающим и лучом, вышедшим из призмы. Показатель преломления стекла п=1,5. 1 729. Сечение стеклянной призмы имеет форму равнобед- ренного треугольника. Одна из равных граней посереб- рена. Луч падает нормально на другую, не посеребренную 141
грань и после двух отражений выходит через основание приз- мы перпендикулярно ему. Найти углы призмы. 730. Падающий на грань призмы луч выходит после преломления через смежную грань. Каково максимально допустимое значение преломляющего угла призмы а, если она сделана из стекла с показателем преломления п=1,5? 731. Луч света входит в стеклянную призму под углом а и выходит из призмы в воздух под углом р, причем, прой- дя призму, отклоняется от первоначального направления на угол у. Найти преломляющий угол призмы ср и показа- тель преломления материала, из которого она сделана. 732. Стороны призмы ABCD, изготовленной из стекла с показателем преломления и, образуют двугранные углы: /А-90°, /В=75°, = (призма Аббе). Луч света падает на грань АВ и после полного отражения от грани ВС выходит через грань AD. Найти угол па- дения а луча на грань АВ, если известно, что луч, про- шедший через призму, перпендикулярен падающему лучу. 733. Если лист бумаги полить канцелярским клеем или водой, то сквозь бумагу можно будет прочесть текст, напе- чатанный на другой стороне листа. Объясните, почему? 734. Пучок параллельных лучей падает на плоскопа- раллельную бесконечную прозрачную пластину. При каж- дом прохождении границы раздела двух сред доля отражен- ной энергии равна р, т. е. £отраж =рЕпадзЮ111, а поглоще- ние в материале пластины отсутствует. Какой частью пол- ной энергии падающего пучка обладает прошедший свет? § 30. Линзы и сферические зеркала 735. Светящаяся точка движется по оси вогнутого сфе- рического зеркала, приближаясь к нему. При каких рас- стояниях точки от зеркала расстояние между точкой и ее изображением в зеркале будет равно 0,75/?, где /? — радиус кривизны зеркала? * 736. Плоско-выпуклая собирательная линза изготов- лена из стекла с показателем преломления п=1,5. Опре- делить соотношение между фокусным расстоянием этой лин- зы f и радиусом кривизны ее выпуклой поверхности /?. 737. Преломляющие поверхности линзы являются кон- центрическими сферическими поверхностями. Большой ра* диус кривизны /?=20 см, толщина линзы /=2 см, показа- тель преломления стекла п=1,6. Собирающей или рассеи- вающей будет линза? Найти ее фокусное расстояние. 142
738. Двояковыпуклая линза, сделанная из стекла с по- казателем преломления п=1,6, имеет фокусное расстояние /=10 см. Чему будет равно фокусное расстояние этой лин- зы, если ее поместить в прозрачную среду, имеющую пока- затель преломления п1=1,5? Найти фокусное расстояние этой линзы в среде с показателем преломления п2=1,7. 739. Короткая металлическая трубка закрыта с одного конца плоско-выпуклой линзой, а с другого—тонкой плос- копараллельной пластинкой. Система погружена в жид- кость с показателем преломления пх. Найти фокусное рас- стояние системы, если радиус кривизны поверхности. лин- зы равен и она изготовлена из вещества с показателем преломления и2. 740. Тонкая стеклянная линза имеет оптическую силу D=5 диоптрий. Когда эту линзу погружают в жидкость с показателем преломления п2, она действует как рассеи- вающая с фокусным расстоянием /=100 см. Определить показатель преломления п2 жидкости, если показатель пре- ломления стекла линзы 1,5. 741. Двояковыпуклая стеклянная линза с одинаковыми радиусами кривизны поверхностей имеет в воздухе фокус- ное расстояние F19 а в воде F2. На каких расстояниях F' и F" от линзы находятся ее фокусы, когда линза располо- жена на границе воздуха и воды? Показатель преломления воздуха равен единице, воды 742. Расстояние между двумя точечными источниками света /=24 см. Где между ними надо поместить собирающую линзу с фокусным расстоянием f=9 см, чтобы изображения обоих источников получились в одной и той же точке? 743. Высота пламени свечи 5 см. Линза дает на экране изображение этого пламени высотой 15 см. Не трогая линзы, свечу отодвинули на Z=l,5 см дальше от линзы и,передви- нув экран, вновь получили резкое изображение пламени вы- сотой 10 см. Определить главное фокусное расстояние линзы, j 744. Сходящийся пучок лучей падает на рассеивающую линзу таким образом, что продолжения всех лучей пересе- каются в точке, лежащей на оптической оси линзы на рас- стоянии 6=15 см от нее. Найти фокусное расстояние линзы в двух случаях: 1) после преломления в линзе лучи собираются в точке, находящейся на расстоянии ах=60 см от линзы; 2) продолжения преломленных лучей пересекаются в точке, находящейся перед линзой на расстоянии а2=60 см от нее. 143
745. Расстояние между электрической лампочкой и эк- раном d=l м. При каких положениях собирающей линзы с фокусным расстоянием f=21 см изображение нити лам- почки будет отчетливым? Можно ли получить изображение, если фокусное расстояние f'=26 см? 746. Тонкая собирающая линза дает изображение неко- торого предмета на экране. Высота изображения равна Ль Не меняя расстояния между предметом и экраном, перемещают линзу и находят, что высота второго четкого изображения равна h2. Определить высоту Н самого предмета. 747. Источник света расположен на некоторой высоте над водоемом. Линза, изготовленная из стекла с показате- лем преломления помещена в воду (пх=4/3) и Дает на дне резкие изображения источника, когда расположе- на на расстоянии fх=20 см или /2=80 см от дна. Найти фокусное расстояние такой V 1---J---- I-------- Л линзы в воздухе. 748. Каков радиус R вог- нутого зеркала, находящего- ся на расстоянии а=2 м от лица, если человек видит в нем свое изображение в пол- А Рис. 249. тора раза большим, чем в плоском зеркале, находящемся на том же расстоянии от лица? 749. На рис. 249 изображен луч АВ, прошедший сквозь рассеивающую линзу. Построить ход луча до линзы, если положение ее фокусов F известно. Рис. 250. ' 750. На рис. 250 изображена светящаяся точка и ее изображение, даваемое линзой, оптическая ось которой Afi#a. Найти положение линзы и ее фокусов. 144
/- 751. На заданной оптической оси NJ^ линзы найти по- строением оптически^ центр линзы и ее главные фокусы, Рис. 251. если известно положение источника S и положение изоб- ражения S' (рис. 251). 752. Дано положение оптической оси Л\М2, ход луча АВ, падающего на линзу, и преломленный луч ВС (рис. 252). Найти построением положение главных фокусов линзы. 753. Собирательная линза дает изображение источника в точке S' на главной оптической оси. Положение центра линзы О и ее фокусов F известно, причем OF<OS'. Найти построением положение источника S. 754. Точка S' есть изображение точечного источника света в сферическом зеркале, оптическая ось которого NiN2 (рис. 253). Найти построением положение центра зеркала и его фокуса. -У Щ Рис. 253. 755. Дано положение оптической оси сфери- ческого зеркала, расположение источника и изображения 145
(рис. 254). Найти построением положения центра зеркала, его фокуса и полюса для случаев: 1) Л — источник, В — изображение; 2) В — источник, А — изображение. • Л &_____________________ Рис. 254. 756. Точечный источник света, помещенный на некото- ром расстоянии от экрана, создает в центре экрана осве- щенность 2,25 лк. Как изменится эта освещенность, если по другую сторону источника на таком же расстоянии по- местить 1) бесконечное плоское зеркало, параллельное экрану? 2) вогнутое зеркало, центр которого совпадает с центром экрана? 3) выпуклое зеркало такого же радиуса кривизны, как и вогнутое? 757. Желая получить снимок зебры,- фотог- Д раф снял белого осла, надев на объектив фото- аппарата стекло с Черными полосками. Что по- I лучилось на снимке? 758. Из стекла двух сортов выполнена слои- v~ стая линза, изображенная на рис. 255. Какое изображение даст эта линза в случае точечного \\ источника, расположенного на оптической оси? Отражение света на границе слоев не учитывать. W 759. Видимые размеры дисков Солнца и Лу- V ны у горизонта кажутся увеличенными по срав- Рис. 255. нению с их видимыми размерами в зените. Как можно с помощью линзы экспериментально до- казать, что это увеличение является кажущимся? §31 . Оптические системы и оптические приборы 760. На собирающую линзу с фокусным расстоянием 40 см падает параллельный пучок лучей. Где следует по- местить рассеивающую линзу с фокусным расстоянием 15 см, чтобы пучок лучей после прохождения двух линз остался параллельным? 146
761. На каком расстоянии от двояковыпуклой линзы с фокусным расстоянием м нужно поместить вогнутое сферическое зеркало, имеющее радиус кривизным, чтобы луч, падающий на линзу параллельно главной опти- ческой оси системы, после отражения от зеркала вышел из линзы, оставаясь параллельным оптической оси? Найти изображения предмета, даваемые данной оптической сис- темой. 762. Оптическая система состоит из двух собирающих линз с фокусными расстояниями /1=20 см и /2=10 см. Расстояние между линзами d=30 см. Предмет находится на расстоянии Лх=30 см от первой линзы. На каком рас- стоянии от второй линзы получится изображение? 763. Определить фокусное расстояние оптической сис- темы, состоящей из двух тонких линз: рассеивающей с фо- кусным расстоянием ft и собирающей с фокусным расстоя- нием Д. Линзы расположены вплотную друг к другу. Опти- ческие оси линз совпадают. 764. Две одинаковые рассеивающие линзы помещены на одной оси так, что передний фокус одной и задний фокус другой располагаются в одной и той же точке А оси. Тре- тью линзу располагают на той же оси так, что в точке А находится ее оптический центр. Фокусное расстояние тре- тьей линзы таково, что система дает действительное изоб- ражение любого предмета, расположенного вне системы на ее оси. Какое увеличение дает система, когда расстояние между предметом и его изображением наименьшее? 765. Параллельный пучок света падает на систему из трех тонких линз с общей оптической осью. Фокусные рас- стояния линз соответственно равны Л= + 10 см, /2=—20 см и /3=+9 см. Расстояние между первой и второй линзами 15 см, между второй и третьей 5 см. Определить положе- ние точки схождения пучка по выходе из системы линз. 766. Некоторая оптическая система создает действитель- ное изображение источника в точке Л. Размер изображения 1 мм. Другая оптическая система состоит из двух собира- ющих линз, расположенных так, что центр одной совпа- дает с фокусом другой и наоборот. Вторую систему распо- лагают на одной оси с первой так, что точка А оказывается посредине между линзами. Найти размер изображения, об- разованного второй системой. 767. Система состоит из двух линз с одинаковыми по абсолютной величине фокусными расстояниями. Одна из линз собирающая, другая рассеивающая. Линзы располо- 147
жены на одной оси на некотором расстоянии друг от друга. Известно, что если поменять линзы местами, то действитель- ное изображение Луны, даваемое этой системой, сместится на Z=20 см. Найти фокусное расстояние каждой из линз. 768. Каким должно быть фокусное расстояние Fx соби- рающей линзы, которая давала бы точно такое же по вели- чине изображение Луны, как и система из двух линз', упо- мянутая в задаче 767? Расстояние между линзами а= 4 см. 769. При каком из двух расположений линз5 указанных в задаче 767, размеры действительного изображения Луны будут больше? 770. Линза с фокусным расстоянием f=30 см дает на экране четкое изображение предмета, расположенного на расстоянии а=40 см от линзы. Между линзой и предметом перпендикулярно оптической оси линзы поместили плоско- параллельную пластинку толщиной d=9 см. На какое рас- стояние нужно сместить экран, чтобы изображение пред- мета на нем осталось четким? Показатель преломления стекла пластинки /2 = 1,8. 771. Предмет АВ находится на расстоянии а=36 см от линзы с фокусным расстоянием /=30 см. На расстоянии /=1 м за линзой расположено плоское зеркало, повернутое по отношению к оптической оси линзы на 45° (рис. 256). На каком расстоянии Н от оптической оси нужно поместить дно кюветы с водой, чтобы получить на нем отчетливое изоб- ражение предмета? Толщина слоя воды в кювете d=20 см. 772. Стеклянный клин с малым преломляющим углом а расположен на некотором расстоянии от собирающей лин- зы с фокусным расстоянием f, причем одна из поверхностей клина перпендикулярна оптической оси линзы. По другую сторону линзы в ее фокусе находится точечный источник 148
света. Отраженные от клина лучи дают после преломления в линзе два изображения источника, смещенные друг от- носительно друга на d. Найти показатель преломления стекла клина. 773. Вогнутое зеркало имеет форму полусферы радиу- сом R=55 см. В это зеркало налит тонкий слой неизвестной прозрачной жидкости. При этом оказалось, что данная оп- тическая система при некотором положении источника дает два действительных изображения, одно из которых совпа- дает с самим источником, а другое отстоит от него на рас- стояние /=30см. Найти показатель преломления п жидкости. 774. Двояковыпуклая линза имеет фокусное расстояние /\ = 10 см. Одна из поверхностей линзы, имеющая радиус кривизны 10 см, посеребрена. Построить изображение предмета, даваемое данной оптической системой, и найти положение изображения, если предмет находится на рас- стоянии а=15 см от линзы. 775. Плоско-выпуклая линза из стекла (показатель пре- ломления п) с посеребренной плоской стороной имеет фо- кусное расстояние F±. Какое фокусное расстояние будет иметь та же линза, если посеребрить не плоскую, а выпук- лую сторону? 776. На плоской поверхности массивного куска стекла (показатель преломления, п) вырезано углубление в виде шарового сегмента. Вынутый из углубления кусок стекла представляет собой тонкую собирающую линзу с фокус- ным расстоянием f. Найти фокусные расстояния и /2 по- лучившейся сферической поверхности. 777. На прозрачный шар, имеющий радиус R и показа- тель преломления п, падает в направлении одного из диа- метров узкий параллельный пучоксветовых лучей. На каком расстоянии f от центра шара лучи будут сфокусированы? 778. Алмазный шарик (п=2,4) радиуса R посеребрен с задней стороны. На каком расстоянии вперед шариком дол- жен быть расположен точечный источник света, чтобы лу- чи, преломившись на передней поверхности, отразившись от задней и вновь преломившись на передней, образовали изображение, совпадающее с источником? 779. Найти положение главных плоскостей прозрачного шара, используемого в качестве линзы. 780. Предмет находится на расстоянии d=2,5 см от поверхности стеклянного шара, имеющего радиус R= 10 см. Найти положение изображения, даваемого шаром. Показа- тель преломления стекла п=1,5. v 149
781. Сферическая колба, толщина стенок которой Л/? значительно меньше ее радиуса 7?, изготовлена из стекла с показателем преломления п. Считая эту колбу оптической системой и рассматривая лишь лучи, близкие к прямой,’ проходящей через центр сферы, определить положение фо- кусов и главных плоскостей системы. , 782. На капельку воды сферической формы под углом i падает луч света. Найти угол 0 отклонения луча от перво- начального направления в случае однократного отражения от внутренней поверхности капли. 783. На капельку воды сферической формы падает па- раллельный пучок лучей. 1) Вычислить значения углов 0 отклонения лучей от первоначального направления для различных углов паде- ния: 0; 20; 40; 50; 55; 60; 65; 70°. 2) Построить график зависимости 0 от i и по графику найти приближенное значение угла наименьшего отклоне- ния 0min. 3) Определить, вблизи каких значений угла 0 лучи, вы- шедшие из капли, идут приблизительно параллельно. Показатель преломления воды считать равным п—1,333. (Это значение поимеет место для красных лучей.) 784. Почему в тех фотоаппаратах, в которых при навод- ке на резкость употребляется матовое стекло, не пользу- ются прозрачным стеклом? 785. Два фонаря одинаковой яркости находятся на разных расстояниях от наблюдателя. 1) Будут ли они казаться наблюдателю одинаково яр- кими? 2) Будут ли их изображения на фотографиях одинаково яркими, если фонари сфотографировать на разных кадрах, чтобы изображения были в фокусе? 786. Один и тот же объект фотографируют с небольшого расстояния двумя фотоаппаратами, имеющими одинаковую светосилу, но различные фокусные расстояния. Одинаковы ли должны быть выдержки? 787. При помощи линзы последовательно получают два изображения одного и того же предмета с увеличениями &i=5 и £2=2. Во сколько раз изменилась освещенность эк- рана в месте получения изображения с переходом от одного увеличения к другому? 788. Точечный источник света расположен на оси рассеи- вающей линзы на расстоянии а=30 см от нее. На экране, расположенном по другую сторону линзы на расстоянии 150
/==10 см, получается световое пятно. Если увеличить рас- стояние между линзой и экраном в 4 раза, то освещенность центра пятна уменьшится во столько же раз. Найти фокус- ное расстояние линзы. 789. Расстояние от точечного источника до собирающей линзы d=30 см, от линзы до экрана 1=60 см. Известно, что освещенность центра светового пятна на экране увеличится в п=4 раза, если экран придвинуть вплотную к линзе. Определить фокусное расстояние линзы. 790. Небольшое количество непрозрачной жидкости по- крывает посеребренный участок на дне черного сферичес- кого сосуда, в центре которого расположен точечный ис- точник света. Когда жидкость заменили на прозрачную, освещенность верхней точки сосуда увеличилась на 25%. Определить показатель преломления прозрачной жидкости. 791. Линзы, о которых говорится в задаче 767, имеют одинаковые диаметры. Сравнить освещенности изображе- ний Луны в случаях первого и второго расположений и при применении эквивалентной линзы. 792. Можно заметить, что освещенная заходящим Солн- цем белая стена кажется ярче поверхности Луны, находя- щейся на той же высоте над горизонтом, что и Солнце. Означает ли это, что поверхность Луны состоит из темных пород? (Миннарт, «Свет и цвет в природе».) 793. Почему, открыв глаза под водой, мы видим только туманные очертания предметов, а в маске для ныряния пред- меты видны совершенно отчетливо? 794. Близорукий человек, пределы аккомодации глаза которого лежат между аг= 12 см и а2=60 см, носит очки, с помощью которых может хорошо видеть удаленные пред- меты. Определить, на каком наименьшем расстоянии а3‘ мо- жет этот человек читать книгу в очках. 795. Два человека, дальнозоркий и близорукий, надев свои очки, видят так же, как человек с нормальным зре- нием. Однажды они случайно поменялись очками. Надев очки близорукого, дальнозоркий обнаружил, что он может отчетливо видеть только бесконечно удаленные предметы. На каком наименьшем расстоянии а сможет читать мелкий шрифт близорукий в очках дальнозоркого? 796. Предмет рассматривают невооруженным глазом с расстояния D. Каково будет угловое увеличение, если тот же предмет рассматривать в лупу, расположенную на рас- стоянии г от глаза и помещенную таким образом, что изоб- ражение находится на расстоянии L от глаза? Фокусное 151
расстояние линзы равно f. Рассмотреть случаи: 1) L=oo; 2) L=D. 797. У оптической трубы, установленной на бесконеч- ность, вынули объектив и заменили его диафрагмой диа- метра D. При этом на некотором расстоянии от окуляра на экране получилось действительное изображение диафрагмы, имеющее диаметр d. Чему было равно увеличение трубы? 798. При изготовлении двухлинзового объектива фото- камеры конструктор использовал рассеивающую линзу с фокусным расстоянием Л=5 см, поместив ее на расстоянии /=45 см от пленки. Где необходимо поместить собираю- щую линзу с фокусным расстоянием /2=8 см, чтобы на плен- ке получалось резкое изображение удаленных предметов? 799. Для трех различных расположений линз, найден- ных в задаче 798, рассчитать диаметр D изображения Луны на негативе. Поперечник Луны виден с Земли в среднем под углом <р=ЗГ5"»0,9-10“2 рад. 800. Главное фокусное расстояние объектива микро- скопа /об=3 мм, окуляра /ок=5 см. Предмет находится от объектива на расстоянии а=3,1 мм. Найти увеличение микроскопа для нормального глаза. Рассмотреть случаи: 1) изображение располагается на расстоянии D=25 см; 2) в глаз из окуляра идут параллельные пучки лучей. 4
Глава VI. ФИЗИЧЕСКАЯ ОПТИКА § 32. Интерференция света 801. Две световые волны, налагаясь друг на друга в оп- ределенном участке пространства, взаимно погашаются. Означает ли это, что световая энергия превращается в дру- гие формы? 802. Два когерентных источника света Si и S2, располо- жены на расстоянии I друг от друга. На расстоянии D^>1 от источников помещаете^ экран (рис. 257). Найти расстояние Рис. 257. между соседними интерференционными полосами вбли- зи середины экрана (точка А), если источники посылают свет длины волны X. 803. Два плоских зеркала образуют между собой угол, близкий к 180° (рис. 258). На равных расстояниях b от зеркал расположен источник света S. Определить интервал между соседними интерференционными полосами на экране MN, расположенном на расстоянии ОА=а от точки пере- 153
сечения зеркал. Длина световой волны известна и равна X. (Ширма С препятствует непосредственному попаданию света источника на экран.) 804. Интерференционный опыт Ллойда состоял в полу- чении на экране картины от источника S и его мнимого изоб- ражения S' в зеркале АО (рис. 259). Чем будет отличаться Рис. 259. интерференционная картина от источников S и S' по срав- нению с картиной, рассмотренной в задаче 802? 805. Два точечных когерентных источника, расстояние между которыми расположены на прямой, перпенди- кулярной экрану. Ближайший источник находится от экрана на расстоянии Ь^>Х. Какой вид будут иметь интерферен- ционные полосы на экране? Каково расстояние на экране от перпендикуляра до ближайшей светлой полосы (при условии /=пХ, п — целое число)? 806. Найти радиус гк для &-го светлого кольца (см. за- дачу 805) при условии, что О=/=пХ, /ф>1, k=n, п—I, п —2, ... 154
807. Как практически можно осуществить опыт, опи- санный в задаче 805? 808. На бипризму Френеля, изображенную на рис. 260, падает свет от источника S. Световые пучки, преломленные различными гранями призмы, частично перекрываются и дают на экране на участке АВ интерференционную кар- тину. Найти расстояние между соседними интерференцион- ными полосами, если расстояние от источника до призмы а=1 м, а от призмы до экрана Ь=4 м; преломляющий угол призмы а=2-10"3 рад. Стекло, из которого изготовлена призма, имеет показатель преломления п= 1,5. Длина све- товой волны 1=6000 А. 809. Сколько интерференционных полос наблюдается на экране в установке с бипризмой, описанной в предыду- щей задаче? 810. Трудность изготовления бипризмы с углом, близ- ким к 180° (см. задачу 808), заставляет прибегнуть к сле- дующему приему. Бипризма с углом 0, сильно отличающим- ся от 180°, помещается в сосуд, заполненный жидкостью с показателем преломления nit или является одной из стенок этого сосуда (рис. 261). Рассчитать угол 6 эквивалентной бипризмы, находящейся в воздухе. Показатель преломле- ния вещества призмы п2. Произвести вычисления для Пг= = 1,5 (бензол), л2=1,52 (стекло), 0=170°. 811. Собирающая линза, имеющая фокусное расстояние /=10 см, разрезана пополам, й половинки раздвинуты на расстояние d=0,5 мм (билинза). Оценить число интерферен- 155
ционных полос на экране, расположенном за линзой на расстоянии D=60 cm, если перед линзой имеется точечный источник монохроматического света (Х=5000 А), удален- ный от нее на а=15 см. 812. Из собирающей линзы с фокусным расстоянием f=10 см вырезана центральная часть ширины d=0,5 мм, как показано на рис. 262. Обе половины сдвинуты вплот- ную. На линзу падает монохроматический свет (Х=5000 А) от точечного источника, расположенного на расстоянии а=5 см от линзы. На каком расстоянии с противоположной стороны линзы нужно поместить экран, чтобы на нем можно было наблюдать три интерференционные полосы? Чему рав- но максимально возможное число интерференционных по- лос, которое можно наблюдать в данной установке? 813. Найти расстояние между соседними полосами ин- терференционной картины, даваемой линзой радиуса /?= = 1 см, которая описана в задаче 812, при условии, что это расстояние не зависит от положения экрана. При каком по- ложении экрана число интерференционных полос будет мак- симальным? Источник света дает монохроматический свет длины волны %=5000 А. 814. Что произойдет с интерференционной картиной в установке, описанной в задаче 812, если ввести в световой пучок, прошедший верхнюю половину линзы, плоскопарал- лельную стеклянную пластинку толщины di=0,l 1 см, а в световой пучок, прошедший нижнюю половину линзы, 156
пластинку толщины d2=0,l см? Показатель преломления стекла п=1,5. Пластинки располагаются нормально к про- ходящим сквозь них световым пучкам. 815. Почему кольца Ньютона образуются только вслед- ствие интерференции лучей 2 и 3, отраженных от границ воздушной прослойки между линзой и стеклом (рис. 263), а луч 4, отраженный от плоской грани линзы, не влияет на характер интерференционной картины? 816. Изменится ли характер интерференционной карти- ны в установке, которая описана в задаче 803, если ширму С убрать? Расстояние а считать большим (равным 1 м). Излучаемые источником волны не являются монохромати- ческими. 817. В каком случае кольца Ньютона видны более от- четливо: в отраженном свете или же в проходящем? 818. Контакт между плоско-выпуклой линзой и стек- лянной пластинкой, на которую она положена, отсутствует вследствие попадания пыли. Радиус пятого темного кольца Ньютона равен при этом гг—0,08 см. Если пыль удалить, то радиус этого кольца увеличится до r2=0,1 см. Найти толщину слоя пыли d, если радиус кривизны выпуклой поверхности линзы /? = 10 см. 819. На поверхность двояковогнутой линзы, имеющую радиус кривизны /?ъ положена своей выпуклой стороной плоско-выпуклая линза с радиусом кривизны поверхности Найти радиусы колец Ньютона, возникающих вокруг точки соприкосновения линз, если на систему па- дает нормально монохроматический свет длины волны X. 820. Чтобы уменьшить коэффициент отражения света от оптических стекол, на их поверхность наносят тонкий слой прозрачного вещества, у которого показатель преломле- 157
ния п меньше, чем у стекла. (Так называемый «метод про-: светления оптики».) Оцените толщину наносимого слоя, считая, что световые лучи падают на оптическое стекло приблизительно нормально. 821. Нормальный глаз способен различать оттенки в цвете при разности длин волн в 100 А. Учитывая это, оце- нить максимальную толщину тонкого воздушного слоя, при которой можно наблюдать в белом свете интерферен- ционную картину, вызванную наложением лучей, отражен- ных от границ этого слоя. 822. На тонкий стеклянный клин от удаленного источ- ника почти нормально падает поток монохроматических волн длины волны X. На расстоянии d от клина расположен экран, на который линза с фокусным расстоянием f проеци- рует возникающую в клине интерференционную картину. Расстояние между интерференционными полосами на экра- не AZ известно. Найти угол а клина, если показатель пре- ломления стекла равен п. § 33. Дифракция света 823. Вычислить радиусы зон Френеля сферической волны радиуса а для точки В, отстоящей от источника монохро- матических волн длины волны % на расстояние а+Ь, учи- тывая, что бф>Х и 824. Вычислить радиусы зон Френеля плоской волны для точки В,.отстоящей от фронта волны на расстояние где X —длина волны источника. Рис. 264. 825. Точечный источник монохроматического света дли- ны волны Х=5000 А находится на расстоянии а=6,75 м от ширмы с отверстием диаметра D=4,5 мм. На расстоянии 158
b=a от ширмы расположен экран (рис. 264). Как изменится освещенность в точке В экрана, лежащей на оси пучка, если диаметр отверстия увеличить до Di=5,2 мм? 826. Как согласовать с законом сохранения энергии тот факт, что увеличение отверстия (см. условие задачи 825) может привести к уменьшению освещенности на оси пучка? Ведь при увеличении отверстия полный световой поток, проникающий за ширму, возрастает. 827. Плоская световая волна (Х=6000 А) падает на ширму с круглой диафрагмой. На расстоянии Ь=2 м за диафрагмой расположен экран. При каком диаметре D диафрагмы освещенность экрана в точке В, лежащей на оси светового пучка, будет максимальна? 828. Считая расстояния от источника до ширмы и от ширмы до экрана примерно одинаковыми и равными а, оценить, при каких условиях дифракция световых волн длины X на отверстии в ширме будет выражена достаточно отчетливо (интенсивность на оси пучка будет зависеть от диаметра отверстия). 829. Показать, что за круглым экраном С в точке В (рис. 265) будет наблюдаться светлое пятно, если размеры экрана достаточно малы. Рис. 265. 830. На каком расстоянии друг от друга должны нахо- диться два человека для того, чтобы глаз смог различить их с расстояния около 11 км? Разрешающая способность нормального глаза составляет примерно Г. 831. Плоская световая волна (длина волны X) падает нормально на узкую щель ширины Ь. Определить направле- ния на минимумы освещенности. 832. Определить оптимальные размеры отверстия «ды- рочной камеры» в зависимости от длины волны, т. е. радиус отверстия г, при котором точечный источник изобразится 159
на стенке камеры кружком минимального диаметра, если рас- стояние от источника света до камеры велико по сравне- нию с ее глубиной d. Направления на минимумы освещен- ности по порядку величины определяются той же формулой, что и в случае щели (см. задачу 831), только вместо ширины щели b нужно взять диаметр отверстия 2г. 833. На дифракционную решетку, имеющую период d=4-10~4 см, нормально падает монохроматическая волна. Оценить длину волны %, если угол между спектрами второ- го и третьего порядков а=2°30'.Углы отклонения считать малыми. 834. На дифракционную решетку, имеющую 500 штри- хов на миллиметр, падает плоская монохроматическая вол- на (Х=5-10“б см). Определить наибольший порядок спект- ра k, который можно наблюдать при нормальном падении лучей на решетку. 835. Определить постоянную решетки d, способной анализировать инфракрасное излучение с длинами волн до %=2-10“2 см. Излучение падает на решетку нор- мально. 836. На дифракционную решетку, имеющую период d=4-10~4 см, падает нормально монохроматическая волна. За решеткой расположена линза, имеющая фокусное рас- стояние f=40 см, которая дает изображение дифракционной картины на экране. Определить длину волны X, если первый максимум получается на расстоянии 1=5 см от централь- ного. 837. Источник белого света, дифракционная решетка и экран помещены в воду. Какие изменения претерпит при этом дифракционная картина, если углы отклонения свето- вых лучей решеткой малы? 838. На дифракционную решетку, имеющую период d=2-10~4 см, падает нормально свет, пропущенный сквозь светофильтр. .Фильтр пропускает длины волн от Х^бООО А до Х2=6000 А. Будут ли спектры различных порядков на- лагаться друг на друга? z 839. Решить задачу 834 в предположении, что плоская волна (Х=5- 10“б см) падает на решетку под углом 30°. 840. Решить задачу 835 в предположении, что падение лучей на решетку может быть наклонным. 841. Найти условие, определяющее направление на главные максимумы при наклонном падении световых волн на решетку, если период решетки d^>H (k — порядок спектра). 160
§ 34. Дисперсия света и цвета тел 842. Луч белого света падает под углом а=30° на приз- му, преломляющий угол которой равен <р=45°. Определить угол 0 между крайними лучами спектра по выходе из призмы, если показатели преломления стекла призмы для крайних лучей видимого спектра равны пк= 1,62, пф=1,67. 843. На двояковыпуклую линзу, радиусы кривизны по- верхностей которой равны /?!=/?2=40 см, падает белый свет от точечного источника, расположенного на оптической оси линзы на расстоянии а=50см от нее. Вплотную перед линзой расположена диафрагма диаметра D = 1 см, ограничивающая поперечное сечение светового пучка. Показатели преломле- ния для крайних лучей видимого спектра равны пк= 1,74 и пф=1,8. Какую картину можно будет наблюдать на эк- ране, расположенном на расстоянии 6=50 см от линзы пер- пендикулярно ее оптической оси? 844. Используя результаты задачи 783, построить эле- ментарную теорию радуги, т. е. показать, что центр радуги находится на прямой, проведенной от Солнца через глаз наблюдателя, и что дуга радуги представляет собой часть окружности, все точки которой видны под углом 42° (для красного света) по отношению к прямой, соединяющей глаз наблюдателя и центр радуги. 845. Объяснить качественно причины появления двой- ной радуги. Каково чередование цветов в первой (основной) и второй радуге? 846. Можно ли в Москве во время летнего солнцестоя- ния (22 июня) наблюдать радугу в полдень? (В это время Солнце в северном полушарии стоит наиболее высоко над горизонтом.) 847. Длина волны в воде уменьшается в п раз, где п w— показатель преломления. Означает ли это, что ныряльщик не может видеть окружающие тела в естественном цвете? 848. На тетради написаны красным карандашом «от- лично» и зеленым «хорошо». Имеются два стекла — зеле- ное и красное. Через какое стекло надо смотреть, чтобы уви- деть оценку «отлично»? 849. Почему объективы с «просветленной оптикой» (см^ задачу 820) имеют пурпурно-фиолетовый (сиреневый) от- тенок? 850. Цвета тонких пленок (например, пленки нефти на воде) и цвета радуги имеют совёршенно различные оттен ки< Почему? 6 Б. Б. Буховцев и др. 161
851. Тонкая мыльная пленка натянута на вертикальную рамку. При освещении белым светом на пленке наблюдаются три цветные полосы: пурпурного (малинового), желтого и голубого (сине-зеленого) цветов. Найти расположение и порядок полос. 852. Почему днем Луна имеет чистый белый цвет, а после захода Солнца принимает желтоватый оттенок? 853. Почему столб дыма, поднимающегося над крышами домов, на темном фоне окружающих предметов кажется синим, а на фоне светлого неба — желтым или даже крас- новатым? 854. Почему цвета влажных предметов кажутся более глубокими, более насыщенными, чем сухих?
ОТВЕТЫ И РЕШЕНИЯ Глава I. МЕХАНИКА § 1. Кинематика равномерного прямолинейного движения 1. В течение 1 часа после встречи лодки с плотами лодка уда- лялась от плотов. В течение 30 мин, когда мотор ремонтировался, расстояние между лодкой и плотами не увеличивалось. Лодка дого- нит плоты через 1 час, так как скорость ее относительно воды, а следовательно и относительно плотов, постоянна: y = S// = 7,5/(1+0,5+1) = 3 км/ч. 2. Расстояние между электропоездами S—vt, с другой стороны, S = trr+«T. Отсюда u~v(t — т)/т = 45 км/ч. 3. На рис. 266 ЛМ/V — график обычного движения машины. CD — график пути инженера до встречи с машиной в точке D. DB —график движения машины после встречи с инженером. Согласно условиям задачи BN — KM = 10 мин. Время движения инженера до встречи с машиной С£==СМ-ЕМ==СМ-КМ/2==55 мин. 4. Так как время оценивается по последнему прибывшему, крат- чайшим оно будет тогда, когда все три туриста прибудут одновременно. 6* 163
График движения туристов изображен на рис. 267. Из него следует, что движение пешком занимало у второго и третьего туристов оди- наковое время (Д^+Д/Д гДе — время обратного движения велосипе- диста. Поэтому (AG~f“ Д^2)4“ 1 ~ — уср(Д^1 + AG + V2^t± — v2M2 = (Д/1 + Д/г)* Отсюда средняя скорость туристов °с₽=ет?У2=10 км/ч- 5. Графики пути катеров, отплы- вающих одновременно, изображены ломаными МЕВ и КЕА, где Е— точ- ка их встречи (рис. 268). Так как скорость катеров относительно воды одинакова, то МА 'и КВ —прямые линии. Оба катера будут находиться в пути одинаковое время, если они встретятся посередине между пристанями. Точка их встречи О лежит на пересечении линии КВ с перпендикуляром, восставленным из середины отрезка КМ. Гра- фики движения катеров изображаются линиями KOD и СОВ. Как видно из рисунка, ДЛ4AF е* &COF, и, следовательно, иско- мое время МС — 45 мин. 6. Скорость катеров относительно воды и скорость реки v2 определяются из уравнений В = /1(у1 + у2) и S — t2 (^i~у2), где tr и t2 — время движения катеров по течению и против течения. Как следует из условия, = 1,5 ч и t2 = 3 ч. Отсюда = км/ч, v, = 5 dry км/4- Место встречи находится на расстоянии 20 км от пристани М. 7. Предположим, что река течет от С к t со скоростью у0. Учи- тывая, что время движения лодки и катера одинаково, можно 164
написать уравнение _s_=2(_£_+^s_\ ^1+^0 Х^г + Ц) ^2—^0 J где S — расстояние между пристанями. Отсюда С’о 4“ 4“ — 1/2 = 0. Следовательно, t/0 = —2о2 ± j/5y| — 4t/1u2 =—20 ± 19,5 км/ч. Решение vb — —39,5 км/ч следует отбросить, так как при этой ско- рости реки ни лодка, ни катер не смогут подняться против течения. Следовательно, vQ = —0,5 км/ч, т. е. река течет от пристани Т к при- стани С. 8. Расстояние R края тени от точки О, находящейся на земле под фонарем, связано с расстоянием г человека от той же точки соотношением R — —-г. Следовательно, траектория тени подобна п —п траектории человека (центр подобия находится в точке О). Поэтому вектор скорости края тени vT направлен так же, как вектор скорости человека v, а по величине превосходит его в —раз (рис. 269). 9. Пусть за время Д/ первая свеча сгорела на величину ДЛ^, а вторая свеча —на ДЛ2 (рис. 270). Тогда тень на левой стене (от первой свечи) опустится на расстояние Дх=Д/ц 4- (Д^! — ДЛ2) = 2ДЛХ — Д/12. Тень на правой стене опустится на расстояние Ду = ДЛ2—(ДЛ1 — ДЛ2) = 2ДЛ2 — ДЛ1> h h Учитывая, что ДЛ1 = — Д/, ДЛ2 = —Д/, получим *1 *2 Дх 2h h — (<мл i \ д/ G ^2 * 1,1 Ду 2h h h = —2. Vt ы h tl h)' 165
t2 > /1, следовательно, > 0, a v2 может быть величиной отрица- тельной, т. е. на правой стене тень может перемещаться вверх. 10. Так как скорость движения в воде меньше скорости движе- ния по берегу, то путь АВ не обязательно займет минимальное время. Предположим, что траектория движения человека—ломаная ли- ния ADB (рис. 271). Необходимо определить, при каком значении х время будет минимальным. Время движения t равно t — । S—x _ ц2 Vrd2A-x2 — v1x-l~v1S “ ~~ VtV2 Это время будет минимальным, если y = v2 }^d2-}~x2—v1x будет иметь наименьшее значение. Очевидно, что х, соответствующее минималь- ному времени /, не зависит от расстояния S. Для нахождения х, соответствующего минимальному значению у, выразим х через у и получим квадратное уравнение х2 , t&^—y8 = 0 vt-v? vl-vj Решение его приводит к следующему выражению: _У1У ± ?2 Кy2 + d2t)8—1>2^8 * 2 2 vl — Vi Так как x не может быть комплексным, то у2 + d2v[ vId2, Мини- мальное значение у равно ymin = d]/ и2. Этому значению у соот- ветствует х = dvj]f v2 — v2. Если S dvJV v% — у2, то следует сразу плыть по прямой АВ к точке В, В противном случае надо пробежать по берегу отрезок, равный AD — S—dvjV v2—Vi, а потом плыть кВ. Отметим, что для пути, соответствующего кратчайшему времени, sin a = vx/v2. 11. Автобус находится в точке А, человек — в точке В (рис. 272). Точка С—место встречи человека с автобусом, а—угол между на- правлением на автобус и направлением, по которому должен бежать человек, AC BC = v2t2t где t± и /2 —время движения автобуса и человека до точки С. 166
Из Д АВС видно, что AC = b sin a/sin р, где sin fi = a/BC. a _ Следовательно, sina=y^-~. По условию задачи /^/2, поэтому sin a^avjbvz = 0,6. Отсюда 36°45'<;a<: 143°15'. Направления, по которым может двигаться человек, заключены в пределах угла DBE. При движении вдоль BD или BE человек Рис. 272. достигнет шоссе одновременно с автобусом. В любую из точек шоссе, находящуюся между точками D и £, человек прибежит раньше автобуса. 12. Наименьшую скорость можно определить из условий = sin a = ay1/&u2= 1* Отсюда t>2= (у ^i==2,4 м/с. При этом a = 90°. Следовательно, направление, по которому должен бежать человек, перпендикулярно направлению на автобус. 13. В произвольную точку D шоссе автобус приходит за время t = l/vv где Z—длина участка AD шоссе. В эту же точку за время, равное или меньшее /, человек может попасть, если он находится в пределах круга радиусом r~v2t с центром в точке D. Начертив подобные круги для других точек, расположенных на шоссе, полу- чаем искомую область. Границами ее являются две общие касатель- ные к окружностям. Угол а, который границы составляют с шоссе, определяется равенством sin a= r{l = y2/yi (рис. 273). 14. Скорость лодки относительно берега v направлена на Л В (рис. 274). Очевидно, v = v0-[-u. Известны: направление вектора v и величина и направление вектора v0. Вектор и будет иметь мини- мальное значение, как видно из чертежа, при uj_v. Следовательно, nmin = v0cosa, cos a==d/V'a2+d2. 15. Пусть скорость и направлена под углом а к берегу (рис. 275). Тогда (и cos а — и) T==BC = at (и sin а)т = АС^Ь^ 167
где т —время движения лодки. Исключив а из этих уравнений, будем иметь (и2—у2) т2 — 2im—(а2 + 6а) = 0. Отсюда т== 15/21 часа. Следовательно, преодолеть расстояние АВ за 30 минут невозможно. 16. Пусть и0—скорость ветра относительно катера. Тогда флаг на катере будет направлен вдоль и0. Если v —скорость катера по отношению к берегу, то u = u0 + v (рис. 276). В £±FCD DCF— =0-f-a—л/2, a £FDC = n — р. По теореме синусов v ________ и sin (а + р —n/2)““sin (л —Р)’ п sin(a+p —л/2) ~ Следовательно, v — u—sjn pj— ‘ Определить скорость течения реки по известной скорости катера относительно берега невозможно, так как неизвестно направление движущегося катера относитель- но воды. 17. Введем обозначения: и 12 — скорость второй автомашины отно- сительно первой, и21—скорость первой автомашины относительно второй. Очевидно, «12 = «21 и Н12 = Di + v2 + cos a (рис. 277). Искомое время 168
18. Примем момент проезда перекрестка первой автомашиной за начало отсчета времени. В системе координат, изображенной на рис. 278, движение автомобилей описы- вается уравнениями X! = — («1 cos a) t, = (г»а sin а) t, xa = —v2x-{-v.2t, уг = 0. Расстояние между автомашинами в лю- бой момент времени Отсюда S2 = -|_ 2V1v2 cos а) /2 — — 2о2 (ох cos а+и2) т/ + о|т2. Найдя минимум квадратного трехчлена, получим наименьшее расстояние: Smin = — т sin а(см. решение задачи 17). &12 19. За время Д/ прямая АВ переместится на расстояние охД/, а прямая CD — на расстояние v2\t. При этом точка пересечения прямых перейдет в положение О' (рис. 279). Перемещение 00' точки Рис. 279. пересечения прямых найдется из треугольника OFO' или ОЕО', где OF = ylA//sin a = EO't ОЕ = v2ht/sin a — FO': 00' = у OF2 + О£г+20 F • ОЕ cos а = v Дt. Отсюда 0 = Vi+fi+SfiOgCosa. 169
заключенной между ломаной § 2. Кинематика неравномерного и равнопеременного прямолинейного движения 20. Путь S, пройденный точкой за 5 с, численно равен площади, Oabcd и осью времени (рис. 280): —10,5 см. Средняя скорость движения точки за 5 с vl = Sl/t1 — =2,1 см/с. Среднее ускорение точ- ки за тот же промежуток времени «! = Ау//1 = 0,8 см/с2. Путь, прой- денный за 10 с, равен S2 = 25 см. Следовательно, средняя скорость и среднее ускорение равны v2 = = S2//2 = 2,5 см/с, аа = 0,2 см/с2. 21. За малый интервал вре- v t =5 t-10 мениА/ нос лодки переместится из 1 точки А в точку В (рис. 281). > AB = v1At, где — скорость лодки. Рис. 280. за это же время будет выбран от- резок веревки О А — О В = С А = v &t. ДЛВС можно считать прямоугольным, так как АС<^ОА. Следова- тельно, v-l — v/cqscl. Рис. 28L 22. Пусть в начальный момент в точке S (рис. 282), а в момент Подобие Л SCD и Л SB А приво- дит к равенству AB—hl/SD— hl/v^. Скорость точки В в данный момент времени у2 = ВВ'/А/ при условии, что время А/, за которое край тени переместится на расстояние ВВ', стремится к нулю. Так как ВВ' = АВ — АВ'= времени / = 0 предмет находился времени t занял положение CD. НО' >7 zz? Рис. 282. что = _hl ( 1 1 \ hl М — V / /4-а/; /(/+а/)г hl то и2 = '—или, учитывая, 170
23. Для равноускоренного движения х = х0 4-у0/4~^2/2. Следо- вательно, у0 = 35 см/с, а — 82 см/с2, х0 = И см —начальная коорди- ната точки. 2 f х3 — xt х2 —xt \ _ м- а’‘Т^77(-1^7-Т^г)=5с“'с - 25. Из графика скорости (см. рис. 8) вытекает, что начальная скорость v0 — 4 см/с (0Л = 4 см/с). Ускорение а — 0А/0В — 1 см/с2. Вначале скорость тела убывает. В момент /х = 4 с скорость тела равна нулю, а затем возрастает по величине. Второй график (см. рис. 9) изображает также равнопеременное движение. До остановки тело проходит путь /i = 10 см. Согласно же первому графику путь до остановки, численно равный площади Л О АВ, составляет 8 см. Сле- довательно, графики изображают разные движения. Второму графику соответствует другая начальная скорость: Vq — 2h/ti — 5 см/с — и дру- гое ускорение: а' = 2h/t% = 1,25 см/с2. 26. График движения второго автомобиля представляет собой параболу, изображенную на рис. 283. Очевидно, что скорость пер- вого автомобиля не может быть слишком большой, иначе обгон совершится всего один раз (точка В на рис. 283, между тем как точка А соответствует встрече машин). Скорость не может быть и слишком малой (прямая ОС на рис. 283), так как в противном слу- чае автомобили вообще не смогут оказаться рядом. Таким образом, уравнение, выражающее равенство координат автомобилей: vrt = l — — vQt + а/2/2, должно иметь два действительных решения, причем оба они соответствуют более поздним моментам времени, чем момент остановки (мгновенной) второго автомобиля, определенный равенством —уо+а/ = О. Оба условия дают i/q-7 al vo V2al — ve < vt < -----s-, или 8 м/с < < 9 м/с. 27. Наибольшая скорость, которую будет иметь шарик при сопри- косновении с подставкой, vmax= ]/ 2gH. При ударе скорость шарика 171
изменяет направление на противоположное, оставаясь неизменной по абсолютной величине. График скорости имеет вид, изображенный на рис. 284,а. На рис. 284, б представлено изменение координаты со временем. Рис. 284. 28. Время падения первого шарика /1== pr2/i1/^ = 0,3 с. Отно- шение максимальных скоростей шариков у2/ух= Vh^i —1 /2. Как следует из графика скоростей (рис. 285), минимальное время т = 0,3 с. Кроме того, второй шарик может начать падение через 0,6; 0,9; 1,2 с и т. д. после начала падения первого шарика. Время в течение которого скорости обоих шариков совпадают, равно 0,3 с. Процесс периодически повторяется через 0,6 с. 29. Исходными являются уравнения gt2/2 — nt g(t — т)2/2 = п — 1, где т — время движения тела на п-м сантиметре пути. Отсюда * = j(/n— Кл-1). 30. Обозначая через xt и координату и скорость первого тела относительно башни, а через х2 и о2 —второго, можно написать сле- дующие уравнения: Х1 = —gt*/2, v^VQ—gt, *)—Я (/ —т)2/2, о2 = —Оо—£(* — *)• (Направление вверх считается здесь положительным.) Скорость пер- вого тела относительно второго равна u = v1—va=2i/o“ gr и не ме- 172
яяется с течением времени. Расстояние между телами равно S = —х2 = (2v0—gr) t—v0T+gr2/2. Друг относительно друга тела движутся равномерно, и, следова- тельно, расстояние между ними изменяется линейно со временем. 31. По условию задачи AA' — vt, СС' = at2/2 (рис. 286). Из подо- бия треугольников ЛД'О, ВВ'О и СС'О имеем А А '/АО = ВВ'/ВО = СС'/СО. Как видно из рис. 286, АО = АВВО, СО—ВС—ВО. Эти соотно- шения позволяют определить (АА'-СС') _ vt at* ВВ =-------2------— Следовательно, точка В движется с начальной скоростью v/2t направ- ленной вверх, и постоянным ускорением а/2, направленным вниз. Достигнув высоты Л = и2/4а, точка будет двигаться вниз. 32. Величина ускорения книги относительно пола лифта зависит не от направления движения лифта (направления его скорости), а от направления ускорения лифта. Если ускорение лифта направлено вверх, то ускорение книги будет равно g+a. Если же его ускорение направлено вниз, то ускорение книги будет g—а. 33. В момент встречи всех автомобилей первые два имеют отно- сительно равномерно движущегося третьего одинаковую скорость v2. Относительно дороги скорости автомобилей (t^ + ^i) и — По- этому остановка (мгновенная) одного из автомобилей произойдет после первой встречи через время (и2 4- vj/a, а другого — через (v2 — v^/a. Остановка одного из автомобилей запаздывает на столько же, на сколько запаздывает его отправление. Следовательно, искомое время запаздывания t = + gl) „ 2^1 . а а а * 34. Если бы скорость лифта не изменялась, то шарик подскочил бы над его полом на высоту Н. В системе отсчета, имеющей посто- янную скорость, равную скорости лифта в момент, когда шарик начал падать, лифт поднимается за время т на высоту Л1 = ат2/2, а за следующий интервал времени т—еще на высоту h2 — ax2—ат2/2. Полная высота подъема Л = Л1-|-/12 = ат2. Искомая высота, на кото- рую подскочит шарик над полом лифта, х = Н — h = H—ат2. 35. При свободном падении за время t тело А пройдет по вер- тикали путь Sx = gt2/2. За это же время клин должен сместиться на 173
расстояние S2 = a/2/2. Если тело все время соприкасается с клином, то, как видно из рис. 287, Sg/S^ctga. Следовательно, искомое' ускорение a=gctga. Если ускорение клина в горизонтальном направ- лении будет больше gctga, то тело оторвется от клина. § 3. Кинематика криволинейного движения 36. Полное ускорение шарика в любой точке траектории равно g (ускорение свободного падения). Нормальное ускорение равно an=gsina, где а—угол, который составляет касательная к траек- тории с вертикалью. Тангенциальное ускорение равно ят —gcosa. Из простых геометрических соображений можно получить sin a = —, cos a = •_ — • V Vo + gb;2 V Vo4-g2T2 Следовательно, flt==^r7===-. V t»o+g2T2 37. Движение тела можно рассматривать как наложение движе- ния по окружности радиуса R в горизонтальной плоскости и паде- ния по вертикали. Соответственно скорость тела v в данный момент можно представить как геометрическую сумму двух составляющих: v± = v cos a — направленной горизонтально, и v2 — = v sin a — направленной вертикально (рис. 288). Здесь а— угол, образованный винтовой линией желоба с горизонтом. Ускорение тела при криволинейном движении равно геометрической сумме тангенциального и нормального ускоре- ний. Нормальное ускорение, соответст- вующее движению по окружности, а1п = = v*/R = v2 cos2 а//?. Движение по вер- тикали прямолинейно, поэтому а2п—в. Искомое ускорение а — —V ^1т+«2т+^1п , где an и а2т—танген- циальные ускорения, соответствующие движению по окружности и вдоль верти- кали. Полное тангенциальное ускорение, очевидно, равно ат=|/ГЯхтН-Дат- Его можно найти, мысленно раз- вернув поверхность цилиндра, на которую навит винтовой желоб, в плоскость. При этом желоб превратится в наклонную плоскость с высотой nh и длиной основания 2л/?п. Очевидно, aT = gsina = = ^/УЛ2+4л2/?2. Для определения а1п найдем v из закона сохранения энергии: mv2[2 = mghn. Следовательно, v2 — 2ghn и aln = 8n2nftg/?/(/i2 + 43T2/?2). Подставив найденные ускорения а% и aJn в выражение для искомого ускорения, найдем _gh VЛ2+ 4л27?2+64я4п27?2 а~ Л*+4л2/?2 174
38. Точку А отправления лодки примем за начало отсчета системы координат. Направление осей указано на рис. 289. Движение лодки в направлении, перпендикулярном течению, происходит с постоянной скоростью и. Поэтому лодка будет находиться на расстоянии у от берега через время t—y/u после отправления. Рассмотрим движение лодки до середины реки (у^с/2). На расстоянии у от берега ско- 2у0 рость течения равна v = —~ У- Подставляя y — ut в выражение для скорости течения, получим v = 2vout/c. Из последнего соотношения следует, что движение лодки в направлении, параллельном берегам, происходит с постоянным ускорением а = 2и0и/с. Лодка достигает середины реки за время Т~с!2и. За это же время она будет снесена вниз по течению на расстояние S = a7'2/2 = t/0c/4rz. При движении от середины реки (точка D) др противоположного берега лодка будет снесена дополни- тельно еще на расстояние S. Таким образом, искомое расстояние равно Vqc!2u. При движении лодки до середины реки a y = ut. Из этих соотношений определяем траекторию лодки от А си до D: у2=—х (парабола). Вторая половина траектории (DB) имеет тот же характер, что и первая. 39. Закон движения тележки A: y — vt. Закон движения тележ- ки В: х— Y l2—v2t2. Движение тележки В вдоль горизонтального рельса можно представить как сумму двух независимых движений: движения вертикально вверх со скоростью v и вращения вокруг точки А с некоторой скоростью w (w J_/). Из простых геометрических соображений следует, что ulvt — v/x. где х—расстояние тележки В от начала координат. Отсюда получим u = — v2t/ У I2—v2t2. 40. Относительно системы отсчета, изображенной на рис. 290, координаты и скорости тела в любой момент времени определяются следующими выражениями: X=voxt, (1) Vx = v9x, (3) y=vOyt — gP/2, (2) vy=vOy— gt. (4) Здесь Vox = t,ocosa и 0oy = yosina—проекции начальной скорости на оси х и у. Уравнения (1)—(4) позволяют ответить на все вопро- сы, поставленные в условии задачи. 175
Время полета Т определяется уравнением (2). При g = 0 (у0 sin а) Т —gT2/2 — 0. Отсюда Т = 2у0 sin a/g. Дальность полета L = (v0 cos a) T = Vq sin 2a/g. Максимальное значение L принимает при a = 45°: lmax = Vo/g. Высота, на которой будет находиться тело спустя время т, равна /i = (yosina)x—gx2/2. Скорость тела в момент времени т равна v = Ух + Уу, где vx = vp cos а, у у = vQ sin a—gr. Отсюда скорость и = J/ Уо+g2x2 — 2u0gT sin а. Она составляет с вертикалью угол 0, определяемый равенством tg 0 = уо cos a/(vo sin ос—gx). 41. Координаты тела x и у меняются с течением времени по закону g = (y0 sin а)/—g/2/2, x = (y0cosa)t Исключив отсюда время, мы получим уравнение: У =---n~2 g , ~ *’+№ а)х. 2у% cos2 a Это уравнение параболы. Обозначая через х0 и у0 координаты вер- шины параболы (точка А на рис. 290), можно записать уравнение траектории в форме y—y0 = k(x—x0)2, где _ yosin2a у __ Уо sin 2a tv — ' о ) у0 । , Л/0 — - - — - 2у? cos2 a 2g 2g 42. Траектория мяча проходит через точку с координатами И и S. Поэтому (см. задачу 41) Н — —gS2/2yoCOs2 a+S tg а. Отсюда имеем: у2 = g S2/2 cos2 a(S tg a — H) = g S2/(S sin 2a — H cos 2a—//) = — g S2/1 Д/S2 4~ #2 sin (2a — <p) — H], где igq = H/S. Наименыиее значение у0 = ^gS^/(V'Si+Н*-Н) = Vg (/S2 + Я2 + Н) дости- Ф I л гается при а== 2—r'j"* 43. (Рис. 291.) (у0 cos a) t = D/2, (y0 sin a) t —gt2j2 = Я, / = . -^-tga--^-(l-tg2a) = W, 2o0cosa 2 8v® ‘ga«-lihga+(-^r+,)=0- 176
При заданном D последнее уравнение дает два значения а, соответ- ствующих навесной и настильной траектории, или два одинаковых значения (критический случай), или ни одного (осколок не попадает на край ямы). Следовательно, яма должна быть такой, чтобы это уравнение не имело решений: Отсюда следует, что если о0 < V 2gH, то D—любое; если же v0> V2gH, то v%—2gH, Dmin = ^K vl-2gH. о И 44. Координаты и скорости тела в любой момент времени отно- сительно системы отсчета, изображенной на рис. 292, определяются теми же уравнениями, что и в задаче 41. В момент падения тела в воду его координата у— — И. Поэтому время полета Т определяется уравнением — H = (v0 sin а) 7—gT2/2. Отсюда у0 sin a j sin2 а~Н 2g//" g Так как Т > 0, следует оставить знак плюс. Расстояние от берега , _ и? sin 2а , v0 cos а —2 . 9—, о L = v0 cos а• Т —-------1—— у Vq sin2 а+2g//. Тело окажется на высоте h над водой спустя время __ v0 sin а ± vq sin2 а+2g (H — h) а — • g Если | h | < | H |, то физический смысл имеет только знак плюс. При h^H имеют смысл оба решения. Тело дважды во время паде- ния окажется на одной высоте над водой. Д77
Конечную скорость v проще всего найти с помощью закона сохра- нения энергии яюо/2+mgH = mv2!2. Отсюда v = V Vo+2gH. 45. В системе отсчета, изображенной на рис. 293, координаты камня в любой момент времени определяются следующими уравне- ниями: x=(v0 cosa)f, у = А0 + (у0 sin a)/—^/2/2. В момент падения камня у = 0 и Рис. 293. х—S, где S—дальность полета камня. Решая эти уравнения относительно угла а, получим tga = . 1/, , 2gh0 g&\ ~gs\,l± У ,+ v2 v* J- Это выражение имеет смысл при 1 + 2ф«_4-Ч0. Vo Vo Отсюда S < v0 К «о + Zghjg. Следовательно, Sn,ax=o0 К Vo 4- 2ghjg. При меньших S каждому значению S соответствуют два значения угла а, разность между которыми тем меньше, чем ближе значение S к максимальному. Следовательно, при максимальной дальности полета tga = ^-=-7=^==-U> a = 30° gSma* vJ+2gh0 У3 Vo 46. Движение тела описывается уравнениями h + (и0 sin a) t — = 0, (y0 cos a) t = S. Отсюда ,2 8S* _gS*_________________1_________= ° 2 cos2a(ft-f-Stga) 2 ft cos2 a 4-S sin a cos a ~g$ h (2 cos2 a— 1)+S sin 2a4-й h4-(hcos 2a4-S sin 2a) =__________g52___________ Й4- />4-S2cos(2a—<p)’ где <p — некоторый угол. Следовательно, 178
47. В любой момент времени труба составляет с горизонтом угол Р такой, что tg £ = г//х, где у = (u0 si и a) t — gt 2/2, х = (v0 cos а) i (координаты тела). Вектор скорости тела составляет с горизонтом угол <р, причем tg ф = (у0 sin a —g/)/(yo cos а)« По условию р~ф=л/2. Воспользовавшись формулой tg (р — <р) = (tg Р—tg <р)/(1 +tg р tgф), придем к уравнению g2/2 —3g (d0 sin a) f+ 2vo = 0. Отсюда / = ^-[3sina± — 9 cos2 а]. Это выражение имеет смысл (дает два разных или одинаковых дейст- вительных значения t) лишь при cos a 1/3. 48. Путь S, пройденный бомбой по горизонтальному направле- нию, равен S = |/Gl2 — Н2 = (у cos a) /, где t — время падения бомбы. Пройденный по вертикали путь H = (v sin a) t -\-gt2/2 (рис. 294). Исключив время из этих уравнений, находим: . v2 , (v2\22Hv2 ~ tga gs± V Us) + gS* 1 • Имеет смысл решение co знаком плюс. Знаку минус соответствует a < 0, т. е. случай сбрасывания бомбы в момент, когда самолет летит вверх. 49. Решение задачи значительно упростится, если оси координат направить вдоль наклонной плоскости и перпендикулярно к ней (рис. 295). Тогда проекции ускорения шарика на оси х и у будут соответственно равны ax~gx~g sin a, Oy=gy =— g cos a. Скорость шарика в момент первого соударения с наклонной плоскостью будет равна Vq— V 2gh. Начальная скорость шарика после первого соуда- рения равна v0 и образует с осью у угол а (рис. 295). Расстояние между точками первого и второго соударений шарика с плоскостью равно 1 / : w I (g sin а) G = (о0 sin a) t! + , 179
где — время полета. Это время определяется уравнением (vQ cos a) tt — (g cos а) /*/2 = 0. Отсюда t1 = 2vQ/g и l1 = 8hsina. Скорость шарика в момент вто- рого соударения определяется равенствами vix==vox~l~ax^i~vo sin а+(£ sin «) / i = 3v0 sin а, у = 1 = cos а—(g cos а) tr = — у0 cos а. После окончания процесса соударения эти скорости равны V2X~V1X^ V2y~ Vly Расстояние между точками второго и третьего соударений равно /2 = (Зу0 sin a) /2 + (g sin а) /2/2, где /2— время полета. Так как начальная скорость вдоль оси у та же, что и при первом соударении, то /2 = /г Поэтому /2 = 16Л sin а. Аналогично можно показать, что расстояние между следующими точ- ками /3 = 24/1 sin а. Следовательно, отношение /t:/2:/3... = 1:2:3... 50. Составляющие скоростей тел вдоль х и у в любой момент времени определяются так: sin ai— gt, v2y — VQ sin а2—gtt = cos аь 1>2х = —vo cos а2. Пусть и — скорость первого тела относительно второго. *Гогда sin at—gt— Vq sin a2 + g/ = y0 (sin ax —sin a2), ux = c0 (cos at 4- cos a2). Следовательно, скорость и равна U. = V «l+up = 2u0cos ”4“?. Тела движутся друг относительно друга с постоянной скоростью. По прошествии времени т расстояние между ними будет 5 = 2 с0 cos aia2-т. 51. Скорость любого камня при подлете к земле укон = = j/" t>o + 2g/i . Камень, подлетевший по наиболее пологой траектории, имеет наибольшую горизонтальную скорость. Но последняя равна "гор max = «о- Поэтому cosq> = a0/K »о + 2&Л. Отсюда h=g^-tg2<p. 52. Первый шарик отражается от пола, имея вертикальную со- ставляющую скорости с0 = V^2gh. Его вертикальная координата yi~vQt —gt2/2t в то время как вертикальная координата второго шарика y2 — h—gt2l2. В момент встречи £/l = «/2- Отсюда следует, что шарики встретились на высоте 0,75/i. 53. Из решения задачи 43 следует, что снаряд не попадает в точку, расположенную на высоте у, если расстояние по горизон- 180
тали от этой точки до пушки Г > V Vo — 2gy . g Следовательно, граница «опасной зоны» определяется уравнением У 2g 2vl Сечение такой поверхности (параболоида) представляет собой парабо- лу, которая совпадает с траекторией снаряда, выпущенного со скоро- стью и0 горизонтально из пушки, установленной на высоте Н — vo/2g. 54. 1) Обозначим через L — na длину гусеницы. Тогда I — = (L — 2л/?)/2 —расстояние между осями колес. В поступательном движении участвуют — — 2nR)/2a звеньев. Такое же число звеньев находится в покое относительно земли. Во вращательном движении участвуют nz = 2nR/a звеньев. 2) Время движения трактора Iq-S/v. За полный оборот гусе- ницы звено пройдет поступательно путь 21, двигаясь со скоростью 2v. Время поступательного движения звена за один оборот l/v. Всего гу- сеница сделает M — S/L оборотов. Следовательно, время поступатель- ного движения звена /j = Столько же времени звено покоится. Звено будет участвовать во вращательном движении в течение времени 2Nl S + 2nRN — NL V to------• При условии S^>L можно считать число оборотов целым, пренебре- гая временем неполного оборота гусеницы. 55. Обозначим искомый радиус через R и угловую скорость движения Рис. 297. трактора по дуге через <о. Тогда о1 = <о (/? — d/2), у0 = св (R-j-d/2) (рис. 296). Отсюда t>i _R—d/2 d v0+v!_ v9 R+d/2 2 Vo—«I 56. Первоначально наблюдатель находится на полюсе (точка О, рис. 297). Земная ось проходит через точку О перпендикулярно .181
чертежу. О А (параллельное ВС) —направление на звезду. Гора нахо- дится справа от точки А. а = соД/— угол, на который повернется земной шар за время Д/, <о — угловая скорость вращения Земли. Чтобы видеть звезду, наблюдатель должен за это время пробежать расстояние ОС «ОЛшДЛ Скорость наблюдателя v = OC/At — ОАа = = 0,7 м/с. 57. 1) За время полного оборота Т диск пройдет путь, равный длине окружности диска, т. е. В = 2лг, где г —радиус диска^-Следо- вательно, поступательная скорость любой точки диска оп = 2ш7Т = о. С другой стороны, линейная скорость вращения относительно центра О точек, лежащих на ободе диска, равна ол = сог, где со — угловая ско- рость вращения. Так как (о = 2л/Т, то ол = 2яг/Т=оп, что и требо- валось доказать. 2) Скорость точек обода относительно неподвижного наблюдателя складывается из двух скоростей: скорости поступательного и ско- рости вращательного движений. Для точки А суммарная скорость будет равна 2v. Для точек В и D суммируемые скорости равны по абсолютной величине, и их сумма составляет Y~2v (рис. 298, а). Для точки С суммарная скорость относительно неподвижного наблюдателя равна нулю, так как скорости поступательного движения и враща- тельного движения равны по абсолютной величине и направлены в противоположные стороны. 3) Мгновенные скорости точек диаметра АС возрастают прямо пропорционально расстоянию от точки С. Поэтому движение диска в данный момент времени можно рассматривать как вращение вокруг точки соприкосновения диска с плоскостью. Ось, проходящая через точку С перпендикулярно плоскости диска, носит название мгновен- ной оси вращения. Она перемещается при движении диска, проходя все время через точку соприкосновения диска с плоскостью. Следо- вательно, в данный момент времени все точки диска, отстоящие от точки С на одно и то же расстояние, будут иметь одинаковую сум- марную скорость относительно неподвижного наблюдателя. Точки, находящиеся на расстоянии радиуса диска от мгновенной оси (от точки С), будут иметь ту же по абсолютной величине скорость, что и скорость оси, т. е. v (рис. 298, б). 58. Составляющая скорости вдоль палочки для всех точек палочки одинакова и равна u = cos а. Следовательно, скорость конца па- лочки В равна од = ц cos а/cos £. Движение палочки можно рас- сматривать как поступательное движение вдоль АВ со скоростью и 182
и одновременное вращение вокруг оси, перпендикулярной плоскости чертежа и проходящей через некоторую точку О палочки. Расстоя- ние точки О от конца палочки А легко определить из соотношения v cos atg р_/ — АО дп—1 tga_____ у sin a “ АО * ~ tga+tgp’ Движение палочки в данный момент времени можно также рассматри- вать как чистое вращение вокруг оси О', перпендикулярной плоскости чертежа и расположенной на перпендикуляре 00' к АВ, Расстоя- ние 00' равно 00' = 40ctga=/~----1 ---о • tga+tgp 59. Распределение скоростей различных точек палочки показано на рис. 299. Скорость vc про- извольной точки С палочки по величине равна vc = Г QQ'i — 1/ v2 cos2 a+o2 sin2 a-jyrx F /1С/Л и направлена перпендикулярно прямой, проведенной из точки О' (см. предыдущую задачу) в дан- ную точку С. 60. Ус = cos 60° = у/2. Рис. 299. 61. Удобнее всего решать эту задачу в системе отсчета, связан- ной с равномерно движущимися автомобилями. В этой системе дорога движется назад со скоростью и = 50 км/ч, автомашины находятся в покое друг относительно друга, а их колеса вращаются. Линейная скорость точек на окружности колеса и линейная скорость зажатого камня также равны и. Камень пролетит наибольшее расстояние, если он вырвется в момент, когда его скорость составляет с горизонтом угол 45°. Найдем это расстояние. Пренебрегая тем обстоятельством, что в момент вылета камень находится несколько выше уровня шоссе, получим I = у2 sin 2a/g = y2/g = 19,6 м. Расстояние между машинами должно быть не меньше 19,6 м. 62. Угол между соседними спицами переднего колеса равен <p = 2n/N1. Колесо будет казаться зрителю невращающимся, если за время между съемками двух соседних кадров (т=1/24 с) колесо повернется на угол a = J?<p, где k — целое положительное число. С дру- гой стороны, угол поворота колеса за время т равен а = сот, где со—угловая скорость колеса. Следовательно, переднее колесо будет казаться невращающимся, если со = 2nZ?/7V1r. При этом скорость повозки у = сог = 2л йг/Л^т. Она будет иметь наименьшее значение ymin = 2n/7/V1T = 8,8 м/с. Задние колеса также будут казаться невра- щающимися, если 2л^1г/Л/1т = 2л^?/?/Д/2т. Отсюда при /г1 = ^2=1 Д/2 = ^/г=9. 63. 1) Спицы кажутся вращающимися против часовой стрелки, если за время т (см. задачу 62) колесо повернется на угол рь удов- летворяющий условию feqp > Рг > fe<p—ф/2, где k — \, 2, 3, ... После- довательные положения спиц колеса для данного случая изображены 183
на рис. 300, а. Зрителю кажется, что каждая спица повернулась на угол а < ср/2 против часовой стрелки. Возможные значения угловой скорости заключены соответственно в интервале Рис. 300. того, чтобы колеса казались вращающимися против часовой стрелки, скорость повозки должна удовлетворять неравенствам *SL>V> т A?cpr <рг т 2т’ kq>R ср/? т 2т * (1) (2) /? = 1,5г, поэтому образом: второе неравенство можно переписать следующим , - kcpr 1,5&фг 1 5 —г- > v > ——— т т 1,5<рг ~2т”’ Оба неравенства, совместимые лишь при k~l, дают допустимые значения скорости повозки в виде срг/т > v > 0,75<рг/т, или, так как ср = 2эт/6, 8,8 м/с > v > 6,6 м/с. 2) Спицы заднего колеса будут казаться вращающимися по ча- совой стрелке, если за время т колесо повернется на угол ра> удов- летворяющий условию (2k — 1) ф/2 > р2 > (£ —1)<р (рис. 300, б). От- сюда для скорости повозки вытекает неравенство 1 > 1,5(А? —1)<рг ’2т т Одновременно должно быть удовлетворено неравенство (1). При А = 1 оба неравенства совместимы, если 0,75 срг/т > v > 0,5 срг/т; при k — 2 они совместимы при условии 2срг/т > v > 1,5срг/т. Если k > 2, то неравенства несовместимы. Следовательно, 6,6 м/с > v > 4,4 м/с или 17,6 м/с > v > 14,2 м/с. > v т 184
Рис. 301. 64, Через точку С (рис. 301) проходит мгновенная ось враще- ния (см. задачу 57). Поэтому точка А имеет скорость, равную = (R + r)/r. Точка В имеет скорость ув = у(^—г)/г. 65. Мгновенную скорость, равную скорости оси катушки, имеют точки, рас- положенные на окружности радиуса г, центром которой является точка С. 66. Траектории точек Л, В и С изо- бражены на рис. 302. Точка В описывает кривую, называемую обыкновенной цик- лоидой; точки Л и С описывают удлинен- ную и укороченную циклоиды. 67. Линейная скорость точек на ок- ружности вала ^ = (0^/2. Линейная ско- рость точек обоймы u2 = QD/2. Так как шарики катятся без скольжения, таковы же будут и мгновенные скорости тех точек шарика, которые в данный момент соприкасаются с валом и обоймой. Но мгновенную скорость любой точки шарика можно рассматривать как сумму двух скоростей: скорости движения его центра и0 и линейной скорости вращательного движения вокруг Рис. 303. центра. Вращение шарика будет происходить с некоторой угловой скоростью со0 (рис. 303). Поэтому ^ = ^0 —coor, v2 = uo + ®or- Отсюда Ц) = (^i +а Vi (wd 4- QD). В этом выражении каждая из угловых скоростей может быть как положительной (вращение по часовой стрелке), так и отрицательной (вращение против часовой стрелки). При Q = 0 yo = cod/4. u 68. Так как конус катится без скольжения, то точки образую- щей О А (рис. 304) должны быть неподвижны. Из этого условия 185
определяется скорость Q вращения конуса вокруг собственной оси. Для точки А это условие дает coA/cos a = QA tg а. Отсюда Q —со/sin a. Скорость произвольной точки Di диаметра АВ основания конуса слагается из двух скоростей: ох = со (h cos a —г sin a)+rco/sin a, где г —расстояние от центра С основания до данной точки. Для точки D2, лежащей ниже центра С, будем иметь о2 = со(Л cos a + r sin a)—rco/sin a. Крайняя нижняя точка имеет скорость, равную нулю, а крайняя верхняя — скорость 2coAcosa. 69. В местах сцепления конических шестерен £ и С, а также шестерен Е и D линейные скорости должны быть одинаковы. Так как шестерни £ вращаются вокруг оси А со скоростью со, а сама ось вращается в другой плоскости со скоростью Q, то для сцепления колес £ и С имеет место равенство Г1Сд1 = гсо + г1й. Для сцепления колес £ и D аналогичное равенство имеет вид r2(o2 = —ra-^-rjQ. Отсюда 2Q = (Oi-[-co2, 2со=у(сд1 — со?). При определенной скорости Q приводимого во вращение двигателем колеса В угловые скорости ве- дущих колес автомобиля могут отличаться друг от друга на величину от нуля до 2Q. 70. Из соображений симметрии очевидно, что в любой момент времени черепахи будут располагаться в углах квадрата, сторона которого все время уменьшается (рис. 305). Скорость каждой черепахи можно разложить на радиальную (направленную к центру) и перпендикулярную ей. Радиальная скорость, т. е. скорость прибли- жения к центру, будет равна vr — vlV' 2. Каждой черепахе предстоит пройти до центра расстояние / = ц/У~2. Следовательно, черепахи встретятся в центре квадрата через время t = I/vr~ a/v. 186
71. Корабль В движется по направлению к кораблю А со ско- ростью v. В то же время корабль А удаляется от корабля В со ско- ростью у cos а (рис. 306). Следовательно, расстояние Л В сокращается со скоростью v (1 — cos а). Точка С (проекция точки В на траекторию корабля Л) движется со скоростью у cos а. Поэтому расстояние АС увеличивается со скоростью и (1 — cos а). Следовательно, сумма рас- стояний S = АВ + АС остается при движении кораблей посто- янной. В начальный момент точка С совпадала с Л и по- тому S = AB = a. Через доста- точно большое время точка С будет совпадать с В. При этом ЛВ = ЛС = В/2 = а/2. Корабли будут двигаться на расстоянии 1,5 км друг от друга. 72. Движение шарика мож- но рассматривать, по обыкно- вению, как результат сложе- ния движений по вертикали Рис. 306. (равноускоренное движение) и горизонтали (равномерное движение). Проще всего решить задачу, построив график зависимости коор- динаты щарика вдоль горизонтали от времени для предельных зна- чений скорости 267 см/с и 200 см/с (рис. 307). Нижняя ломаная соответствует максимальной скорости, а верхняя—-минимальной. С течением времени, как видно из графика, неопределенность коор- динаты шарика х, даваемая отрезком горизонтальной прямой, заклю- ченной между линиями графика, увеличивается. Вертикальная штри- ховка на рис. 307 соответствует движению шарика от М к W, а горизонтальная — от N к Af. Области пересечения штриховок соот- ветствуют неопределенности в направлении горизонтальной скорости. 1) Непосредственно из графика видно, что направление скорости шарика по горизонтали после того, как он один раз отскочит от 13?
плиты N, будет неопределенным при времени падения OK ^OL или t > АВ (ОК = ^Л^ с; 0£ = 0,2 с; АВ — 0,225 с). Следовательно, 10 см Ж 20 см или /7^ag/2/2« 26 см. 2) Шарик может попасть в любую точку основания, на котором покоятся плиты, если время падения шарика AF = 0,3 с. Следо- вательно, //min = 44 см. § 4. Динамика прямолинейного движения 73. T-Fx/l. 74. Т = F xy/lh — действует в горизонтальном направлении КТ КАХУ вправо. N — M — действует вертикально вверх. М М 75. Масса левой части стержня т2=—/, а правой т2=-£-(£—-/), где М— масса всего стержня. Под действием приложенных к ним сил каждая часть стержня движется с одним и тем же ускорением а. Поэтому Fr—F = m1ai F — F2 = m2a. Отсюда растягивающая сила Тупг + Туп! L — l , р I mi + тг 1 L ' 2 L' 76. К бруску приложены две силы mg и 7V. Источником силы N является деформированный пол лифта. Из уравнения движения бруска ma = mg—N следует, что N — m(g — a). По третьему закону Нью- тона брусок (вследствие того, что он деформирован) действует на пол лифта с силой, равной 2V. Если a — gt то N = 0, т. е. брусок пере- стает действовать на пол (в бруске исчезли деформации). При уско- ренном движении вверх 77. Чтобы доска не съезжала, составляющая силы, приложенной к доске со стороны мальчика, должна быть направлена вдоль доски вверх. Следовательно, на мальчика со стороны доски действует рав- ная и противоположная сила, направленная вдоль доски вниз. Вы- брав положи1ельное направление координатной оси вдоль наклонной плоскости вверх, запишем уравнения движения мальчика: — Mg sin а—F — Ма, S = ~Ь "ёг > at. Доска находится в равновесии, поэтому/7 — mg sin а = 0. Решив дан- ную систему, находим 8(М 4~m)g sin а' 78. Большее ускорение (а > g) у частиц бруска в верхнем сече- нии. Частицы бруска в нижнем сечении в начальный момент времени имеют ускорение a = g. 188
79. В верхнем сечении в начальный момент времени частицы бруска имеют ускорение a—g, в нижнем сечении—ускорение а > g. 80. Показания весов уменьшаются. 81. При движении крыльев воздух сжимается под крыльями и разрежается над крыльями. Вследствие деформации воздуха возни- кает подъемная сила W. Из уравнения движения ma = N — mg сле- дует, что N — ma-\-mg. По третьему закону Ньютона крылья мухи действуют на воздух с силой N, направленной вниз. Вследствие этого чашка весов, на которой находится бутылка с мухой, будет опускаться. 82. На рис. 308 изображены силы, действующие на грузы. Урав- нения движения для грузов запишутся следующим образом: гща — Т — m^g, m2a — m2g — Tt где Г —натяжение нити, а —ускорение. (Ускорения грузов одина- ковы, так как нить считается нерастяжимой. Невесомость нити и блока определяет постоянство 7.) Отсюда а — [(т2 — + ^2)] g= =327 см/с2, Т — тг (a + g) = 1,3 Н. Время движения / = У^Н/а « 1 с. Рис. 308. Рис. 309. 83. Если масса блоков и нити пренебрежимо мала (рис. 309), то 27 — 7=0, Т — Р — та. Отсюда F = (1 + alg)Pl2. При а — 0 F = Р/2. 84. Уравнения движения для тел с массами mlt m2t т3 имеют вид mla = m1g — Tli m2b = m2g—T2, т3с = т^ — Т3, где а, Ь, с—ускорения относительно неподвижного блока А. Уско- рение считается положительным, если оно направлено вниз. Если масса нити ничтожно мала по сравнению с массами mlt т2, т3, то натяжение постоянно вдоль всей нити. Отсюда следует, что 72=78, и сила, с которой нить, перекинутая через блок Л, действует на блок В, равна 7\ (рис. 310). 189
Рассмотрим ту часть нити, которая находится в данный момент времени на блоке В, На эту часть вертикально свисающий левый конец нити действует с силой Г2, а правый—с силой Т3 (Т2 — Т3). Так как масса любой части нити ничтожно мала, то сумма всех дей- ствующих на нее сил должна стремиться к нулю. Следовательно, блок В действует на лежащую на нем часть нити с силой F — Та + Т'з, направленной вверх. По третьему закону Ньютона деформированная Рис. 310. нить в свою очередь действует на блок с си- лой Т2 + Т3. Так как масса блока В ничтожно мала, то Т\== 7*2 —j-Tg. По прошествии некоторого времени (весьма малого) после начала движения тел деформа- ция нитей прекращается, и длины после этого не изменяются с течением времени. Это оз- начает, что ускорение блока В будет равно ( — а), а ускорения грузов т2 и т3 относительно блока В равны и противоположны по на- правлению. Обозначив через d ускорение тела т2 относительно блока В, получим Ъ = (— а) 4- d, c = (-a) + (-d), откуда b-\-c= — 2а. Таким образом, оконча- тельно имеем следующую систему уравнений: = —7\ т2Ь ~ т^ — Т /2, m3c = m3g—Т/2, Ь-\-с — —2а. Решая эту систему уравнений, получим (при условии m1 = m2-j-m3) а_ (т2 — т3)2 ь— mj—4ml (ml 4- т3 + 6m2m3) ’ ml-j- 4т2т3 с ml — 4m3 Т 8тгт2т3 & mf + 4m2m3 ’ ml + 4т2т3 В общем случае _____8тгт2т3_____ 4т2т3 —|— т,-^ (т2 —|- ^з) 85. Показание динамометра вначале равно F = 30H. Если пока- зания динамометра не меняются, то на груз 20 Н действует направ- ленная вверх сила натяжения нити, равная ЗОН. Следовательно, этот груз движется вверх с ускорением a = g/2. С тем же ускорением движется вниз другой груз. Дополнительный груз на второй чашке находится из уравнения P±b£=P+Pl-P, s 2 откуда Рх=30Н. 190
^1+^2 86. Чтобы восстановить равновесие, надо с правой 'чашки весов _ (т,—tn,}2 снять груз весом Р— -7-7—~~g. /721 —j~ 77/2 87. Так как массой блоков и нити мы пренебрегаем, натяжение нити будет Поэтому всюду одинаково (рис. 311). m2g — 2Т = m2a2, m3g—T = m3a3, аг— —(ai4-a3)/2. Отсюда «1 4m1m8—3/ngm3 -f- 4тхт34-m2tn3-\-тхпгк _ — 4m1m3 + m2m3 a* ~ 4m1m3 4- т2т3 + mAm2 _ 4/n1m3—Зт^пц + m2m3 3 4/n1m3+m2m34-mi/722 Рис. 311. 88. Так как масса блоков и нити пренебрежимо мала, натяже- ние нити всюду одно и то же. Поэтому mzg — 2T — nirfizi 2Т — Т = 0, Отсюда Т = 0 и a1 = a2 = g- Оба груза свободно падают с ускоре- нием g Блоки В и С вращаются против часовой стрелки, блок А — по часовой стрелке. 89. Пусть в некоторый момент i-й груз находится на расстоя- нии Х( от потолка. Тогда из нерастяжимости всех нитей следует, что сумма всех таких расстояний остается во время движения постоян- 8 ной: — За время Д/ произойдет изменение каждого из этих i = l расстояний, поэтому через время Д/ о Z=1 L Вычитая одно уравнение из другого, получим после сокращения на Д/: 8 , 1 X 8 1 8 i=l 4 i=l i=l Поскольку это равенство должно быть справедливым при любом Д/, 8 ускорения грузов связаны соотношением Так как массы [191
блоков и трение в их осях пренебрежимо малы, то натяжение одной и той же нити, перекинутой через любой блок, по обе стороны блока одинаково и равно половине натяжения той нити, на которой этот блок висит. Отсюда следует, что натяжения всех нитей, на которых висят грузы, равны между собой. Обозначив это натяжение через 7, запишем уравнение движения f-го груза в виде mjg — Т = или Сложив все 8 таких уравнений, получим 8 1 8 1=1 1=1 Отсюда т—8g 8 Sa/mz) 1=1 90. Обозначим натяжение нити, на которой висят грузы и /п2, через Т. Тогда натяжение нити, на которой висит груз т3, равно 27, а натяжение нити, на которой висит груз /и4, равно 47. Уравнения движения для грузов запишутся в виде 'n1g — 7 = m1a1, /n2g —7 = m2a2, m3g—2T = m3a3t m4g — 4T = m4a4. Из-за нерастяжимости нитей ускорения грузов связаны соотношением ai 4~ а2 4- — 0* (Это соотношение можно получить тем же способом, как в решении задачи 89.) Решив полученную систему уравнений при заданных значениях mlt т2, m3i т4, найдем a^ — g/ЗЗ « 0,3 м/с2. 91. Для данного случая уравнения динамики запишутся в виде т __ vt~=v0—ai. mg — 7 = та, Т — Ма, где 7 — натяжение нити. Отсюда а 2 =—g. Уравнения кинематики дают x=vot —at2/2, Решая данную систему уравнений, найдем, что через 5 с тележка будет находиться на том же месте (х=₽0) и будет иметь скорость = 7 м/с, направленную вправо. Тележка пройдет путь S = 2|vel = 17,5 м. о«/2)« 2 192
92. На рис. 312 изображены стрелками все силы, действующие на тела mlt т2, т3. Обозначим через а ускорение тел mi и т2, а через с и b — горизонтальное и вертикальное ускорения тела т3. Запишем проекции уравнений движения на горизонтальное и верти- кальное направления: m1a = m1g—Т, m2a — T-\-N sin a, m3b = m3g—Af cos a, m3c = 2Vsina. Из геометрических соображений следует соотношение bl(a-\-c)= tga. Решая эти уравнения, находим а, Ь, с: __+ sin а cos а а mi + т2+тз sin2 а __тг sin a cos a -f- + т2 + т3) sin2 а m1 + m2 + zn3 sin2 а & (m1+m2)_sin a cos a —m1sin2a С==\ т1^-т2-^т3 sin2 а Решение справедливо при tga — \^т2/т1 (см. задачу 35). 93. На рис. 313 изображены все^силы, действующие на стержень и клин. Обозначим через а ускорение стержня относительно непо- движного стола, а через b — ускорение клина. Запишем проекции Рис. 313. уравнений движения стержня и клина на горизонтальное и верти- кальное направления: та —mg — N cos a, 0 = А\ —N2-^N sin a, Mb = TV sin a, 0 = TV3~ N cos a—Mg. Из геометрических соображений следует уравнение кинематической 7 Б. Б. Буховцев и др. 193
связи между ускорениями стержня и клина: a/6 = tga. Решая урав- нения, получаем _ mtg2a . mtga М~{-т tg2a tg2a 94. Единственной силой, действующей на бусинку, является сила реакции стержня Л\ которая направлена перпендикулярно стержню. Абсолютное ускорение wa бусинки (ускорение относительно непо- движного наблюдателя) будет направлено в сторону действия силы реакции W. Относительное ускорение wo направлено вдоль стержня (рис. 314); wa = a4-w0. Из треугольника ускорений следует: t0o = acosa, t0a = asina. На основании второго закона Ньютона сила реакции* равна W = /nasina. Время движения бусинки по стержню т определяется из уравнения I — (a cos а)т2/2. Отсюда т = K2//(acosa). 95. Рассмотрим элемент нити, находящийся в щели. Пусть нить движется вниз. Тогда на элемент нити действуют силы натяжения нити с обеих сторон и сила трения (рис. 315). Так как массой рас- сматриваемого кусочка нити мы пренебрегаем, то F—Т2 = в. Уравнения динамики запишутся следующим образом: —Т^ m^a, m2g — Т2= — т2а. Отсюда т1 + 96. 1) Если F <kmg, то бруски не будут двигаться. 2) Если kMg^ F > kmg, то будет двигаться меньший брусок. 3) Если F > kMg, то бруски будут двигаться в разные'стороны. (Предполагается, что пока действовали силы F, трение отсут- ствовало.) 97. 1) Если скорость вагона возрастает, картина будет скользить вдоль задней стенки вагона вниз с ускорением g—ka. (Если ka > g, то картина будет находиться в покое.) 2) Если скорость вагона убывает, картина будет двигаться вниз с ускорением g и вперед с ускорением а, 98. По второму закону Ньютона (т1+т2) а — m^g sin a—m^g sin £ — kmtg cos a — km^g cos £. 194
Г рузы окажутся на одном уровне после прохождения пути S, удовле- творяющего следующим уравнениям: S sin a = /z — S sin 0, 5 = ат2/2. Исключая S и а из системы трех уравнений, получим ml _ gr2(sin a+sin 0)(fc cos 0+sin 0)+2Л m2 gT2 (sin a+sin 0) (sin a —& cos a)—2h * 99. Рассмотрим желоб, составляющий произвольный угол ф с вер- тикалью (рис. 316). Если k < ctg ф, то путь, пройденный песчинкой, S = a/2/2, где а=£(созф— — k sin ф). Координаты песчин- ки: % = 3 5тф, ^ = 5созф. От- сюда x2-]~y2 = S2 = или (х2+у2)2 = (^2/2)2(у-^х)2. Учтем, что k < у/х, и обозна- чим A~gt2/4. Тогда х2 + у2 = 2А (y—kx). Последнее уравнение можно за- писать в виде (х + Ы)2 + (у—А)2 == Л 2( 1 + £2). Это — уравнение окружности радиуса R = А У 1 + k2 = ГJ У 1+&2 с центром, лежащим ниже точки О на А=-^— и левее вертикали на kA — kgt2j4. По другую сторону вертикали геометрическое место искомых точек составляет аналогичную дугу. 100. Уравнения движения дают для ускорения камня следующие выражения: ai = £f(sin a+fc cos а) при движении вверх, a2 = g(sin a—k cosa) при движении вниз.* Кинематические уравнения запишутся следующим образом: I = Ц)Л — 0i^i/2, / = а2^1/2, Ц) —* «Л = 0. Из этих пяти уравнений находим k = 2Z—sin a w 0 37> = l/_ - Z ~ se 4,2 c. gHcosa r gGsina —1 101. Тележка первоначально движется равнозамедленно. Ско- рость тележки равна у=у0— где/— сила трения, равная king. Тело движется равноускоренно. Скорость тела u = (//zn)/. Если 7* 195 I
тележка длинная, то скорости тела и телёжки могут сравняться. Это произойдет в момент времени т —и0/(//m + //Af).* После этого и тело, и тележка начнут двигаться с постоянной скоростью, равной Mvol(M + /п). Тележка к этому моменту времени пройдет путь S = — vox—(//2А4)т2,-а тело — путь s = (f/2m) х2. Путь, пройденный телом относительно тележки, равен S—s. Этот путь должен быть меньше /. Таким образом, тело не покинет тележку при условии S — s^l, т. е. ^0 < I. • 2g£(Af4-m) 102. Если k (m-}-M)g, движения нет. Пусть/7 > k(m-\-M)g. Исследуем случай отсутствия скольжения тела по бруску. Уравне- ния движения при этом будут идоеть вид ma = f, Ma = F— f — k (т^-M)g; f^kmg. Отсюда F и f mF l l a ——k8> f = —r-^~kmg^kmg, что возможно, если k (m-\-M)g < F < 2k (m+ Al)g. Если же F > 2k (m-|-Af)g, то тело будет скользить по бруску. В этом случае уравнения движения имеют вид та —kmg, Mb = F— kmg— k(M-\-m) g, откуда , . F u (2m + M) a=kg> —g. Легко убедиться в том, что b > а. 103. Уравнения движения бруска и тела имеют вид ma = f, (1) Mb = F-f, '(2) где /—сила тпения, а и Ь — ускорения. Предположим, что проскаль- зывания нет, тогда а = Ь. Из уравнений движения можно определить ускорение и силу трения. Сила трения f — mFI(M + m). Чтобы не было проскальзывания, сила трения должна удовлетворять неравен- ству f^kmg, т. е. Fkg. Если F > k(M-\-m) g, то воз- никает скольжение. Уравнения (1) и (2) в этом случае примут вид та —kmg, Mb = F — kmg. Из этих уравнений находим а и b\ a —kg, b = (F — kmg)!M. Оче- видно, что b > а. Ускорение тела относительно бруска будет на- правлено w сторону, противоположную движению, и по величине равно (F — kmg)lM — kg. Время движения тела по бруску т== = 1/ 2Ш V F -kg(M-\-m) 196
104. 1) Силы, действующие на стол и „груз, изображены на рис. 317. Уравнения движения по горизонтали имеют следующий вид: для стола с блоками F-F + FTp=Afll; для груза F-F^^a2. Предположим, что сила F настолько мала, что груз не скользит по столу. Тогда ai = a2 и ^тр = ^1/(^1 +Увеличивая постепенно силу F, мы тем самым будем постепенно увеличивать и силу трения Гтр. Однако если стол и груз неподвижны друг относительно друга, то сила трения между ними не может превысить значения FTp щах = kPz- Поэтому скольжение груза по столу начнется при F > F^maK^f^ = k^(Pt+Pt)= 100Н. В нашем случае F = 80H, следовательно, груз проскальзывать не будет и F 8 01 = а2 = 7\+Р7*=25*№3’14 М/с2' 2) Уравнения движения для* стола с блоками и груза в этом случае имеют вид F ~ ai> Р Рт$ — '~а2> Ускорения стола и груза направлены в противоположные стороны, поэтому проскальзывание обязательно будет. Следовательно, Лтр = kP2. Ускорение стола равно --F kPа 2 101/2 «1 =----—?g = -j5^ «-1,31 м/с2. Стол будет двигаться влево. .197
105. При движении бусинки на нее действуют две силы: сила трения kN и сила реакции W. Абсолютное ускорение будет направ- лено по результирующей силе F. Из рис. 318 следует: N = ma sin а, до0 = а cos a—kN/т —a (cos а — k sin а). ~ 1/ 2Z Отсюда т = 1/ —--------г-т--г . V a (cos a—k sin а) Если k ctg а, то бусинка не будет двигаться относительно стерж- ня, и в этом случае сила трения равна та cos а. 106. По второму закону Ньютона изменение "количества движе- ния системы пушка—снаряд за время выстрела т должно равняться импульсу действующих на систему сил: по горизонтали * mv0 cos а — Mvt = FTpT, где ГТрт—импульс сил трения; по вертикали mv0 sin a —Nt —(Mg-]-mg) т, где Nt—импульс сил нормального давления (реакции горизонталь- ной площадки), (Mg-]-mg)T—импульс сил тяжести. Учитывая, что FTQ = kN, получим т , т . , М4-т »i =-^осоз a—k -^-v0 sin а — fe- fa- gt или, так как £т<^ц0, ~ (cos a —ft sin а). Данное решение пригодно для fcCctga. При k > ctg а пушка оста- нется неподвижной. 107. 1) Ускорение в начальный момент aQ — FlM » 13,3 м/с2. В зависимости от времени ускорение меняется по закону a—Fl(M—р/), 198
где ц —200 кг/с — масса топлива, расходуемая ракетой в секунду. График ускорения имеет вид, изображенный на рис. 319. Скорость спустя 20 с численно равна площади, заштрихованной на чертеже, v 300 м/с. 2) Второй Закон Ньютона запишется так: (М — p,t)a = F —(М —— f- По условию / = 20 с, a — 0,8g. Отсюда сила сопротивления воздуха f = F-(M-iit)g-(M-iLt)0,8g& 125-103 Н. 3) Уравнение Ньютона для груза дает m^a-kx—где tnt — масса груза на конце пружины, а — ускорение ракеты, & —коэф- фициент упругости пружины, х — удлинение пружины. По условию задачи т& — klQ. . Следовательно, х = -~- (а + g). Шкала прибора должна быть равномерной. Делению в один сантиметр соответствует ускорение g. 108. В системе отсчета, связанной со вторым кубиком, имеем у?—Уо = 2аг/, yj — vl = 2#2Л где а1^= — F (2М-\-т)/2Мт— ускорение пули в первом кубике, — ее скорость при выходе из него, а2 = — F (М-\-т)/Мт — ус- корение пули во втором кубике, v2 — ее скорость при выходе из него. Положив 1^ = 0, найдем нижний предел х скорости и0, а поло- жив v2 = 0, получим верхний предел. Следовательно, VFl (2М-\-т)/тМ <: У Fl (4/И + Зт)/т^1. § 5. Закон сохранения количества движения 109. Разобьем массу диска на пары одинаковых элементов, лежа- щих на одном диаметре на равных расстояниях от центра. Количество движения каждой пары равно нулю, так как количества движения обеих масс равны, но направлены в противоположные стороны. Следовательно, количество движения всего диска равно нулю. НО. Пусть масса лодки М, масса мешка т, начальная скорость лодок у0- При выбрасывании мешка с лодки на нее действует неко- торая сила в направлении, перпендикулярном у0. Однако изменения количества движения лодки не происходит, так как сила сопротивле- ния воды препятствует поперечному движению лодок. Количество движения лодки изменяется только при попадании в нее мешка. Применяя закон сохранения количества движения к системе мешок—лодка, в первом случае можно написать: — (M-{~2m)v1 для одной лодки, # —— v2 для другой» Здесь и v2 — конечные скорости лодок. Из данной системы уравне- М нии имеем = —v2 = v0. 199
В случае, когда мешки перебрасываются одновременно, конечные скорости лодок v[ и v2 определяются из уравнений Mvq — = v'lt —— _ г г M —*т Отсюда цх = —v2= уо- Таким образом, конечная скорость ло- док в первом случае будет больше. 111. Количество движения системы доска — лягушка в горизон- тальном направлении не изменяется. Следовательно, можно написать mv0 cos а — Ми, где и — скорость доски относительно неподвижной поверхности пруда. Чтобы лягушка оказалась на другом конце доски, должно выпол- няться условие L — ин = (vQ cos a) Т, где fr —время, в течение которого лягушка находилась в воздухе; оно равно т = 2у0 sin a/g. Из имеющихся уравнений можно опре- делить V° V (m/M 4-l)sin 2a ‘ 112. ' Поскольку в горизонтальном направлении система жук — клин замкнута, для определения скорости v клина можно воспользо- ваться законом сохранения количества движения Mv-\~m(v — u cos a) = О, где (у—и cos а) — горизонтальная составляющая скорости жука отно- сительно неподвижной системы отсчета. Отсюда ц cos a. 113. Так как внешние силы, действующие на систему по гори- зонтали, отсутствуют, проекция общего количества движения системы клин —грузы на горизонтальное направление должна оставаться по- стоянной (равной нулю). Отсюда следует, что клин начнет двигаться только в том случае, если ^5удут двигаться грузы. Чтобы груз т2 . двигался вправо, должно выполняться условие tn2g sin a mYg 4- ktn2g cos a. Отсюда m1/m2^sina—fccosa. При этом условии клин будет дви- гаться влево. Чтобы груз т2 двигался влево, должно выполняться условие m2g sin a 4-km^g cos a или m1lm2'^ sin a + k cos a. Клин при этом будет двигаться вправо. Следовательно, для равновесия клина отношение масс грузов должно удовлетворять неравенству sin a — k cos a tn1lm2 «С sin a 4- k cos a. 114. Для направления вдоль наклонной плоскости можно на- писать mv cos a 4- Mw = 0, 200
где « — скорость ящика в момент попадания в него камня; она равна и = 2gS sin а. Следовательно, — V sin « ~ т cos а 115. В высшей точке подъема скорость ракеты равна нулю. Из- менение общего количества движения частей ракеты под действием внешних сил (силы тяжести) крайне незначительно, так как импульс этих сил весьма мал ввиду кратковременности взрыва. Поэтому об- щее количество движения частей ракеты до и сразу после взрыва остается постоянным и равным нулю. Между тем три вектора m3v3) могут в сумме дать нуль только тогда, когда они лежат в одной плоскости. Отсюда следует, что и векторы vb v2, v3 лежат в одной плоскости. 116. Скорость лодки относительно берега и связана со скоростью m ~ человека относительно лодки у соотношением и = —г-т? v. Отношение т-\- М скоростей во время движения остается постоянным. Поэтому отношение прой- денных путей будет равно отноше- нию скоростей: S/l = М), где S—-путь, пройденный лодкой, а / — длина лоДки (расстояние, пройден- ное человеком относительно лодки). Следовательно, для того чтобы лодка причалила, ее длина должна быть не менее I == S = 2,5 м. Лодка не т причалит. 117. Примем за начало коорди- нат ту точку, откуда начинал дви- гаться взрослый. Тогда начальная координата центра масс будет равна Рис. 320. х т^/2+т^ 1 nh + mz + nis' Обозначим через х2 координату центра масс,в момент, когда взрос- лый добегает до края платформы. Тогда х ^mJZ/2 — s)4-m2(/ — s) + m3(//2 — s) 2 m1 + m2 + m3 где s—перемещение платформы. Так как в горизонтальном направ- лении система взрослый — ребенок — платформа замкнута, <го х1 = х2. Из этого равенства находим s: 2т2 —/п3 I mi + m2 + m3 2 ’ При заданных числовых значениях это дает s = 0,6 м. 118. На систему обруч —жук в горизонтальном направлении внешние силы не действуют. Поэтому центр тяжести системы (точка^С на рис. 320) не будет перемещаться в горизонтальной плоскости. 201
Расстояние от центра тяжести системы до центра обруча равно СО — — —R. Так как оно постоянно, центр обруча О будет опи- т-\-М сывать относительно неподвижной точки С окружность радиуса СО. Легко видеть, что траектория жука представляет собой окруж- ность радиуса АС=—т~тг Взаимное расположение, а также на- правление движения жука и обруча указаны на рис. 320. 119. Обозначим через скорость ракеты в конце k-й секунды. В конце (£-|-1)-й секунды из ракеты выбрасывается газ массы /и, который уносит с собой количество движения, равное т(—u-}~v^). Из закона сохранения количества движения следует, что (М — km)vk = \M — (А?+ l)m] vk + 1 + m (—u+vk). Изменение скорости ракеты за одну секунду равно Vk + l Ok ти Зная изменение скорости за одну секунду, можно написать выраже- ние для скорости в конце n-й секунды: . / т т . .. т \ \ vn — _ т+ м __ М — пт)' 120. Скорость ракеты будет увеличиваться. Это становится оче- видным, если перейти к системе отсчета, относительно которой ракета в данный момент покоится: Давление вытекающих газов будет толкать ракету вперед. 121. Для рассмотрения всех четырех возможностей обозначим: и, —начальная скорость снаряда относительно пушки, ^ — начальная скорость снаряда относительно земли, р —угол наклона ствола пушки, у — угол наклона вектора начальной ско- рости v2 (рис. 321). Соотношения меж- ду указанными величинами определя- ются равенствами Vi cos р + v = v2 cos у, (4) Vi sin p = v2 sin y, (2) mv2 cos у4-Л4о = 0. (3) Если под понимается начальная скорость снаряда относительно’земли, а угол а—угол наклона век- тора v0 (т. е. ^0 = ^2, а = у), то из уравнения (3) имеем т т * —гг cos у = —'-тг cos а. М ’ М Пусть теперь ^ — начальная скорость снаряда относительно ствола пушки, который направлен под углом а к горизонту (Уо = иь а = Р). Тогда из (1) и (3) получим Рис. 321. т о т У = ~ ТТЛ- V1 COS Р = — -77-— vQ cos а. М~^т 1 r М-\-т w Если задана начальная скорость относительно земли и угол наклона 202
пушки (Vq — v^ а = Р), то /*(/«+7И)2 sin2 р+Л!2 cos2 р * т =-----у0 cos а. У (m-|-Af)2 sin2a+M2 cos2 а Наконец, если у0 — начальная скорость относительно ствола пушки и задан угол наклона вектора начальной скорости относительно* земли (начальный угол траектории), то у0 = ух и ® = у. Тогда т v==-----yr - • .......—- - vL cos у == у М)2 cos2 у + М2 sin2 у т =.................................v0 cos a. M)2 cos2 a+ Af2 sin2 a Во всех случаях знак минус означает, что пушка откатится в сто- рону, противоположную движению снаряда. § 6. Статика 122. Рассмотрим часть нити, прилегающую к цилиндру. Разобь- ем угол 0 на большое число п частей и обозначим 0/п = а. Предста- вим нить как п связанных между собой кусочков и выделим /-й ку- сочек (рис. 322). На него действуют соседние части нити с силами Ту и Ту+1, реакция цилиндра Nj и сила тре- ния kNj. Учтя, что угол а мал, полу- чим следующие уравнения равновесия: Nj — Ту ос/ 2 — Ту+xa/2 = 0. Отсюда Ту+х = Ту (1 + ka), т. е. . 7\ = T0(l + ka), T^TJl+^a), ..., T^T^-Hl+M. Следовательно, Тп = Те (1 -f- ka)n = То (1 + кв/п)п, и при п -> оо получим Т = lim Тп= lim То(1 + ktyn)n ==ТоеЫ, * п -► 00 П -+ СО где 6 = 2,71828...—основание натуральных логарифмов. 123. В положении равновесия (рис. 323) mg —2mg cos a = 0. Следовательно, a = 60°. Искомое расстояние h = l ctg a=// ]Лз. Равновесие установится после того, как затухнут колебания, возни- кающие при опускании груза. ~ 124. Равенство проекций сил на направление вертикали (рис. 324) приводит к уравнению 2^ sin (a/2) — 2F'Tp cos (a/2) = 0, 203
где 2V —сила нормального давления, a Ртр«С&Л^ —сила трения. (Массой клина обычно можно пренебречь.) Следовательно, tg(a/2)<# й a<2 arctgfc. 125. Если груз Р опустится на высоту h, то точка # опустится на 1/3h. Груз Q при этом поднимается на 2/3Л. Применяя «золотое» правило механики, имеем Ph—Q2[3h. Отсюда P = 2/3Q. 126. Если ящик не опрокидывается, то момент силы F, вращаю- щий его против часовой стрелки вокруг ребра при основании, меньше или равен моменту силы тяжести, вращающему ящик по часовой стрелке. Чтобы ящик скользил, сила F должна -быть больше макси- мальной силы трения, приложенной к ящику. Следовательно, Fh^mgll2, F^kmg., Отсюда 127. Для поворота балки необходимо, чтобы момент сил, прило- женных к концам балки, был больше момента сил трения, когда эти силы достигают своего максимального значения. Силы трения рас- пределены равномерно вдоль балки (рис. 325). Среднее плечо сил Рис. 325. у трения, действующих на левую или правую часть балки, равно //4, если длину всей балки обозначить через /. Момент всех сил трения относительно центра балки равен 2(АгР/2) (Z/4). Следовательно, для поворота балки необходимо, чтобы приложенные силы F удовлет- воряли неравенству 2F//2 > kPlft. Отсюда F > kPfi. Для посту- пательного перемещения балки необходимо, чтобы 2F > kP. Следова- тельно, балку легче повернуть. 128. Уравнение движения груза имеет вид P^afg—F— Ро (рис. 326). Сумма сил, действующих на кран по вертикали, равна нулю. Поэтому P1-f-P2 = ^ + ^. Равенство нулю суммы моментов сил относительно точки А дает уравнение Fl-\-PLl2 — LPz. Решая данную систему уравнений, найдем Pi « 2,23.104 Н, Р2 « 1,77.104 Н. 204
129. Для равновесия рычага приложенная к точке D сила должна* создать момент, равный Р-АВ. Сила будет минимальной при максимальном плече, равном BD, Следовательно, F — Р -AB/BD— — Р)У 2 и направлена перпендикулярно BD. 130. Если трение между полом и ящиками отсутствует, то ящики сдвинутся одновременно. Если же коэффициент трения не равен нулю, то раньше сдвинется правый ящик (см. рис. 49), так как сила, приложенная к нему со стороны палки, будет больше, чем сила, приложенная к левому ящику. Действительно, со стороны правого ящика на палку действует сила Fu направленная против F, а со стороны левого zящика —сила Л2, направленная по F. Сумма сил при равновесии равна нулю. Следовательно, F1 — F-\-F2t и сила раньше, чем Л2, достигнет значения максимальной силы трения покоя. 131. Равенство нулю суммы моментов сид, действующих на шар, относительно точки А (рис. 327) приводит к уравнению F^R-NR=0^ Так как F^^kN, то k^\. 132. Для того чтобы тело находилось в покое, необходимо, чтобы суммарный момент сил, стремящийся повернуть тело по часо- вой стрелке, был равен моменту сил, стремящемуся повернуть тело против часовой стрелки вокруг какой-либо точки (например, вокруг центра тяжести). В данном случае момент сил трения, вращающий кирпич по часовой стрелке, должен быть равен моменту сил давле- ния плоскости на кирпич. Отсюда следует, что сила давления плос- кости на правую часть кирпича должна быть больше, чем на левую. По третьему закону Ньютона и сила давления правой половины кирпича на плоскость должна быть больше силы давления левой. 133« Для того чтобы каток поднялся иа ступеньку, необходимо, чтобы момент сил, вращающих каток вокруг точки А (рис. 328) против часовой стрелки, был бы по крайней мере равен моменту сил, вращающих его по часовой стрелке: P(R-h) = P //?2-(/?-/i)2. Отсюда Л = (2 ± У 2)7?/2. Так как h < /?, то ft=(l- К 2/2) 7? «0,29/?. 205
134. Так как на одной из плоскостей сила трения равна нулю, то она равна нулю и на другой плоскости. В прдтивном случае шар вращался бы вокруг своего центра, ибо момент всех остальных сил относительно этого центра равен нулю (ввиду равенства нулю плеча каждой из этих сил относительно центра шара). Суммы проекций сил на вертикальное и горизонтальное направления равны нулю (рис. 329). Поэтому N} cos а2 — cos ах = О, P — Nr sin а2 — W2 sin ах = О, где и 2V2 —искомые силы давления.* Отсюда Р Р Ni = -----г—----7----, N2 = —т-----. 1 sin а2+cos «a-tg»! * sin ах + cos artg а2 135. Обозначим через F силу, приложенную к одной ручке. Из-за действия силы F произойдет поворот ящика, вследствие чего возникнут в точках Л и В упругие силы и W2 (рис. 330), дей-' ствующие на ящик со стороны шкафа. Эти приложенные к ящику силы равны друг другу: W 2 = Из условия равенства нулю момента всех действующих сил относительно центра ящика С сле- дует, что N — Fl/Za. Ящик может быть выдвинут, если приложен- 206
ная сила F больше максимальной силы трения покоя: Z7 > + = = 2kN. Для того чтобы последнее неравенство выполнялось, необхо- димо, чтобы k < а/1. 136. Доска, отклоненная на угол а от горизонтального положе- ния на шероховатом бревне, аналогична телу, удерживаемому силами трения на наклонной плоскости с углом при основании а. Поэтому при равновесии FTp = mgsina. Учитывая, что FTp kmg cos a, имеем tgas^Tj. 137. Силы, приложенные к лестнице, изображены на рис. 331. В случае равновесия суммы проекций сил вдоль вертикали и гори- зонтали равны нулю. Следовательно, ^ = FTp, N2 = mg. Равенство нулю суммы моментов сил относительно точки В дает еще одно уравнение: cos a = (mg sin a)/2. Отсюда FTp = (mg tg a)/2. Так как сила трения удовлетворяет нера- венству FTp^/eX^2, то Для равновесия получаем следующее необхо- димое и достаточное условие: tga«C2£. 138. Приложенные к лестнице силы изображены на рис. 332. Из условия равенства нулю суммы сил и суммы моментов сил имеем f+tf2 = mg, (1) #i=FTp, (2) f sin a + cos a —(mg sin a)/2. (3) Силы трения / и FTp удовлетворяют неравенствам f^kN± и Frp^kN2. Используя первое неравенство и уравнения (1) и (3), получаем ctg а //2/2^1 — ^/2. Так как k^Ni/N2t то ctg a ^5 (1 — k2)/2k. Введя обозначение k — данному нера- венству можно придать более удобную для вычислений форму: ctg a ctg 2£, или а 2р. Рис. 332. 139. Если в момент, когда конец палочки В начнет приподни- маться, значение силы трения FTp^kN окажется достаточным для того, чтобы конец А не проскальзывал, то палочка начнет вращаться вокруг точки А. В противном случае конец А начнет скользить до тех пор, пока сила трения Frp=kN не окажется способной удержать 207
палочку в равновесии (рис. 333). После этого палочка начнет вра- щаться вокруг конца А. Найдем, какие значения должен иметь коэффициент трения k, чтобы скольжение прекратилось при определенном угле а между ч палочкой и нитью. Равенство сил в момент, когда палочка почти 4 горизонтальна, приводит к уравнениям Ртр=Т cos а, , P = iV-f-T sin а. Равенство моментов сил относительно точки А запишется следующим образом: Plft — Tl sin а. Используя данную систему уравнений, найдем A»==PTp//V==ctg а. Для того чтобы палочка вообще не начала скользить, необходимо, чтобы k ctg 60° = 1/ /“3. 140. Сумма моментов сил, действующих на человека относительно его центра тяжести, равна нулю. Поэтому сила F, действующая со стороны земли, обязательно направлена к центру тяжести человека С (рис. 334). Горизонтальная составляющая этой силы не может быть больше максимальной силы трения покоя: F silf (X kF cos а. Отсю- Рис. 334. 141. На лестницу действуют сила тяжести Р, сила F со стороны земли и реакция опоры ЛЛ Так как стена гладкая, сила Л/ (рис. 335) перпендикулярна ей. Направление силы F проще всего определить, если найти точку, относительно которой моменты сил Р и Л/ равны нулю. Такой точкой будет точка пересечения прямых CW и ОР. Тогда и момент сцлы F относительно этой точки тоже должен быть равен нулю. Следовательно, сила должна быть направлена таким образом, чтобы ее продолжение прошло через точку О. Как вытекает из рис. 335, направление силы F составляет с лестницей угол Р = 30°—arctg (1/2 ]/~ 3) ^ 14°10'. Сила, действующая на лестницу со стороны земли, будет направлена вдоль лестницы только в том 208
случае, если все остальные силы будут приложены к центру масс лестницы или же будут действовать вдоль нее. 142. Лестницу нельзя удержать от падения с помощью веревки, привязанной к ее середине. Моменты сил реакции пола и стены, а также момент силы натяжения веревки относительно точки О равны нулю при любом натяжении Т (рис. 336). Момент силы тяжести относительно той же точки отличен от нуля. Поэтому лестница обя- зательно упадет. N Рис. 338. 143. Со стороны стены на лестницу действует реакция пер- пендикулярная стене. На нижний конец лестницы действуют силы N2 (реакция опоры) и FTp (сила трения) (рис. 337). Если для простоты не учитывать массу лестни- цы, то кроме этих сил на нее действует еще вес человека Р. Равенство проекций сил по горизон- тали и вертикали дает = и ^ = Ртр. Пусть вначале человек находится в нижней части лестницы (точка Л). Равенство моментов сил отно- сительно точки О приводит к уравнению N1CB = — Р cos а-АО. Следовательно, чем выше находит- ся человек, тем большее значение будет иметь сила Но PTp = /Vi. Поэтому и сила трения, удержи- вающая лестницу, растет по мере того, как чело- век поднимается вверх. Когда Pfp достигнет сво- его максимального значения, равного kP, лестница начнет скользить. 144. При равновесии сумма сил, действующих на картину (рис. 338), равна нулю. Следовательно, ^^тр+Т cos a, W = Tsina. Сила трения долж- на удовлетворять неравенству F^^kN или ^^PTpW. Равенство моментов относительно точки В дает уравнение р ___________ -J-1 sina = T (/ cosa+ ]/d2—-Z2 sin2 a) sin a. 209
Отсюда £тр__ /cosa+2 У^2 —Z2 sin2 ос N ““ I sin a * ‘ Z cosa + 2 —/2 sin2cx • I sin a 145. Найдем первоначально направление силы f, с которой стер- жень ВС действует на стержень CD. Предположим, что у этой силы имеется вертикальная составляющая, направленная вверх. Тогда по третьему закону Ньютона стержень CD действует на стержень ВС с силой, у которой вертикальная составляющая направлена вниз. Однако это противоречит симметрии задачи. Следовательно, верти- кальная составляющая силы / должна быть равна нулю. Сила, дейст- вующая на стержень CD со стороны стержня DE, будет иметь и горизонтальную и вертикальную составляющие, как указано на рис. 339, а. Из равенства нулю-всех сил, действующих на CD, следует, что F=mg и f = f'. Равенство нулю момента сил относительно D дает f sin р • CD — mg CD (cos p)/2, или tg p = mg/2J. На рис. 339,6 изображены силы, действующие на стержень DE. Из условия равенства нулю момента сил относительно Е вытекает f sina*D£ = F cos a>DE-^-mg^~ DE, или tga^=^~» Следовательно, tg a — 3 tg p. 146. Действующие на ящик силы изображены на рис. 340. Усло- вия равновесия имеют вид Q cos a = Ртр и Р = N^Q sin о. В момент нарушения равновесия сила трения достигает максимальной вели- чины Гтр=^. Отсюда Q=kP/(cosa+k sin а). -Величина Q будет минимальной при значении угла а, соответствующем максимуму знаменателя дроби. Для нахождения максимума преобразуем зна- менатель, введя вместо k новую величину (р, так что tgq>=£. Тогда cos a+k sin a = cos (a —- <p)/cos ф, или cos a+£ sin a = 210
= +Zs2 cos (а—(р). Так как максимальное значение cos (a — ф) равно 1, то Qmin = ^71^14-&2- Отсюда k = QI Vе Р2 — Q2 = 0,75. 147. Силы, действующие на цилиндр, изображены на рис. 341. Так как цилиндр поступательно не движется, то Лгр—F cos a=0, F sin a—mg-{-N = 0. Сила трения F^ = kN. Отсюда F = kmg/(c'os a + & sin a). Знамена- тель этого выражения можно представить в виде Лз1п(а + ф), где А== F" 1 + k2. Следовательно, минимальная сила, с которой нужно тянуть нить, = kmg! У 1 + &2. Угол ах найдется из уравнения cos ax + /г sin ax = 'К 1 -f- £2, откуда tgar = ^. 148. Силы, действующие на поршень и заднюю крышку цилиндра, равны F1 — F2 = pS (рис. 342, а). На точку колеса А в горизонталь- ном направлении действует также сила F3, передаваемая от поршня через кривошипно-шатунный.механизм.^ Сумма моментов сил, действующих на колесо, относитёльно его оси равна нулю. (Массой колеса мы пренебрегаем.) Следовательно, FJpR = F2rt где Гтр—-сила трения. Так как сумма сил, действующих Рис. 342. на колесо, также равна нулю, то сила F3, действующая на ось со стороны подшипников паровоза, равна F3 — FTp+F2. По третьему закону Ньютона на паровоз со стороны оси действует сила Г4 = Г3. Следовательно, сила тяги F=iF4t—F1 — FTp=pSr/R. Во втором положении поршня и кривошипно-шатунного меха- низма интересующие нас силы изображены на рис. 342, б. Flv~F2rfR по тем же соображениям, что и в предыдущем случае. Сила тяги F = F^—F^==FT?~pSr/R. Как и следовало ожидать, сила тяги равна силе трения, ибо сила трения — единственная внешняя сила, действующая на паровоз. 211
149. Наибольшая длина выступающей части самого верхнего кирпича равна //2. Центр тяжести двух верхних кирпичей С2 на- ЗР Рис. 343. 2 которое правый край ходится на расстоянии Z/4 от края вто- рого кирпича (рис. 343). Следовательно, на эту длину второй кирпич может быть вы- двинут относительно третьего. Центр тяжести трех верхних кирпичей С3 определяется равенством моментов сил тяжести относительно С3: Р (//2—х) = 2Рх. Отсюда х = //6, Т. е. третий кирпич может выступать над четвертым не более чем на //6. Аналогично можно найти, что четвертый кирпич выступает над пятым на Z/8, и т. д. Характер изменения длины выступа- ющей части с увеличением числа кирпичей очевиден. Максимальное расстояние, на верхнего кирпича может выступать над пра- вым краем кирпича, служащего основанием, записывается в виде ряда l=4(i+4+4’+z+-"); При неограниченном увеличении числа кирпичей эта сумма стремится к бесконечности. Действительно, сумма ряда больше суммы ряда 1/2 1/2 1+4+т+т+4+4'+4+4+• • - а эта последняя сумма заведомо бесконечно велика в случае беско- нечного числа членов. Центр тяжести всех кирпичей проходит через правый край самого нижнего кирпича. Равновесие будет неустойчи- вым. (Данная картина имела бы местб в случае, если бы Земля была плоской.) 150. В окружность радиуса г впишем правильный многоугольник (рис. 344). Найдем далее момент (относительно оси АК) сил тяжести, приложенных к серединам сторон многоугольника АВ, ВС, CD, DE и т. д., считая, что сила тяжести действует перпендикулярно чер- тежу. Этот момент равен р^(АВх14-ВСх2+СРхз4-£>Вх44-£Рхб+РКхв), где р— масса единицы длины проволоки. Рассматривая подобие соответствующих треугольников, можно показать, что произведения ABxlf ВСх^ CDx3 и т. д. равад соот- ветственно АВ'й, B’C'h, C'D'h и т. д., где Л —апофема многоуголь- ника. Таким образом, момент равен pgh (АВ' + В'С' + C'D' + D'E' + EfF' + F'K) = pgh2r. Если число сторон безгранично возрастает, то величийа h стремится 212
к г, а момент—к 2r2pg. С другой стороны, момент равен произве- дению силы тяжести проволоки nrpg на расстояние х центра тяжести от оси АК. Итак, 2r2pg=nrpgx, откуда х = 2г/л. Рис. 344. 151. Разбиваем полукруг на треугольники и сегменты, как показано на рис. 345. Центр тяжести треугольника лежит, как известно, на пересечении медиан. В данном случае центр тяжести каждого треугольника находится на расстоянии 2/3h от точки О (h — апофема). При безграничном увеличении числа сторон центры тяжести треугольников будут ле- жать на окружности радиуса 2/3г, а площади сегментов будут стре- миться к нулю. Таким образом, задача сводится к тому, чтобы оп- ределить центр тяжести полуок- ружности радиуса 2/3г. Из решения задачи 150 следует, что х — расстояние центра тяжести полукруга от точки О — равняется 2 2 4 Х~п 3 Г~3п Г' 152. Применяя метод, изложенный в решении задач 150 и 15L, можно показать, что центр тяжести находится в точке С, располо- _ „ 2 sin (а/2) женной на расстоянии СО=----------- т от центра кривизны дуги (см. рис. 59). 153. Используя решения задач 150, 151, 152, можно показать, что центр тяжести находится в точке С, отстоящей от точки О на 4 sin (а/2) расстояние С0=-т-------v 7 г. о 4 а 154. При определении положения центра тяжести пластинку с вырезом формально можно, рассматривать как сплошную, если 213
считать, что на нее наложен полукруг отрицательной массы, равной по величине массе вырезаемой части. Момент сил тяжести положи- тельной и отрицательной масс относительно оси АВ равен а г яг2 4 \ 1 р^(2г2у— если сила тяжести действует перпендикулярно чертежу, р — масса единицы площади пластинки (см. решение задачи 151). С другой стороны, этот момент равен произведению силы тяжести пластинки на расстояние х—ОС от центра ее тяжести до оси'ЛВ. Следова- 1 2 тельно, xpg(2r2—nr2/2)==-^-r3pg. Отсюда * = ---г г. . о о (4 — Я) § 7. Работа и энергия 155. Работа силы не зависит от маСсы того тела,‘'на которое дей- ствует данная сила. Сила в 30 Н совершит работу Л=ГЛ=150 Дж. Эта работа пойдет на увеличение потенциальной энергии’ (50 Дж) и кинетической энергий (100 Дж) груза. 156. Прежде всего нужно найти силу давления воздуха на одно из полушарий. Представим себе, что его основание закрыто плоской крышкой—диском радиуса R. Тогда, если из получившегося сосуда откачать воздух, сила давления на плоскую крышку будет Fl~pS = = pnR2. Очевидно, такова же будет и сила давления воздуха на полушарие. В противном случае силы взаимно не уравновесятся и сосуд должен будет вечно двигаться в сторону, большей силы. Число лошадей должно быть равно Fj/F, так как другое - полушарие может быть просто привязано к столбу. Натянутая веревка создаст точно- такую же силу, как и упряжка лошадей, тянущая с другой стороны. 157. Изменение количества движения тела равно импульсу силы тяжести. Так как силы, действующие на камень и Землю, равны и действуют одинаковое время, то равны и изменения количества дви- жения этих тел. Изменение кинетической энергии тела равно работе сил тяготения. Силы равны, но пути, пройденные камнем й Землей, обратно пропорциональны их массам. Именно поэтому закон сохране- ния энергии можно записать в форме, не учитывающей изменения кинетической энергии Земли: mv2/2 + W ~ const, где т —масса камня, a W — потенциальная энергия взаимодействия. 158. По закону сохранения энергии /и1§й = /п1п2/2, где т1 — масса копра, h — высота падения копра, скорость копра перед ударом. Ввиду краткрвременности удара сила сопротивления не может заметно изменить общего количества движения системы. Так как удар неупругий, то m1o1=(m1+m2)n2, где т2—масса сваи, v2 — скорость копра и сваи в первый момент после удара. Механическая энергия копра и сваи расходуется на работу против силы сопротив- ления грунта F: к (m1-f-/n2) nJ -s+(mi+^)g5=f5, где S —глубина погружения сваи в грунт. Отсюда " F ’ 4" mlg+ff!lg+J7Isg ~ 24 500 Н— /7*1 ”f 771а о 214
159. В результате неупругого удара линейная скорость ящик& с находящейся й нем пулей в первый момент будет равна и = > где v — скорость пули. Угол отклонения а на основании закона сох- ранения энергии связан со скоростью v соотношением (М-\-т)и2 m2v2 v7/1 ' -2 =2(7rM) = (A1+m)L(|-C0Sg)g- Отсюда _ . a M-\-m _fT- v = 2sin-g- T- V Lg. 160. Ввиду кратковременности взрыва внешние горизонтальные силы (силы трения) не могут за время взрыва заметно изменить общего количества движения системы. До взрыва и сразу после него- оно остается равным нулю. Следовательно, т1р1+/п2у2 = 0. Отсюда v1/v2 = — Так как тележки в конце концов останавливаются, их начальные кинетические энергии тратятся на работу против сил трения: m^/2 = m2v*/2 = km^S2. Отсюда оа/цз = Sj/Sa, и, следовательно, 52=2 м. 161. Обозначим через и скорость тела и тележки после прекра- щения их относительного движения. На основании закона сохране- ния количества движения (M + m)u = Alt>0. (1) Тележка дернет кинетическую энергию ввиду того, что сила трения /, действующая на тележку, совершает отрицательную работу: Mv2/2 — — Mu2l2 = fSt где S—путь, пройденный тележкой. Тело приобретает кинетическую энергию за счет того, что сила трения, действующая на тело, совершает положительную работу: mu2/2 = fs. Здесь s — путь, пройденный телом. Изменение кинетической энергии системы Д4г»2 Г Ми2 /пи2~1 -г-- =r<s-sb <2> как легко видеть, равняется силе трения, умноженной на относитель- ное перемещение тела вдоль тележки. Из уравнений (1) и (2) сле- mMv2 mMv* дует, что = . Так как S-s<l, Mv* Учитывая, что f — kmg, получаем / 2^g(Af+’т)' * 162. В результате сгорания второй порции топлива скорость v ракеты увеличится на величину До. По закону сохранения коли- чества движения (так как сгорание происходит мгновенно) _ (УИ + т)и=Л4 (и +Av)-J-m (v—и), где т— масса порции топ дива, М— масса'ракеты без топлива, и — скорость истечения газов относительно ракеты. Приращение скорости / 215
ракеты Ди=^и не зависит от скорости и ракеты перед сжиганием- второй порции. Напротив, приращение кинетической энергии ракеты (без топлива) А„ Af (v-f-Av)2 Л4и2 ( т . \ = v.2 ----2---mu (jmU+v) будет тем больше, чем больше v. Высота подъема ракеты определяется полученной ею энергией. Поэтому вторую порцию топлива выгоднее сжечь в момент, когда скорость ракеты наибольшая, т. е. непосредственно вслед за первой порцией. При этом наибольшая часть механической энергии, возни- кающей при сгорании топлива, будет сообщена ракете, а механичес- кая энергия продуктов сгорания минимальна. 163. Достаточно рассмотреть последовательное сгорание двух порций топлива. Пусть вначале масса ракеты с топливом равна М + 2т. После сгорания первой порции скорость ракеты v = — тихК2И4-/П), где —скорость газов относительно ракеты. Началь- ная скорость ракеты принимается равной нулю. Приращение ско- рости ракеты после сгорания второй порции Ди = /уи2/Л1, где w2— новое значение скорости газов относительно .ракеты. При сгорании первой порции выделилась механическая энергия ДЕ1 = (Л14-^) u2/2 + mu|/2. При сгорании второй порции выделяется энергия Л4(и4-Ду)2 . m(v — u2)2 (Af + /n)w2 Д^2“ 2 1 Т 2 • . z По условию‘задачи Д£1 = ДЕ2. Отсюда 2 / m2 — f т2 .т\ “х \2(Л14-т) \2М'~Т ) • Следовательно, и± > и2—скорость газов отнокительно ракеты умень- шается. Это связано с уменьшением массы ракеты в процессе сгора- ния топлива. 164. Оба склона можно разбить на множество сколь угодно малых наклонных плоскостей с различными углами наклона. Рассмотрим одну из них (рис. 346). Работа по подъему тела на такой наклон- ной плоскости равна работе против сил тяжести mgkh плюс работа против сил трения FTpAS, Но F^ = kmg cos а и Д5 = Д//соэа 216
Отсюда Гтр Д5 = kmg &l. Общая работа ДА — mg (АЛ + k &1). Если рассмотреть все наклонные плоскости и сложить элементарные работы, то общая работа будет равна А = ДЛ = mg(^ АЛ + k 2 А/) = mgh+kmgl. Работа определяется только высотой горы h и длиной I ее осно- вания. 165. Сила, приложенная к рукоятке, будет минимальна, если она составляет с ней прямой угол. Обозначая через F значение искомой силы, на основании «золотого» правила механики будем иметь 2nRF — Ph. Отсюда F — Ph/2nR. 166. По определению коэффициент полезного действия т) = = Ai/tAiA-Л2), где Ay —PH — работа по поднятию груза Р на высоту Я, а Л2 —работа, совершаемая при этом против сил трения. Так как сила трения способна удержать груз в равновесии, то ра- бота этой силы не может быть меньшей, чем работа Лх. Минималь- ное значение, работы сил трения' 742 = 24!. Следовательно, т]^50%. 167. За время подъема человека по лестнице шар опустится на некоторую высоту Л. Следовательно, работа, совершенная человеком, пойдет на увеличение потенциальной энергии человека на величину mg (/—Л) и на увеличение потенциальной энергии шара на вели- чину mgh (на шар без человека действует подъемная сила mgt на- прав.гёенная вверх); отсюда А — mg (I — h) + mgh = mgl. Этот результат можно получить сразу, рассчитывая работу человека в системе, связанной с лестницей. Если человек поднимается со скоростью v относительно лест- ницы, то относительно Земли он имеет скорость v—vlt где vr—ско- рость шара во время подъема человека. По закону сохранения коли- чества движения (и—Vi)m — Mvv Отсюда М-\~т V' 168. Для того чтобы прогнать за единицу времени вдвое большее количество воды, нужно сообщить вдвое большей массе воды вдвое большую скорость. (Работа мотора идет на сообщение воде кинети- ческой энергии то2/2.) Поэтому мощность мотора должна быть уве- личена в восемь раз. 169. 1) На подъем воды из ямы затрачивается работа A^pg^-S = где р —плотность воды. На сообщение воде кинетической энергии затрачена работа ' 1 н Q 2 ^2=2-Р ~2Sv- Скорость -v, с которой вытекает вода из трубы на поверхность земли, определяется из соотношения (Я/2) 5 = я/?2ут. Полная работа равна л 3 с„2 , 1 Я3£3 А- 8 PgSH +l6Pn2Rixt- 217
2) Работа, затраченная на подъем воды во втором случае, мень- Лше Aj на величину ДЛ'==р^51Л(Я—-Л/2). Работа, затраченная на х „ А, 1 (HS/2-hSJ* сообщение воде кинетическом энергии, равна Л2 =ур—^2^4-2 . Полная работа Л' = Ai ДА'х + Л' . 170. Проще всего решить задачу в системе координат, связанной с эскалатором. Человек пройдет относительно него расстояние I = = A/sina+vT, где vr—путь, пройденный эскалатором. При этом он должен совершить работу A = (h/sin a-j-ur) mg sin a, так как во время подъема сила mg была приложена на пути I и составляла с ним угол 90° —а. Часть работы sngh идет на увеличение потенциальной энергии человека, другая часть работы, mguv sin а, вместе с работой мотора, приводящего эскалатор в движение, идет на преодоление сил трения. ‘ t 171. Энергия пружины, оттянутой за среднюю точку О, равна U71==(2fe}x2/2. Когда пружину отпустили, ее энергия стала равной №2 = £х2/2, так как за время перераспределения упругих деформаций в пружине масса т не успевает сдвинуться. Следовательно, потери энергии в пружине — W2— kx2/2. Это, конечно, грубая оценка. 172. Человек, действуя с силой F на пружину, совершает ра- боту Aj = —FL. Одновременно на пол вагона се стороны человека действует сила трения А Работа этой силы A2=FL. Следовательно^ полная работа, совершенная человеком в системе координат, связан- ной с Землей, равна нулю, так же как н в системе, связанной с поездом. ( * 173. В системе поезда совершенная работа равна потенциальной энергии растянутой пружины А = kl2/2, так как сила трения между человеком и полом вагона в этой системе не совершает работы. В системе, связанной с Землей, работа человека по растяжению пру- жины равна произведению средней силы kl/2 на пройденный путь L — 1, т. е. A1 = (kl!2)(L—‘l). На-пол вагона человек действует с той же средней силой kl/2. Ее работа A2 = (kl/2) L. Полная работа в дан- ной системе координат Л = Ах-|- A2 = kl2/2 та же, что и в системе вагона. 174. На основании законов сохранения количества движения и энергии можно записать следующие уравнения: + m2v2 == т^ + m2v miti« гщ'* 2 + 2 — 2 + 2 ’ где и' и и'—скорости шаров после соударения. Решая данную сис- тему уравнений, получим v, (^1—^2) 0i + 2mavg = (m2 — m1)v2-\-2m1v1 1 т1-\~т2 ’ 2 tfh + m2 1) Если второй шар до удара покоился (п2 = 0), то = (.Щ — При т± > т2 первый шар продолжает двигаться в том же направ- лении, что и до удара, но с меньшей скоростью. Если тг < /и2, то 218
первый шар отскакивает после удара назад. Второй шар будет дви- гаться в ту же сторону, в которую двигался до удара первый шар. 2) Если m1 = m2, то ' v't = 2mo2/2m — у2, у' = 2ту±/2т = . Шары при ударе обмениваются скоростями. 175. Энергия упругой деформации станет максимальной тогда, когда относительная скорость шаров станет равной нулю. Для этого момента времени закон сохранения механической энергии и закон сохранения количества движения можно записать следующим об- разом: mlvl + m2V2 — (т1 4“ m?) U> где и—абсолютная скорость шаров в тот момент, когда они облада- ют максимальной энергией W упругой деформации. Следовательно, ' ц? I т^2 (miV1 + —1>2)2 2*2 2(m1 + m2) 2(nh+nh) 176. В результате упругого соударения левый брусок приобре- тает скорость у. Правый брусок в этот .момент времени еще покоится, так как пружина не деформирована. Обозначим через ut и й2 ско- рости левого и правого брусков в произвольный момент времени, а через х—абсолютное удлинение пружины в тот же момент времени. На основании законов сохранения количества движения и энергии имеем m (u1 + u2) = /nu, m«i t mul t kx2__mu2 ~2 f”“2 г-у— — > или kx2 = m]y2 — (ui+nt)]• Заменяя в последнем уравнении и через «i + u2, получим &х2=2лш1ив. Следовательно* u1u2=£x2/2/n и Из двух последних выражений видно, что и и2 будут иметь один и тот же знак —оба бруска движутся в одну сторону. Величина х2 будет иметь максимальное значение тогда, когда произведение скоростей и± и и2 будет максимальным. Следовательно, для ответа на второй вопрос, поставленный в задаче, нужно найти максимальное значение произведения uxu2 при условии, что сумма «1 + ^2 постоянна и равна у. Рассмотрим очевидное неравенство. (цх —ца)2^0, или а?—-2u1u2 + u2^0. Прибавим Тс правой и левой частям неравенства 4и2и?. Тогда «1+2ц1ц24-н2 4щц2, или (и± + + w2)24uxu2. Так как Wi4-wa — и, то 4щи2^у2. Следовательно, наибольшее значение и±и2 равно v2J4, и оно достигается при и1=м2=ц/2. Расстояние между брусками в этот момент равно I ± *тах = * ± vVm/2k. 177. До тех пор, .пока нижняя пластинка лежит на столе, из закона сохранения механической энергии следует: kx2 ku2 . . /тш2 (1) 219 -
где у—удлинение пружины, ^ — скорость верхней пластинки. В мо- мент отрыва нижней пластинки от стола ky = mg, у = mg Ik, причем v > 0. Учитывая эти соотношения, получаем из (1) my2 kx2 3 m2g2 -=—-mgX-r^->0. (2) Для того чтобы выполнялось (2), должно быть х > 3>mg[k. Скорость vc центра масс в момент отрыва нижней пластинки от стола равна = После отрыва нижней пластинки от стола центр масс будет двигаться вверх равнозамедленно с ускорением g и на- чальной скоростью vc. Следовательно, максимальная высота подъема центра масс равна Я=±^-2^-З^У, 2g 8g 8g \ т 6 k J где Н отсчитывается от положения центра масс в момент отрыва нижней пластинки от стола. (Разобранная задача дает представление о процессах, происхо- дящих при прыжках в высоту.) 178. В системе отсчета, связанной со стенкой, скорость шарика равна v4- и. После удара в той же системе отсчета скорость шарика будет —(у+w). Скорость шарика после удара относительно непод- вижной системы отсчета равна * — (у + u) — и =— (у + 2ц). Кинетическая энергия после удара m(v-\-2u)2/2. Кинетическая энер- гия до удара mv2/2. Изменение кинетической энергии равно 2mu(u-\-'v). Теперь подсчитаем работу упругих сил, действующих на шарик при .ударе. Пусть удар длится время т; предполагаем для про- стоты, что во время удара упругая сила постоянна (результат не зависит от этого предположения). Так как в результате удара количество движения изменилось на величину 2т (v+и), то упругая сила равна F— 2т (и-[-и)/т. Работа этой силы А = FS = Fun — 2m(v-\- и) их/х = 2т (v + и) и'. Как легко видеть, эта работа равна изменению кинетической энергии. 179. 1) До момента, когда веревка натянется, камни падают сво- бодно: S1 = g/2/2, S2 = g (/— т)2/2. Момент натяжения веревки определяется из условия l — S1 — S2. Отсюда /=3с, = 44,1 м, S2 = 4,9m. Время отсчитывается с мо- мента падения первого камня. При натяжении веревки происходит упругий удар, и камни обмениваются скоростями (см. задачу 174). В момент удара —gt — 29,4 м/с, v2 = g(/ —т) = 9,8м/с. Время падения первого камня (после того как веревка натя- нется) находится из условия Л—S1 = o2/j4 gtl/2. 220
Время t2 падения второго камня —из условия h—S2=^1^2 4" g^/2. Отсюда tr « 1,6 с, /2«1,8с. Первый камень падает 4,6 с, второй 2,8 с. 2) В случае неупругой веревки скорости камней после ее натя- жения выравниваются (неупругий удар): и = (у14-у2)/2= 19,6 м/с. Время падения камней после того, как веревка натянется, опреде- ляется уравнениями h—= /2, h — S2=vt2-\-gt2 /2. Si и S2 те же, что и в первом случае. Отсюда « 1,2 с, t2 «3,3 с. Первый камень падает 4,2 с, второй 4,3 с. 180. Если отклонить один правый шар, то после удара слева отскочит крайний левый шар на угол, равный углу отклонения пра- вого шара. Если отклонить одновременно два шара и отпустить их, то после удара слева отскочат два крайних левых шара. Если от- клонить три правых шара, отскочат три левых и т. д. При ударе первого шара о второй первый шар остановится, передав свое количество движения второму шару (см. решение за- дачи 174); второй передаст это же количество движения третьему, третий — четвертому и т. д. У крайнего левого шара нет «соседа» слева, поэтому шар отскочит (если г?ет трения и потерь энергии) на тот же угол, на который был отклонен крайний правый шар. Когда левый шар, после отклонения на максимальный угол, ударит пред- последний шар, процесс передачи количества движения по цепочке шаров повторится в обратном направлении. При отклонении одновременно двух правых шаров они переда- дут свое количество движения цепочке не одновременно, а по оче- реди, через очень малый (неуловимый на взгляд) промежуток вре- мени. Таким образом, цепочка шаров получит не один «двойной» импульс, а два, которые будут распространяться по цепочке с не- которым временным интервалом. Крайний левый шар отскочит, по- лучив «первую порцию» количества движения. Следом за ним Откло- нится его «сосед», получив следующую порцию количества движения, переданную ему от крайнего правого шара. При отклонении трех правых шаров цепочка получит три сле- дующих один за другим через очень малые промежутки времени порции количества движения соответственно от третьего, второго и первого шаров. Если отклонить и одновременно ^отпустить четыре шара, то отскочат слева также четыре шара, а два останутся непо- движными. 181. Ударяющий шарик отскочит назад, следующие шарики до стального останутся неподвижными. Стальной шарик и все после- дующие начнут двигаться влево, причем скорости их будут различны. Наиболее быстро будет двигаться крайний левый шарик. Следующий будет двигаться медленнее и т. д. Шарики разойдутся (см. решение задач 174 и 180). 182. Пусть груз 2т опустился на высоту Н. Тогда грузы т под- нимутся на высоту h (рис. 347). На основании закона сохранения энергии 2mgh-]—g—I—или vf -{-vf — 2g (И — h), 221
NHK и NFK тем где Vi—скорость грузов m, a v2—скорость груза,массы 2т. По мере опускания груза 2т его скорость v2 приближается к скоро- сти так -как углы между участка- ми нити, перекинутыми через блоки, стремится к нулю. В пределе v2 « Одновременно H — h^l. Следова- ' тельнов предельное значение скорости грузов равно 183. Скорости грузов равны, если пути, пройденные ими за равные малые промежутки времени, одина- ковы.-Эти пути одинаковы при таком значении угла AN В, при котором опускание груза на As=W^ (рис. 348) будет сопровождаться увеличени- ем длины участка нити AN В также не величину As. Поэтому при равен- стве скоростей ЯК=ВК—BW = As/2 и FK — AK — AN = ks!2. Треуголь- ближе к прямоугольным, чем меньше мы ' ники выберем отрезок As. При As->0 углы и NFK стремятся к прямым, а углы KNH и KNF — к 30°. Следовательно, скорости будут равны при /ЛМВ=120°. Используя закон сохранения энергии, найдем значения этих скоростей: =2 (2 — У 3) v3. Отсюда v2 = 2gh я о Грузы будут совершать колебания окола положения равновесия, которому соответствует значение ут*ла ANB = 2arccos(/n1/2m2) « 149°. * Углу ANB—120° соответствует максимальное отклонение от поло- ( жения равновесия. ' Н 184. Так как проскальзывание доски по каткам и катков по го- < ризонтальной поверхности отсутствует, то расстояние между осями Ч катков во время движения останется постоянным. Поэтому движение 222
доски будет поступательным. Доска будет перемещаться в горизон- тальном направлении и одновременно двигаться вниз вдоль катков. Если катки сместятся на некоторое расстояние /, то каждая точка доски (в частности, ее центр тяжести Л) пройдет вдоль горизонтали то же расстояние I и одндвременно переместится на это же рас- стояние вдоль катков: АВ — ВС — 1 (рис. 349). (Последнее становится особенно очевидным, если рассмотреть движение катков в системе координат, перемещающейся вместе с катками.) В результате центр тяжести доски будет двигаться вдоль прямой АС, наклоненной к горизонту под углом а/2, так как треугольник АВС равнобедренный. Движение будет равноускоренным. Доска приобретет кинетическую энергию за счет уменьшения потенциальной: mv2/2 = mgl sin а, или t>2 = 2g/sina. С другой стороны, при равноускоренном движении v2 — 2aS, где S = АС = 21 cos (а/2). Следовательно, ускорение а — — y2/2S = gsin (а/2). 185. Подсчитаем разность потенциальных энергий для двух по- ложений цепочки: цепочка полностью лежит на доске и часть це- почки длины х свешивается с доски. Эта разность равна силе тяжести (M/2l)xg свешивающейся части, умноженной на х/2, . поскольку цепочка однородна и центр тяжести свешивающегося конца нахо- дится на расстоянии х/2 от края доски. На основании закона сохра- нения энергии имеем Mv2/2 — (Mg/41) x2t или v = gx2/2l. Ускорение в этот же момент времени можно найти из второ- го закона Ньютона: Ма = (М /21) gx. Следовательно, a=gx/2l. Для подсчета реакции края доски най- -» дем первоначально натяжение цепочки в точке [J соприкосновения с доской. Оно равно произведе- [ нию массы части цепочки, лежащей ,на- доске, на !/• ускорение цепочки: Mxg(2l—х)/4/2. * Рассмотрим теперь очень малый элемент це- Рис. 350. почки, соприкасающийся с краем доски. На этот малый элемент цепочки действуют* три силы (рис. 350). Они вызы- вают изменение его количества движения по горизонтали и верти- кали: (N cos а — F) Ы = — /И у2 Д t/2l, (F — N sin а) Д/ == Мv2 &t/2l. 223
Следовательно, _угол наклона силы # к горизонту а = 45° и М = = Mgx(/—x)V"2/P. 186. Обозначим через v скорость тележки. Горизонтальная со- ставляющая скорости маятника относительно тележки равна и cos 0 (рис. 351), а относительно рельсов равна v + ucos0. В горизонталь- ном направлении на систему внешние силы не действуют. Поэтому на основании закона сохранения количества движения имеем m (u-J-u cos 0) +Л4и = 0, (1) так как вначале система покоилась. Вертикальная составляющая ско- рости маятника относительно тележки и рельсов равна и sin 0. На Рис. 351. основании теоремы Пифагора квадрат скорости маятника относи- тельно рельсов равен (y-f-tt cos 0)24-н2 sin2 0. Используя закон со- хранения энергии, получаем второе уравнение, связывающее ско- ч рости v и и: [(и cos 0 + v)2+ и2 sin2 0^ +-у v2 = mgl (cos 0 —cos a). (2) Из уравнений (1) и (2) можно найти g _ 2m2gl (cos 0 — cos a) cos2 0 V (M sin2 0) В частном случае при 0 = 0 (считая /и/7И<^1) получаем tn2 v2 = 2APg/(,“COSa)’ или -=2Ss4^ 187. Обозначим через v скорость клина, а через их и иу — гори- зонтальную и вертикальную составляющие скорости и бруска отно- сительно неподвижной системы отсчета (рис. 352). На основании законов сохранения количества движения и энергии можно написать: — Лк/-|-/пих = 0, —+у (4+Uy) = mgh. Заметим, что угол а с горизонтальной плоскостью составляет не абсолютная скорость бруска, и (под абсолютной скоростью в данном 224 *
случае понимаем скорость относительно неподвижной горизонтальной плоскости), а относительная скорость н0ТН, т. е. скорость бруска относительно движущегося клина. Из треугольника скоростей (рис. 353) следует, что uy/(v+ux) *= — tg а. Решая данные уравнения относительно и, получим Абсолютная скорость бруска в тот же момент времени равна В случае, когда масса клина много больше массы бруска, и стре- мится, как и следовало ожидать, к величине е» 188. Скорость стержня относительно движущегося клина на- правлена под углом а к горизонту. Если к этой относительной ско- рости прибавить скорость клина, то в результате получим абсолют- ную скорость стержня и (рис. 354). Очевидно, что отношение ско- ростей равно u/y = tga. „ Mv* , ти2 . Из закона сохранения энергии следует: ——|— 8 Б. Б. Буховцев и др. 225
Исключая из этих двух уравнений и, получим выражение для и: v= y^2mghl(M4-.*rrtg2a). Тогда для относительной скорости стержня мы можем написать и * 1 1/ 2m^h отн cos а V A14-mtg2a‘ Скорость стержнд. и 2mgh m + Af tg2a ^gtg2a m + Aftg2a Из последнего выражения видно, что скорость стержня меняется с пройденным путем h по закону равноускоренного движения: г- mjcrtg2a u= у 2ап. Следовательно, ускорение стержня a==^JTjr^rtg2a ’ § 8. Динамика криволинейного движения 189. На основании второго закона Ньютона (М-\-т)и2 zp /лх , ч Л . a -——=Т —(M +m)g, где и = 2 sin -у у Lg JLt Z (см. задачу 159). Отсюда T — (M-}-m)g I 4 sin2 \ 190. T1 = 10mcD2/; Т2==9тш2/; Т3=7тсо2/; Т4 = 4тсо2/. 191. Расстояния от центра тяжести до грузов с массами т1 и т2 равны соответственно Х =----2 I [/ =--£—— I, тг + т2 т1-^т2 Обозначим через и скорость центра тяжести, а через со — угловую скорость вращения. Тогда а+(вх=у1 и ц—сог/ = и2- Отсюда _t>i—о, miVr+m^ I * «i+лц 192. Скорость вращения замедлится. Платформа сообщает сна- ряду дополнительное количество движения по касательной л траек- тории конца ствола пушки. По третьему закону Ньютона вылетающий из ствола снаряд будет оказывать давление- на внутреннюю часть ствола, направленное против вращения. Рис. 355. 193. В момент соприкосновения тела с горизонтальной плос- костью вертикальная и горизонтальная составляющие скорости тела 226
будут иметь значения ^в = У 2gH sin а и vr= )/* 2gff cos а. В случае абсолютно упругого удара вертикальная составляющая изменит знак, а горизонтальная составляющая останется без изменений. Траектория тела будет представлять собой куски парабол (рис. 355), причем h = Н sin2 а, а /==2# sin 2а. Если удар абсолютно неупругий, то вертикальная составляющая скорости станет равной нулю и тело будет двигаться равномерно по горизонтальной плоскости со ско- ростью о== У 2gH cos а. 194. Со стороны земли на мотоцикл действуют две силы (рис. 356): N — реакция опоры и / — сила трения. Сумма этих сил Т направлена вдоль мотоцикла. (В противном случае относительно центра тяжести О действовал бы момент сил, опрокидывающий мотоцикл.) На центр тяжести тела, таким образом, действует результирующая сила F = T + ₽, где Р —сила тяжести. И так как F— Т cos а = /, то центростремительное ускорение мотоциклу сообщает только сила трения /. По второму закону Ньютона f—mv^/R, причем f^kmg. Как видно из рис. 356, mg = ftga. Минимальное значение R из данной системы уравнений равно /?min = v2/kg —147 м, при этом iga — Rg/v* » 3,33 и, следовательно, а « 73°20'. , 195. Рассмотрим промежуточное положение стержня, когда он отклонился от вертикали на угол а. По закону сохранения энергии MgR — MgR cos а + 7Исо2/?2/2, где —расстояние от конца стержня до центра тяжести шара. Отсюда угловая скорость ю выражается следующим образом: о — 2 sin (a/2) Уg/R. При данном а она тем меньше, чем больше R. Следовательно, стер- жень упадет скорее, если он поставлен на конец В. ч Рис. 357. F96. По второму закону Ньютона moPR — mg cos a — Nt где TV —сила, с которой деформированный стержень действует на шар. В момент прекращения давления стержня на пол деформация стержня исчезнет и N — 0. Как показано в задаче 195, © = 2 Уg/R sin (a/2). Подставляя это значение © в уравнение движения, найдем cos a=2/3. Отсюда a = 48°10'. Чтобы стержень не проскользнул, необходимо выполнение условия TV sin а «С kN cos а (рис. 357). Следовательно, k tg а. Отсюда k У Ъ/2. 8* 227
197. Если k > V 5/2, то стержень не проскользнет до тех пор, пока 7V не станет равным нулю, т. е. до тех пор, пока a^arccos (2/3). При а > arc cos (2/3) уравнение m^R — mg cos a—N дает N < 0. Это означает, что если бы конец стержня был прикреплен к полу, то стержень был бы растянут. При незакрепленном стержне шар начнет падать свободно с момента, когда угол достигнет значения aQ = arccos (2/3). В этот момент скорость у = со/? = V2/3gR состав- ляет угол а0 с горизонтом, а высота шара над полом CD=2I2R (рис. 358). Используя законы свободного падения, найдем искомое расстояние: ЛВ = АР + ОВ = /?(5 V'5-f-4 /23)/27 » 1,12/?. 198. На участке ADB (рис. 359) бусинка движется под действием силы тяжести. Для того чтобы она, покинув проволоку в точке Д, , попала в точку В, необходимо, чтобы пройденный ею по горизонтали путь был равен 2R sin а. Для этого скорость бусинки в точке А должна удовлетворять условию (2u2 sin a cos a)/g = 2R sin а (см. задачу 40). Отсюда и2 = gR/cos а. Бусинка будет иметь в точке А скорость «, если в точке О ей сообщена скорость и, равная по закону сохранения энергии v — y/ru2 + 2gR(\4-cos а)= 1/ gR (24-2 cos а 199. Пусть выреза нет. Тогда в точке С (рис. 360) mg=mv2/R. (1) По закону сохранения энергии , mgh = m£2R-\-mv2l2. (2) 228
Из (1) и (2) получаем = Скорость в точке А определится из закона сохранения энергии wg б/2 R = mvA/2 + mgR (1 + cos а). (3) Тело, брошенное под углом а к расстояние AB — (va sin 2a)/g. (4) С другой стороны, AB = 2R sin а.' (5) Из (4) и (5) следует: va = = Rg/cos а. Подставляя это значение в (3), получим + mgR + mgR cos a. Отсюда cos a = (3 ± l)/4, и, следовательно, ах = 0, a2 = 60°. Нетрудно убедиться, что, если а > 60°, тело упадет внутрь петли; если а < 60°, тело вылетит наружу. 200. Рассмотрим силы, действующие на нить, перекинутую через левый гвоздь (рис. 361). Вертикальные составляющие сил натяже- ния 7, действующих на тела, равны mg, если нить закреплена на гвозде. На узел (точку О) по третьему закону Ньютона действуют те же силы Т. Их сумма направлена вертикально вниз и равна 2mg. В случае одного вращающегося тела вертикальная составляющая натяжения нити Т' равна 2mg (если тело не опускается вниз). Но само натяжение нити Т’ > 2mg (рис. 361). Следовательно, наша система не будет в равновесии. Правый груз перетянет. 201. Направление ускорения совпадает с направлением резуль- тирующей силы. Вниз ускорение направлено при крайних верхних положениях В и С шарика (рис. 362). Вверх ускорение направлено при крайнем нижнем положении А и горизонтально при положе- ниях D и L, определяемых углом а. Найдем а. По второму закону Ньютона произведение массы на центростремительное ускорение 229
равно сумме проекций сил на направление радиуса вращения: пшЩ = Т — mg cos а. С другой стороны, как видно из рве. 362, Т = mg/cos а. На осно- вании закона сохранения энергии имеем mv2/2 — mgl cos а. Из этих уравнений находим, что cosa=l/)/' 3, и, следовательно, a « 54°45'. Рис. 362. 202. Обозначим через о угловую скорость стержня в момент прохождения стержня через вертикаль. На основании закона сохра- нения энергии имеем у ("Vi + =g(l — cos a)+ m2r2), или Z ‘ /71x71+m2r; откуда t>x = = 2rx sin /П1Гх + т2Г2 ^х + ^г’ р2==шга=2г2 sin miri + m2rg. ш^г’ + лцг** 203. Равнодействующая сил, приложенных к шарику, F — rngtg a должна создавать центростремительное ускорение а = о2г, где г = / sin а (рис. 363). Отсюда mg tg a=mco2/ sin a. Это уравнение имеет два решения: г «1 = 0, сбз = агссов (g/co2/). 230
Во втором случае имеют место оба решения: ах = 0 (при этом шарик находится в состоянии неустойчивого равновесия) и а2 = 60°. В пер- вом случае имеет место только решение 04 = 0. 204. Силу F, действующую со стороны стержня на груз т, раз- ложим на взаимно перпендикулярные составляющие Т и 2V (рис. 364). Спроецируем силы на вертикаль и горизонталь и запишем уравне- ния Ньютона для этих направлений: mw2/ sin ф = Т sin ф — N cos ф, mg—T cos ф -f- W sin ф. Из этих уравнений определяем Т и Т = т(ы21 эт2ф4-£ cos ф), N = m(g—со2/ cos ф) sin ф. Следовательно, F = КТ2 + № = т /g2 4-w4Psin2<p. 205. Действующие на бусинку силы изображены на рис. 365: / — сила трения, mg—сила тяжести, У — сила нормальной реакции. Уравнения Ньютона для проекций сил на горизонтальное и верти- кальное направления имеют вид mco2/ sin ф = / sin ф — W cos ф, / cos ф+W sin ф = ф£. Из этих уравнений находим, что / = mo)2/ sin2 ф -|_ mg COS ф, N — mgsmq) — тиМ sin ф cos ф. При равновесии или /ио)2/ sin2 ф + mg cos ф < k (mg sin ф — ты21 яп ф cos ф). 231
Отсюда k sin ф—-cos ф g sin ф (sin ф+ k cos ф)(о2 * В частном случае ф = л/2 206. На рис. 366 изображены силы, действующие на грузы. 7\ и Т2 —натяжения нитей. Запишем уравнения Ньютона для проекций на горизонтальное и вертикальное направления: для первого груза: 7\ sin ф — Т2 sin ар = mo2/ sin ф, Тх cos ф — Т2 cos ар — mg — Q; для второго груза: /по2/ (sin ф + sin гр) = Т2 sin ар, Т2 cos ty = mg. Исключив из системы уравнений (1) и (2) 7\ и Т2, приходим к урав- нениям a sin ф = 2 tg ф—tg ар, a (sin ф + sin ар) = tg ар, где Из этих уравнений вытекает, что 2 tg ф —tg ар < tg ар и, следовательно, ф < ар. 207. Действующие на грузы силы изображены на рис. 367. Т19 N-l и Т2, N2 — составляющие сил, действующих со стороны стержня на грузы m и М. и Л/2 направлены в противоположные стороны, так как сумма моментов сил, действующих на стержень, относи- тельно точки О равна нулю вследствие невесомости стержня: N2 (&4-а) = 0. Уравнения движения грузов для проекций на 232
горизонтальное и вертикальное направления имеют вид ты2Ь sin ср = sin ф — cos ф, 7\ cos ф + sin ф — mg, М(д2 (b-\- a) sin ф == Т2 sin ф + М> cos ф, Т2 cos ф —W2 sin Ф = Л1£. Исключив из системы неизвестные Tlt TZi и найдем: 1) Ф = 0; 2) cos ф = to2 mb2-\- M (a-\-b)2 Первое решение справедливо при любых угловых скоростях враще- mbМ (а-\-Ь) . тЬЧМ^+Ф (CU- РеШеН“е ЗЭДа‘ ния, второе — при co^s чи 203). 208. В состоянии равновесия ma>2x — kx, где х — расстояние тела от оси. Отсюда ясно, что при любом х пружина сообщает телу необхо- димое для вращения центростремительное ускорение. Поэтому после толчка тело будет двигаться с постоянной скоростью до упора А или до тех пор, пока для пружины выполняется закон прямой про- порциональности между силой и деформацией. 209. Запишем второй закон Ньютона для малого участка це- почки массы (т//)7?Да, изображенного на рис. 368: (m/l) R Да (2шг)2 R~2T sin (Да/2). Так как угол Да мал, то sin (Да/2) « Да/2; отсюда Т = mln2 »90Н. 210. Выделим малый элемент трубки длиной R Да (рис. 369). Растянутые стенки трубки сообщают жидкости, протекающей по этому элементу, ускорение a = v2/R. По третьему закону Ньютона на элемент трубки со стороны жидкости будет действовать сила а п nd2 в а 1,2 = Да£. где р— плотность жидкости. Сила ДГ уравновешивается силами на- тяжения кольца Т. Из условия равновесия, учитывая, что Да мало, 233
имеем AF = 2T sin (Да/2) а Т Да. Следовательно, искомая сила Т = (pnd2/4) у2. \ / \ / \ / V Рис. 369. 211. Разобьем стержень на п участков одинаковой длины и рассмотрим произвольный участок с номером i (рис. 370). Ускоре- ние различных точек этого участка будет неодинаково, поскольку расстояния точек до оси вращения различны. Однако если разность —г/ мала, то мы можем считать, что ускорение i-ro участка равно со2 (г/+1 + г/)/2, и это тем точнее, чем меньше длина участка. На i-й участок действует упругая сила Тсо стороны деформи- рованного участка i+1 и сила Т/со стороны участка i — 1. Так как масса* i-го участка равна (т//) (г<+1—/7), то на основании второго за- кона Ньютона можно написать Ti-Ti+1 (rz+1 -п) <->2 , или 234
Запишем уравнения движения для участков от п до k включительно, считая, что гл+1 = /, а rk—x: т т — m<j)2 (г2 г2 1 1 п~ 1 п-1—-----21“УП — Гп^1) ^Л + 2“^Л + 1== —(г|+3— rl+l)> Т^Т^^^-Х2). В первом уравнении этой системы учтено, что упругая сила на конец стержня не действует, т. е. Тл+1 = 0. Сложив уравнения си- стемы, получим, что искомое натяжение Тх=^- (12—х2). Чем ближе участки стержня к оси вращения, тем в большей степени они растянуты. 212. В неподвижной относительно оси системе отсчета сила на- тяжения стержня не совершает работы, так как она все время перпендикулярна скорости шарика. В движущейся системе эта сила совершает работу, отличную от нуля, и за счет ее меняется кинети- ческая энергия шарика. 213. Участок обруча АВ массы m обладает в наивысшем поло- жении энергией mg2R-}-m (2и)2/2. При движении кинетическая й потенциальная энергии участка АВ начинают уменьшаться. Умень- шение энергии происходит за счет работы сил упругой деформации Рис. 371. Рис. 372. обруча, равнодействующая которых дает центростремительную силу, направленную всегда к центру. Скорость участка АВ составляет тупой угол а с силой F (рис. 371). Поэтому работа силы Лх = = F AS cos а отрицательна, и, следовательно, уменьшается энергия участка массы т. После того, как участок АВ пройдет крайнее нижнее положение, работа силы F, как легко видеть, станет поло- жительной и энергия участка АВ начнет возрастать. 214. Проведем из точки Л, являющейся «мгновенной осью вра- щения» (см. задачу 57), касательную к внутренней окружности ка- тушки (рис. 372). Если направление нити будет совпадать с направ- 235
лением касательной АС, то момент сил, вращающих катушку отно- сительно мгновенной оси, будет равен нулю. Поэтому покоящаяся катушка не начнет поворачиваться вокруг мгновенной оси, и, сле- довательно, катушка не будет катиться. Значение угла а, при ко- тором происходит изменение направления движения катушки, опре- Рис. 373. деляется из треугольника АОВ: sin а = r/R. Если наклон нити больше а, катушка покатится вправо, если меньше, то влево, при условии, что нет проскальзывания. Если натяжение нити Т удовле- творяет условию Tr^fR, где / — сила трения, то катушка оста- нется неподвижной. В противном случае при sina = r/7? она начнет вращаться на месте против часо- вой стрелки вокруг, точки О. 215. Разобьем весь обруч на равные малые участки массы Дт каждый. Рассмотрим два симмет- ричных (относительно центра) уча- стка. Все частицы обруча участвуют одновременно в двух движениях— поступательном со скоростью у и вращательном со скоростью vA=®R. Результирующая скорость v2 верхнего участка обруча найдется как геометрическая сумма скоростей v и (рис. 373): vl = vl+v2+cos a. Для симметричного участка из = у2 + v2—2vv х cos a. Суммарная кинетическая энергия обоих участков ДЕ = Дт + Дт у2/2 = Д/n у2 4- Д/n о2/?2. Так как это выражение имеет место для любых двух участков, то для всего обруча можно записать E = Mv2/2+MRW/2. Если обруч катится без проскальзывания, то v = (i)R и, следова- тельно, E — Mv2. 2Pv2 216. Е=^у-(лг + /). 217. Цилиндр из более плотного материала, очевидно, будет полым. При одинаковых скоростях поступательного движения кине- тическая энергия вращательного движения будет больше у полого цилиндра, так как частички его массы дальше отстоят от центра и, следовательно, имеют большие скорости. Поэтому при скатывании без проскальзывания с наклонной плоскости полый цилиндр при- обретет меньшую скорость, чем сплошной. Полные кинетические энергии обоих цилиндров в конце пути одинаковы, что возможно только при различных скоростях, так как при одинаковых скоро- стях энергии поступательного движения равны, а энергия враща- 236
тельного движения сплошного цилиндра обязательно меньше, чем полого. 218. Прй движении катушки сила трения не совершает работы, так как нет проскальзывания кабеля и катушки. Следовательно, энергия системы не изменяется: = U2+(p_px) R1 s s где и — искомая скорость. Отсюда и = V(Ру2 + pRgx)/(P — рх) и оказывается равной бесконечности при Р = рх вследствие того, что мы не учитывали массу катушки. Количество движения уменьшает- ся в результате действия силы трения, направленной в сторону, противоположную движению. 219. Так как сила трения постоянна, движение будет равноза- медленным. Развиваемая силой трения мощность равна fv, где у = сог — мгновенная скорость той точки шкива, к которой приложе- на сила f. Работа за время t равна средней мощности, умноженной на время /: A = f<^r+^rt Изменение кинетической энергии шкива равно этой работе: =^(..+0», Отсюда = — 220. Сила трения f постоянна, поэтому изменение количества движения обруча за время t равно mv = ft. В случае качения без проскальзывания скорость точки обруча, к которой приложена сила трения, равна нулю. Приравнивая работу сил трения разно- сти кинетических энергий, имеем (см. задачу 215). Решая уравнение относительно у, найдем: у = соог/2. 221. Уравнения, выражающие изменение количества движения и кинетической энергии обруча, имеют вид ту2 уЛ4-0 где у = (вг—- скорость центра обруча при качении без проскальзыва- ния. Решая эти уравнения относительно у, имеем у = у0/2. Следова- тельно, искомая величина (d = v0/2r. 222. Уравнения, выражающие изменение количества движения и кинетической энергии обруча, имеют вид m(v0—v) = ft, тв>*г* mv2 fa+M+lH-arL 2 2 2 2' 2 237
где v—скорость центра обруча в любой последующий момент вре- мени. Решая данную систему уравнений, найдем v = и0 — (f/m) t, to = to0 — (flmr) t* Если v0 < то в момент времени x — обруч останавливает- ся, вращаясь при этом с угловой скоростью to = to0 — v0/r, Затем обруч начнет двигаться в обратную сторону с проскальзыванием. Спустя некоторое время проскальзывание прекратится и обруч будет катиться без проскальзывания влево с поступательной скоростью i> = (toor — и0)/2 (см. задачу 221). Если же > toor, то через x = mr&df обруч перестанет вра- щаться, передвигаясь вправо с поступательной скоростью у = = ув—-г©0. В дальнейшем вращение обруча будет происходить в обратную сторону, и спустя некоторое время обруч будет катиться без проскальзывания вправо; его угловая скорость to = (y0 —rto0)/2r. Заметим, что, как показывает опыт, обруч тормозится и при отсут- ствии проскальзывания. Мы не получили данного результата, так как не учитывали специфического трения качения. 223. Так как обручи не проскальзывают, то у0, скорость центра тяжести обручей, и у, скорость груза, связаны соотношением R Vo = VR^-r- Пусть груз опустится на высоту h. Считая, что в начальный момент система покоилась, цз закона сохранения энергии имеем mgh = mv2/2 + М у* (см. задачу 215). Из последнего соотношения находим скорость груза; 2mgh --------( R~\*' Отсюда ускорение груза Груз движется вниз с ускорением а под действием двух сил: силы тяжести mg и натяжения нити Т. Искомое натяжение нити Т равно * V Т = m (g-а) =-------V . Так как центр тяжести обруча движется с ускорением, равным r а-=---, под действием силы Т и силы трения F, то на основании К —г второго закона Ньютона для силы F получаем равенство R~r 238
или / Р \ 2 / г\2 т (1—21) 4-2М \R — rJ \RJ Значение силы трения покоя не может Поэтому проскальзывание наступит тогда, -------у-- р "\а > или т + 2М {—££_) —г/ превышать величину kMg. когда '+S- 224. Центр тяжести катушки не будет перемещаться, если натя- жение нити удовлетворяет равенству Т — Mg sin а. Для определения натяжения нити Т найдем ускорение груза массы /п. Пусть груз опустился на h. Так как центр тяжести катушки по условию должен оставаться в покое, то изменение потенциальной энергии равно mgh. Если v—скорость движения груза массы т, то скорость точек ка- тушки, отстоящих на расстояние R от оси вращения, равна vR[r. Следовательно, кинетическая энергия системы равна „ mv* , М& R2 £=—+—75-- Из закона сохранения энергии следует: (m+AfW)^/2=mgft, или —2^2 Отсюда ускорение груза а ходим натяжение нити: mg m+Af/?2/r2* Зная ускорение груза, на- , ч MR*/r2 Таким образом, для sin а получаем выражение 8>п «—M/m+rW Центр тяжести катушки может покоиться только в том случае, если 225. Если скорость доски равна о, то скорость центра тяжести каждого катка v/2 (см. задачу 57). Кинетическая энергия системы (доски и обоих катков) равна М v2 ( 2mv2__M -$-m л “2 1 4“ 2“V’ 239
Приравнивая кинетическую энергию работе силы Q на пути S, по- лучим "+=«>-<№ -/Ж 2 г М-\-т (рис. 374). (Силы трения работы не' совершают, так как проскаль- J зывание отсутствует.) Й Из выражения для скорости движения доски следует, что уско- Ж рение ее равно а~ Я Для определения силы трения, действующей со стороны катка Ж на доску, напишем уравнение движения доски: Ma—Q — 2F. Под- Ж ставляя сюда значение ускорения а, получим F = mQ/2(M+m). S Так как скорость центра тяжести катка в два раза меньше скорости доски, то и ускорение центра тяжести катка будет в два раза, меньше ускорения доски. Поэтому уравнение движения цент- ра тяжести катка будет иметь вид mal2 — F—f. Из этого урав- нения вытекает, что / = 0. 226. Предположим для определенности, что > т2г. В этом случае первый груз будет опускаться, а второй —подниматься. Пусть первый груз опустится на h. Тогда второй груз поднимется на hr{R. Убыль потенциальной энергии при этом будет равна m1gh — m^ghrlR = gh —m^r/R). Если абсолютное значение скорости первого груза v, то скорость второго груза будет равна vrjR. Все точки первой ступени блока имеют скорость у, а все точки второй ступени блока имеют скорость vr!R. Кинетическая энергия системы будет равна 2 v "1 2 № V' Из закона сохранения энергии следует: mt + Mi тг + М2 г2 г\. —------°2+—2------ R2 или 2(ml—m<tr/R)gh Отсюда ускорение первого груза _______«1—_____________ 1 (mx+ Mx)+(m2+ Mt) r2/R* 8‘ 240
Из соотношения a^a^ — RIr, где а2—ускорение второго груза, найдем я2=;----г?/ \~~гХ । \ 2/D2 • Натяжения нитей 7\ и Т2 на осно- ("h4- Afx) + (m2+ Л42) r2/R2 вании второго закона Ньютона равны г г2 Mi + mt -^-+-^-(т2 + Л1а) 7'1=---------------------— mig> т1 + М1+(т2+М2)-^~ Л =---------------*-------т&- mi + М х 4- (т2 + Л12) Сила F, с которой система действует на ось блока, равна /7 = Г1 + Т2 + (М1 + 7И2)^ 227. Пусть путь, пройденный время /, равен S, а скорость цент- ра тяжести к этому моменту вре- мени равна v (рис. 375). Тогда на основании закона сохранения энергии имеем Mv2 — MgS sin а. Отсюда скорость равна и = = TrgSsina, и, следовательно, ускорение a=gsina/2. Скорость центром тяжести цилиндра за центра тяжести цилиндра и угловая скорость его вращения будут равны v = (g sin a/2) /, со — (g sin a/27?) t § 9. Закон всемирного тяготения 228. По второму закону Ньютона mig = F, где пц—инертная масса, т. е. величина, характеризующая способность тел приобретать то или иное ускорение под влиянием определенной силы. С другой стороны, по закону всемирного тяготения F = ymgMg/R2, где коэф- фициент пропорциональности у —так называемая гравитационная постоянная, а mg и Mg—гравитационные массы взаимодействующих тел. Гравитационная масса определяет силу гравитационного притя- жения и в этом смысле может быть названа «гравитационным за- рядом». Заранее не очевидно, что mi-nig. Однако лишь при выполнении этого равенства (достаточно и пропорциональности) ускорение свобод ного падения одинаково для всех тел, так как при подстановке силы тяготения во второй закон Ньютона массы т/ и mg можно сократить, и g = yM{R2. Только сила тяготения сообщает всем телам одинако- вые ускорения, не зависящие от их масс. 229. Ускорение g — yM/R2 (см. задачу 228). Принимая g = — 9,81 м/с2, найдем у = 6,67* 10“11 м3/кг*с2. 230. Тела внутри космического корабля перестанут оказывать Давление на его стенки, если они будут иметь такое же ускорение как и корабль. Одинаковое ускорение в данном участке простран 241
ства всем телам независимо от их массы может сообщить только сила тяготения. Следовательно, необходимо, чтобы двигатель корабля был выключен и* сопротивление внешней среды отсутствовало. Дви- жение же корабля может происходить в любом направлении по отношению к направлению сил тяготения. 231. Сила тяжести сообщает одинаковое ускорение маятнику и рамке. Никаких деформаций за счет тяготения при свободном паде- нии в системе не возникает. Поэтому относительно рамки маятник будет двигаться так, как если бы тяготения не было (см. решение задачи 230). Он будет вращаться с постоянной угловой скоростью до тех пор, пока длится падение рамки. 232. На участке ВСА (рис. 376) сила тяготения совершает по- ложительную работу (угол 0j острый) и, следовательно, скорость планеты возрастает. В точке А скорость достигает своего максималь- ного значения. На участке ADB сила тяготения совершает отрица- тельную работу (угол 02 тупой), и, следовательно, при движении по этому участку скорость планеты убывает, "достигая минимального значения в точке В. 233. Чтобы спутник двигался по замкнутой орбите (окружности радиуса /?+Л), на него должна действовать сила, направленная к центру. Этой силой в данном случае является сила притяжения Земли. По второму закону Ньютона ту2/(7? 4- h)=утЛ4/(₽ 4- й)2, где М —масса Земли, 7? = 6370 км —радиус земного шара, у —гра- витационная постоянная. На поверхности Земли ymM/R2 = mg. Сле- 234. Под. влиянием сопротивления атмосферы спутник постепенно с течением времени приближается к Земле. Радиус его орбиты умень- шается. Так как в верхних слоях сопротивление' мало, то за один оборот это уменьшение радиуса незначительно. Считая орбиту при- ближенно круговой, можно записать m&IR—ymM/R\ где R — радиус орбиты. Отсюда v— VуМ/Я, т. е. скорость спутника возрастает при уменьшении R. Наглядно этот результат можно пояснить следующим образом. Вследствие сопротивления атмосферы движение спутника, выведенного, например, на круговую орбиту 242 , >
(пунктир на рис. 377), будет в действительности происходить по не- которой спирали (сплошная линия на рис! 377). Поэтому проекция F силы Тяготения на направление скорости спутника v отлична от нуля. Работа силы F (сила F больше силы сопротивления атмо- сферы f) и приводит к увеличению скорости. При движении в атмосфере полная механическая энергия спут- ника уменьшается, но потенциальная энергия при приблйжении к Земле убывает быстрее, чем полная. За счет этого кинетическая энергия растет. Следует подчеркнуть, что в плотных слоях атмо- сферы из-за большой величины силы сопротивления мы не можем даже приближенно рассматривать движение спутника как вращение по окружности, и наш вывод несправедлив. 235. Если бросить контейнер против движения спутника Л, то он начнет двигаться по некоторому эллипсу 2, расположенному внутри орбиты спутника (рис. 378). Период обращения контейнера будет немного меньше периода обращения спутника В. Поэтому они могут встретиться в точке соприкосновения орбит только после со- вершения большого числа оборотов. Контейнер нужно бросить в направлении движения спутника А. Он начнет двигаться по эллипсу 3. Скорость и нужно подобрать так, чтобы за время одного оборота контейнера спутник В также сделал один оборот и дополнительно прошел путь АВ. Это вполне возможно, так как период обращения по эллипсу 3 несколько больше периода обращения по круговой орбите /. Контейнер встретится со спутником в точке соприкосновения орбит 3 и /. 236. Считая приближенно орбиту Земли круговой, для силы тяготения можно записать выражение F = rm^R, где т —масса Земли, а (й — 2я1Т— угловая скорость Земли (Т — 365 дней).' С дру- гой стороны, согласно закону тяготения F — ymM/R2, где М—масса Солнца. Отсюда ^mM/Rz==m&2Rt или М = й)2/?3/у ~ 2-1030 кг. 237. Поскольку как Луна, так и спутник движутся в поле тя- жести Земли, применим третий закон Кеплера: Т?/Т| = (Л+Я + 2/?о)8/8₽3 (рис. 379). Отсюда ft=2R(T1/7’2)‘/« —Я—2«,=220 км. 243
238. Так как масса шарика больше массы воды в том же объеме, то поле тяготения больше вблизи шарика, чем вдали от него. Соот- ветственно вода возле шарика дополнительно сжата. Сила давления жидкости, действующая на пузырек слева, будет несколько меньше, чем сила, действующая справа. С другой стороны, сила тяготения между воздухом в пузырьке и шариком больше силы притяжения между воздухом и выделенным пунктиром объемом воды (объем а на рис. 380). Так как масса воз- духа в пузырьке очень мала, то решающим оказывается действие первого фактора. Пузырек будет отталкиваться от шарика. Движение железного шарика, напротив, будет определяться тем, Рис. 380. что сила притяжения между воздухом в пузырьке и шариком много меньше силы притяжения между шариком и выделенным пунктиром объемом воды (рис. 380, объем Ь), Вычисление силы проводится путем следующих рассуждений. В однородной среде (вода) имеется сфера с почти полным отсутствием массы (пузырек) и сфера с избы- точной массой (шарик). Формально это можно рассматривать как наличие отрицательной и положительной масс. Сила взаимодействия между сферами в жидкости равна силе взаимодействия в пустоте отрицательной массы, равной массе воды в объеме пузырька, и положительной массы, равной избытку массы железного шарика над массой воды в том же объеме. Следовательно, F== — — m^/R2. Здесь т1 — масса воды в сфере радиуса г, т2 —масса железного шарика. 239. Вблизи пузырька поле тяготения меньше, чем в однородной жидкости. Жидкость соответственно сжата здесь меньше. Поэтому в участок жидкости возле одного пузырька устремляется другой, и наоборот. Пузырьки будут притягиваться. Два пузырька в одно- родной жидкости, массы которых пренебрежимо малы, можно рас- сматривать формально как отрицательные массы, наложенные на положительную массу т среды в объеме пузырька: F = у (— tn) (— m)!R2 = ym2lR2. 240. Если бы шар был* сплошным, то сила тяготения Ft — уМтЦ2, где М = 4/3л/?3р — масса шара без полости. Наличие полости экви- валентно появлению силы отталкивания F2 — ym'm[S\ где т' ==Л/Злг3р, a S — расстояние между центром полости и материальной точкой. 244
Искомая сила F является геометрической суммой сил и Г2 (рис. 381). По теореме косинусов F = Vf? + FI - ZFtFi cos р = 4 //?« re 2/?3r3cosP — 3 ЯТтР У Z4 + (P_rf2)2 /2(/2_d2) 5,7.10-«H. 241. Искомая сила притяжения будет являться геометрической суммой сил притяжения, создаваемых отдельными элементами сферы. Малые элементы ах и а2 (рис. 382) вырезаются из сферы конусами с вершиной в точке Л, которые получаются при вращении образующей ВС вокруг оси 3Х32. Площади элементов равны соответственно (ЛЗх)2й)/со8 ах и (AS2)2co/cos а2, а их массы (ЛЗ^2 cop/cos ах и (Л32)2сор/соз а2, где со —телесный угол, под которым видны оба элемента из точки А; р — поверхностная плотность сферы (масса, приходящаяся на единицу площади); Z ai= Z а2» так как треуголь- ник ЗдО32 равнобедренный. Силы притяжения, создаваемые элемен- тами, соответственно равны т (ЛЗХ)2 сор_ тсор т (Л32)2сор_________ тсор ^(ЛЗХ)2 cos ах””^ cos ах ’ (Л32)2 cos cos а/ где т — масса тела, и направлены в противоположные стороны. Йх равнодействующая равна нулю. Проводя аналогичные рассуждения для других соответствующих элементов сферы, убеждаемся, что все они попарно компенсируют Друг Друга. Следовательно, сила притяжения, которая действует со стороны сферы на тело, помещенное внутри нее, равна нулю. Заме- тим, что данный результат справедлив и для сферы конечной тол- щины, так как ее можно разбить на сколь угодно тонкие сферические оболочки, для каждой из которых справедливо доказанное выше утверждение. 242. Сила притяжения равна силе, с которой тело массы т притягивается к шару радиуса г и плотности р. Внешние слои толщи Земли не оказывают, как доказано в задаче 241, на тело никакого 245
действия. Поэтому искомая сила „ (4л/3) рг3/и 4л f = r /2 —=Т -3-риг. Эта сила убывает пропорционально г по мере приближения к центру Земли. § 10. Гидро- и аэростатика 243. Уровень воды не изменится, так как количество вытеснен- ной воды остается тем же. 244. Равновесие не нарушится, так как согласно закону Паскаля давление на дно сосуда будет всюду одинаковым. 245. 1) Так как кусок льда плавает, вес воды, вытесненной им, равен весу самого льда или получившейся из него воды. Поэтому вода, образовавшаяся после таяния льда, займет объем, равный объему погруженной части куска, и, следовательно, уровень воды не изменится. 2) Объем погруженной части куска с камнем больше суммы объемов камня и воды, получившейся после таяния льда. Следова- тельно, уровень воды в стакане понизится. 3) Вес вытесненной воды равен весу льда (вес воздуха в пу- зырьке можно не принимать во внимание). Поэтому, как и в слу- чае 1), уровень воды не изменится. 246. Вес тела, погруженного в жидкость, в первом случае равен P1==(d—di)V\ во втором случае P* — (d—d2)Vt где V—объем тела. Отсюда d = (P2dl-P1d2)/(P2-P1). 247. Только в небольших водоемах лед может удерживаться на весу с помощью береговой кромки, В центре большого озера он обязательно плавает. Отношение плотностей льда и воды равно 0,9. Следовательно, 0,9 всей толщины льда находится в воде. Расстояние от поверхности льда до воды равно 1м. 248. После снятия камня коробка стала легче на вес камня, и, следовательно, объем вытесненной ею воды уменьшился на вели- чину V1 = P/d1, где Р—-вес камня, dx — удельный вес воды. При погружении в воду камень вытеснит объем воды, равный своему объему V2 = P/d2, где d2 — удельный вес вещества камня. Так как d2 > dlt то Vi> F2. Следовательно, уровень воды в чаше понизится. 249. В обоих случаях работа насосов одинакова, так как одно и то же количество перекачанной воды поднимается на одну и ту же высоту. 250. Г-образная фигура устойчиво стоит на дне пустого сосуда, так как перпендикуляр, опущенный из центра тяжести фигуры, не выходит за пределы площади опоры. По мере наливания воды в со- суд будет расти выталкивающая сила, действующая на прямоуголь- ник (предполагается, что вода имеет возможность подтекать под фигуру). При глубине воды в сосуде, равной 0,5а, сумма моментов сил, стремящихся повернуть тело по часовой стрелке, будет равна сумме моментов сил, стремящихся повернуть тело против часовой стрелки. При дальнейшем заполнении сосуда фигура упадет. 246
251. Длина трубы х найдется из условия dx = d0(x—h), выра- жающего равенство давлений на глубине нижнего конца трубы. Здесь d'o —удельный вес воды. Отсюда Z=do/i/(do—d) = 50 см. 252. Давление на дно равно р— pg(H4-Л) (рис. 383). С другой стороны, так как сосуд цилиндрический p — (P-\-mg)/nR2. Высоту h можно определить, приравнивая друг Другу силы, действующие на поршень: pghn(R2— — г2) = Р. Отсюда „ _ 1 < Р г2 л/?2р \т g R*-r* CM. 253. Используя закон сохранения, энергии и закон Архимеда, приходим к уравнению mgx=-. где р —плотность воды, а х — искомая высота .Отсюда Jc = (4/3n/?8p — —т) him. 254. Из равенства моментов относительно точки А (рис. 384) действующих на доску сил имеем Pi (1—а—х/2) cosa = Р (//2—a) cos а, где Px = SxdQt a P = Sldt S —площадь поперечного сечения доски, d0 — удельный вес воды. Отсюда х = (/-а)± /(/-a)a-(d/d0)Z(/-2a). Так как х< 1 — а, то имеет смысл только одно решение: 255. Человек не достиг своей цели, так как, увеличив выталки- вающую силу, он вместе с тем более значительно увеличил вес своей ноши (плотность сжатого воздуха в камере больше плотности наруж- ного воздуха). 256. Показания весов .увеличатся, если средняя плотность взве- шиваемого тела меньше плотности разновесок. Показания весов уменьшатся, если средняя плотность тела больше плотности разно- весок. В случае, если разновески и тело имеют одинаковую среднюю плотность, равновесие весов не нарушится. 257. Истинная масса тела М = Л*! + do (V - Мj/di) » 801,16 гг 247
Допущенная относительная ошибка в процентах равна лд__м 100% ~ °>*4%. 258. Нормальное атмосферное давление равно приблизительно 10® Па. Значит, вес атмосферного столба воздуха площадью в 1 м2 равен 10® Н. Зная поверхность земного шара, можно подсчитать массу всей атмосферы Земли. Поверхность Земли 5 = 4л/?2, где # = 6370 км —средний радиус Земли. Масса атмосферы М « 4лР2х Х1 кг/см2 « 5*1015 тонн. 259. Представим себе, что внутренность бутылки заполнена стеклом. Если производить давление на наружную поверхность, то такое же давление возникнет во всех участках внутри стекла. При этом произойдет сжатие и объем внутренней части бутылки умень- шится. Не важно, что производит давление на внутреннюю поверх- ность бутылки —вода или стекло, заполняющее ее внутренность. Если бутылка подвергнется с наружной и внутренней стороны дав- лению р, то вместимость ее уменьшится. 260. Давление жидкости в точке D равно нулю, а в точке А равно pgh. Так как давление на боковую стенку линейно возрастает, то среднее давление равно Рсред—(0+ р^)/2. Сила, с которой действует жидкость на бокрвую наклонную стенку, равна 2 sin а г 2 sin а Сила Д с которой боковые стенки действуют на дно, направлена вверх и равна f — 2F cos а = pgh2b ctg а. (Вертикальная составляющая полной силы, действующей на дно сосуда, будет равна, разумеется, силе тяжести налитой жидкости. В самом деле, pghab — f = pghb (a — h ctg a) = pgV, где V—объем налитой жидкости.) 261. Сила, с которой жидкость поднимает вверх сосуд, равна F = n(R2 — r2) pgh. Следовательно, Я(^-г2)р^Л = Р, 262. Давление на «дно» сосуда равно pgh. Сила, с которой заштрихованная часть жидкости (рис. 385) давит на стол, равна pghn(2Rhtga--h2tg2a). По третьему закону Ньютона такая же сила действует на жидкость. Условие равновесия жидкости в момент, когда сосуд перестает давить на стол, имеет вид Р + = pghn (2Rh tg a - Л2 tg2 a), 248
где Pi —вес заштрихованной части жидкости (усеченный конус минус объем цилиндра): Рх=-2^- {л/?2 + + л(Р —/itga)2 + ^P(P —htg a)} — — pg/иг (/^— h tg a)2. Отсюда __________3P__________ rcgh2 tg a(3P —htg a) ’ 263. В цилиндрическом сосуде дно отпадет во всех трех случаях, так как сила давления на дно сосуда сверху будет каждый раз одной и той же. В сосуде, суживающемся Рис. 385. кверху, дно отпадет только при наливании масла, 'Так как уровень масла здесь будет выше, чем в цилиндрическом сосуде. В сосуде, расширяющемся кверху, дно отпадет при наливании ртути, которая будет стоять выше, чем в цилиндрическом сосуде, а также при опускании гири, вес которой распределится в данном случае на меньшую площадь, чем в осталь- ных двух случаях. 264. Если уровень воды в сосудах одинаков, то и уровень ртути до того, как положили кусочек дерева, будет одинаков. Внесение в сосуд кусочка дерева совершенно равносильно доливанию количества воды, которое вытесняется этим кусочком, т. е. количества воды, рав- ного ему по весу. Следовательно, если сечение сосудов одинаковое, уровни воды и ртути в обоих сосудах будут совпадать. Если же сечения не одинаковы, то вода будет стоять выше, а ртуть ниже в том сосуде, сечение которого меньше. Это произойдет потому, что добавление одинаковых по весу (и по объему) количеств воды в сосуды с разным сечением приведет к различному увеличению давле- ния на поверхность ртути. 265. После опускания кубика во второй сосуд уровень ртути в обоих сосудах повысится на величину х и займет положение АВ (рис. 386). Необходимая высота столба воды во втором сосуде опре- деляется равенством давлений, например, на уровне CD: (y+x)pig— =/jP2g, где рг— плотность ртути, р2 —плотность воды, у можно найти из условия сохранения объема ртути: (x+t/)S1 = V2, где V2— объем ртути, вытесняемый кубиком после вливания воды. 249
Если вода целиком покрывает кубик, то по закону Архимеда Vopog = V2Pig+ (Vo - V2) p2g, где p0 —плотность железа. Решая написанные уравнения, получим А=Р1 (Ро — Р2Ж«/р2 ((h—р2) Если же вода не покрывает кубик, то закон Архимеда запишется в виде VoPog==V2pig+№()£, где S = V*/s — площадь грани кубика. В этом случае искомая высота /ip0V0/p2 + V*/3). Первое решение справедливо при Si=C Pi (po-p2)V2/8 Р2(Р1-Ро) ’ Н Р2(Р1-Ро) ° 266. При изменении атмосферного давления архимедова сила, действующая на барометры со стороны воздуха, меняется как из-за изменения плотности воздуха, так и из-за изменения объема баро- метров при изменении уровней ртути в их открытых частях. При учете всех условий задачи барометры имеют не только оди- наковый вес, но и одинаковый объем. Поэтому для каждого из них изменение выталкивающей силы из-за первой причины одинаково. Изменение же объемов будет различно. В U-образном барометре для изменения разности уровней на определенную величину уровень ртути в каждом колене должен измениться только на половину этой величины. В чашечном барометре уровень ртути в чаше меняется незначительно, а в трубкепрактически на всю величину изменения разностй уровней. При этом, на сколько изменится объем ртути в трубке, на столько же он должен измениться и в чаше. Следова- тельно, для чашечного барометра изменение объема будет вдвое больше, чем для U-образного (при одинаковых диаметрах трубок). При повышении давления объем чашечного барометра станет меньше объема U-образного, сила Архимеда, действующая на чашечный барометр, также станет меньше, и поэтому он перевесит. 267. В случае, когда человек станет на матрац, его вес распре- делится на меньшую площадь (площадь ступней), чем в случае, когда он ляжет. Поэтому состояние равновесия наступит в первом случае при большем давлении воздуха в матраце, чем во втором. 268. Рассмотрим первоначально накачанную воздухом камеру (рис. 387, а, изображающий камеру в разрезе). Для равновесия участков камеры АВ и CD, очевидно, необходимо, чтобы натяжение растянутых стенок камеры Т уравновешивало избыточное* давление внутри камеры р. Теперь рассмотрим силы, действующие на участки камеры АВ и CD в том случае, когда камера надета на нагружен- ное колесо (рис. 387,6). В верхней части камеры распределение сил, действующих на участок АВ, существенным образов не меняется. Внизу положение будет теперь иным. На участок CD будет действо- вать упругая сила со стороны обода, равная нагрузке, приложенной к колесу (вес колеса и четверть веса автомобиля). Под влиянием этой дополнительной силы камера сплющивается и угол между силами натяжения резины Т увеличивается. Суммарная сила натя- жения, действующая на участок CD, уменьшается, и поэтому избы- точное давление воздуха в камере уравновешивает как силу натя- жения, так я вес колеса и части автомобиля. 250
Итак, обод не падает вниз, так как он поддерживается избы- точным давлением воздуха в камере. В верхней части камеры это избыточное давление уравновешивается натяжением стенок камеры, в нижней же части оно уравновешивает как уменьшившееся натя- 269. Сила на единицу длины, с которой растянута стенка цилин- дрической части котла в направлении, перпендикулярном оси котла ООЪ равна f± = (2RI/21}р — pR, где 2RI — площадь сечения котла ABCD, а р—давление внутри котла (рис. 388); 2/?/р—сила, дей- ствующая на половину цилиндра (см. задачу 156). Максимальное значение силы, приходящейся на единицу длины сферических днищ, можно найти по формуле f2=(aR*/2nR) р =» pR/2 = Л/2. Следовательно, сферические днища могут выдержать вдвое большее давление, чем цилиндрическая часть котла (при одинаковой толщине стенок). Чтобы прочность котла была одинаковой во всех частях, толщина днищ может быть в два раза меньше толщины цилиндри- ческих стенок, т. е. 0,25 см. 270. Форма котла должна быть такой, чтобы сила, приходящаяся на единицу длины сечения котлн, была на- именьшей. Эта сила равна f — pS/l, где S — площадь сечения котла, / — периметр сечения, р—давление пара. Сила f будет минимальной, если отношение площади сечения к периметру сечения минимально. Как известно, это отно- шение имеет минимальное значение для круга. Известно также, что сечение сферы любой пло- скостью дает круг. Поэтому наивыгоднёйшая форма котла в смысле прочности—это сфера. 271. Выделим внутри жидкости столбик высотой h (рис. 389). Уравнение движения этого столбика имеет вид та—mg—pSt где m — pSh—масса жидкости, а р—давление на глубине h. Следова- тельно, p = p&(g—а). 251
272. В соответствии с решением задачи 271 выталкивающая сила может быть записана следующим образом: F = pV(g — а), где V— объем погруженной части тела. Уравнение движения плавающего тела массы М имеет вид Ma — Mg—pV(g—а). Отсюда V = Af/p, как и в неподвижном сосуде. Тело не всплывет. 273. Давление жидкости на крышку цистерны на расстоянии х от передней стенки равно р — рха. Так как это давление линейно возрастает по мере удаления от передней стенки, то искомая сила „ pa/ + 0., l2d равна F-—~—/d = p-^-a. 274. F = phldg + p(l2d/2)a. 275. Если бы бак покоился или двигался равномерно, то дав- ление на глубине h равнялось бы p^pgh. С другой стороны, если бы бак двигался ускоренно, а сила тяжести отсутствовала, то дав- ление в точке А было бы равно р2 = ра/. Именно такое давление, в соответствии со вторым законом Ньютона, сообщило бы столбику жидкости длины I требуемое ускорение а. При ускоренном движении бака в поле тяжести возникает как давление рх, так и давление р2. Согласно закону Паскаля давление в жидкости одинаково по всем направлениям. Поэтому давления и р2 складываются, и результи- рующее давление в точке А равно р = р(gh-]-al). 276. Имеем (рис. 390) . a H—h а tg а——7==- , —< ....... VV-1-а2 1/2 /й2+а2 277. Чтобы жидкость не выливалась, сосуду должно быть сооб- щено такое ускорение, при котором поверхность жидкости займет /V положение, изображенное на рис.*391. Максимальный объем жидкости bcS равен bcS/ll. Масса всей системы равна М-\-—р. Необходимое ус- корение определяется из условия, что сумма сил, действующих на малый элемент жидкости массы Ат у поверхности, направлена гори- зонтально (рис. 391). По второму закону Ньютона A/n а — кт gig а. Следовательно, искомая сила г ( м । bcS \ ь 278. Движение жидкости в сифоне обеспечивается силами сцеп- ления между элементами жидкости. Жидкость в длинном колене перевешивает жидкость в коротком колене, что и приводит к ее пе- 252
рекачке. На основании этого можно было бы предположить, что с помощью сифона можно перекачивать воду через стенку любой вы- соты. Однако это не так. При высоте подъема в 10 м давление внутри жидкости становится равным нулю. При этом пузырьки воз* духа, всегда имеющиеся в воде, начнут расширяться и водяной столб окажется разорванным. Как только это произойдет, сифон переста- нет работать. 279. Сначала прибор будет действовать, как сифон. Вода будет выливаться через узкую трубку в водоем. Затем через А проскочит пузырек воздуха и разделит в верхнем колене жидкость на две части. Жидкость после этого перестанет выливаться. 280. Давление воды непосредственно под поршнем каждого насоса меньше атмосферного на величину pg(H-}-h), где р —плот- ность воды. Поэтому, чтобы удержать поршень, нужно тянуть его кверху с силой F = pg (Н-]-h) S, где S—площадь поршня. Следова- тельно, с большей силой необходимо тянуть те поршни, площадь которых больше. 281. В нижней половине камеры заполняются более плотным воздухом. В верхней половине он покидает камеры. Вследствие этого давление постепенно выравнивается, и машина будет работать лишь до тех пор, пока разности давлений между половинами сосуда будет достаточно для поднятия воды по трубке в верхнюю полови- ну сосуда. 282. В данном случае колесо не симметрично и давление воз- духа на правую часть колеса больше, чем на левую. Избыточная сила давления, действующая на правую часть колеса, равна F — (pi — p2)St где S —площадь поперечного сечения камеры. Вес же заполненных водой камер не может превышать P — pgSh. Так как h » то F^P. Колесо начнет вращаться против часовой стрелки. Поэтому камеры будут подниматься из нижней части сосуда в верхнюю наполненные воздухом. Колесо будет вра- щаться против часовой стрелки до тех пор, пока уменьшившаяся разность давлений не станет недостаточной для того, чтобы поднять воду на высоту h. 283. «Потолок» стратостата определяется не максимальной высо- той, на которую может подняться стратостат, а той высотой, при спуске с которой обеспечивается «безопасная» скорость приземления. Как известно, оболочку стратостата заполняют легким газом (водо- родом или гелием) только частично, так как по мере подъема стра- тостата заполняющий его оболочку газ расширяется, вытесняя из оболочки воздух. Это позволяет поддерживать подъемную силу при- мерно постоянной. На некоторой высоте газ заполнит весь объем оболочки. Однако и после этого подъемная сила стратостата продол- жает возрастать за счет вытекающего из нижнего отверстия оболочки газа. При этом вес стратостата уменьшается. Только после утечки некоторого количества газа из оболочки стратостат достигнет «по- толка». Для спуска стратостата Необходимо дополнительно выпустить некоторое количество газа через верхний клапан оболочки. Для плавного спуска подъемная сила должна быть лишь немного меньше веса стратостата. На малой высоте скорость снижения окажется слишком большой, так как объем газа уменьшится, а количество его будет меньшим, чем при подъеме. Сбрасыванием балласта достигают уменьшения скорости снижения. 253
§11. Гидро- и аэродинамика 284. Обозначим через h расстояние от_уровня воды до верхнего отверстия, через х—искомое расстояние от сосуда до точки пересе- чения струй по горизонтали и через у — расстояние от уровня воды в сосуде до той же точки (рис. 392). Точка пересечения струй будет оставаться на одном месте, если уровень воды в сосуде не изменяется. Для этого необходимо, чтобы Q = Sy1 + Su2, где и у2 — = (Я+А)—скорости истечения струй из отверстий. На основании законов кинематики x=v1/1 = v,/a> у = Л+г/1/2=Л + Я4-^1/2, где и —времена «падения» воды от отверстий до точки пересе- чения струй. Отсюда 285. Скорость истечения воды из отверстия v — У 2gh. Импульс силы, действующей со стороны сосуда на вытекающую воду, FA/ = = A/nt\ где Am=pSo Л/—масса воды, вытекающая за время А/. Следовательно, F—pu2S—2pgAS. Давление у дна p—pgh, и поэтому F = 2pS. Такая же сила действует со стороны струи на сосуд. Таким образом, вода действует на с¥енку с отверстием с силой на 2р8 меньшей, чем на противоположную, а не на pS, как могло бы пока- заться на первый взгляд. Это связано с уменьшением давления на стенку с отверстием вследствие большей скорости течения воды у этой стенки. Сосуд придет в движение, если kQ < 2pS, или k < 2pghSlQ. 286. По второму закону Ньютона обязательно должно иметь место равенство pS0=2pS. Следовательно, при истечении жидкости через трубку площадь поперечного сечения струи должна умень- шиться в два раза: S=S0/2. Это сжатие струя объясняется следую- щим образом. Крайние струйки жидкости, подходящие к трубке 254
сверху, не могут вследствие инерции обогнуть край трубки, вплот- ную прилегая к стенкам, и стремятся к центру струи. Под давле- нием частиц, идущих ближе к центру струи, линии тока выпрям- ляются, и суженная струя жидкости течет вдоль трубки. 287> Пренебрегая разбрызгиванием воды, мы тем самым считаем удар струи о стенку абсолютно неупругим. По второму закону Ньютона изменение количества движения воды за время Д/ равно hmv-FM, где Д/и = р (лх/2/4) у Д/— масса воды, протекшая за время Д/ через поперечное сечение трубки. Отсюда F — (ртР/4) и2^0,08 Н. 288. Прц движении газа по трубе (рис. 393) количество движе- ния не меняется по величине, но меняется по направлению. За еди- ницу времени через поперечное сече- 7 ние I вертикального колена трубы ' проходит масса pSy, которая прино- сит с собой количество движения Pi ~ pSvvj, где - Vj — вектор скорости течения газа в вертикальном колене, численно равный заданной скорости у. За то же время через сечение II уносится количество движения р2 = = pSuv2, где v2—вектор скорости в горизонтальном колене, также числен- но равный v. Изменение количества движения равно импульсу силы F, действующей со стороны трубы на газ:- F = == pSu (v2—ух). По величине сила F = = pSv2 У"2. По третьему закону Ньюто- на с той же силой газ действует на трубу. Эта сила направлена в сторону, противоположную изгибу трубы. 289. Начальная скорость воды относительно лопасти v = V 2gh— —&R. Поэтому за единицу времени о лопасть ударяет масса воды m — pS( y ^gh—(dR). Скорость воды относительно лопасти после удара равна О, поэтому изменение количества движения воды за единицу времени равно mv. По второму закону Ньютона. 290. В первый момент судно начнет двигаться вправо, так как давление на правый борт уменьшается на величину 2pS, где р—дав- ление на глубине пробоины ht .aS—ее площадь (см. задачу 285). После того, как струя воды достигнет противоположной стенки, на эту с/енку начнет действовать сила f^pSo2, где и—-скорость струи относительно судна (см. задачу 287). F несколько больше 2pS, так как v > У 2gh из-за того, что судно движется навстречу струе. В результате движение начнет замедляться. 291. Скорость течения жидкости в трубке постоянна по всему сечению в силу малой сжимаемости жидкости и неразрывности струи. Эта скорость равна v= У 2gH. Скорость жидкости в сосуде очень мала и практически равна нулю, так. как площадь сосуда во много раз больше площади сечения трубки. Следовательно, на границе сосуд—трубка должен быть скачок давления, который мы обозначим через Pi—ра» Работа сил давления вызывает изменение- скорости от 255
нуля до V2gH. На основании закона сохранения энергии Дт и2/2 = (pi — р2) 5 ДА, где S —площадь сечения трубки, ДЛ—высота малого элемента жид- кости и Д/и = рЗДЛ—масса этого элемента. Следовательно, ри2/2 — = Pi — p2==pgff^ Из-за постоянства скорости течения давление в трубке меняется по закону p — po — pg(h—x)t как и в неподвижной жидкости. р0 — атмосферное давление, а х—расстояние, отсчитываемое от верхнего конца трубки. Изменение давления по высоте изображено на рис. 394. По оси ординат отложено давление, по оси абсцисс —рас- стояние от поверхности жидкости в сосуде. 292. Вытекающая из трубки вода за малый промежуток времени Д/ унесет с собой количество движения Др=р5о2 Д/, где v— V^2gH— скорость вытекающей струи (см. задачу 291). По второму закону Ньютона F At = 2pgHS At. Такая же сила будет действовать со сто- роны вытекающей струи на сосуд с водой. Следовательно, в началь- ный момент показание весов уменьшается на 2pgHS, 293. В первый момент, пока струя еще не достигла чашки, рав- новесие нарушится. Чашка качнется вверх, так как вытекшая из сосуда вода перестанет оказывать давление на дно сосуда. Однако после того, как струя достигнет чашки, равновесие восстановится. Рассмотрим элемент струи массы Д/n. Падая на чашку, он сообщает ей в вертикальном направлении импульс Д/n }/^2g/z, где Л —высота крана над чашкой, С другой стороны, этот элемент, покинув сосуд, перестанет оказывать давление на его дно и на чашку в течение времени падения t— V2h/g. Это эквивалентно появлению импульса силы, действующего на сосуд вертикально вверх во время падения элемента жидкости. Среднее за время падения значение этого им- пульса равно Amg 2Л/^ = Д/n V2gh. Таким образом, с каждым элементом жидкости Дт связано в среднем за время падения появление двух равных и противоположно направленных импульсов силы. Так как струя течет непрерывно, весы будут находиться в равновесии. В момент, когда струя пре- кращается, чашка качнется вниз, так как последние элементы жид- кости, падая на чашку, действуют на нее с силой, превышающей их вес, а уменьшение давления на дно сосуда прекратится. 256
' 294. На основании закона сохранения энергии можно написать Mv2[2=fmght где М — масса воды, останавливающейся в трубе при закрытии клапана #2, т—масса воды, поднявшейся при этом на высоту h. Отсюда — •—= pV0 ght где Vo —объем массы т. За 2 с в среднем поднимается объем Vo = lfui2v2l&gh — 1,7* 10-3 м3. За час работы тарана поднимется V—1,7-10-3-30-60 3 м3. 295. Давление в обтекающем крышу воздушном потоке меньше, чем в покоящемся воздухе. Это избыточное давление неподвижного воздуха под крышей и вызывает описанные явления. 296. За счет большой скдрости течения газа в струе давление внутри струи меньше атмосферного. Снизу шарик будет поддержи- ваться напором струи, а/с боков —статическим атмосферным давле- нием. 297. При течении воздуха между дисками скорость его убывает по мере приближения к краям дисков. У краев она минимальна. Давление в струе газа тем меньше, чем больше его скорость. Поэтому давление между дисками меньше атмосферного. Атмосферное давле- ние прижимает нижнюю пластинку к верхней, и течение газа пре- кращается. После этого статическое давление газа снова отодвигает пластинку, и процесс повторяется. 298. В потоке текущей жидкости давление уменьшается с уве- личением скорости течения. Скорость течения воды в сосуде значи- тельно меньше скорости' течения в трубе, и, следовательно, давление воды в сосуде больше/чем в трубе. На границе сосуд—труба ско- рость . течения увеличивается, а давление уменьшается; вследствие этого шарик, помещенный на сетку-, оказывается прижатым к ней и не всплывает. 299. За промежуток времени t поршень переместится на рас- стояние их (рис. 395). При этом сила F совершит работу A = Fux. Рис. 395. Масса жидкости, вытекающей за время т, равна pSux. Скорость истечений жидкости v определится из соотношения Su = sv. Измене- ние кинетической энергии жидкости за время т равно pS их (у2/2 — и2/2). Это изменение энергии должно равняться работе силы F: Fux ~ pSux (и2/2 — и2/2). 2F 1 Исключая отсюда и, находим v2 — -77-1------57™ • Если s S, то ______ op 1 —s£/S* v — У 2F/Sp, 300. При решении задачи 299 мы считали, что скорость любого элемента жидкости, находящегося в насосе, постоянна. Изменение 9 Б. Б. Буховцев и др. 257
скорости от и до v происходит при выходе жидкости из насоса. Однако это имеет место не сразу после того, как сила начнет действовать на поршень. Необходимо некоторое время, за которое процесс уста- навливается, т. е. частицы жидкости в цилиндре приобретают посто- янную скорость. При s S это время стремится к бесконечности, и поэтому скорость, приобретаемая жидкостью под действием посто- янной силы, оказывается бесконечно большой. 301. Введем систему координат, изображенную на рис. 396. Скорость истечения жидкости по формуле Торричелли V = V 2gу, где у—толщина слоя воды в верхнем сосуде. Вследствие несжимаемости воды sV = Sv, где ц—скорость опускания верхнего уровня воды, S—его площадь, a s —площадь отверстия. Если принять, что сосуд имеет соевую симметрию, то £ = лх2, где х—горизонтальная коорди- ната стенки сосуда. Следовательно, тех2/ Y2gy ~s/v — const, так как по условию уровень воды должен опускаться с постоянной скоростью. Отсюда форма сосуда определяется уравнением y=kx^, где k » n2v2/2gs2. 302. В горизонтальном сечении давление в зависимости от рас- стояния г до оси изменяется по закону р=ро-|-(рсо2/2) г2, где р0 — давление на оси сосуда, а р —плотность жидкости. Деформация сжатия жидкости будет наибольшей у стенок сосуда, в то время как деформация растяжения вращающегося стержня (задача 211) макси- мальна у оси. 303. На расстоянии г от оси вращения избыточное давление р==(рй>2/2)г2 (см. решение задачи 302). С другой стороны, это дав- ление определяется превышением уровня жидкости в данном участке по сравнению с уровнем на оси: p — pgh (рис. 397). Приравнивая эти выражения, имеем h = (a)2/2g) г2. Это —уравнение параболы. Соответственно, поверхность жидкости во вращающемся сосуде при- нимает форму параболоида вращения. 304. Размешивание придает частицам воды в стакане некоторую угловую скорость со. Распределение давлений в жидкости будет при- мерно таким, какое получено в решении задачи 302. Избыточное давление внутри жидкости уравновешивает давление, вызванное пре- вышением уровня жидкости у краев стакана (см. задачу 303). После того, как размешивание прекращено, вследствие трения о дно ско- рость вращения жидкости у дна начнет уменьшаться, причем тем значительнее, чем дальше элемент жидкости находится от оси. Теперь 258
уже избыточное давление, вызванное вращением, не уравновесит вес столба жидкости у краев сосуда. Вследствие этого возникнет цирку- ляция жидкости, изображенная схематически на рие, 398. Поэтому чаинки собираются в центре дна стакана. Глава II. ТЕПЛОТА. МОЛЕКУЛЯРНАЯ ФИЗИКА § 12. Тепловое расширение твердых и жидких тел 305. Ы « 420 °C. 306. Большая прочность железобетонных конструкций возможна только благодаря тому, что коэффициент расширения бетона очень близок к коэффициенту расширения железа. 307. Количество тепла, переданного в единицу времени от одцого тела к другому, пропорционально разности температур этих тел. При большой разности температур термометра и окружающих его предметов изменение объема ртути идет быстро. Если же показания термометра близки к температуре окружающих тел, изменение объема ртути идет медленно. Поэтому при измерении температуры прихо- дится ждать довольно длительное время, пока термометр не примет точно температуру тела. Если же нагретый термометр соприкасается со сравнительно холодным воздухом комнаты, столбик ртути из-за большой разности температур «падает» настолько быстро, что термо- метр удается «стряхнуть» почти тотчас же. 308. При охлаждении шкалы от до /о = О°С цена каждого деления уменьшается. Поэтому высота столба ртути, отсчитанная по шкале, имеющей температуру /о = О°С, будет иметь иное значение, равное Н = Высоты столбов ртути при различных тем* пературах и одинаковых давлениях обратно пропорциональны плот-» ностям: Яо/Я1 = р1/Ро=1/(1+Ю. Отсюда ЯО = ЯХ (1 +00(1 +₽/х) » Н, (1 +а/1-р/1). 309. Можно предварительно охладить термометр в холодильнике и стряхнуть. Если холодильника нет, то нужно подержать термометр под мышкой долгое время, извлечь его и сразу же стряхнуть. Тер- мометр покажет температуру тела. 310. Разность длин Линеек при температуре t± равна l'o (1 + + — (1 +a2/i) = /. При температуре t2 эта разность (1 +аЛ) — fo 0 +а2^г) = ± Знак плюс соответствует случаю, когда разность длин линеек остается неизменной (рис. 399, л). Знаку минус соответствует зависимость длин линеек от температуры, изображенная на рис. 399, б. В первом случае система уравнений приводит к следующим результатам: *0 (1> =++ 1 = 6-8 СМ’ 1"о <1> 1 =4)8 СМ- 9* 259
Во втором случае r 2+a2(/i+/2)- 1 олоя^гм /* , — 2+а1(^1+^) /_олпАгм <2>~(/2—(а2—а1} ^~200^,5сМ> Z« (2> tj (а2- а1)/““2006 см- При / = 0°С длина железной линейки должна быть больше медной. 311. Возможная схема подвеса изображена на рис. 400. 1 и 2— стержни с' малым коэффициентом линейного расширения (напри- мер, стальные), а 3 — стержни с большим коэффициентом расшире- ния а2 (например, цинковые или латунные). Длины стержней можно подобрать так, что длина маятника не будет изменяться с темпера- турой. Для этого необходимо, чтобы ах (Zi + /2) = a2Z3. 312. При нагревании баллона объем его полости увеличивается по тому же закону, что и объем стекла: (1+0G), где 0— коэффициент объемного расширения стекла. Если через р0 и рх обо- значить плотности ртути при температурах /0 и tlt то можно записать: mo = t,oPo и mi — ujPi> причем pi = p0/(l+fMi)- Данная система уравнений дает для 0 следующее выражение: р = о + Р1А>—т0 - зло-5 град-1 Линейный коэффициент расширения a = 0/3 ~ 10“6 град-1. 260
313. Пусть маятник часов, идущих точно, совершает /V колеба- ний в сутки. Тогда при температрре tt маятник наших часов сделает /V колебаний за п—-5 секунд (где л —86 400 — число секунд в сутках), а при температуре /2—за п+10 секунд. Периоды колебаний будут соответственно равны Tr — (п —5)/Ы и Та = (п+10)/ЛЛ Отсюда отно- шение периодов Т1/Т2= (п—5)/(п +10) » 1 — 15/n. С другой стороны, учитывая, что период колебаний маятника Т — 2л V 11g, получаем £= » У *+“<'-'> “1 Приравнивая выражения для отношения периодов, найдем а 30/(/2 — п 2,3-10~6 град-1. § 13. Закон сохранения энергии. Теплопровбдность 314. По закену 'сохранения энергии выделившееся тепло равно убыли кинетической энергии: Q = Mvl/2 -(M + m) и*/2, где v — скорость тележки, после того как на нее опустили кирпич. Эта скорость находится из закона сохранения количества движения: t> = Afu0/(Af + /n). Следовательно, Q = Mmvl/2 (М+/п). 315. На основании закона сохранения энергии mgl = mv2/2 + k (I - Z0)2/2 + Q, где Z —длина шнура в момент, когда шайба покидает его. С другой стороны, изменение механической энергии шайбы равно работе сил трения: #ш2/2 — mgl — Л, где А = — fl. Следовательно, Q = — A—k(l —1^/2. Используя закон Гука f = k(l — Zo), найдем Q — fl0 + f2/2k. 316. Электрический ток совершает работу А=шт. За счет этой работы от холодильного шкафа будет отнято количество тепла Q2 = = где с~теплоемкость воды, а Л—теплота плавления льда. Количество тепла, выделенное в комнате, по закону сохранения энер- гии будет равно Qi = Л + Q3=orc + так как энергия тока в конечном счете превращается в тепло. 317. Температура в комнате повысится. Количество выделенного в единицу времени тепла будет равно мощности, потребляемой холо-, дильником, так как в конечном счете энергия электрического тока переходит- в тепло, а тепло, отбираемое у холодильного шкафа, опять возвращается в комнату. 318. Более выгодно использовать холодильник, забирающий тепло от наружного воздуха и выделяющий его в комнате. В этом случае тепло, выделяемое в комнате в единицу времени, равно и/+02, где - w—мощность, потребляемая холодильником, a Q2—тепло, отнятое от наружного воздуха в единицу времени (см. задачу 316). Только 261
дороговизна и сложность оборудования препятствуют в настоящее время использованию подобного рода тепловых насосов для отоп- ления. 319. При растворении кристаллическая решетка соли разру- шается. Этот процесс требует затраты некоторого колйчества энергии, которое заимствуется из растворителя. Во втором случае часть меж- молекулярных связей кристаллической решетки уже разрушена при измельчении кристалла. Поэтому для растворения порошка требуется меньше энергии, и вода после растворения во втором сосуде будет обладать более высокой температурой. Эффект будет, правда, крайне незначительным. 320. Количество тепла, отданное охлаждающейся водой, равно m2c(t2— В), где 0 —конечная температура. Холодная вода получает тепло пцс (Q — t]). Тепло, полученное калориметром, равно q (0— На основании закона сохранения энергии т^с (0 — G) + q (0 — tr) = mg (t2 — 0). Отсюда е (mi/1 + /n2f2)c + ^i ~ 4 оС (m^m^c + q 321. Мощность, затрачиваемая на нагревание воды в калориметре, Ni — DVctli;, где D — плотность воды, с—удельная теплоемкость воды. Искомое отношение DVct ~ 50/ Q—N~ -1—WT~5/o' h 322. Q=~-(Ti-T0)S/ «4-10? Дж. 323. Количество тепла Q, проходящего через, первую пластину за одну секунду, равно Q = где S —площадь пластины. Поскольку процесс является стационарным, такое же количество ч р _р тепла проходит через вторую пластину: Q = k2 —- S. Из условия а2 k, s — k S находим что Т — + Ui d% ^2^1 "Г ^1^2 324. Подставляя в выражение для Q значение температуры Т2 (см. задачу 323), при d1==d2 = d найдем О__ ^^1^2 Л Т, ~ 4 k± + k2 2d Следовательно, коэффициент теплопроводности стенки k = . "1 “Г «2 325. Количество тепла, проходящего в одну секунду через попереч- ные сечения брусков с коэффициентами теплопроводности и k2t равно соответственно <21 = ^(Л-То)5 и Q2 = ^(Tx-T0)S. 262
Количество тепла, проходящего через два бруска, полная площадь сечения которых 2S, равно Q = Ql + Q, = 2S. Отсюда вытекает, что коэффициент теплопроводности стенки k = = (^i + W/2. 326. Коэффициенты теплопроводности стенок Л и В равны h а____2k]k2 А 2 ’ B~kt+k2 (см. решения задач 324 и 325). Из очевидного неравенства (£i~62)2 > 0 вытекает, что (&i + £2)2 > > 4&1&2. Отсюда т е ki > kn 2 > kt+k^' *А>Ив- 327. Количество тепла, ежесекундно поступающего от нагревателя через дно кастрюли в воду, равно ъ Q=-j(T-T1)S = mki где 7\—температура кипения воды, Л—удельная теплота парообра- зования. Отсюда Т — T^rnkd/kS. § 14. Свойства газов 328. Снимаемый с ручки колпачок действует, как насос. Под ним возникает разреженное пространство, высасывающее чернила из резервуара. Отверстие служит для поддержания постоянного давле- ния под колпачком. 329. Считая, что температура остается постоянной, применяем к объему воздуха над ртутью закон Бойля—Мариотта: (Poi — Pi) (/ — 748 мм) = (Ро2“Р2) (/ — 736 мм). Отсюда I = 764 мм. 330. В положении равновесия f — tng —-F = 0, где / — выталкиваю- щая сила: f^dl^S, где d — удельный вес воды, hx — высота столба воздуха в пробирке после погружения. В данном случае выталкиваю- щая сила создается разностью давлений на запаянный конец трубки снизу и сверху: f = p1S — (p0+dh) S, где р±—давление воздуха в трубке после погружения. По закону Бойля —Мариотта PqIS—p^S. Из данной системы уравнений имеем ' с F = у [(Ро+^)2+W/-(Po + ^)] —=0,87 Н. 331. Первоначально, за счет понижения уровня воды в сосуде, давление р воздуха будет уменьшаться приблизительно изотерми- чески. Так будет происходить до тех пор, пока суммарное давление на уровне нижнего конца трубки не станет равным атмосферному 263
давлению Ро(:р + р£^=Ро> где Л —высота столба жидкости в сосуде над уровнем нижнего конца трубки. С этого момента в сосуд нач- нут проскакивать пузырьки воздуха. Давление на уровне нижнего конца трубки будет оставаться равным атмосферному, а давление воздуха р = р0 —pgh—линейно расти с понижением уровня воды. При этом скорость вытекания жидкорти из сосуда будет постоянной. Зависимость р от Q изображена на рис. 401. Незначительные колебания давления при проскакивании отдельных пузырьков воз- духа на рис. 401 не учтены. 332. При откачке воздуха из сосуда после одного качания дав- ление в сосуде станет равным pi = p0V/(V/+D0). После второго кача- ( V \2 ния PiV = pa(^+^o) и» следовательно, р2 = р0( —— и т- Д- \v-rtW / у \п После п качаний давление в сосуде будет равно р' = р0 [ ту--) . „ „ \ и/ При нагнетании воздуха в сосуд после п качаний установится давление р-р +— р > р0 при любом п по той причине, что во время нагнетания при каждом качании насос захватывает воздух, имеющий давление р0, а при откачке удаляются объемы воздуха v0 при давлениях, мень- ших До- 333. Применяя закон Бойля—Мариотта к двум объемам газа в закрытой трубке, получим AZ ) S, Здесь р—давление при горизонтальном положении трубки, рх и р2— давления в нижней и верхней частях трубки при вертикальном ее 264 -
положении с закрытыми концами, d—-удельный вес ртути, S — по- перечное сечение трубки. Отсюда первоначальное давление в трубке P~d2\bl 10)’ Здесь для краткости введено обозначение Z0 = (L —Z)/2. Если открыть один из концов трубки при ее горизонтальном положении, то давление газа в трубке станет равным атмосферному. По закону Бойля—Мариотта pl0S=dHl1S (здесь Я —атмосферное давление), откуда Х~2Я \ AZ z0 . Столбик ртути переместится на расстояние AZ1 = Z0-Zl = Zo J Г Для того чтобы ртуть не выливалась из трубки, необходимо выпол- нение условия AZ При открывании верхнего конца вертикально расположенной трубки ^Z0S=d(/7 + Z)/2S. Отсюда Д/з = /о—/з= 2 (Я 4-/) Ртуть не выливается из трубки, если А^ ~2(я+о р ,;, - AZ I J + "Г При открывании нижнего конца pZ0S = d(tf-Z) Z3S. 2(Я + /) Z ы Отсюда Д/з = /о-/з= 2(Я-/) При этом, чтобы столбик ртути не был выдавлен из трубки, должно выполняться условие -|/W-Q2 , 2(Н^1) AZ^ V /2 -Н+ i 334. Так как для любого газа при р= 1 атм и T = 273JK Уц = = 22,4 л/моль, то для всех газов, взятых в количестве одного моля, C = pVyJT = 0,082 ; л‘аТМ-;< . Эта постоянная обычно обозначается н (моль-К) # и называется универсальной газовой постоянной. Значение R в единицах СИ: # = 8,31 Дж/(моль-К). 26&
335. При фиксированном давлении и температуре объем, зани- ’ маемый газом, пропорционален его массе. Одному молю соответ- ствует объем Уц,, произвольной массе hi — объем V. Очевидно, что уц=Ур,//п, где р,—молярная масса. Подставляя это выражение в уравнение состояния для одного моля, будем иметь pV = (т/р) RT. 336. При исчезновении притяжения между молекулами давление должно увеличиться. Для доказательства этого выделим мысленно внутри газа или жидкости два слоя I и II (рис. 402). Молекулы, проникая из слоя / в слой II вследствие теплового движения, стал- киваются с молекулами слоя //, и в результате на этот слой дейст- вуют силы давления рт, зависящие от температуры. Силы притя- жения действуют на слой II со стороны молекул слоя / в противо- положном направлении. Результирующее давление слоя / на слой // p = pT—pit где р( — давление, обусловлен- ное внутренними силами притяжения. Ис- чезновение pt- увеличивает давление. 337. При исчезновении сил взаимодей- ствия между молекулам!? вода преврати- лась бы в идеальный газ. Давление можно найти по уравнению состояния идеального tn RT 1О_Л газа: р-—-— ^ 1370 атм. р V Рис. 402. 338. Выделим цилиндрический объем газа, прилегающий непо- средственно к стенке (рис. 403). Силы, действующие на боковую поверхность цилиндра, взаимно уравновешиваются. Так как объем находится в равновесии, то давление на газ со стороны стенки должно быть обязательно равно "давлению на другое основание ци- линдра со стороны газа. На основании третьего закона Ньютона отсюда мы можем заключить, что давление газа на стенку равно давлению внутри сосуда. 339. Давление в газе зависит от сил взаимодействия между моле- кулами (см. задачу 336). Силы же взаимодействия молекул газа друг с другом и молекул газа и стенки различны. Равенство давле- ний внутри газа и у стенки сосуда (см. задачу 338) может осущест- виться поэтому только за счет различия в концентрациях. 340. Вследствие постоянства объема Рг/Pi= ^2/^1, или (Рг— Pi)/Pi = (^2 Т1)/7'1 = 0,004. Отсюда ^^(Тг — 7\)/0,004 = 250 К. 266
341. На основании закона Архимеда mg-}-P = dVt где d — удель- ный вес воды, а V—объем шара. Уравнение состояния дает (pQ + dh)V = (m/ii)RT. Исключая из этих уравнений V, найдем т Рр (РоЧ-^) dRT — pg (pn+dh) « 0,666 г. Равновесие будет неустойчивым. , 342. При горизонтальном расположении трубки устройство слу- i жить термометром не может, так как при любой температуре давле- ния справа и слева на капельку будут уравновешены. Если же трубка вертикальна, то давление газа в нижнем шарике будет больше давления в верхнем шарике на постоянную величину. При неизмен- ном объеме давление с ростом температуры растет тем быстрее, чем больше начальное давление. Для поддержания постоянства разно- стей давлений в шариках капелька начнет перемещаться вверх. Устройство может служить термометром. 343. Так как массы газа в обеих половинах цилиндра одина- ковы и поршень находится, в равновесии, то уОтсюда T2 = (V2/V1) Т1 = ЗЗО К. Применяя закон Бойля—Мариотта к объему газа, температура которого не меняется, имеем р = роИо/У1 = 1,О5 атм. 344. В равных объемах различных газов при одинаковых внеш- них условиях содержится одинаковое число' молекул (закон Авогад- ро). Поэтому Vi:V2:V3:^ = ^i:^2^3:^4, гДе V/—объем соответ- ствующего газа, Nj — число молекул этого газа. Масса некоторого количества газа пропорциональна числу его молекул и молярной массе газа: > С другой стороны, обозначая через nt-=(V7/V) 100% относительный объем данного газа в процентах, имеем п -п -п Л1-Л2-Лз-Л4— у .• у • у • у — ft ^2 .^3.^4 N ' N ' N * Если характеризовать процентный состав воздуха величинами n't = (tni/m) 100% (состав по массе), то на основании предыдущих соотношений . ^2.^з. ^4_ #1И1. ^2 . JV3H3 . #4М4 _ 1 2 3 4~ от ‘ от ’ от ’ от ~ N ' N ’ N ’ N ~ = Л1Ц1: лаН2 • пзНз: n4M<- Отсюда * ni + n2+ns + «4 nt — 1 " 1 ** I । 1 nt My • И1М1 + Л2Ш + ЛзМз + Л4М4 Учитывая, что ni + n2+л3 +n<= 100%, получим п' =________________________100%___________ 1 ЛЯЧ+'ЫЧ + "зНз+'МЧ' 267
Следовательно, < = 75,52%, < = 23,15%, < = 1,28%, < = 0,05%. 345. Для каждого. газа можно написать уравнение состояния: PiV = (m1/p(1)₽T, p^^(rnJ^RT. p3V = (m3/p3) RT, p4V = (m4/g4) RT. Отсюда , . . . ч v / I m2 I Л4 Kt (pi+p2+Рз+P4) V=(*t. \ Hi Pa Из P4 / С другой стороны, для смеси газов pV=(m/p,)RT, где m = /n1 + m24- + m3 + /n4, а |i — искомая молярная масса. По закону Дальтона р = р14-р24-р34-р4. Следовательно, = /п1-Ьт2+/п3+/п4 = п'г+п^ + пз+п^ = 28 96б тз <_< <_ , <_д_^£_ - ’ Hi Ц2ТНзТН4 Hi где щ = (т;/т) 100% — процентный состав воздуха по массе. Резуль- тат, полученный в предыдущей задаче, позволяет найти р по извест- ному составу воздуха по объему: И1”1 + Рз”2 +1Х3И3 4~ Н4”4 28 96бв п1 4“ п2 + п3 + Л4 346. На основании уравнения Клапейрона p, = mRT/pV — pRT/p = 72 г/моль. Искомая формула: СбН12 (один из изомеров пентана). 347. При сжатии газа в теплонепроницаемой оболочке работа, совершаемая внешними силами, идет на увеличение внутренней энер- гии газа. При этом его температура растет. Давление в' газе будет возрастать как за счет уменьшения объема, так и за счет увеличе- ния его температуры. При изотермическом сжатии давление растет только за счет уменьшения объема. Следовательно, в первом случае давление увеличится на большую величину, чем во втором. 348. Зависимость р от V изображена на рис. 404. Наибольшая работа, равная заштрихованной на рис. 404 площади, совершена при изотермическом процессе (/—2). На участке 1 — 2 температура не меняется. На участке 2—3 температура уменьшается в два раза. В дальнейшем температура растет, и при V4 = 4 л Т^ = ТГ. 349. 1—2 —изобара (рис. 405). Газ нагревается при постоянном давлении, поглощая тепло. 2—3—изохора. Газ охлаждается при постоянном объеме; дав- ление задает, тепло выделяется. 268
3 — / — изотерма. Газ уменьшает объем при постоянной темпера- туре. Давление растет. Газ не нагревается, хотя внешнее силы совершают над ним работу. Следовательно, на этом участке газ отдает тепло. 350. Количество тепла, выделяющееся при сгорании метана в 'час, Qi = rpV№/RTt где р=16 г/моль—молярная масса газа, Т = 284 К— его температура. Количество тепла, полученное водой в час, лЛ2 Qa = 2J-vpc(/a —/^>3600, где р —плотность воды, с —удельная теплоемкость. По условию задачи — — Решая полученную систему уравнений, найдем / __У I___^ДУрРТ) QQ Op 900лЛ2урс/?Т ~ 351. В начальном состоянии р1У = (/и/р1)/??!, где. —моляр- ная масса озона. В конечном состоянии p2V = (/п/р2) где р2 — молярная масса кислорода. Уравнение теплового баланса дает Pi р2 Решая данную систему уравнений, найдем —=-Дя"1-—= ю. Р1 ' (12 352. Ввиду линейной зависимости давления от объема можно записать: р = аУ4-6. Постоянные а и b находятся из условия задачи: а=(Р1—P«)/(^i——0.5 атм/л, 6=(p'2v1—PiVaMV, — V2)« 20 атм. Подставляя выражение для р в уравнение состояния идеального газа pV = (m/p) Z?T = const«T, найдем aV* + bV= const-Т. (1) 269
График зависимости Г от V (рис. 406) представляет собой параболу. Кривая достигает максимума при Vmax =—^/2а « 20 л, когда корни квадратного уравнения (1) совпадают. При этом Ртах = «Утах+ & ==*/2^ Ю ЗТМ. Следовательно, ^тах —iPmax^maxf1/^^ ~ ^90 К» 353. Энергия единицы объема газа ц1 = СТр, где р — плотность воздуха. Согласно уравнению состояния идеального газа pVjT = mB (В —постоянная). Так как p=m/V, то отсюда рТ ==р/В. Следова- тельно, — (С/В) р определяется только давлением. Энергия всего воздуха в комнате также определяется только давлением. Давление же в комнате равно атмосферному и не меняется при нагревании воздуха. Поэтому не меняется и энергия воздуха в комнате. Нагре- ваясь, воздух частично вытекает через щели наружу, что и обеспе- чивает постоянство энергии, несмотря на нагревание. Только в гер- метически закрытой комнате энергия возрастала бы с нагреванием. 354. На основании уравнения состояния искомая масса газа . JipV ^*2 — Г1 - « » 1,3 кг. А 1 I1 2 355. Пусть первоначально пробирка находится у дна в состоя- нии устойчивого равновесия. По мере нагревания давление воздуха в ней и, соответственно, выталкивающая сила возрастают. При не- которой температуре 7\ пробирка начнет всплывать. Так как давле- ние жидкости убывает по мере удаления от дна, то объем воздуха в пробирке и, следовательно, выталкивающая сила продолжают возрастать. Пробирка быстро достигнет поверхности воды. При даль- нейшем увеличении температуры пробирка будет находиться у по- верхности. Если температуру уменьшать, то пробирка не начнет тонуть при температуре 7\. Дело в том, что у нее имеется большой запас плавучести, вызванной значительным увеличением выталки- вающей силы при всплывании пробирки. Только при некоторой 270
температуре Т2 < 7\ пробирка начнет тонуть. При этом выталкиваю- щая сила будет падать из-за того, что воздух в пробирке по мере погружения в воду будет занимать меньший объем. Пробирка дости- гает дна очень быстро. Зависимость положения h пробирки (по отношению к дну сосуда) от температуры Т изображена на рис. 407. При Т < Т2 пробирка Рис. 407. обязательно будет находиться на дне, при Т > Тг—у поверх- " ности. Если Т2 < Т < Tlt то пробирка будет либо у дна, ли- бо у поверхности в зависимо- сти от того, каковы были пред- шествующие значения темпера- туры. 356. Газ расширяется при некотором постоянном давле- нии р, которое создается порш- нем. В этом случае работа A = p(V3~V1), где V, и Из- начальный и конечный объемы газа. Используя уравнение состояния, выразим произведение pV через, температуру Т. Тогда А = (т/ц) 7? (7\ —7\) 330 Дж. 357. Сообщаемое газу тепло идет на нагревание газа и на совер- шение механической работы. Согласно закону сохранения энергии (?=-£- Cv(Ta-Г1)+^-«(Г2-7’1) = - Л)(СГ+Я) « 1480 Дж. §15. Свойства жидкостей 358. «Труднее» сжать литр воздуха в том смысле, что для этого нужно совершить большую работу. Вода мало сжимаема, и для повышения давления внутри нее до трех атмосфер нужно небольшое уменьшение объема. 359. Максимальный термометр можно осуществить следующим образом. Над столбиком жидкости горизонтально расположенного Рис. 408. термометра (рис. 408) поместить небольшое не смачиваемое жид- костью тело, которое может свободно перемещаться вдоль труб- ки термометра. Положение тела укажет максимальную темпера- туру, так как при расширении жидкости тело будет перемещаться вдоль трубки, а при сжатии останется на месте. Аналогично можно 271
осуществить и минимальный термометр. Для этого смачиваемое жид? костью термометра тело нужно поместить внутрь жидкости. 360. При растяжении упругой резиновой пленки сила натяже- ния зависит от величины деформации пленки. Сила же поверхност- ного натяжения определяется только свойствами жидкости и не ме- няется с увеличением ее поверхности. 361. Поверхностное натяжение чистого бензина меньше, чем поверхностное натяжение бензина, в котором растворен жир. По- этому при смачивании бензином краев пятна оно стягивается к центру. При смачивании же самого пятна оно будет растекаться по ткани. 362. В уплотненном поверхностном слое почвьг образуются ка- пилляры типа изображенных на рис. 409. Они сужаются кверху, и вода в них поднимается вверх. С поверхности вода" интенсивно испаряется. Разрыхление почвы при бороновании разрушает указан- ную структуру капилляров, и вла- га лучше сохраняется в земле. 363. .Внутри кожи имеется большое количество капилляров. Внутри капилляра постоянного сечения капелька смачивающей жидкости будет находиться в рав- новесии. Нагревание жидкости уменьшает поверхностное натяже- ние. Поэтому жидкость втягивается в стороцу холодной части капилляра. Мазь будет втягиваться внутрь кожи при нагревании ее снаружи. 364. Жир расплавляется, и капиллярные силы перемещают его на поверхность холодной ткани, подложенной под костюм (см. за- дачу 363). 365. Часть полена, в тени холоднее. Поэтому капиллярные силы перемещают воду в этом направлении. 366. Гидростатическое давление должно уравновешиваться ка- пиллярным давлением: pgh~4ald. Отсюда h = 30 см. 367. На участок пленки abed действуют следующие силы: сила тяжести, поверхностное натяжение Fa^t приложенное к линии abt и поверхностное натяжение приложенное к линии cd. Равно- весие возможно только в том случае, если Fab больше Fcd на ве- личину, равную силе тяжести рассматриваемого участка пленки. Различие сил поверхностного натяжения объясняется разницей кон- центрации мыла в поверхностных слоях пленки. 368. Выталкивающая сила уравновешивает силу тяжести кубика mg и силу" поверхностного натяжения 4аа, т. е. a2xpg = mg -j- 4аа, где х — искомое расстояние. Отсюда х = (т^4-4аа)/о2р^ « 2,3 см. Силы поверхностного натяжения вносят поправку около 0,1 см. 369. Жидкость поднимается на высоту /i = 2a/pgr. Потенциальная энергия столбика жидкости W = mgh/2 = 2na2/pg. Силы поверхност- ного натяжения совершают работу A ~2nrah = 4na2/pg. Половина этой работы идет на увеличение потенциальной энергии, а другая половина —на выделение тепла. Следовательно, Q'=2na2/pg. 370. Давление внутри жидкости в точке, находящейся выше некоторого уровня на расстояние Л, меньше давления на этом уровне на величину pgh. На уровне жидкости в сосуде давление равно 272
нулю. Следовательно, давление на высоте h отрицательно (жидкость растянута) и равно p — ^-pgh. 371. Силы притяжения, действующие на' молекулу у поверх- ностного слоя со стороны всех остальных молекул, дают равнодей- ствующую, направленную вниз. Однако со стороны ближайших соседей на молекулу действуют силы отталкивания. Благодаря этому молекула находится в равновесии. В результате действия как сил притяжения, так и сил оттал- кивания плотность' жидкости в поверхностном- слое меньше, чем внутри. В самом деле, на молекулу 1 (рис. 410) действует сила отталкивания со стороны молекулы 2 и силы притяжения всех ос- тальных молекул (5, 4, ...). На молекулу 2 действуют силы оттал- кивания со стороны 3 и 1 и силы притяжения со стороны лежащих в глубине молекул. В результате расстояние 1—2 должно быть больше расстояния 2—3 и т. д. Указанная схема рассуждений является весьма грубой (напри- мер, не учитывается тепловое движение и т. д.), но дает качественно правильный результат. Увеличение поверхности жидкости сопровож- дается возникновением новых участков разреженного поверхностного слоя. При этом должна быть совершена работа против сил притя- жения между молекулами. Эта работа представляет собой поверх- ностную энергию. 372. Необходимое давление должно превышать- атмосферное на величину, способную уравновесить гидростатическое давление столба жидкости и капиллярное давление в пузырьке воздуха радиуса г. Превышение давления p = pgh-j-2a/r — 4S4 Н/м2. Рис. 410. © 373. Так как в этом случае pgh < 2а./г > то жидкость поднимается до верхнего конца трубки. Мениск будет представлять собой часть сферического сегмента (рис. 411). Радиус кривизны сегмента опре- деляется из условия, что силы поверхностного натяжения урав- новешивают вес столба жидкости: 2jtracos <p = nr2hpg. Отсюда 273
cos ф = Mpg72a; Как видно из рис. 411, радиус кривизны сегмента 7? = r/cos (р = 2a//igp =^0,74 мм. 374. После открывания трубки на нижнем ее конце образуется выпуклый мениск такой же формы, как и верхний. Поэтому длина столбика воды, оставшегося в трубке, будет равна 2h, если l^h, и будет равна / + если l<h, 375. 1) Силы поверхностного натяжения могут удержать в данном капилляре столбик воды высотой не более h. Поэтому вода будет выливаться из капилляра. 2) Вода не выливается. Мениск выпуклый. Для абсолютно сма- чивающей жидкости он будет иметь форму полусферы. 3) Вода выливаться не будет. Мениск выпуклый. Кривизна его меньше, чем во втором случае. 4) Вода не выливается.. Мениск плоский. 5) Вода не выливается. Мениск вогнутый. 376. Внутри мыльного пузыря радиуса R давление р превышает атмосферное на величину удвоенного капиллярного давления, так как пленка пузыря двойная: р = р0 + 4а/$. Давление внутри пузыря радиуса R вместе с давлением участка пленки между пузырями должно уравновесить давление внутри меньшего 'пузыря. Следова- тельно, 4а/^Ц-4а/7?д. = 4а/г, где Rx — радиус кривизны участка пленки АВ. Отсюда Rx~Rr/(R — r). Силы поверхностного натяже- ния в любой точке поверхности соприкосновения пузырей уравнове- шивают друг друга и равны между собой. Это возможно только том случае, когда утлы между ними равны 120°. 377. По закону сохранения энергии крест не может прийти во вращение. Составляющие сил поверхностного натяжения уравнове- ш ива юте я силами гидростатического давления, так как гидростати- ческое давление воды, находящейся выше горизонтального уровня жидкости, отрицательно (см. задачу 370). Рис. 412. 378. Если тела смачиваются водой, то форма поверхности воды примет вид, изображенный на рис. 412,а. Между спичками выше 27<
уровня MN вода растянута капиллярными силами, и давление внутри нее меньше атмосферного. Спички сближаются, так как дав- ление на них с боков равно атмосферному. Для не смачиваемых водой спичек форма поверхности изображена на рис. 412,6. Давле- ние между спичками равно атмосферному, а с боков ниже уровня MN больше атмосферного. При сближении спичек в последнем случае заданным краевым углам соответствуют две различные формы поверхности (рис. 413) жидкости. Однако одна из них (рис. 413^а) не может реализоваться. Рис. 413. Давление на уровне KL должно быть, везде одним и тем же. В част- ности, давление столбиков АВ и CD разной высоты должно быть одинаковым. Но это невозможно, так как положение столбиков можно выбрать с таким расчетом, чтобы их поверхности имели одинаковую форму. Тогда дополнительное.давление поверхностных сил одинаково, а гидростатическое давление различно. Следовательно, при сближе- нии спичек поверхность воды между ними будет стремиться к гори- зонтальной форме (рис. 413,6). В этом случае, как видно из ри- сунка, давление между спичками на уровне MN равно атмосферному. Давление же слева на первую спичку равно атмосферному и ниже уровня MN. На вторую спичку давление справа меньше атмосфер- ного выше уровня MN. В результате спички будут отталкиваться. § 16. ^Взаимные превращения жидких и твердых тел 379. Замерзание при нуле произойдет только при наличии цент- ров кристаллизации. Ими могут служить любые нерастворимые час- тицы. Когда масса воды велика, то в ней всегда найдется хотя бы один такой центр, а этого уже достаточно, чтобы замерзла вся вода. Если же масса воды раздроблена на мельчайшие капли, то лишь в сравнительно небольшом числе капель будут иметься центры крис- таллизации и замерзнут только эти капли. 275
380. Количество тепла, получаемое в единицу времени водой и льдом, примерно одинаково, так как разность температур воды и комнатного воздуха примерно такая же, как и льда и воздуха. За 15 минут вода получила 840 Дж тепла. Следовательно, лед за 10 часов получил 33 600 Дж. Отсюда Л,=336 Дж/г. 381. и = 2464 м/с. 382. Уравнения теплового баланса имеют вид At + С А/, Q2 = m1c1 Д//2-Ь^1^ + ^1С2Д//2 + С А/, где т1 и — масса и теплоемкость льда, С—теплоемкость калори- метра, с2 — теплоемкость воды, А/= 2 °C. Отсюда - «(й'+Л-(+4)-«= е= —=63“ Дж'гр‘д- q 2 2 ’ сг 383. Количество тепла, которое может быть выделено водой при охлаждении ее до 0 °C, равно 16 800 Дж. Для нагревания льда до 0°С требуется 50400 Дж. Следовательно, лед может нагреваться только за счет тепла, выделяющегося при замерзании воды. Для выделения недостающих 33 600 Дж должно замерзнуть 108 г воды. В результате в калориметре образуется смесь из 500 г воды и 500 г льда, находящихся при температуре 0 °C. — ' 384. Конечная температура содержимого сосуда 0 = 0 °C. Урав- нение теплового баланса имеет вид mici G1 9) = т^2 (— 12) +-(m2 ~ тз) К ! где т1 — искомая масса сосуда, с2 — теплоемкость льда. Отсюда =WjjC, (8—/а)+(m, — m3) А, г М/1-0) 385. 1) Искомую массу льда т можно найти из уравнения tnk — Mc(— t). Отсюда m=100 г. - 2) Уравнение теплового баланса запишется в данном случае следующим образом: Л4А = Мс (— /). Отсюда / =— 80 °C. 386. Температура плавления льда при сжатии до давления 1200 а^м понизится на А/ = 8,8 °C. Лед будет плавиться до тех пор, пока не произойдет охлаждения до —8,8 °C. При этом поглощается количество тепла Q = tfz1X, где тг — масса расхаявшего льда, а X —удельная теплота плавления. На основании уравнения теплового баланса /п1Х = /псА/, где с—теплоемкость льда. Отсюда тг = = ст М/к & 5,6 г. § 17. Упругость и прочность 387. F=SE (Я-г)/г = 600 Н. 388. При нагревании стержня с закрепленными концами на А/ в нем возникает упругая сила F, равная по закону Гука F=SE M/l — SEaAt, где Е — модуль Юнга стали, а а —ее коэффи- циент расширения. Если постепенно освобождать один из концов 276
стержня, то длина его увеличится на Д/ = /аД/. При этом сила будет линейно уменьшаться от F до нуля, так что ее среднеё значение будет равно F/2. Искомая работа Л =(F/2)AZ = 1/2 SEZa2(A/)2. 389. Натяжение проволоки Т = Afg/2 sin а. Из закона Гука сле- дует, что T = (AZ/2Z) ES. Так как Д/ = 2 (Z/cos а—/), то T = '-C0™SE=-^-. cos а 2 sin а При малых углах sin а « а, a cos а = 1 — 2 sin2 (а/2) « 1 — а2/2. Учитывая это, получим а= Mg/SE. 390. Нагретый на AZ стержень в свободном состоянии удли- нился бы на AZ = ZoaAZ, где Zo—первоначальна'я длина стержня. Чтобы вставить нагретый стержень между стенками, его придется сжать на AZ. По закону Гука Д/ = IF/ES. Отсюда F = ESa Ы = 1100 Н. 391. При нагревании стержней в свободном состоянии их общая длина увеличится на Д/ = AZt-f-A/2 = (ai^i + a2^2)Ы* Сжатие их на ту же величину AZ приведет к сокращению длин стержней на Д/J и AZ2, причем Д/^ + Д/2 = Д/. Для этого необходима сила f=e1saz;/z1==e2s az;/z2. Решая полученную систему уравнений, найдем р_._ ~Ha2^2 С этой силой стержни действуют друг на друга. 392. Из соображений симметрии очевидно, что удлинение про- волок будет одинаковым. Обозначим это удлинение через AZ. На основании закона Гука натяжение стальной проволоки FC = (AZ/Z) SEC, а медной FM = (AZ/Z)SEM. Отсюда вытекает, что отношение натяже* ний равно отношению соответствующих модулей Юнга: FMlFz — = Ем/Ес=1/2. При равновесии 2FM-f-Fc = /ng. Следовательно, FM = /ng/4 = 250H и Fc = 2FM = 500H. 393. На основании закона Гука имеем F6 = (AZ/Z)S6E6 и Еж = (Д^05жЕж.4 Отсюда следует, что F^F^ = 2t Таким образом 2/3 нагрузки прихо- дятся на бетон, а 1/3 —на железо. 394. Под действием сжимающей силы F трубка укорачивается на Fl/SMEU, а под действием растягивающей силы F болт удлиняется на величину Fl/SCEC. Сумма Fl/ScEc-\-Fl/SMEW равна перемещению гайки вдоль болта: Fl/ScEc+Fl/SME„=h. Отсюда F— z Sc£c+Sm£m • • 395. Так как коэффициент теплового расширения меди ам больше, чем у стали ас, то увеличение температуры вызовет сжатие медной пластинки и растяжение стальных. Вследствие симметрии относи- тельные удлинения всех трех пластинок одинаковы. Обозначив через F силу сжатия, действующую на медную пластинку со стороны 277
обеих стальных, для относительного удлинения медной пластинки имеем —F/SEM. На стальную пластинку со стороны медной действует растягивающая сила F/2. Приравнивая относительные удлинения пластинок, получим а“* — S£7=“cZ/+2S5^ ’ ~ „ 2SEy.Ec (ам—ас) i ОгеюиГ= 2J-.+е. • 396. При вращении кольца в нем возникает натяжение Т — mv2f2nr ”‘см. задачу 209). Для тонкого кольца /n = 2nrSp, где S — поперечное сечение кольца. Следовательно, T/S = pt/2. Отсюда максимальное значение скорости п= ргР/р^41 м/с. 397. Первоначально со стороны растянутого болта на каждую гайку действует упругая сила Fo. Груз Р ^Fo не в состоянии уве- 'На верхнюю зайку [ На нижнюю Зайку Р личить длину той части болта, ко- торая находится между гайками, и, следовательно, изменить его на- тяжение.-Поэтому и сила, дейст- вующая на верхнюю гайку со сто- роны бруска, не будет меняться до тех пор, пока P^FQ. На'ниж- нюю гайку со стороны верхней части болта действует сила Fo, а со стороны нижней части —сила Р. Так как гайка находится в рав- новесии, то сила, действующая на нее со стороны бруска, F = F0 — P. Такйм образом, действие груза сводится только к уменьшению давления нижней гайки на брусок. При Р > Fo длина болта увеличится и сила, действующая на нижнюю гайку со стороны бруска, исчезнет. На верхнюю гайку будет действовать сила Р. Зависимость действующих на гайки сил от величин груза изображена на рис. 414. - Рис. 414. о § 18. Свойства паров 398. В калориметре будут вода и пар при температуре 100 °C. 399. Сам водяной пар невидим. Мы можем наблюдать только облако мельчайших капель, возникающих после конденсации. При выключении газа исчезают струи нагретого воздуха, ранее обтекав- шие чайник. При этом выходящий из! чайника водяной пар охлаж- дается и конденсируется. 400. На основании уравнения состояния идеального газа p = m/V — pp/RT. Если давление выражать в мм рт. ст., а объем в м3, т<^ мм рт. ст.*м3/(К*моль). Отсюда р = = 1,06р273/Т. При температурах, близких к комнатным, т. е. при Т « 290 К, будет р « р. 401. На первый взгляд кажется, что уравнение состояния иде- ального газа не может дать близких к действительным значений плот- ности или удельного объема насыщенных паров. Однако это не так. 278
Если подсчитать плотность пара по формуле р = m/V = pp/RT и сравнить полученные значения с приведенными в табл. I, то обна- ружится xopotnee совпадение. Объясняется это следующим образом. Давление идеального газа растет прямо пропорционально температуре при постоянном объеме газа и, следовательно, при постоянной плотности. Изображенная же на рис. 155 зависимость давления насыщенных паров от температуры соответствует постоянному объему насыщенного пара и жидкости, с которой он находится в равновесии. При возрастании температуры плотность пара увеличивается, так как жидкость частично переходит в пар. При этом малому изменению объема, который занимает пар, соответствует значительное увеличение его массы. Отношение давле- ния к плотности оказывается, приближенно, пропорциональным тем- пературе, как и в случае идеального газа. Уравнение Клапейрона—Менделеева дает в основном правиль- ную зависимость между р, V и Т для водяного пара вплоть до зна- чений этих параметров, которые соответствуют началу конденсации. Но оно не способно описать процесс перехода пара в жидкость и, в частности, указать, при каких значениях р, V и Т начнется этот переход. 402. При 30 °C давление насыщенных паров р = 31,82 мм рт. ст. Согласно уравнению состояния идеального газа l7 m RT о Л V=------« 296 л. И Р 403. При медленном увеличении температуры давление водяных паров в комнате можно считать неизменным. Влажности Ло=1О% со- ответствует давление паров р — Лоро/(1ОО%), где р0 = 12,79 мм рт. ст.— давление насыщенных паров при 15 °C. При температуре 25 °C дав- ление насыщенных паров р±~23,76 мм-рт. ст. Следовательно, иско- мая относительная влажность A=S- 100% =-^^=5,4%. Pl Pl 404. По условиям задачи относительная влажность и на улице, и в комнате близка к 100%. Однако давление насыщенных паров воды на улице гораздо меньше, чем в комнате, так как температура воздуха в комнате выше, а для выравнивания давлений за счет про- никновения паров сквозь щели наружу требуется значительное время. Поэтому при открывании форточки пары начнут быстро про- никать из комнаты наружу, и белье высохнет быстрее. 405. 1) Уровни воды сравняются, как у сообщающихся сосудов. Водяные пары в левом сосуде будут частично конденсироваться, а в правом сосуде часть воды испарится. 2) Уровни сравняются в результате перетекания паров из одного сосуда в другой. При данной температуре давление насыщенных па- ров одинаково в обоих сосудах у поверхности жидкости и убывает одинаковым образом с высотой. Поэтому давление паров на одном и том же уровне в сосудах неодинаково, что и приводит к перете- канию пара и последующей конденсации его в сосуде с' низким уровнем воды. ' 406. При /2 = 30 °C давление паров оказалось равным давле- нию р2о насыщенных паров (р20 = 31,8 мм рт. ст.) только при дав- лении воздуха в 10 атм. При изотермическом уменьшении давления 279
воздуха в 10 раз объем его увеличится также в 10 раз. Следова- тельно, при атмосферном давлении и температуре 30 °C давление водяного лара было равно р = 3,18 мм рт. ст. Из уравнения Кла- пейрона вытекает, что при температуре ^ = 10 °C давление пара р^рТ^Т* где 7\=283К, а Т2 = 303К. Искомая относительная влажность равна А = ^- 100%=— -J1 100% ю 32,6%, Ро Ро 1 2 где р0 = 9,2 мм рт. ст.—давление насыщенных паров при /1 = 10°С. 407. Давление р = 6,5 мм рт. ст.— это давление насыщенных водяных паров при / = 5°С. Резкое падение давления свидетель- ствует о том, что вся вода перешла в пар. Объем пара, откачанный насосом до лолного испарения воды, У = 3600 л. На основании урав- нения состояния Клапейрона—Менделеева искомая масса воды fn = pVp,/RT к 23,4 г. 408. На нагревание воды до 100 °C необходимо количество тепла Q1~mc Ы = 12 600 Дж. Следовательно, на парообразование будет затрачено Qa = Q — Q. = 11 600 Дж. Количество воды, перешедшей в пар, равно tf{i = Q2A = 5,l г. По уравнению состояния идеального газа это количество пара займет объем RT/p. Пренебрегая уменьшением объема, занятого водой, найдем высоту поднятия поршня: /i = V/S=17 см. Глава ///. ЭЛЕКТРИЧЕСТВО И МАГНЕТИЗМ § 19. Электростатика 409. F = (?2/4ле0г2 = 9000 Н. Сила очень велика. Сообщить телу небольших размеров заряд в один кулон невозможно, так как электростатические силы отталкивания настолько велики, что заряд не сможет удержаться на теле. /Ир/Пр '1 е2 Frp=T-^, f.«=4^7F. Ггр/Рвл=4,5.10-з». 411. Если заряды одного знака, то при а>2 точечный заряд будет двигаться к точке О; при а < 2 — по направлению к точке В (рис. 415). Если заряды разных знаков, то направление движения будет обратным. 412. Условия равновесия подвешенного шарика дают для обоих рассматриваемых случаев уравнения 7\ sin «1—-$т-К^-=0, 1 * 4ле0 2а4 2 ‘_«£±2_ 4ле0 а2 ® 1 Qq К2 4ле0 2а2 2 ’ Q? К2 - а8 4ле0 2а2 2 ’ Г1 cos 4ле0 2а2 2 Т2 sin а2 Tt cos ------- nJlCQ 280
где Tlt Та— натяжения нити, qlt Og —углы отклонения нити, + Qt — Q — заряды закрепленных шариков, -\-q — заряд подвешен- ного шарика, /и—масса подвешенного шарика (рис. 416). Исключая Рис. 416. неизвестные из данной системы уравнений, получим ctg ах—ctg а2 = сtg ах—ctg 2ах = 2 (2 /~2 — 1). / Отсюда ctga1==2(2 У' 2 — 1) £ И 35—16 2 . Следовательно, а1 = 7°56', О2 = 15°52' при mg > ~f 1—~ 1 . 4ле0 а2 \ 4 / а, =82°04', а2= 164°08' при mg < -Д- (1 —. 4TC8q \ 4 / 413. Индуцированные отрицательные заряды на поверхности проводника распределяются таким образом, что результирующая напряженность поля внутри проводника от положительного точеч- ного заряда и индуцированных отрицательных зарядов равна нулю. (Индуцированные положительные заряды уйдут на удаленные края пластинки, и их полем можно будет пренебречь.) Это распределение индуцированных зарядов не зависит от "толщины пластинки. Поместим слева от пластинки на том же расстояний d заряд — q. Ясно, что на левой стороне пластинки индуцированные положитель- ные заряды распределяются таким же образом, как и отрицательные на правой стороне пластинки. От. того, что му поместили слева от пластинки заряд — q, электрическое поле справа от пластинки не изменится. Таким образом, справа от пластинки электрическое поле от заряда -\~q и отрицательных индуцированных зарядов совпадает с полем, создаваемым зарядами -\-q и —q и зарядами, индуциро- ванными на поверхностях пластинки (рис. 417). Если толщина пластинки очень мала по сравнению с d, то мы можем пластцнку — считать бесконечно тонкой, а в таком случае поле, создаваемое ин- дуцированными зарядами, вне пластинки отсутствует. Итак, мы показали, что поле справа от пластинки, создаваемое зарядом + q и индуцированными отрицательными зарядами, совпа- дает с полем, создаваемым точечными зарядами ^-q и — q. Поскольку ,261
в точке нахождения заряда -\-q напряженность поля от индуциро- ванных отрицательных зарядов равна напряженности поля от точеч- ного заряда — qt находящегося на расстоянии 2d от +q, то иско- мая сила притяжения равна F— ^-^ . 414. Т=ла I^nsoma/Qq. 415. Так как Q^>q, то взаимодействием между отдельными элементами кольца можно пренебречь. Выделим малый элемент кольца длины R Да (рис. 418). Со стороны заряда Q на него дейст- . вует сила ДГ=-г-!—> где Д^ = ^Да/2л. Силы натяжения кольца Т уравновешивают ДГ. Из условия равновесия, учитывая* что Да мало, имеем AF = 2T sin (Да/2) « Т Да. Искомая сила является натяжением Т = QqjBnle0Ri, 416. При отклонении нити на. угол ф на заряд q действует сила F, перпендикулярная пластине и равная F=_!____________£_______ z 4л 80 4 [/ (L— cos ф) + яр (см. решение задачи 413). При малых углах отклонения I (1 — cos ф) h 1 <72 и, следовательно, F=-j-гтг^ • При этом период колебаний будет 4Л8() 4ft равен Т = 2я V 92 9 417. Задача формально сводится к определению периода малых колебаний математического маятника. длины Х/2 около положения равновесия в поле силы qEz Т=2л т/^^ИО-^с. Г 2 qE 282
418. Каждый заряд создает в точке D напряженность поля £1==^/4л80а2. Полная напряженность будет суммой трех векторов (рис. 419). Горизонтальные составляющие этих' векторов в сумме дадут нуль, так как они равны по величине и составляют друг с другом углы по 120°. Сами векторы образуют с вертикалью углы 90°—а, где а —угол между ребром тетраэдра и высотой h треуголь- ника АВС. Вертикальные составляющие одинаковы и равны каждая Рис. 419. Рис. 420. q sin а/4ле0а2. Из £±ADE очевидно, что sin а =1^2/3. Отсюда иско- мая напряженность поля F . Кб <7 ' 4ле0 а2 ’ 419. Рассмотрим случай разноименных зарядов qv >0, q2 < 0. Напряженности, созданные зарядами qt и q2, равны соответственно и Е2 = ^2/4л80г2- Как видно из рис. 420, E2 = El~\~ I f2__^2 + Е2 — 2ЕхЕ2 cos ф. Из Л АВС со&ф= 1»Х-8---------. Следовательно, Если заряды одноименны, то £~т~ 4лв0 л г4 .1 2р/ 4ле0 га * 1 Р 420. 1) - q . 4л80 £‘~’ 4л80 (г2+ P/4)s/e ~ 4ле0 f3 ’ (См. рис. 421.) 421. Напряженность поля Е в произвольной точке А на оси кольца может. быть найдена как геометрическая сумма напряжен- I 283
ностей* создаваемых отдельными малыми элементами заряженного кольца (рис. 422). Суммируя векторы напряженности в точке А, следует учитывать только составляющие, направленные вдоль оси кольца. Составляющие векторов напряженности, направленные пер- пендикулярно оси, при сложении дадут нуль вследствие симметрии. Следовательно, напряженность поля в точке А равна n 1 Q 1 Qr 4ле0 У?2 4- г2 4ле0 f2)s/« 422. Сила, действующая на заряд — q, равна F— 1 <&х 4ле#(£»+х«)’/» и направлена всегда к центру кольца; так как то, пренебре- гая в знаменателе х по сравнению с /?, получим р— 4ле0 Я8 Таким образов, сила пропорциональна х и направлена к центру кольца. Под влиянием этой силы заряд совершает колебательное движение, период которого равен Т = 4л }/rns0m^3/qQ. ' 423; В обоих случаях напряженность электрического поля в точке, отстоящей на расстояние г от пластины или нити, может зависеть только от о или т и г. Зависимость от а и т должна быть линейной (принцип суперпозиции), т. е. E — af(r) или £’ = тф(г). Здесь f(r) и <р (г)—некоторые, пока неизвестные функции г. Примечание. Принцип суперпозиции в данном случае за- ключаете^ в том, что напряженность от суммы зарядов равна сумме напряженностей, создаваемых каждым зарядом в отдельности. Как известно, [£]=гЦ-Я* • Размерности о ц- т: lo] = Q/L2, 1®о1 L [t] = Q/L. Очевидно, в случае пластины E — k^!^ а в случае нити E—k^l^r, где kr и &2-"некотоРые безразмерные ко эффици- 284
енты. На основании соображений симметрии легко определить направление напряженности. Напряженность Е направлена перпен- дикулярно к пластине или нити. Теория дает для коэффициентов X и k2 значения ^ = 1/2, k2=l/2n. 424. Ввиду очень малой толщины пластинки мы можем считать, что заряд распределен равномерно на двух поверхностях, площадь каждой из которых равна ab. Таким образом, поверхностная плот- ность заряда о = ^/2оЛ. Поле внутри металла будет равно нулю, вне металла напряженность равна р — 2 2е02а6 2ъ0аЬ * F р _(Qi+<W р (Qi+Qi) 425- Ев~ 2e0S • Ес~-’ Еа =----------------2^S~ • Напряженность считается положительной, если она направлена слева направо. 426. Выделим на проводнике столь малый участок АВ, чтобы его можно было считать плоским (рис. 423). Поле вблизи этого участка можно представить как суперпозицию двух полей:того, г 1 If что создается зарядами этого Т Т 1 участка (векторы Elt Ej), и I ’ R того, что создается остальными 4^*- . Г зарядами проводника (векторы Е*, Е2). Так как участок АВ lrF можно считать плоским, то | ♦ ч Е1=Е'1 = о/2г0. Кроме того, рис 423 так как поле, создаваемое заря- - ‘ дами, находящимися вне участка Л В, является непрерывным, то E2 = eJ. Наконец, так как внутри про- водника поле отсутствует, то Е1 = Е2, откуда, учитывая предыдущие равенства, получаем Е1 = Е2 = о/2е0. Следовательно, искомая напря- женность Е — Ej-f- Еа = o/28q -|~ о/2е0 = о/вд. 427. Если | х | С а/2, то Е — рх/е0. Если | х | > а/2, то Е = ра/2в0. 428. Найдем напряженность поля на расстоянии г < R от центра шара. С этой целью проведем из центра шара сферу радиуса г. Все заряды, которые находятся внутри сферы, дают в интересующей нас точке такую напряженность, как если бы они находились в центре, а от зарядов, находящихся вне сферы, напряженность поля равна нулю. Таким образом, г 1 4 я 1 _ Рг 3 ПГ р г2 ~ Зе0 ‘ Вне шара (г > R) напряженность равна . Е = Q/4ji€€r2, где Q —заряд шара. (График E — f(r) см. на рис. 424г) 429. Найдем напряженность поля на расстоянии г < R от оси цилиндра. Заряды, которые находятся внутри цилиндра радиуса г, ' х 285
создают на расстоянии г от оси цилиндра такую же напряженность, как если бы они находились на оси. Заряды же, находящиеся вне цилиндра, поля не создают. Следовательно, если г < R, то Е __ т лг2р _ рг ~~ 2леог —2л80г ~~ * Если же г > R, то Е = т ______л/?2р/?2р 2ле0г 2леог 2е0г* 430. E^ = pR?|/3e0 —рг/3ео = ра/3ео. (См. рис. 425 и решение задачи 428.) 431. См. решения задач 430 и 429. I 432. Молекула будет притягиваться к заряженному цилиндру. Сила притяжения равна Р __ ту / 1_____1 \ тдК “2Л80 \ Г г+Ху =3=2л80Г (г-|-^) * В этом выражении мы можем пренебречь величиной Х (Х^ 10~8 см) по сравнению с г (г не может быть меньше радиуса цилиндра). Окончательно для F получаем выражение F=T(/X/2n80r2. 433. В начальный момент силы, действующие на обе молекулы, одинаковы. При приближении к цилиндру сила Flt действующая на молекулу, с постоянным электрическим моментом, растет пропорцио- нально 1/г2: F1 = 2nqk/r2 (см. задачу 432). Сила F2t действующая на «упругую» молекулу, растет быстрее, пропорционально IJr3 (за счет непрерывного увеличения электрического момента этой молекулы). Массы молекул одинаковы, поэтому ускорение второй молекулы при приближении к цилиндру нарастает быстрее, чем у первой, и она быстрее достигнет поверхности цилиндра. 434. Ввиду того, что а и b много больше с и dt мы можем счи- тать пластинку бесконечно большой. Учитывая, что напряженность поля' от нескольких зарядов равна сумме напряженностей, создавае- мых каждым из этих зарядов, и воспользовавшись результатами 286
решения задач 413 и 424, получаем значение искомой силы: 2еоа6 I6ji£Qd2 Первое слагаемое соответствует силе отталкивания, второе — силе притяжения. Положительно заряженная пластинка будет притягивать точечный положительный заряд, если Q2/16ne0d2 > ^Q/2eoa^, т. е. если Q/d2 > Sttqlab. 435. Вне шара потенциал <р = Q/4ne0r = ^3р/3еог. Для того чтобы определить потенциал внутри шара (при г < 7?), надо к потенциалу ср = Q/4jt80/? добавить величину, численно рав- ную- работе, производимой полем над единичным положительным зарядом при его перемещении по радиусу от г до /?. Эта работа равна заштрихованной площади на рис. 424 (см. задачу 428). Вычис- ляя, получим ср=^(3/?2—г2) р/6е0. 436. Потенциал всех точек шара одинаков. Для решения задачи достаточно найти потенциал одной точки. Проще всего найти потен- циал центра шара. Он равен потенциалу, созданному,в центре шара точечным зарядом q>=^/4n8od, плюс потенциал, созданный зарядами, возникающими на поверхности шара вследствие электростатической индукции. Но этот последний потенциал равен нулю, так как сум- марный заряд на сфере равген нулю и все элементы заряда нахо- дятся на равном расстоянии от центра. Следовательно, потенциал шара m = ^/4ne0d. ' 437. Потенциал шара ср равен 1 Г <7_______ 'Р 4л80 \ R 4- г г ) ’ Поскольку шар заземлен, его потенциал равен нулю, т. е. q/(R + r)~ — Q/r = O. Следовательно, Q = n“i—q. К -\-r 438. На суживающейся части трубы возникнут положительные индуцированные заряды. Под их влиянием электрон начнет уско- ^Яггься. Кинетическая энергия электрона будет увеличиваться за счет уменьшения потенциальной энергии системы электрон —труба. 439. <7 = -Q -К Лз — А1 Лз 440. Для равновесия заряда q необходимо, чтобы заряды (—Q) находились на равных расстояниях а от него (рис. 426). Сумма сил, действующих на зяряд (—Q), также равна нулю: Q2/4a2—Q^/a2=0. Отсюда q — Q/4. Расстояние а может быть любым. Равновесие не- устойчиво, так как при смещении заряда — Q вдоль ООГ на отре- 1 О2 зок х от заряда q сила притяжения гу=f действую- щая со стороны заряда q, меньше силы отталкивания Р 1 Q 4л80 (2а 4-х)^ 287
и заряд —Q уходит еще дальше от положения равновесия.. При смещении заряда —Q вдоль ООХ на х к заряду q Eq > Fq для . x^at и система не возвращается к положению равновесия. Также нарушает равновесие, как нетрудно видеть, произвольное перемеще- ние заряда q< Потенциальная энергия заряда —Q в поле двух других заря- дов равна w =_______________Q Q8 Зу-а 1 4л8о \ у _а+у) 16л80у(а+у)’ где г/—расстояние между зарядом q и одним из зарядов — Q. За- висимость Wr от у при изображена кривой А ВС для одного заряда и кривой DEF для другого (рис. 426). Энергия заряда q при неподвижных зарядах —Q равна w ___<L_V______91____“ 2 4ле0 \a—z a-\-z) 8ле0 a2— z** где z —смещение заряда q от положения равновесия. При измене- нии z от 0 до а энергия меняется в соответствии с кривой MNP (рис. 426). Характерно, что максимумы всех трех потенциальных кривых соответствуют положению зарядов при равновесии. Именно с этим связана неустойчивость равновесия. 441. Нет, не может. Для того чтобы, к примеру, положитель- ный заряд находился в состоянии устойчивого равновесия, необхо- димо, чтобы при смещении заряда в любом направлении на него действовала сила, возвращающая в положение равновесия. Следова- тельно, силовые линии электрического поля должны сходиться в точке, в которой расположен заряд. Но силовые линии электриче- ского поля начинаются на положительных зарядах и оканчиваются на отрицательных. В точке же, где расположен рассматриваемый заряд, отрицательных зарядов нет, и, следовательно, силовые линии внешнего по отношению к заряду поля не могут сходиться в точке, где он расположен. 442. Энергия заряженного шара равна работе, которую могут совершить заряды, находящиеся на шаре, если они покинут его и удалятся на бесконечно большое расстояние. Пусть с шара каждый раз удаляется на бесконечность порция заряда в q единиц (<?<^Q). 288'
При удалении л-й порции, когда заряд шара станет равным Q— nqt электрическое поле совершит работу, равную ДД = » (Q-n^ 4ле0 R Работа, затраченная на удаление М порций заряда, где N = Q/q, равна л__1_ГЗ^+ 4ле0 [ К 4 При N—>оо (q—>0) Л = О2/8лео/?. Следовательно, энергия заря- женного шара равна W = Q2/8n&QR. (Эта энергия называется соб- ственной.) Тот же результат можно получить, используя график изменения потенциала шара при уменьшении заряда. График будет представлять собой прямую линию, проходящую под некоторым углом к оси абсцисс, а работа будет численно равна площади, огра- ниченной графиком и осями координат. ' 443. Энергия всей системы зарядов равна сумме собственных энер- гий зарядов, находящихся на первой сфере (^71 = Qj/влео/?!) и на второй сфере (ТГ2 = Q2/8ne0/?2)» а также энергии взаимодействия заря- дов первой сферы с зарядами второй сферы. Эта энергия взаимо- действия равна произведению заряда Qa на потенциал, создаваемый на поверхности сферы радиуса R2 зарядом QP Таким образом, иско- мая энергия W всей системы равна W = 1 4ле0 В случае, когда Qx —— Q2 = Q (сферический конденсатор), W=QL(1_______LV 8ле0 \ Ri Ra )' < Г „S „8 Л» - \ Г2 7 \ , Г3 J \ ГП 7 J 445. Как всегда, считаем потенциал на бесконечности равным нулю. Тогда потенциалы пластин равны соответственно +£7/2 и —£7/2, причем U = (ЦС. Потенциалы в точках первоначального положения электрона соответственно равны 0, +£7/4, —£7/4. Начальные значе- ния полной энергии электрона равны: 1Ч /пц© о mvi eU mvi , eU О 2 ’ 2 ' 4 ’ 2 10 б. Б. Буховцев и др. 289
Конечные скорости v2, v3 определяются на основании закона сохранения энергии: 2 2 . ч ти0 mvf i) ‘’x=v.: 0 mvQ eV mv3 mvl-~eQ/2C 2) ~~2 T““’ V*~ ? 1 o mvf eU mvf -if mvo-\-eQ/2C 3) ^3= v ----------------. В первом случае конечная скорость равна начальной, во втором случае меньше ее, а в последнем больше. Во всех случаях скорость первоначально растет (во время движения внутри конденсатора), а затем начинает убывать. 446. Расстояние между зарядами станет минимальным в момент, когда их скорости сравняются, т. е. когда относительная скорость зарядов станет равной нулю. Очевидно, что скорость зарядов при минимальном расстоянии между ними, согласно закону сохранения импульса, равна v = (m1v1—/и2У2)/(т1 + тг)- Используя также закон сохранения энергии • I msv* I __(/»!+ , <71<7г 2 2 ^4^ 2 "Г" 4л80г2* определим минимальное расстояние г2 между зарядами: 2 1 глерГП^ (Е1+^)2 г * "Г (mi + m2) qrq3 1 447. Энергия взаимодействия равна половине энергии взаимодей- ствия дйух точечных зарядов +q и —q, находящихся на расстоя- нии 2d, т, е. W = — q2/8ne0d = —q2/16n80d. Скорость найдем из закона сохранения энергии. Когда заряд нахо- дится на бесконечно большом расстоянии от пластины, его скорость равна нулю и энергия взаимодействия W тоже равна нулю. Таким образом, mv2 2 _____е_\ 16n80d J mvo 2 Я* 8n&omd ’ 448. Работа по перемещению заряда —q пропорциональна раз- ности потенциалов между точкой О и весьма удаленной от кольца точкой Л, лежащей на оси (рис. 160). Потенциал на бесконеч- ности принимаем равным нулю. Потенциал точки если расстоя- ние ОЛ^>/?, мы можем считать равным нулю. Потенциал в точке О найдется суммированием потенциалов, созданных отдельными ма- лыми элементами кольца: _ 1 уч Д? __ я То 4ле02-4 2? 4л80/?‘ 290 ч
Используя закон сохранения энергии ту2/2 = ?2/4л807?, найдем V — У ?2/2л80т/?. 449. Полная энергия заряда равна W =mv2l2 —qQ/4neQR. Если W 0, то заряд уйдет ,в бесконечность. При IF=0 скорость заряда на бесконечно большом расстоянии от центра кольца будет равна нулю, а при W > 0 —отлична от нуля. Если же W < 0, то заряд будет совершать периодическое движение вдоль оси кольца. Наиболь- шее расстояние г, на которое при этом удалится заряд от центра кольца, -можно найти из закона сохранения энергии: my2 qQ_________ _______qQ______ 2 4ne0R 4лв0 |Л/?2+г2 * г о - /"__________J__________1 1/ /2л80/пу27? .у Г I qd - ) 450. Энергия заряженного шара W — Q2/8ne0R = 2ле07?ф2, где 7? —радиус шара, а ф—его потенциал. При разряде эта энергия выделится в форме тепла. Вычислив, найдем 1Г = 0,55 Дж. 451. Пусть первоначально заряды шариков были qt и q2. Тогда работа Л1 = ^2/4л80/, гДе I—расстояние между шариками. Заряды шариков после соединения стали одинаковыми: ^ = (^1 + ^2)/2, а ра- бота Л2 = (qt4-q2)2/ 16jt80Z. Нетрудно видеть, что А2> АКроме того, в проводнике при соединении шариков выделяется тепло Q. Однако полный запас энергии шариков по закону сохранения энер- гии должен быть одинаков в обоих случаях. Так как работа А± и, соответственно, А2 представляет собой потенциальную энергию вто- рого шарика в поле первого (в первом и во втором случаях), то >4Х+^1 = Л2+0 + 1Г2, где = собственная 1 ( Q2 q2 \ энергия шариков до соединения, ^2 — 4^ ( 27’4“ 2r J— собствен- ная энергия шариков после перераспределения зарядов (см. за- дачу 442)* Энергия, выделившаяся в форме тепла, равна !-)• 452. Предположим, что радиус оболочки увеличился на д, где 6 —сколь угодно малая величина. Тогда растягивающая сила совер- шат работу Л = 4л7?2/6, где сила, приходящаяся на единицу площади. Эта работа совершается за счет уменьшения электростати- ческой энергии. Вначале электростатическая энергия равна Q2/8n807?t после растяжения С2/8ле0 Изменение энергии Q2 Q2 _ Q2 6 8ле07? ~ 8л80 (7? + 6) ~ 8л80 7? (7? + 6) равно работе Л, т. е. 4л7?2/6 = Р26/8л807?(7?+6). Учитывая, что величина 6 сколь угодно мала, получаем для силы следующее выра- жение: f — Q2/32n2s07?4 = a2/280. Здесь через о = С/4л7?з обозначена плотность электричества, т. е. заряд, приходящийся на единицу площади. 10* 291
Можно определить искомую силу и непосредственно. Рассмотрим на сфере малую площадку S (рис. 427). Найдем напряженность электрического поля на рассматриваемой площадке, создаваёмую всеми зарядами, за исключением зарядов, находящихся на самой площадке. Для определенности рассмотрим случай, когда сфера несет положительный заряд. Обозначим через Е2 напряженность электри- ческого поля, создаваемого зарядами, находящимися на рассматри- ваемой площадке. Так как внутри сферы результирующая напряжен- ность равна нулю, то Е1 — Е2. Результирующая напряженность на сфере E1JrE2 = С/4л80/?2. Следовательно, 2Et — Q/4ne0/?2 = o/80. Отсюда Е1 = о/2е0. Для того чтобы определить силу, действующую со стороны всех зарядов, не находящихся на площадке, на заряды, находящиеся на площадке, надо напряженность Ег умножить на величину электрического за- ряда площадки aS: F = E1oS = a2S/2eo. Сила, приходящаяся на еди- ницу площади, будет равна f — a2!2^. 453. Q < 8л/? Ve0Ra. Рис. 428. 5r=^ 454. Пусть разность потенциалов на клеммах батареи равна U, а заряд батареи равен Q. Найти емкость батареи — значит найти емкость такого конденсатора, который имел бы при напряжении U тот же заряд Q на пластинах, что и батарея. Следовательно, Со = = С/£Л причем Q=<7i4-^2 + ^4 = ^4 + ^ + ^e (рис. 428), а ^ = (/4 = = q^C. Работа сил электростатического поля при обходе по замкну- тому контуру равна нулю. Отсюда ^1/С—q2l С—q${ С = 0, q2/С — q4/C-^- qb/С = О, <7з/С—<7б/С+qdC =0. Кроме -того, проводник, соединяющий второй, третий и пятый кон- денсаторы, электронейтрален. Следовательно, ?з+?5 ~~<72=О- Решая эти уравнения, получим qr =q2 = q5 — ^3 = 0. Следова- тельно, С0 = 2С. 455. Пусть батарея конденсаторов заряжена. Тогда точки /, 2, 3 будут иметь одинаковый потенциал и их можно будет соединить между собой. Так же можно соединить точки 4, 5, 6 (рис. 162). В результате получим эквивалентную схему, изображенную на рис. 429. Емкость отдельных участков ЗС, 6С, ЗС. Общая емкость найдется из формулы 1/С0 = 2/ЗС+ 1/6С. Отсюда С0=1,2С. 456. При пробое искровых промежутков происходит автоматиче- ское переключение параллельного соединения конденсаторов на по- 292 ,
следовательное. При этом напряжение между соответствующими обкладками конденсаторов растет, так как емкость системы падает. Действительно, из-за большого сопротивления проводников АВ и CD можно пренебречь токами, протекающими по ним за время разряда, и рассматривать их как изоляторы, через которые конденсаторы не разряжаются. Эквивалентная схема после пробоя первого искрового проме- жутка изображена на рис. 430. В результате пробоя первого проме- жутка разность потенциалов на втором промежутке будет равна сумме напряжений на первом и втором конденсаторах, т. е. увели- чится вдвое. Вследствие этого начнется пробой второго промежутка. В момент пробоя n-го промежутка напряжение на нем достигнет величины V~nVQ. Сопротивления проводников АВ и CD должны быть большими, чтобы за время последовательного соединения пла- стин конденсаторов при пробое промежутков конденсаторы не успевали разряжаться через эти проводники. 457. Да, будет. Каждая из пластин об- ладает определенной, обычно небольшой ем- костью относительно земли (вблизи краев пла- стин силовые линии искривляются и достигают земли). Эквивалентная схема изображена на рис. 431. Емкость пластин конденсатора отно- - . . сительно земли изображена в виде малых ем- ь/ггг* ТТТ2^ костей Ci и С2. При замыкании левой пласти- ны нейтрализуется часть заряда, находящегося на ней. Эго же произойдет при замыкании пра- ' вой пластины. Конденсатор будет разряжаться Рис. 431. тем медленнее, чем больше емкость конден- сатора по сравнению с емкостью пластин относительно земли. 458. Полная энергия двух конденсаторов до соединения равна «7o = y(C1t>?+C2U?). После соединения 1 Q* 1 2 C, + Ct~ 2 293
Легко показать, что > IF. Разность энергий =27§+сЗ(^+и*~2и^^> > 0: при U1 — U2 Wq — W = 0, а при С1 — С2 и t/2 = 0 IF0 = 2^. Электро- статическая энергия уменьшилась вследствие того, что при соедине- нии этих конденсаторов проводниками заряды перетекали с одного конденсатора на другой. В проводниках, соединяющих конденсаторы, выделялось при этом тепло. Количество выделенного тепла не зави- сит от сопротивления соединительных проводов. При малом сопро- тивлении проводов в них протекают большие токи, и наоборот. 459. Рассмотрим для простоты диэлектрик в форме однородного сильно вытянутого параллелепипеда (рис. 432). Разложим поле Ео, в которое помещен диэлектрик, на составляющие, направленные вдоль стержня и перпендикулярно ему. Эти составляющие вызовут появ- ление связанных зарядов на поверхностях ДВ, CD, ВС и AD. Поле Рис. 432. связанных зарядов между поверхностями AD, ВС и АВ, DC ослаб- ляет составляющие поля Ео внутри диэлектрика, причем составляю- щая, перпендикулярная стержню, ослабляется сильнее, так как связанные заряды на поверхностях AD и ВС расположены близко друг к другу и их поле подобно однородному полю плоского конден- сатора, в то время как заряды на поверхностях малой площади раздвинуты далеко друг от друга. Поэтому полное поле внутри ди- электрика не будет совпадать по направлению с полем Ео. Следова- тельно, возникающие диполи будут ориентированы не вдоль Ео, а вдоль некоторого направления ОР, составляющего угол Р с Ео. (Это относится как к обычным, так и к дипольным молекулам.) В элект- рическом отношении поляризованный диэлектрик можно рассматривать как большой диполь, составляющий угол р с полем Ео. В этом поле он будет поворачиваться до тех пор, пока не установится вдоль поля. Поле связанных зарядов является внутренней силой и не может вызвать поворота диэлектрика. 460. Вследствие поляризации заряды раздвинулись. Внутри шара с центром О' содержатся отрицательные заряды, а с центром О — положительные заряды. Объемная плотность зарядов p — Nq. Рас- стояние между центрами О' и О равно I. Напряженность в про- извольной точке, находящейся в области перекрытия двух шаров, можно найти путем простого геометрического построения, приведен- ного рис. 433. Из этого построения следует, что напряженность Е = р//Зе0. Она постоянна ц направлена против вектора р. Это
однородное поле создается, в сущности, отрицательными и поло- жительными зарядами, находящимися вне области перекрытия. Так как 10~8 см то можно считать, что это поле создается поверхностными зарядами, плотность которых равна а — Nql cos 0 = Np cos 0. 461. Нетрудно заметить, что диэлектрик будет поляризован одно- родно и дипольный момент р любой молекулы направлен вдоль внеш- него поля. Результирующее поле внутри диэлектрика имеет напря- женность Е' — E — Np/Зе^. Так как р = ае0£', где а—коэффициент поляризуемости молекулы, то р = а80Е/(1 -f-Wa/3), Е' = Е/(1 + aAf/3). Если учесть, что. е = 1 + Na, то окончательно для Е' получаем вы- 3 ражение следующего вида: Е' = Поле вне шара, создаваемое поляризованным диэлектриком, экви- валентно полю двух точечных зарядов — Q и + Q (Q = 4/3nR3Nq), помещенных в точках О' и О, Так как расстояние O'O — l<^.R (/ « 10"8 см), то для вычисления полной напряженности в точках А, В, С и D можно воспользоваться ответом к задаче 420: 1 4 Ч 462. Поверхностная плотность зарядов изменяется на сфере по закону o = 3Ee0cosO, где 0 —угол между радиусом-вектором и век- тором (— Е). 463. a —4^2“l“38o£ cos 0, <у0 = ^р = Зе0Е, 0 2 4 О Еа = Ес = Е+т-^+nR3Np = ЗЕ + -т-—. л с 1 4ле0/?2 1 4ле0/?3 3 4л80/?2 9 * г _ с _ с । Q 380Е___ Q Зе° — 4jig^2 . 464. Е = —#р/280. г 295
W. p _F -F4-^1(L_____1 \ -£ | 1 - E 1 NP Ea-Ec~E+ 2eo ^ + zj-£+/? 2g# ^(/? + x)“£ + 2e0 ’ Wp = (8-l)eo£'=2^|8o£, ea=ec=e+^e=^-e, p p _p Np E 2 466. (j = 2s0£ cos 04-х/2л/?, £л = Ес = х/2л8()7? +2£, £^ = £/) = х/2л80£. 467. Поле в диэлектрике создается зарядом q и поляризацион- ным зарядом q'. Напряженность поля в произвольной точке Л, нахо- дящейся вне шара на расстоянии г от его центра, равна е=( \ Г2 Г2 ) 4Л80 * Напряженность Е связана с напряженностью £0 поля в вакууме, создаваемого зарядом qt соотношением Е==Е. = Я_____1 е ег2 4яе0‘ Следовательно, 1 ( Я Я' \ Я 1 4л80 \ Г2 Г2 J 8Г2 4л80 ’ гх ,8—1 Отсюда имеем q =~—Я- * 468. Обозначим через q величину поляризационных зарядов. Применяя принцип суперпозиции, получаем при г > г2\ При Г1 < г < г2 E-(q И 1 Q \ Г2 Г2 / 4л80 4эТ880Г2 * g___J Из (1) следует, что Q. Следовательно, в области rt < г < г2 г Q , <?(«-!)! 4ле. 8Г2 296
При г < Г1 4л80Г2 ’ \ _L= 1 Г Q I Q(e-i)f JL-JL\1 * \г Г1 ‘/’а) 4лв0 4лв0 [ г ' в \.r9 fi/J ' 469. Напряженность поля в пространстве между сферами равна - Е= Q ~(Q 1 ег24ле0 \ г2 г2 J 4ле0 ’ откуда (/= Q (е—- 1)/е. Следовательно, плотность Q(8-l) Q (в-1) 1 4лг28 ’ 2 4jir*e Поле вне сфер отсутствует: £ = 0; <р=0. Потенциал <р в области гх < г < г2 равен Т 4Л80 \ Г Г ) 4л808Г Из последнего выражения легко найти разность потенциалов сфер: « (1-1V ' т 4лев8 \ гг г2 J 4лв0 в г±г9 Следовательно, емкость, являющаяся коэффициентом пропорциональ- ности между зарядом и разностью потенциалов обкладок конденса- тора,, равна 470. а) Емкость конденсатора будет равна емкости параллельно соединенных конденсаторов, из которых один заполнен диэлектри- ком, а,другой нет, т. е. eoeS/i j S(l— Zi)e0____b0S 11 t /n 1Ч /x | ~ dl + dl — d |1 + I p б) Электрическое поле между обкладками конденсатора не изме- нится, а следовательно, не изменится и емкость, если верхнюю по- верхность диэлектрика покрыть бесконечно тонким слоем проводника. Поэтому искомая емкость будет равна емкости двух последовательно соединенных конденсаторов: г_ СОСХ р__ fyS __еое^ С~Со+С,’ д С® d—di Cl dx ’ Следовательно, с_ bobS d1+e(d-d^' 297
471. Плотность поляризационных зарядов о равна а-2— z г+1 S 472. Обозначим искомую полную напряженность поля в диэлект- рике через Е. Расстояние %, на которое раздвинулись заряды в каждой молекуле, определяется из соотношения kk~qE. Следова- тельно, £ = £0-£1 = £0-^£. • Сл« Qi___£ 8q 8q Диэлектрическая проницаемость 8 определяется из соотношения * 0^ E = Eq/&. Отсюда е = 1+-~гл. 8q/c 473. При раздвижении в молекуле зарядов — q на расстоя- ние % совершается работа kk2/2. Энергия, запасенная в диэлектрике, = (k№/2) N, где N = Sin — Vn—число молекул в объеме V диэлек- трика, находящегося между пластинами конденсатора. Таким образом, Wr = n (kk2/2)V, Так как к—qE/k, то W\ — n(q2E2l2k)V. Выражая nq2lk через 8 (8—1 = ?2п/е0£), получаем для значение Полная энергия конденсатора равна W'=^=^£2V. ZL> z Эту полную энергию W можно представить в виде суммы чистЬ g £2 электростатической энергии и энергии, запасенной в 8 — 1 диэлектрике, Wr = 80 —• 474» Сила, действующая на единицу площади диэлектрика, равна е2__ 1 Р = — 475. Предположим для простоты рассуждений, что две парал- лельные металлические пластины, несущие заряды+Q и —Q, поме- щены в жидкий диэлектрик. Напряженность электрического поля между пластинами £ = Q/8qsS. Напряженность поля, создаваемая каждой пластиной, будет равна Е1==£2= Q/288^S. Определим силу, действующую со стороны., например, первой пластины на вторую. Для этого надо напряженность поля, создавае- мого первой пластиной, умножить на величину заряда, находящегося на второй пластине. Таким образом, Е = Q2/2soeS. Предположим, что первая пластина закреплена, а вторая может очень медленно перемещаться (изменением механической энергии диэлектрика пренебрежем). Работа, которую может совершить элект- рическое поле при перемещении пластин до непосредственного сопри- косновения, будет равна произведению силы F (сила F постоянна) 298
на величину перемещения d, т. е. A ~Fd~Q2d/2&0€,S. Эта работа совершается за счет убыли электрической энергии конденсатора. Таким образом, электростатическая энергия будет равна W = Q2d/2e^S = Qi/2Ci или W = QU/2, где (/ — разность потенциалов. Полученная формула справедлива для любых диэлектриков. 476. Энергия конденсатора в случае, когда диэлектрик втянут на расстояние х внутрь конденсатора, будет равна 1 1 . 2С М 1 + (е- 1)у ’ так ка^ 1 + (е—, a Q=^- U (см. задачу 470). Если х увеличится на б, то энергия уменьшится и будет равна HZ 1 * * 1 + (е-1Г-Н‘ Разность энергий (_____________________________________Li_____________ будет равна работе искомой силы F на пути 6. На этом пути зна- чение силы, вообще говоря, изменяется, но если 6 взять достаточно малым, то можно написать Отсюда следует, что e0S£/2 е-1 2dl р+(е-1)±р ’ если пренебречь 6 в знаменателе. Надо оговориться, что, в то время как при подсчете энергии мы считали, что поле внутри конденсатора однородно, и пренебрегали краевыми эффектами, для физического объяснения силы, действующей на диэлектрик, обязательно надо принять во внимание неоднородность поля у краев. 477. Энергия конденсатора в случае., когда диэлектрик втянулся на расстояние х внутрь конденсатора, будет равна W1 =1 U*C = 1 (/’ /1 +(8- 1) у 1 . Если х увеличится на б, то энергия конденсатора увеличится на . ₽99
Заряд на обкладках конденсатора при перемещении диэлектрика на расстояние 6 увеличится при этом на Q4-Qi=8-^(e-Dy. Работа, совершаемая батареей при перемещении такого количе- ства электричества, будет равна 4=(Qt_Qt)i/=£!^!£(e_l)l. Эта работа частично идет на приращение электростатической энергии конденсатора, а частично —на втягивание диэлектрика. Обозначим, как и в предыдущей задаче, через F силу, с которой втягивается диэлектрик в конденсатор. Тогда на основании закона сохранения энергии имеем Я = №2 —+ т. е. (8 -1) 7-=^^ (е - 1) 7 + Р Отсюда В этом случае, как [ мы видим, сила постоянна и не зависит от х. 478. Жидкость между обкладками конденсатора (рис. 434) подни- мается под действием силы v F = S°U.2.— (8—1) 2dl ( h Когда жидкость достигнет максималь- ной высоты Я, ее кинетическая энер- гия будет равна нулю, а потенциаль- ная энергия увеличится на величину FH. Из соотношения FH^Sd^-pg^- находим Н: Н = г»и*(г-\)1(Р pg. В этом положении сила тяжести больше силы F, и вследствие этого жидкость начнет опускаться. При высоте столба жидкости /1 = Я/2 = 80^2(е-1)/2б/2р^ кинетическая энергия максимальна, а сила F равна силе тяжести. Жидкость между пластинами конденсатора, продолжая опускаться, достигнет уровня жидкости в сосуде. Уровень жидкости в сосуде вследствие больших размеров сосуда практически не перемещается. Таким образом, высота столба изменяется периодически в пределах от 0 /До Я. 300
Вследствие трения амплитуда колебаний жидкости будет умень- шаться, и уровень столба в конце концов установится на высоте A = e0t/2(8-l)/2d2pg. Измерив высоту подъема ht можно определить диэлектрическую про- ницаемость жидкости 8. 479. Свободный заряд 4~ Q создает в диэлектрике однородное электрическое поле, напряженность которого E0 = Q/2sqS. Вслед- ствие этого слой диэлектрика толщины d поляризуется. На двух плоскостях, ограничивающих этот слой снизу и сверху, появляются поляризационные заряды. Плотность поляризационных зарядов равна е-l „ е —1 Q а‘---— 8»£«------- 2S ‘ 8 — 1 О Заряды Q и отрицательный заряд —-—действуют на поло- жительный поляризационный заряд, находящийся на поверхности жидкости, с силой F, направленной вверх: Q2 82—1 8S80 82 * Вследствие этого уровень жидкости над пластиной поднимается на высоту h: Q2 е2-1 •480pgS2 82 ’ § 20. Постоянный электрический ток 480. Докажем сначала, что напряженность электрического поля во всех точках, лежащих в плоскости сечения 00', направлена пер- пендикулярно этой плоскости. Для доказательства возьмем произвольную точку в плоскости сечения и две малые площадки, произвольно, но симметрично рас- положенные на цилиндре относительно сечения 00'. Легко видеть, что результирующая напряженность поля, создаваемая зарядами на этих площадках, будет направлена вдоль оси цилиндра (рис. 435). Так как для каждого элемента найдется симметрично расположенный относительно плоскости сечения другой элемент, то из этого следует, что напряженность, создаваемая всеми элементами, будет, парал- лельна оси цилиндра. Покажем теперь, что напряженность будет одинакова во всех точках, которые отстоят от оси цилиндра на одно и то же расстоя- ние. Пусть А и В—две такие точки (рис. 436). Напряженность поля внутри цилиндра не изменится, если, помимо имеющегося заряда, каждой единице площади .поверхности цилиндра сообщить один и тот же дополнительный отрицательный заряд, чтобы плотность зарядов в точке С была равна нулю. Это очевидно из того факта, что поле внутри бесконечного, равномерно заряженного цилиндра равно нулю. В этом случае распределение плотности зарядов на 301
поверхности цилиндра (рис. 436) будет иметь тот же самый вид, что и на рис. 167. Следовательно, напряженность в точках А и В одинакова. Остается показать, что напряженность поля в точках, отстоящих на разные расстояния от оси цилиндра, одинакова. Для доказатель- ства рассмотрим контур fi/fLD (рис. 437). Как известно, работа по замкнутому контуру в случае электростатического поля равна нулю. 0Г С' Рис. 436. Работа на участках KL и DB равна нулю, поскольку напряженность поля перпендикулярна пути, работа на участке ВЛГ равняется — а на участке LD равна E^l (по доказанному выше Eq-Er, Eq=El). Следовательно, —E^l-]- Ед1 = 0, т. е. Eq = Ejj. Таким образом, доказано, что напряженность электрического поля внутри цилиндра будет одинакова во всех точках и эта напря- женность направлена вдоль оси цилиндра. Заметим, что подобное распределение заряда на поверхности проводника возникает при прохождении по нему постоянного тока. Рис. 437. Рис.- 438. 481. При прохождении по проводнику постоянного тока электри- ческое поле внутри проводника постоянно и направлено вдоль него. Работа электрического поля при перемещении заряда вдоль замкну- того контура abed (рис. 438) равна нулю. Участки ad и be будем считать столь малыми, что работой на них можно пренебречь. Зна- чит, работа вдоль ab равна работе вдоль de. Поэтому тангенциаль- ная составляющая напряженности поля вблизи поверхности провод- ника должна равняться напряженности поля внутри него. 482. Распределение силовых линий изображено на рис. 439. Увеличение наклона линий вблизи закругления объясняется тем, что тангенциальная составляющая напряженности поля у поверх- ности проводника постоянного сечения постоянна, а нормальная со- 302
ставляющая убывает по мере приближения к закруглению, так как разность потенциалов между соответствующими участками, лежащими на противоположных сторонах дуги, уменьшается. 483. При последовательном соединении сопротивление цепи R — *01 + *02 + «1*01* + «2*02*• С другой стороны, можно написать R = (14-а7), где *o=*oi+*O2> а а' —искомый температурный коэффициент. Отсюда / _ *01«1 + *02«2 *01+*02 ' При параллельном соединении п *01*02 U +«!*) U +«2*) ____n /1 f „ГЦ. \ flei(l+ai0+*os(l + a»0-Ко( + ’’ где *о = *о1*ог/(*о1 + *02)- Опуская члены,-пропорциональные про- изведениям температурных коэффициентов, как малые, получим а" — *02а* + *oia2 *01+ *02 484. Точки А и С имеют одинаковые потенциалы, так как со- единены проводом, сопротивлением которого можно пренебречь. Рис. 439. Точно так же одинаковы потенциалы точек В и D. Поэтому концы сопротивлений А, С и, соответственно, В, D можно считать соеди- ненными вместе. Таким образом, сопротивления АВ, СВ и CD соеди- нены параллельно. Соответствующая эквивалентная схема представ- лена на рис. 440. Полное сопротивление равно /?/3. 485. По закону Джоуля—Ленца количество тепла при заданном U будет тем больше, чем меньше сопротивление. Минимальное со- противление куска проволоки, который может быть включен в сеть с напряжением U, равно r — Длина этого куска l = (L/R) г = = LU/IqR. Если разрезать теперь проволоку на такие куски и включить их параллельно, то к каждому из участков будет прило- жено наивысшее возможное в схеме напряжение U и по каждому будет протекать наибольший возможный ток /0. Поэтому на каждом из участков будет выделяться максимальная тепловая мощность ?0 = U^/r = I$r — IQU, а на всех участках —мощность q = nq0, где п — наибольшее целое число кусков длиной I, которое можно полу- чить из проволоки. 303
Остаток проволоки с длиной, меньшей Z, если он имеется, дол- жен быть отброшен, так как при параллельном включении через него пойдет ток больший, чем /^, и он перегорит, а при последова- тельном включении с каким-либо из других кусков получится кусок с сопротивлением, большим г, на котором выделится тепла меньше qQ. 486. В силу симметрии точки D и С имеют одинаковый потен- циал. По проводнику DC ток не течет; поэтому его можно удалить из схемы, не изменяя общего сопротивления цепи, которое после этого легко подсчитывается: r = R/2. 487. Вследствие симметрии очевидно, что ток в проводнике 1—7 равен току в проводнике'7—4, ток 2—7 равен току 7—5, ток 6—7 равен току 7—5 (рис. 170). Поэтому распределение токов и, следо- вательно, сопротивление шестиугольника не изменится, если отсоеди- нить проводники 2—7, 7—5, 6—7 и 7—5 от центра (рис. 441). Со- противление же этой схемы, которая эквивалентна исходной, легко вычислить. Сопротивление верхней части схемы равно 8/3/?. Таково же сопротивление нижней части. Полное сопротивление) Rx найдется из соотношения }/Rx=\/2R + 6/8R. Отсюда RX = */&R. 488. В силу симметрии очевидно, что потенциалы вершин куба 2, 3 и 6 равны. Точно так же равны потенциалы вершин 4, 5 и .7 (рис. 171). Поэтому вершины 2, 3, 6 и 4, 5, 7 мы можем соединить проводниками, лишенными сопротивления»,—«шинами». Сопротивле- ние куба от этого не изменится.' Таким образом, шины соединены друг с другом шестью проводниками: 2—7, 2—4У 3—5, -3—4, 6—7 и 6—5. Сопротивление схемы (рис. 442) равно искомому сопротив- лению куба: *х~ 3 + 6 + 3 489. Сопротивление между точками А и В R ЛО==_______*_________(ra+rb)rc АВ V(ra + rb)+\!rc ra-^rb+2rc * где гa—pa/S, rb — pb/S, гс=*р Va* + b*/S. 304
Сопротивление между точками С и D найдем, рассмотрев токи, текущие в ветвях цепи (рис. 443). Из соображений симметрии оче- видно, что токи в проводниках DB и ACt а также AD и ВС равны, чем ток в проводнике AD равен Ч+Ч» так как сумма токов в уз- ле А равна нулю. На участке DAC 01 + Ч) rd + *irb = U DC* и на участке DABC <2 014- ц) Гд4"12гс=: U DC’ Отсюда I________га 4~ гс____у 1 - 2гагь+rarc+rbrc DC’ Искомое сопротивление n Vcd U CD ;_______ГЬ га_______rj 2 2гвгь+гагс+гбГс D ‘ _________________2rari, + rc (ra+~rb) / 2ir +-ia ^a^rb~t~^rc 490. Если ток через гальванометр не идет, то потенциалы точек С и D одинаковы и ток 1г через сопротивление Rx равен току через сопротивление /?0, а ток /2 вдоль реохорда АВ одинаков во всех сечениях. По закону Ома гд& р —удельное сопротивление, a S — поперечное сечение реохорда. Отсюда . 491. Между точками С и D необходимо включить такое сопро- тивление г, чтобы сопротивление последней ячейки (рис. 444) было равно г. В этом случае последнюю ячейку можно будет заменить сопротивлением г, затем то же сделать с предпоследней ячейкой и т. д. Тогда общее сопротивление цепочки не будет зависеть от числа ячеек и будет равно г. Для г можно составить уравнение (2й+г)Л/(3/? + г) = г. Отсюда r = J?(/3-1) яа 0,73Я. Рис. 444. Рис. 445. 492. Последняя ячейка представляет собой делитель напряжения, уменьшающий потенциал л-й точки по сравнению с (л-1)-й в k раз. Следовательно, Un— R3 = или Ri/R3 == k — 1 (рис. 445). 305
Соотношение Ui~Ui-i/k должно выполняться для любой ячейки. | Поэтому сопротивление всей последней ячейки, двух последних, трех I последних и т. д. также должно равняться 7?3 (см. задачу 491). - 1 Отсюда 1 п ^з(*1+#з)_ „ k #3 /?2^1 + ^з’ 2 R1 Окончательно 493. Использовать только приборы, действие которых основано, например, на отклонении проводника с током в магнитном > поле, нельзя. Угол, на который отклоняется стрелка в таком приборе, про- порционален силе тока, протекающего через прибор. Определение же разности потенциалов с помощью такого рода приборов, так же как и любых токовых приборов, основано на законе Ома: ток, протекаю- щий через вольтметр, пропорционален приложенной разности потен- циалов. Для проверки закона Ома, следовательно, необходим элек- j тростатический вольтметр, наряду с амперметром обычного типа. 494. Обозначим через qr и q2 заряды на первом и втором кон- денсаторах к моменту времени t. qr и q2 связаны соотношениями ^1 + ^2— Q» ^l/Ci = ^2/^2* ч 1 Так как * z-i _ ___ EpS + 2~d0 — vt9 то Я1 ^d^-vt q* do + vt' Отсюда вытекает, что nd0— vt nd0 + vt ' " < <71=Q-2dT’ Убыль заряда на первом конденсаторе равна увеличению заряда на втором конденсаторе. Сила тока / = —А^/Д/= Д^2/Д/ = Qp/2d0. Ток будет течь в направлении от положительно заряженной пластины первого конденсатора к положительно заряженной пластине второго . конденсатора. ( 495. Силы притяжения, действующие между пластинами конден- саторов, равны соответственно -j F g LQ4d0-vty J 1 2e0S?1 8e0Sd? 1 для первого конденсатора и f ^(d0 + vty 2 8eaSdl j. для второго конденсатора (см. задачу 494). Так как пластины пер- У вого конденсатора раздвигаются, силы электростатического поля W 306
совершают отрицательную работу Яр Во втором конденсаторе эти силы совершают положительную работу Л2. Работа ДЛ, совершаемая полем при перемещении каждой из пластин на малое расстояние Дх, равна О2 х ДА = ДЛ14-ДА2=(Fa-Л) 4 Д*. ZOQ О UQ где х=у/. Таким образом, работа на малом участке пропорциональна перемещению х, как и в случае растяжения пружины. Следовательно, полная работа равна Я = Q2a2/48o»Sdo. Работу Л можно подсчитать и другим способом. Так как сопро- тивление соединительных проводов равно нулю, количество выделив- шегося тепла также равно нулю. Поэтому изменение электростатиче- ской энергии двух конденсаторов будет равно работе электростатиче- ского поля. К моменту времени^ / энергии первого и второго конденсаторов будут иметь соответственно значения /72 Q2 zu2 обфдао Полная энергия б)2 Следовательно, энергия за время t уменьшится на величину Q2a2/4e0Sd0. Это изменение будет равно работе Л электростатического поля. 496. При трении одежды о сиденье стула происходит электри- зация. Тело экспериментатора и сиденье образуют своеобразный кон- денсатор. Когда экспериментатор вста- ет, емкость этого конденсатора резко уменьшается, и вследствие этого рез- ко возрастает разность потенциалов между стулом (т. е. «землей») и телом экспериментатора. Очевидно, для этого необходимо, чтобы тело было хорошо изолирована от земли (резиновая подошва). В момент касания стола разность потенциалов между рукой и землей выравнивается. Образуется электри- ческий ток, ничтожная часть кото- рого ответвляется в гальванометр. Для отброса зайчика необходимо, чтобы сопротивление между одним из концов катушки гальванометра и землей было меньше, чем сопротивле- ние между другим концом и землей. Схематически цепь тока изображена на рис. 446. О — обмотка гальванометра, К — ключ, R условно изображает очень большое, но конечное сопротивление между одним из концов обмотки и землей. 307
Отброс гальванометра наблюдается, несмотря на колоссальное сопро- тивление цепи, ввиду большой разности потенциалов, возникшей при уменьшении емкости. 497. Очевидно, имеется определенная асимметрия между про- водниками, к которым подключены концы обмотки гальванометра. Это может быть, если сопротивление изоляции между одним из кон- цов катушки и землей меньше, чем между землей и другим концом. Кроме того, нужно учесть, что сопротивление между проводниками, идущими от катушки гальванометра, несмотря на хорошую изоляцию, отлично от бесконечности. Схема, поясняющая цепь тока, приведена на рис. 447. О —об- мотка гальванометра, П1 и П2—проводники, отходящие от концов обмотки, 3 —земля, R19 R2 и R3 схематически изображают очень большие, но все же конечные сопротивления, возникающие из-за неидеальности изоляции; /?3 Rt + R2. Пунктир изображает цепь тока в случае, если заряженное отрицательно тело поднесено к про- воднику /72- Если тело поднесено к проводнику 77х, то цепь изоб- ражается точками. Видно, что в обоих случаях ток через обмотку гальванометра течет в одну сторону. Эта задача иллюстрирует наличие проводимости у всех тел. При работе с чувствительными приборами это обстоятельство становится существенным. Рис. 447. Рис. 448. 7Г£ LL 498. На рис. 448 точка А изображает потенциал положительного (медного) электрода, а точка D — отрицательного (цинкового). В ра- створе ZnSO4 цинковый электрод заряжается отрицательно в" резуль- тате выделения положительных ионов Zn, а медный в растворе CuSO4 заряжается положительно за счет поступающих на него положмтель- рых ионов Си. Потенциал электролита изображен линией ВС. АВ = (§х и CD=^ изображают скачки потенциала на границах электрод — электролит. Э. д. с., равная разности потенциалов на концах разомкнутого элемента, равна $ = $г + $2. 499. Соответствующие распределения потенциала изображены на рис. 449, а, б, в, г. а) 1 __^1 + ^2 Г1 + Г2 у BA =*VB~VA = > О- 308
б) /=^1/Г1, Vba—Q- Потенциалы проводников, соединяющих элементы, одинаковы, но ток не равен нулю. «) т/г , VBA = <£, - /г, =^г+Ггг^Г^^ . Г1 + ^2* Г1“ГГ2 г) 1—0, Vqa — Разность потенциалов между провод- никами существует, но ток не идет. 500. На каждый моль прореагировавших в элементе веществ выделяется энергия № = 445-103 — 235* 103 = 21 • 104 Дж. За счет этой энергии электрический ток совершает работу A = Q$, где — э.д.с. элемента, a Q— количеству перенесенного электричества. Так как медь и цинк двухвалентны, заряды их ионов по величине равны удвоенному заряду электрона. Один моль вещества содержит 6,02-1023 атомов. Следовательно, Q — 2* 1,6» 10*’1е-6,02*1023 Кл. Отсюда = « 1,05 В. 501. Отношение сил токов, текущих через элементы, /i//2 = f2/ri> так как э.д.с. элементов одинаковы. По закону Фарадея массы растворившегося цинка пропорциональны токам: —1^1к 1,625. 502. Каждый атом цинка, переходя в раствор в виде иона Zn + + , отдает во внешнюю цепь два электрона, несущие заряд q — 2e — = —3,2* 10”13 Кл. В то же время ионы меди Си+* осаждаются на медной пластине в виде нейтральных атомов, вследствие чего кон- центрация раствора CuSO4 падает. Для поддержания постоянной концентрации необходимо непрерывно растворять кристаллы CuS04-5H2O в количестве, компенсирующем уход ионов Си++ и SO4-- из раствора. 309
По условиям задачи через элемент прошел заряд <2=2880Кл. Это соответствует переходу в раствор п— Q/q = 9* 1021 атомов цинка, что составляет около 0,98 г цинка. Соответственно из раствора выде- лится столько же атомов меди (около 0,95 г), и для восстановления концентрации раствора CuSO4 придется растворить 3,73 г кристаллов медного купороса. 503. При растворении цинка положительные ионы Zn++ пере- ходят в раствор, а освободившиеся электроны по проволоке перехо- дят на графитовый слой и нейтрализуют положительные ионы меди в растворе CuSO4. Поэтому графит покрывается слоем меди. Явление может быть использовано для гальванопластики. 504. Изменение э.д.с. батареи зависит от соотношения между размерами электродов и размерами сосуда. Ъсли два средних элект- рода почти равны по размерам сечению сосуда, э.д.с. батареи изме- нится незначительно. Если электроды невелики, э.д.с. уменьшится приблизительно в два раза. 505. С каждой из половин угольного стержня цинковый стержень образует замкнутый гальванический элемент. Внешним сопротивле- нием элемента служит сопротивление половины угольного стержня, сопро- тивление цинкового стержня и кон- такта цйнк —уголь (см. эквивалент- ную схему на рис. 450). Когда цинковый стержень стоит вертикально, токи i2 в обеих поло- винах угольного стержня равны и вольтметр покажет нуль. Если стер- жень наклонить, то внутреннее сопро- тивление одного из элементов умень- шится, а другого —увеличится. Токи ii и 12 -не будут равны друг другу, и между концами угольного стержня возникнет разность потенциалов, ко- торую и покажет вольтметр. 506. Та^ как г<^₽, то поле внутри сферы практически отсут- ствует и тока на внутреннюю поверхность сферы нет. Следовательно, масса выделившейся меди п F где А/п—электрохимический эквивалент , меди, a F — число Фа- радея . 507. Дело в том, что при электролизе происходит поляризация электродов и каждая ванна приобретает э.д.с., направленную против тока, идущего от конденсатора. Вследствие этого конденсатор не может разрядиться целиком. Чем больше мы возьмем ванн, тей больше суммарная э.д.с. поляризации и, следовательно, тем больший заряд останется на конденсаторе. Энергия гремучего газа будет всегда меньше энергии заряженного конденсатора. 508. При электролизе воды электроды поляризуются и возникает э.д.с. поляризации £ру направленная противэ.д.с. батареи. Поэтому электролиз идет лишь в том случае, когда э.д.с. батареи больше (§р. При прохождении через электролит заряда Q батарея совершает работу против э.д.с. поляризации; За счет этой работы 310
происходит разложение воды с образованием гремучего газа. На основании закона сохранения энергии химическая энергия гремучего газа W, выделившегося при прохождении заряда Q, рав- на $pQ. Согласно закону Фарадея выделение одного грамма водорода на катоде сопровождается прохождением количества электричества Q = m^F = 96 500 Кл. /I Следовательно, $p = Wl(i ~ 1,5 В. Э.д.с. батареи должна превы- шать 1,5 В. 509. Определенная концентрация ионов есть результат динами- ческого равновесия: количество ионов, возникающих вследствие электролитической диссоциации-, равно убыли числа ионов вследствие обратного процесса — рекомбинации (ионы противоположных знаков,, столкнувшись, могут образовать нейтральную, молекулу). Вблизи электродов концентрация ионов падает, и это равновесие нарушается. Число ионов, возникших вследствие диссоциации, больше числа рекомбинировавших ионов. Именно этот процесс поставляет ионы в электролит. Процесс происходит вблизи электродов. Внутри элек- тролита динамическое равновесие не нарушается. 510. За секунду к катоду прибывает и на нем выделяется положительных ионов (S—площадь катода). Одновременно удаляется n_v_S отрицательных ионов. В процессе ухода отрицательных ионов динамическое равновесие между нейтральными молекулами электро- лита и ионами, Tia которые они распадаются., нарушается (см. зада- чу 509). Вновь возникает n--v_S отрицательных и столько же поло- жительных ионов. Положительные ионы выделяются также на катоде, и в результате количество положительных ионов, выделившихся на катоде в секунду, будет равно полному току. 511. Наибольший теоретически возможный коэффициент полезного действия термобатареи п _ <§я___Т2 л Q т\ ’ где Q — количество тепла, поглощаемое в ^единицу времени горячими спаями, (/ — протекающий в единицу времени по цепи заряд, 7\ и Т2 — абсолютные температуры спаев. По закону Фарадея масса выделив- „ А Я г-г шеися за секунду на катоде меди ш—----Подставляя значение а п г 7 из первого уравнения, имеем m _j ую-i r nF£ T± r‘ 512. Разность потенциалов между шариками должна равняться Следовательно, qi/r1—q2/r2 = (§1 где qt и (/2 —заряды шариков. Со- гласно закону сохранения заряда <7i+?2 = 0. Отсюда ql = — q2 = + По закону Кулона F = 1 S2rlrl 4ле0 /?2 (ri+r2)2 44.1С-9 Н, 311
613. В результате перемещения пластин величина заряда на конденсаторе увеличится на AQ = Qg— Qi — — W- При этом батарея совершит работу А = S AQ = S2e0S(1/Ъ- 1/dx). Электростатическая энергия конденсатора увеличится на AU7 = U72_ '---' \ X Z <4 у ^*2 Uj у При сближении пластин была совершена механическая работа А±. На основании закона сохранения энергии А = Ах+Д Следовательйо, 41 = 4-AU7=^^f J---------------------------L\ 1 2 \ d2 di J За счет работы батареи произошло увеличение электростатической энергии конденсатора и была совершена механическая работа Аг. 514. Работа сил электростатического поля при перемещении заряда по замкнутому контуру равна нулю. Поэтому Si — ^1 + ^2—U9 = 0. Заряды на конденсаторах одинаковы, ибо сумма зарядов, находя- щихся как на верхнем, так и на нижнем проводниках, равна нулю. Следовательно, ^ = C1t/1 = Cat/2. Отсюда ^=г“Гг ^1+^) = 17’5‘10’в’ ^=г4^(^1+^)=7.5-*озв- kl"f-C9 515. Не изменятся. При попеременном заземлении пластин будут происходить те же процессы, что и при отсутствии батареи. Един- ственное различие в том, что разность потенциалов между пластинами все время, поддерживается постоянной. . 516. Если ток течет в направлении, ука- . занном на рис. 451 (аккумулятор разряжа- I х — ется), то V = & — IR. Если же ток течет в I / противоположном направлении (аккумулятор заряжается), то V — ^-^fR (сц. ответ к зада- Рис 451 че 499> в)- 517. Внутреннее сопротивление гальвани- ческого элемента невелико, а электростатической машины —громадно. Это — сопротивление изоляторов (десятки и сотни миллионов ом). 518. Для двух элементов / = Si+Si ri + R * где S и г—э.д.с. и внутреннее сопротивление элементов, R — внеш- нее сопротивление. 312
Для одного элемента (например, первого) По условию задачи / < /х, т. е. ^1 + ^2 £1 Отсюда необходимо, чтобы ^2 £1 Г2 # + 519. На основании закона Ома /зР — £» где / — расстояние от батареи до места повреждения, а р —внутрен- нее сопротивление батареи. Из данной системы уравнений находим (f-4+2O’4±30"' Значение /? = 1 Ом должно быть отброшено, так как при этом полу- чается, что место повреждения удалено от батареи на 5,9 км. Дей- ствительно, при /? = 1 Ом / = км. 2г /х Искомое сопротивление /? = 7 Ом. 520. На участке А$2В V А — ЛГ2» где /1 — ($14- ^2)/(ri+гг + Я), ri и г2 — внутренние сопротивления элементов. Согласно условию задачи потенциал точки А ниже потен- циала точки В. Поэтому Ux — Vr — Va > 0. Для другой схемы £/2 = УА-Ид=(^2 + /2г2), где = — ^>2)/(/’1 + г2 + ^)- Решая данную систему уравнений, находим // ^1^2+ (^1—^2) II ofi R 521. В данном случае потенциал точки А при замкнутом ключе выше потенциала точки В, так как при разомкнутом ключе Vb > Va- 313
Поэтому Остальные уравнения имеют тот же вид, что и при решении задачи 520. Следовательно, 522. Предполагая (произвольно), что токи направлены так, как указано на рис. 452, можно на основе закона Ома записать равенства: = Iiri> UAB = <§2 — I*r2i ' UAB = I^ Так как ни в одной точке цепи не происходит накопления заряда, Л + ^2 = ^3- Решая эту систему уравнений, находим токи /1, /2 и /3: Л=5/4А, /3=3/2А. Получившиеся положительные значения токов говорят о правильности первоначально выбран- ных направлений токов. 523. Если /2 = 0, то /1 = /3, Uав = $2 и по-прежнему UAB — Si~hri- Отсюда R — =(O2ri/(Si—Если ток /2 направлен против ^2> то си- стема уравнений примет вид Uab = $i — Лг1> U АВ — &2~\~ U АВ — I3R& Л = + Отсюда / ——<^2 — $2ri/R Наше условие выполняется, если /2 > 0. Следовательно, должно иметь место неравенство ^i—<£2—д’ > °> или % Ом’ 524. Возможны два способа соединения аккумуляторов. Можно внутри отдельных групп соединить аккумуляторы последовательно, а сами группы —параллельно или же, наоборот, внутри группы — параллельно, а сами группы — последовательно. Обозначая через N полное число аккумуляторов, а через п — число аккумуляторов внутри отдельной группы, в первом случае имеем: / — __ 1 R + rn2lN Rln-\- rn/N * так как э.д.с. одной группы равна п<§0, сопротивление группы гп, а число параллельно соединенных групп N/n, достигает максимума, если Rln-\-rnlN минимально, 314Х
Минимум выражения типа ах-[-Ь/х может быть найден так. Зависимость y = ax+b/x (1) графически изображается кривой рис. 453, имеющей минимум в точке х0, в которой корни квадратного уравнения (1) совпадают. Поэтому х0 = V'bja. Следовательно, и = У RN}r — 4 и / _о ________20 А hmax- 2 у Rr~ ' Во втором случае j К So 2 /?4~гА//г2 nR-\-rN/n' Ток достигает максимума при п— У rN/R~6. Следовательно, __ So i/" — 7 гшах—~~2~ У — ylmax* Такйм образом, получить ток, превышающий 20 А, невозможно. 525. Способ переделки ясен из рис. 454. 526. Мощность, потребляемая прибором в первый момент, во много раз больше номинальной, так как сопротивление холодной Рис. 454. Рис. 453. спирали мало. Соответственно велико падение напряжения на прово- дах, идущих от магистрали в квартиру. По мере нагревания спирали потребляемая мощность падает, приближаясь к номинальной. 527. Так как чайник во всех случаях включается в одну и ту же электрическую сеть, удобнее применять формулу для количества U2 U2 выделенного тепла в форме Q——t. Отсюда /? = —/. Так как U и Q одни и те же для всех случаев, последнее равенство можно переписать в виде R = at, где a=C/2/Q« £15
Обозначая через /?t й R2 сопротивления обмоток, имеем = = a/j и fl2 = a/2. При параллельном соединении обмоток г> _ , а tfi + tf2~a(G + *2) а' При последовательном соединении ^h = ^i + ^2 = a(^i + Отсюда + ^ = G+^2- 528. 1) ta w 57 мин; 2) /& = 3 мин 30 с (см. решение задачи 527). 529. При вычислении потерь тепла в проводах линии высокого (/2 напряжения по формуле Q=—/ величина U есть разность потен- К циалов на концах линии (падение напряжения на проводах), но не напряжение на вторичной обмотке повышающего трансформатора. Эта разность потенциалов невелика (в отличие от напряжения на обмотке трансформатора) и уменьшается с уменьшением текущего по линии тока. 530. Выделяющаяся на внешнем сопротивлении R мощность № = /£/. В данном случае U = $ —1г и, следовательно, I = ((§ — U)lr. Таким образом, W = (SU — V2)/r. Отсюда Ui~9 В или U2 = 1 В. Неоднозначность результата связана с тем, что одна и та же мощность может быть выделена на различных внешних сопротивле- ниях /?, причем каждому R соответствует свой ток: при U1 — 9 В, Ц = 1 А = = 9 Ом; при (72=1 В, /2 = 9А Z?e = Ц7//| = Ом. 531. При параллельном соединении При последовательном * NiNt NijN-NJ + N / В числителе последнего выражения стоит произведение двух вели- чин, сумма которых постоянна (равна AZ). Это произведение макси- мально, когда рассматриваемые величины равны. Отсюда Nt = N — Nlt т. е. ^ = ^ = ^2. 532. Полезная мощность (см. задачу 530) равна W = (£U — U2)lr. Обозначим £U -—U2 — x. Нужно определить, при каком U величина х достигнет максимума. Графически зависимость х от U изображена на* рис. 455. Кривая имеет форму параболы, причем каждому х соответствуют два значе- ЗЮ
ния U. При данном х мы имеем квадратное уравнение относи- тельно- U, х достигает максимума, когда оба корня уравнения сов- падают. Следовательно^ при максимальном х дискриминант урав- нения должен быть равен нулю: •»шах = ^>2/4- Отсюда l/ = <£/2, Wmax = ^/4r = 2bBT. При этом R = ^max/'2 = S2 4г2/4г<£2 = г> т. е. внешнее сопротивление равно внутреннему. 533, По определению коэф- фициент полезного действия т) есть отношение полезной мощности ко всей мощности, выделяемой акку- мулятором: 4] = — где U = $R/(R + г) — разность потен- циалов на внешнем сопротивлении R. Следовательно, т) — R/(r-{-R). В задаче 530 тц = 90%; т|2= 10%. В задаче 532 т) =50%: т)—►! при R —► ос, но при этом выделяемая полезная мощность W = ~ (как и полная) стремится к нулю (рис. 456).. Рис. 456. 534. По закону Ома U = <£ + //’. Следовательно, / ~(U — £)lr. Полезная мощность, расходуемая на зарядку аккумулятора, равна Количество тепла, выделенного в единицу времени, ^2 = /2Г=(^-^)2/г. 317
Полный расход мощности 535. Полезная мощности (см. задачу 534). Выделяемое в единицу времени тепло w2=(u-sy/r. Обычно при зарядке U — Следовательно, W±^>W2. На выде- ление тепла расходуется небольшая часть мощности зарядной станции. 535. За время t через поперечное сечение А проводника пройдут все те электроны, которые содержатся в объеме S-v-t (рис. 457). Следовательно, сила тока I = S'V*n*e(e—заряд электрона). Отсюда v~ llSne^i 10-3 мм/с. 537. Электроны в металле можно считать свободными. Пере- распределение электронов внутри бруска закончится тогда, когда возникшее вследствие перераспределения Электрическое поле будет в состоянии сообщать электронам ускорение а. Таким образом, искомая напряженность поля может быть найдена из соотношения та = еЕ (tn и е—масса и заряд электрона). Отсюда Е — (т!е)а. Боковые поверхности бруска, перпендикулярные движению, бу- дут заряжены: передняя поверхность положительно, а задняя — от- рицательно. Плотность зарядов равна о = е0Е = е0 (m/е) а. 538. Свободные электроны вращаются вместе с цилиндром. Сле- довательно, электрон, находящийся на расстоянии г от оси, имеет ускорение а = со2г. Это ускорение может возникнуть' только под действием электрического поля, направленного вдоль радиуса от центра цилиндра и равного Е — та^г/е, Здесь е и/и —заряд и масса электрона. Разность потенциалов =~^~Г ®2^2»так как средняя сила, дейст- вующая на единичный заряд при перемещении его от оси к поверх- 1 tn 9п ности цилиндра, равна у у со2/?. 539. Во вращающемся диске происходит перераспределение заря- дов и возникает электрическое поле, напряженность которого равна Е-та^г/е, где г — расстояние от центра диска, е—заряд электрона, tn—его масса. График зависимости Е = f (г) изображен на рис. 458. Разность потенциалов <р между центром и краем диска численно равна площади заштрихованного на рис. 458 треугольника, т. е. ф = тсо2/?2/2е, Количество тепла Q, выделяющееся на сопротивлении 318
Ro в единицу времени, равно Q = /<p = .L™^, (1) где I —ток, текущий в цепи (находится по закону Ома). Вращение диска тормозят движущиеся от центра к периферии электроны. Пусть на расстоянии R имеется N ионов, расположенных на равных расстояниях друг от друга. При каждом соударении с ионом электрон приобретает количество движения р под действием импульса силы Ft, действующей на электрон со стороны иона: p = m(oR/N = Ft. Момент количества движения, приобретаемый электроном при каждом соударении, равен R R „ R NN N Если ток в цепи равен /, то момент количества движения, сооб- щаемый электронам всеми N ионами за единицу времени, равен IЛ № Па 1 Сумму слева можно преобразовать следующим образом: TTm(d~N^--------- При N —► оо получается I Л, — т® — —М. е 2 Работа, совершаемая всеми N ионами за единицу времени, есть W =—ш2—=М(д, (2) е 2 ' Из сравнения (1) и (2) получаем Q = W\ § 21. Электрический ток в газах и вакууме 540. Законы сохранения энергии и импульса в данном случае можно записать в виде * м _,2 f.2 w „2 о _ , т2и2 , q ©1 n n “1 п “Г © О? 2 2 Z ~ + w2u2, где 0!—скорость первой частицы до соударения, и2— скорости первой и второй частиц соответственно после соударения. Решив i 319
систему уравнений (1), получим ^i—у 1—j U1=vx------- т\ + т2 /пх «2 = ^1 — mx-J-m2 Нетрудно видеть, что после соударения и± не может быть больше ulmax=tf2 ПРИ выполнении следующего соотношения: min* ~Г т2 Это соотношение определяет наибольшую долю энергии, которая может быть превращена в энергию ионизации. Другими словами, если для осуществления ионизации требуемся энергия ^0, то для этого требуется минимальная энергия первой частицы большая, чем ко- нели первая частица много легче второй (например, электрон и атом), то мх ~ м2 ~ vi [У~ И* При этом ^>1 min ~ So, И если Si = Simla, то л mi “1«0, иа /п2 т. е. практически вся энергия первой частицы затрачивается на ионизацию, и обе частицы после сбударения почти покоятся. В случае тк » т2 (например, ион и атом) min ~ 541. До начала разряда напряжение на счетчике равно э.д.с. источника В момент разряда по цепи идет ток и напряжение между корпусом и нитью становится равным U ~(§ — IR. Сопро- тивление R очень велико, и падение напряжения IR настолько зна- чительно, что разряд прекращается. 542. По закону Ома искомое падение напряжения U — IRt где / —сила тока в цепи. Ток одинаков во всех сечениях внутри кон- денсатора. На положительную пластину этот ток обусловлен только отрицательными ионами, а на отрицательную—только положитель- ными. Через произвольное сечение внутри конденсатора проходит некоторая доля как положительных, так и отрицательных ионов. I—enSd, где е—заряд электрона, a S—площадь пластин. Для _, Cd* _ плоского конденсатора Sd — —'. Следовательно, 8о у=епС^/? 1б 75.10_1,В( «о 543. Электроны, созданные внешним ионизатором, будут исче- зать в результате рекомбинации. В единице объема за единицу вре- 320
мени рекомбинирует* ионов =упо- Кроме того, если между элект- родами ток /, то из единицы объема газа в- единицу времени на электродах нейтрализуется число ионов, равное Апо = I/qSl. Сле- довательно, чтобы число ионов в объеме не изменялось, внешний ионизатор должен создавать в единицу времени в каждой единице объема число ионов, равное An0 = ynl + I/qSl. 544. Условие I(qSl упо означает, что числом ионов, исчезаю- щих вследствие наличия тока, можно пренебречь по сравнению с числом ионов, исчезающих в результате рекомбинации. Условие равновесия (см. задачу 543) запишется в виде Дп0 = = у/го- Из этого условия вытекает равенство n0 = Y Ап^/у, которое означает, что число ионов п0 в единице объема газа постоянно. В единицу времени к катоду подойдет nou+S положительных ионов. Одновременно с катода отойдет nou_S отрицательных ионов. Следовательно, общее число положительных ионов, выделившихся на катоде в единицу времени, равнр n0(n+ + n_)S, и столько же отрицательных ионов выделится в единицу времени на аноде. Таким образом, плотность тока равна j + nQq, и так как u+—b+E, и_ — Ь_Е> то j — nQ(b+~\-b_)qE. Это равенство выра- жает закон Ома j = oE, поскольку a = n0(^+ + ^-)^ = const. 545. Условие IqSt означает, что плотность тока j = //S настолько велика, что вся убыль ионов фактически определяется их нейтрализацией на электродах, а убылью в результате рекомбинации можно пренебречь. Условие равновесия принимает вид (см. задачу 543) Aiib = IlqSl — jlql. Отсюда j = j^ — An^qh и /н не зависит от напря- женности поля Е, а следовательно, и от разности потенциалов между электродами. Эта плотность тока является максимально возможной при данных условиях (при данных An0, q и /) и называется плот- ностью тока насыщения. /н тем больше, чем больше /. Этот вывод справедлив при условии, что ионизация производится во всем объеме между электродами и, следовательно, на участке тем большем, чем больше I. 546. График распределения напряжения U приведен на рис. 459. Электроны пробегают темное катодное пространство значительно быстрее, чем положительные ионы. Вследствие этого в любой момент времени положительных ионов в темном катодном пространстве гораздо больше, чем электронов. Наибольшая концентрация поло- 11 Б. Б. Буховцев и др. 321
жительных ионов приходится на начало тлеющего свечения. Наимень- шая их концентрация имеет место около катода, где скорость их Движения наибольшая. Около самого катода имеется слой отрица- тельного пространственного заряда, образуемый электронами, начи- нающими свое движение с катода с очень малыми скоростями. 547. 1) Если в трубке, в которой происходит тлеющий разряд, постепенно передвигать анод в направлении катода, то катодные части разряда остаются почти неизменными по своей длине и распо- ложению. При передвижении анода лишь уменьшается длина поло- жительного столба, пока этот столб не исчезнет совершенно. Затем при дальнейшем приближении анода к катоду укорачивается темное фарадеево пространство, а потом тлеющее свечение, причем положе- ние резкой границы этого свечения со стороны катода остается неиз- менным. Наконец, когда расстояние от этой границы до анода ста- новится очень малым, тлеющий разряд прекращается. 2) Если анод остается неподвижным, а передвигается катод по направлению к аноду, то все катодные части разряда, включая гра- ницу положительного столба, передвигаются вместе с катодом, оста- ваясь неизменными по своим размерам и взаимному расположению. Положительный столб -л фарадеево темное пространство, а затем тлеющее свечение постепенно исчезают. Когда головка тлеющего све- чения доходит до анода, разряд прекращается. 548. При охлаждении отрицательного угля дуга погаснет, так как горение дуги обеспечивается сильной термоэлектронной эмиссией с катода, которая прекращается при охлаждении. Охлаждение поло- жительного угля не будет влиять на работу дуги. 549. Заряд, накопленный лейденской банкой за 30 секунд работы электростатической машины, равен q — It. Следовательно, напряже- ние зажигания искрового разряда равно U8—qlC — ItlC = 3-104 В. Ток в разряде /р = ^/т = ///т = 300 А. Такой большой ток обуслов- ливает сильное нагревание воздуха, в результате которого возникает звуковая волна (треск). 550. Обозначим заряд, сообщаемый лейденской банке электро- статической машиной в единицу времени, через Q. Искра между шариками проскакивает тогда, когда напряжение между шариками разрядника достигает величины напряжения зажигания искрового разряда Us. Величина U9 зависит от расстояния между шариками, от их радиусов и от свойств воздуха. Когда параллельно разряд- нику подключена одна лейденская банка, то U^ — Qt^/C. При двух параллельно соединенных банках C/8 = Qt1/2C. При двух последо- вательно соединенных банках £/3 = 2Qt2/C. Следовательно, т1==2т0, т2 = т0/2. 551. Один электрон-вольт равен 1,6* 10“1е Дж. 552. Не совпадает. Касательные к траектории дают направление скорости частицы, а касательные к силовой линии дают направление силы, действующей на частицу, и, следовательно, направление уско- рения. Только в поле, силовые линии которого — прямые, траекто- рия частицы совпадает с силовой линией, если начальная скорость этой частицы направлена по силовой линии. 553. Полная энергия электрона равна сумме кинетической и по- тенциальной энергий. При приближении к кольцу потенциальная энергия электрона в поле кольца уменьшается, и за счет этого рас- тет кинетическая энергия. Пройдя сквозь кольцо, электрон удаляется от него. При этом потенциальная энергия электрона увеличивается, а скорость постепенно уменьшается до нуля. 322
554. Анода достигают электроны с энергиями от 80 электрон- вольт до 74 электрон-вольт, так как вдоль нити накала существует падение напряжения в 6 В. Энергия электронов у анода опреде- ляется только пройденной разностью потенциалов и не зависит от потенциала сетки. Потенциал сетки изменяет распределение скоростей электронов в проме- жуточных точках пути и влияет на число элек- тронов, достигающих анода. 555. На основании закона Ома = /а₽а + + Ua (рис. 460). Сила тока Ia = AUa + BUa. Отсюда / , (Л7?а+1)-Г(^а+1)2+4^а 2BR*B — 5 мА. Второй корень квадратного уравнения не иМеет физического смысла, так как соответствует Ua < 0. 556. Система уравнений, определяющих токи i’x и i2t имеет вид Отсюда < = Ч + Ч» Ч— i2 = A2Ua + B2U2ai Ua = <§-iR. 2^+BJR =60 B. Отрицательное значение Ua отбрасываем, как не соответствующее смыслу задачи. Искомые токи равны Й = (B1^B,)R № + (Л1В2- Л2В1)RU*- = 22,2 мА, •а = _|_в^ R + (Л2^1 — Ai.B2) RUа—B2Uа] = 37,8 мА. 557. При потенциале сетки ^2 = — 6 В ток, протекающий через лампу, I2 = U2[R, а при потенциале сетки = — 3 В /х = U^R, Следовательно, увеличение потенциала сетки на — <£2 = 3 В вы- зывает’ увеличение анодного тока лампы на /х-—/2 = ([/1 —(72)//? = 3,5 мА. Так как сеточная характеристика лампы в рассматриваемой области предполагается линейной, то увеличение потенциала сетки относи- тельно катода еще на 3 В (от — 3 В до нуля при замкнутых нако- ротко сетке и катоде) вызовет увеличение анодного тока еще на 3,5 мА. Тогда падение напряжения на сопротивлении R увеличится еще ira U1 — (У2 = 35 В, т. е. достигнет U'Q — ^/i~b(^/i — t/3) = 130 В, а разность потенциалов между анодом и катодом лампы будет равна <£-(/в=120В. 558. Первый диод начинает проводить ток только при Ua > 0, т. е. при V>$lt второй —при V > <£2 и третий —при V > ^3. 11* 323
Поэтому график зависимости полного тока от напряжения представ- ляет собой ломаную линию (рис. 461): / = 0 при I = k(V — при l = k(V-Si)+k(V-Si} при <£a<V=C<£3. /=fe(V-^i)+*(V-^2)+fe(V-^3) при <§3<V. Подобные схемы иногда используются в радиоустройствах для полу- чения заданной функциональной зависимости тока от напряжения. .559. На рис. 462 А и В — управляющие пластины, Af/V —экран, ОС—траектория электрона; Начало системы координат находится в точке О. При движении между пластинами в направлении оси у электрон перемещается равноускоренно с ускорением a — eUfind, где О —раз- ность потенциалов между А и В. Расстояние I вдоль оси х он проходит за время = //ух; —горизонтальная составля- ющая скорости электрона, оп- ределяемая из условия тух/2 — =еГ70. За время электрон отклоняется в направлении оси у на величину atl eUl2 2 2dmvx Движение электрона вне пластин происходит с постоян- ной скоростью и длится в течение времени t2 = Llvx. Скорость вдол^ у равна vy — atv Отклонение в области вне пластин , eUlL dmvx 324
Полное отклонение , eUl f I . r \ eUlL UlL У — У1 + У2—---2 \ ~ 3--2~9IJ Я* dmvx \ 2 / dmvi ^0“ Чувствительность q = yllJ = lLl2UQd. § 22. Магнитное поле тока. Действие магнитного поля на ток и движущиеся заряды 560. В системе СИ имеем [Я] = //£, [/] = /, [r]=L, [/]=//£, где / и L — символы размерностей тока и длины. Отсюда следует: в случае (1) H=>CtIlr, в случае (2) H = C2jt где и С2 — некоторые константы. Теоретический расчет дает сле- дующие точные формулы: . в случае (1) H — Iftnr, в случае (2) Н — Ц1!. 561. Ток, протекающий по трубе, можно рассматривать как сум- му множества одинаковых прямо- линейных токов, равномерно рас- пределенных по поверхности трубы. Напряженность магнитного поля в любой точке пространства можно представить как сумму напряжен- ностей полей, создаваемых этими токами. На рис. 463 изображено попе- речное сечение трубы, вдоль ко- торой проходит ток. Сравним на- пряженности магнитных полей Нг и Я2, создаваемых в точке А ли- нейными токами Л и /2, проходя- щими через малые дуги и S2. Длины дуг равны = a/^/cos и S2 = a/?2/cos <р2, где и /?2 — расстояния до точки А. Но, как видно из чертежа, фх = ф2« Следо- вательно, S1/S2 = 7?1/7?2. Ток по трубе распределен равномерно, поэтому Zi//2 = S1/S2, откуда Zi/₽i = - /2//?2. Напряженности магнитных полей, создаваемых в точке А токами /j и /2, пропорциональны этим токам и обратно пропорциональны соответствующим расстояниям. Следовательно, Н х — kl 1/=^fe/2//?2 = f/2, причем Нг и Н2 направлены в противоположные стороны. Так как для каждого элемента поперечного сечения трубы можно подобрать соответствующий элемент, полностью компенсирующий магнитное поле первого элемента в точке Д, то результирующее магнитное поле 325
тока, протекающего по трубе, в любой точке внутри трубы будет иметь напряженность, равную нулю. 562. Проводник с полостью эквивалентен сплошному проводнику, по которому течет ток плотности j, а по объему, соответствующему полости, кроме тога, течет ток той же плотности в обратном направ- лении. Суммарный ток в указанном объеме будет равен нулю, а это соответствует наличию полости в сплошном проводнике. Поле, созданное током плотности j в произвольной точке А по- лости, равно Bi = k*2njR (рис. 464). Здесь R — расстояние от оси проводника до точки А. (Предполагается, что ток течет к нам.) В той же точке ток, текущий по объему, соответствующему полости, в об- ратном направлении, создает поле Ва = ^-2эт/г. Как видно из рис. 464, полная индукция B = J/ В$+ В1-2В& cos а. Очевидно, /?24-r2-d2 cos а—----!------ 2Rr Отсюда индукция B = k-2njd одинакова для всех точек полости. Рис. 464. Рис. 465. 563. Л АОС со Л BAD (рис. 464), так как эти треугольники имеют по одному равному углу, а стороны, заключающие эти углы, пропорциональны. Значит, £ АОС — £ BAD. Но и, следо- вательно, В J_d. Индукция магнитного поля в любой точке поло- сти перпендикулярна линии, соединяющей центры проводника и поло- сти. Соответствующее распределение линий изображено на рис. 465. 564. Любой элемент кругового контура Д/ находится на одном и том же расстоянии R от центра. Кроме того, для любого эле- мента Д/ радиус-вектор R перпендикулярен к Д/, т. е. а = эт/2, sina=l. Таким образом, индукция магнитного поля ДВ, создавае- мого в центре круга элементом Д/, равна &B = kl &1/R2. Индукция ДВ направлена перпендикулярно к плоскости круга, и так как все элементы Д/ создают в центре одинаково направлен- ные ДВ, то суммарная индукция магнитного поля выразится суммой Учтя, что 2д/ = 2л/?, получаем B — k2nI{R. 326
565. Определим индукцию магнитного поля в точке А, отстоя- щей на расстояние d от плоскости контура (рис. 466). Расстояние элементов Д/ от точки А обозначим через г. Рассмотрим индукции ДВх и Д&2, создаваемые двумя элементами контура All и Д/2, на- ходящимися на противоположных концах диаметра. Так как угол а между г и Д/ равен я/2 (как угол между образующей конуса и эле- ментом окружности его основания), то (см. задачу 564) = kl Д/х/г?, ДВ2 = ki М2/г^ Выбрав Д/1 = Д/2 = Д/ и замечая, что Г1 = г2, получим ДВ1 = ДВ2 = А?/Д//г2. Геометрическая сумма ДВ векторов ДВх и ДВ2 будет направ- лена вдоль оси кругового тока и численно равна сумме проекций ДВХ и ДВ2 на ось ОА: ДВ = ДВ1з1п 04-ДВ2 sin Р = sin 0. Поскольку sin 0 = /?/г, то Г3 Разбивая весь круговой контур на соответствующие пары эле- ментов Д/, получим, что результирующая индукция магнитного поля направлена вдоль оси кругового тока и численно равна сумме Поскольку 2^&l = 2nR, то о ’2л^/ В — к „ — к , • Г3 (R^+d2)313 566. Проводник ВС не создает поля в точке Л4. Согласно дан- ному в примечании к задаче 564 правилу магнитное поле от любых элементов проводника ВС должно быть перпендикулярно линий ВМ. Поэтому наличие отличного от нуля поля в М противоречило бы симметрии задачи, ибо все направления, перпендикулярные ВМ, 327
равноправны. Так как напряженность поля пропорциональна силе тока, то до присоединения провода — Поля от проводников АВ и BD складываются. Следовательно, после присоединения про- водника BD H2 — kI-\-kI}2. Отсюда Я2/#1 = 3/2. 567. В произвольной точке линии АВ любой малый элемент тока проводника АСВ создает магнитное поле, перпендикулярное пло- скости чертежа (см. задачу 566). Симметричный ему элемент провод- ника ADB создает такое же поле, но направленное в противополож- ную сторону. Поле от двух любых симметрично расположенных эле- ментов поэтому будет равно нулю. Следовательно, поле в произволь- ной точке АВ, созданное всем проводником, равно нулю, так как прямолинейные участки проводника также не создают поля на АВ. 568. 1) Токи / и i текут в одну сторону. Сила взаимного при- тяжения между шиной и проводом при расстоянии х между ними равна 2лх ' Равнодействующая силы тяжести и сил упругости пружин направ- лена вниз и равна f = 2k(h—х). В положении равновесия F — f. Отсюда получаем квадратное уравнение относительно х. Решение уравнения дает у ~4~4nk (Устоичивое равновесие); _h_ /г2 Цр/17 Xi~ 2 V 4 4л* (неустойчивое равновесие). Если h2/4 < p.0/il/4nk, т! e. fe < [i0/il/jih2, провод притянется к шине. 2) Токи / и i текут в противоположных направлениях. Провод отталкивается и будет в устойчивом равновесии на расстоянии г—Ал. 1/"। 2 + V 4 + 4л* ’ 569. Силы, действующие на стороны АВ и DC, равны по вели- чине и противоположны по направлению, их сумма равна нулю. Сила Flt действующая со стороны тока / на AD, равна Л 2пх Сила F2, действующая на ВС, равна М(/ К а 2n(x-\-h) Силы Fr и F2 направлены вдоль одной прямой в противоположные стороны, причем Fr > F2. Следовательно, контур будет притягиваться к проводу с силой Р — Р __р __ 1 2 2лх(х4-Л) 328
570. Обозначим длину стороны квадрата через I. Момент сил магнитного поля, выводящий рамку из вертикального положения, равен М = В//2 cos а. На рамку, отклоненную от вертикали на угол а, действует, кроме того, момент сил тяжести, стремящийся вернуть рамку в вертикаль- ное положение. Этот момент равен M' = 2pgS/2 sin а. Условие равновесия рамки: , М — М', ВП2 cos a = 2pgSl2 sin а. Отсюда находим индукцию магнитного поля: В = ^^- tg а. • 571. Магнитный момент М, действующий на короткую катушку со стороны поля соленоида, равен M — INSBt где В = рол/. Из ус- /PL \TnNS ‘ 572. Под влиянием магнитного поля кольцо повернется таким образом, что силовые линии поля станут перпендикулярными пло- скости кольца и будут образовывать с направлением тока правый винт. При этом натяжение кольца станет максимальным. Применяя метод, использованный при решении задачи 415, получим F — BIR. 573. На элемент кольца Д/ действует сила AF = BI Ы (рис. 467). Разложим ее на составляющие ДГХ и Д/; Д/7! лежит в плоскости кольца, a A/ = AFsina нормальна к плоскости кольца. Равнодейст- вующая сил Д/7!, действующих на отдельные элементы кольца, равна нулю. Эти силы только растягивают кольцо. Полная сила f, дейст- вующая на кольцо, равна сумме сил Д/: f = ^BI sin a>&li — BI 2tcR sin a. i * 574. Силы, действующие на стороны ВС и AD, перпендикулярны перемещению этих сторон, поэтому эти силы не совершают работы. Силы, действующие на стороны АВ и CD, постоянны, составляют 329
прямой угол с направлением поля и численно равны (рис. 468). Искомая работа будет равна удвоенному произведению силы на перемещение стороны АВ или CD в направлении силы. Это перемещение при повороте контура на 180° равно Ь. Следовательно, A — 2BIab- 576. Разложим скорость электрона на составляющие: v н — па- раллельную В и перпендикулярную В (рис. 469). не ме- няется ни по величине, ни по направлению, так как сила Лоренца не действует^на частицу, имеющую скорость вдоль поля, меняется по направлению, так как за счет этой составляющей на электрон действует сила Лоренца, постоянная по величине и перпендикуляр- ная скорости Oj_. Поэтому ускорение электрона тоже постоянно по величине и перпендикулярно скорости Но движение с постоян- ной скоростью и постоянным ускорением, перпендикулярным этой скорости, ecfb равномерное движение по окружности. Таким образом, на равномерное поступательное движение вдоль В накладывается вращение по окружности в плоскости, перпендику- лярной В. В результате возникает движение по винтовой линии с шагом Л = УцТ, где т —время одного оборота электрона по окружности, радиус которой, как легко найти, равен R = tnv sin ajBe. Так как т *= 2nR/v^ — 2пт/Ве, то ft = (2пт/Ве) v cos а. 576. Вследствие действия силы Лоренца электроны будут пере- мещаться к краю ленты. Поэтому один край ленты приобретет отри- цательный заряд, а другой —положительный; внутри ленты возникнет дополнительное электрическое поле, напряженность Е которого направлена перпендикулярно току. Перемещение электронов продол- жится до тех пор, пока сила Лоренца не будет уравновешена силой, действующей на электрон со стороны электрического поля: еЕ == Bev. Отсюда E = Bv. Разность потенциалов фя==£а===Вра, или, так как I — nevS, уд — ув — ВаЦпеВ. 577. Сила Лоренца действует как на свободные электроны, так и на положительные ионы, находящиеся в узлах кристаллической решетки, поскольку и те и другие движутся в магнитном поле. Сила f, действующая на свободные электроны, согласно правилу левой руки будет направлена так, как это показано на рис. 470. Электроны относительно решетки смещаются, и одна боковая сторона паралле- лепипеда заряжается отрицательным электричеством, другая —поло- жительным. В бруске возникает электрическое поле, и, когда напря- 330
женность этого поля будет удовлетворять соотношению eE~Bev> перемещение электронов относительно решетки прекратится. Искомая напряженность E — Bv. Плотность зарядов о находим из соотноше- ния (У/е0 = Е. Следовательно, <у = Вуе0. 578. Для того чтобы электростатическое поле не возникало, элек- троны при вращении цилиндра не должны перемещаться относительно кристаллической решетки. Это перемещение будет отсутствовать, если действующая на электроны сила Лоренца равна /исо2г, т. е. /исо2г = = Bev. Так как w = cor, то В — пиа/е. Поле должно быть направлено в ст'орону поступательного перемещения буравчика, рукоятка которого вращается в том же направлении, что и цилиндр. 579. Е = т(д г согВ. Здесь заряд электрона равен (— е). Е поло- жительно, если направлено от оси цилиндра. Если направление В и направление вращения цилиндра составляют правый винт, то надо брать знак минус, в противном случае —знак плюс. 580. Так как ионы проходят скрещенные поля без отклонения, то Ее—Bev — О, откуда v — E/B = 5000 м/с. В дальнейшем каждый ион будет двигаться по окружности диаметра 2R — 2mv/B,et где т — масса иона. Следовательно, искомое расстояние равно Д (2R) — = • Так как Д/и = 2 атомные единицы массы =2 -1,66* 10“27 кг, е — 1,6-Ю-i» Кл, у=5000 м/с, В'=0,09 Н/(А-м), то л/от-2’1’66’10”27 5000-^ irin з 1 Д(2/?)— 16.JQ-1, 0,09~1,15' ° м —Ыбмм. § 23. Электромагнитная индукция. Переменный ток 581. 1,07 милливольта; более высокий потенциал —на западном крыле. На все элементарные заряды самолета действует сила Лоренца. Эта сила вызовет движение электронов проводимости с запада на восток. Движение электронов прекратится, когда работа силы Лоренца станет равна работе сил электрического поля, создаваемого перемещенными электронами, т. е. когда будет выполнено условие U = — vBl (1) (знак минус означает, что в направлении движения электронов потен- циал понижается), где Z—длина крыльев, В —индукция магнитного поля, V—скорость движения самолета, U — разность потенциалов между концами крыльев. Заметим, что равенство (1) эквивалентно соотношению U — — ибо vl = &S/M — площадь, очерченная крыльями самолета за единицу времени, и при В = const Ву/= В’ДВ/Д/= ДФ/Д/ есть величина потока магнитной индукции, пересеченного крыльями самолета за единицу времени. 582. Электроны проводимости проводника АВ движутся сосско- ростью vt находясь в .магнитном поле, следовательно, на них дейст- вует сила Лоренца, направленная вдоль проводника, от точки В к точке А. Сила Лоренца, приходящаяся на единицу заряда, в дан- ном случае, поскольку yj_B, равна F==Bv. Электродвижущая сила по определению, численно равна работе, совершаемой над 331
единичным положительным зарядом при движении его по замкнутому контуру. Так как сила Лоренца действует только на участке АВ, то $ — Bvl. В цепи возникает ток ' / = <£/(' +Я)-+ Легко убедиться в том, что механическая работа, совершаемая внешними силами, превращается полностью в джоулево тепло. Элек- трическое поле в цепи имеет электростатическую природу, т. е. источником этого поля являются поверхностные заряды. Для того чтобы определить разность потенциалов фл —фя, применим закон Ома к участку цепи ДВ: <£+фл—фв = /г. фя—фв = — vBlR/(R + r). Следовательно, фл < фв- Напряженность Е электростатического поля в подвижном проводнике равна Е = (фв—Ч>а)/1 = EvR/(R+г) и направлена от В к А. ' 583. Пусть скорость движения проводника в какой-то момент вре- мени равна v. Тогда э. д. с. в этот же момент времени будет равна <§ = Blv, а сила тока I = BlvlR. Вследствие действия магнитного поля на проводник с током появится сила /, препятствующая сво- бодному падению проводника: f = B42v/R. Следовательно, в рассмат- риваемый момент времени ускорение определится из соотношения ma = mg — f=mg — B2l2vjR. Легко видеть, что ускорение а по мере увеличения скорости будет * уменьшаться, и в момент, когда наступит равенство сил f = tng, уско- рение станет равным нулю. Проводник, начиная с этого момента, будет двигаться с постоянной скоростью равной Vk — mgR[B2l2. 584. Э.д.с. индукции, возникающая в проводнике, <§ = Blv. Заряд на обкладках конденсатора находится из соотношения Q = — (gC — BlvC. Текущий в цепи ток / == Дф/Д/= В/С Ду/Д/= В/Са, где а—искомое ускорение. Вследствие взаимодействия этого тока с магнитным полем появится сила Fr, действующая на подвижный проводник. На основании правила Ленца эта сила будет направлена в сторону, противоположную силе F. По величине = ВП = В212Са. Искомое ускорение может быть найдено из соотношения ma = F — Fv Отсюда a = F/(m + B2/2C) есть величина постоянная. Работа силы F на пути S пойдет на приращение кинетической энергии проводника и на увеличение электростатической энергии конденсатора. 585. При изменении магнитного потока, пронизывающего рамку, на малую величину ДФ за малое время Д/ в рамке индуцируется э. д. с. = — ДФ/Д/ и протекает ток, который можно считать посто- янным благодаря малости Д/. Следовательно, заряд, протекший через прибор за время Д/, равен Д^ = ^Д//7? = -ДФ/7?, где R— сопротивление рамки; он зависит только от изменения потока за время Д/. Полный заряд, протекший через прибор, равен сумме элементарных зарядов Д<?: q=2 =- 2 (дф/#)=- (2 дф)/я- 332
Изменение магнитного потока в обоих случаях имеет один и тот же знак (поток уменьшается); но в первом случае поток изменяется от некоторого положительного значения до другого, меньшего поло- жительного значения. Во втором случае изменение потока происхо- дит от того же начального значения до нуля и далее до некоторого отрицательного значения.* Таким образом, во втором случае полное изменение потока больше, чем в первом, а следовательно, во втором случае больше и заряд, протекший через прибор. 586. На основании закона электромагнитной индукции и закона Ома имеем Д(? = /Д/= ДФ//?, или р = (Ф — Фо)/7?. Так как начальный магнитный поток Фо = BSn, а конечный Ф = 0, то количество протекшего электричества будет равно Q = BSn/R. 587. Так как э. д. с. индукции $ — —ДФ/ДЛ то инд~ R Af * ♦где Ф — магнитный поток, пронизывающий контур ABCD. Если пре- небречь индуктивностью этого контура, то w Ир/дб Ах 2лх (х-{-Ь)’ где Дх —расстояние, на которое перемещается контур за время ДЛ Положив в этом равенстве Дх = и Д/ и подставив полученное выра- жение в (1), найдем / /v инд 2nx(x-j-b) R Ток /инд направлен по часовой стрелке. 588. Согласно закону Фарадея ДФ/Д/ = kS. Э. д. с. индукции численно равна работе, совершаемой электрическим полем при пере- мещении единичного положительного заряда вдоль витка, т. е. = ~2пг*Е. Отсюда E — $il2nr. Таким образом, окончательно получаем Е = knr2!2nr — kr/2. Надо отметить, что данное электрическое поле создается не электри- ческими зарядами, а меняющимся во времени магнитным полем. Напомним, что работа при перемещении электрического заряда по замкнутому контуру в случае электростатического поля всегда равна нулю. Под электростатическим полем мы понимаем электрическое поле, создаваемое электростатическими зарядами. 589. Разобьем кольцо на п = (Ь—а)/д колечек шириной 6 каж- дое. Рассмотрим колечко высоты h, внутренний радиус которого х, а внешний х-|-д. Если 6 ма/ю по сравнению с х, то сопротивление такого кольца можно выразить формулой R = р 2лх/6/г. Величина э. д. с. индукции, действующей в этом кольце (при условии, что 6<^х), равна = ДФ/Д/=этх2£. Сила тока, текущего 333
по такому кольцу, _ six*k$h kbhx р-2лх ~ 2р Для того чтобы найти силу тока, текущего по всему кольцу, надо найти, чему равняется сумма следующего вида: l=^r {a+(«+S)+(«+26)+ • • • +[« + (л-О6]}. Выражение в скобках представляет собой арифметическую сию. Поэтому прогрес- , kh.. ,2a-j-b—а—6 / = 2^(&-«)------2----• Этот результат будет тем точнее, чем меньше величина б. 6 стремящимся к нулю, получим 590. На рис. 471 указано направление индукции $ (для k > 0). Правая половина ние /?2, а левая Э. д. с. индукции была ^ = /глг2. Сила тока согласно закону Ома Полагая и э. д. с. магнитного поля кольца имеет сопротивле- определена в задаче 588: равна / = S/(Ri + ₽2) = + ₽2). Как известно, электрическое поле создается электрическими зарядами (электростатическое поле), а также меняющимся во времени магнит- ным полем (вихревое поле). Обозна- чим через и фь потенциалы элект- ростатического поля в точках а и b закона можно соответственно. На основании Ома для правого полукольца написать Фа“ Ф6 + 1/, = а для левого полукольца фь —фа4-1/а^,==^1> Отсюда находим __ _ _ L 9. <ра-<Рь-Ьпг 2(R!s + Ri)- (1) 51 Таким образом, если /?2 > #i, то фа > Фь- Из (1) легко найти напря- женность электростатического поля Ех: <Pa-<Pb_f.r nr 2(Я2+/?1) • Напряженность электрическогр поля Е2, созданного изменением во времени магнитного поля, вычислена в задаче 588. Она равна Ё2 = = kr/2. (На рисунке указано направление напряженностей в левом и правом полукольце для случая, когда Т?2 > /?х.) 334
591. В контуре ABCD действует э.д.с. индукции ^1 = Ла2, а в контуре BEFC <§2~ка2/2. Простейшая эквивалентная схема с гальваническими элементами, заменяющими э. д. с. индукции, для Рис. 472. нашей цепи имеет вид, изображенный на рис. 472. На основании закона Ома = — Л Заг = /22аг —^2. Вследствие сохранения заряда и постоянства потенциала /1 = /2+^з- Из данной системы уравнений легко найти все три тока: Л = + / 2<£i+8<& , ,2^-3^ 1 22аг ’ 2 22аг ’ ? Наг Учитывая выражения для и ^2, будем иметь I — -L / — 3_ka . _ 1 ka 1~22ri /з“227* 592. Разность потенциалов между любыми точками кольца должна быть равной нулю. В противном случае мы придем к противоречию, применяя закон Ома к короткому и длинному участкам кольца. Кроме того, это очевидно и из соображений симметрии. Отсутствие разности потенциалов означает, что электростатическое поле внутри кольца равно нулю. Ток возникает вследствие наличия э. д. с. индук- ции, распределенной равномерно вдоль кольца: I = е;1г = $i/R, где ei и —э.д.с. индукции на коротком и длинном участках кольца, аги £ —соответственно сопротивления участков. Несмотря на отсутствие разности потенциалов между точками А и В, электрометр обнаружит разность потенциалов между стерж- нем и корпусом. Дело в том, что в проводниках АС и BD ток равен нулю.' Следовательно, в каждой точке этих проводников стороннее электрическое поле индукционного происхождения уравновешивается напряженностью электростатического поля, возникающего вследствие перераспределения зарядов в проводниках под влиянием э. д. с. индук- ции. Работа электростатических сил при перемещении по замкнутому контуру ACDBA равна нулю. На участке АВ электростатическое поле отсутствует. При перемещении заряда по проводникам АС и BD работа электростатических сил равна э. д. с. индукции в этих проводниках и имеет противоположный знак. Следовательно, для равенства нулю работы электростатических сил вдоль замкнутого контура необходимо, чтобы разность потенциалов между точками С и D равнялась э. д. с. индукции в проводниках АС и DB и сов па- 335
дала с ней по знаку. Так как э. д. с. индукции в замкнутом кон- туре ACDBA равна нулю (магнитное поле не пронизывает этот контур), то на участке АВ э. д. с. индукции равца по величине и противоположна по знаку э. д. с. в проводниках АС и BD, если пренебречь работой сторонних сил индукции на участке между стержнем и корпусом электрометра по сравнению с работой в про- водниках АС и BD. Поэтому электрометр пркажет разность потен- циалов, приблизительно равную э. д. с. на участке АВ. 593. В отличие от задачи 592 разность потенциалов Ua —Ur не равна нулю. Запишем закон Ома для всех трех участков про- водника, обозначив токи на участках ADB, ARB и АС В через 119 /2, /3, а соответствующие э. д. с. индукции — через ^2, ^з: j ^14-^л— Ur т в—UA г Ur ‘1 Р ‘ D » *8 п А1 А 2 АЗ В силу сохранения заряда и’условия стационарности/2== Л 4~ h- Складывая два первых уравнения, будем иметь + ^2^2 = (*?14~ Si ~ S' Вычитая первое уравнение из третьего, получим /3Л3— Л^1==<§>з — Si* Но э. д. с. индукции в контуре ACBDA равна нулю, так как он не пронизывается магнитным полем. Следовательно, Si~S&* 'з^з— /1/?1 = 0. Система уравнений дает следующее значение искомого тока: Г _ _______S^l_______ 3 ^1-^2 4“ ^1^3. + ^2-^3 594. При отличном от нуля сопротивлении R3 из уравнений задачи 593 можно найти 7 <^з , S(R1+Rs) 1 ^1^2“}“ ^2^3 4~ ^1*3 * 2 ^1^2 4" ^2^3 4“ ^1^3 * i _________SRi____ ^1^г4~ ^2^з4" ^х^з Если /?з = 0, то /i = 0, /2 = — В общем случае У ___Ur —____________S^l^i________ При /?3 = 0 UA-UB=-^^, Ai-h а2 причем UA — Ur — — (так как на участке ADB /г = 0), где^— э.д.с. индукции на участке ADB. 595. В проводниках, составляющих цепь, вследствие изменения во времени магнитного поля возникает электрическое поле (вихре- 336
вое, т. е. не электростатическое). Работа этого электрического поля по перемещению единичного положительного заряда вдоль всей цепи численно равна э.д.с. индукции, которую мы обозначим через Ее можно вычислить на основании закона Фарадея, учитывая изме- нение магнитного потока, связанное с изменением во времени маг- нитной индукции. Тогда получим S^ktSv+lvt). Кроме того, при движении проводника в магнитном поле возникает э.д.с. в результате действия силы Лоренца на заряды проводника. Эта э.д.с. равна ^2== vBl = vl (Во4~ ^0’ Полная э.д.с. действующая в цепи, равна S=Si4"Si~ k (*^о + ivt)4"(Во“Ь^0* $ можно получить из закона Фарадея, учитывая одновременно обе причины ее возникновения. Тогда следует написать, что ^ = ДФ/Д/, где Ф = (В0+ Так как Дф = [Во + k (t 4- ДО] [So4- vl (t + Д/)] - (Bo 4- kt) (So 4- vlt) = = Bfll Д/ 4- kSQ M + 2kvlt M+kvl (ДО2, TO ДФ/Д/ = B^vl 4- kSQ-{~2kvlt 4- Д/ kvl. При Д/, стремящемся к нулю, ДФ/Д t = B^vl 4- kS0 4- 2kvl t. Таким образом, ^ = ^(So+^0 + ^(Bo4-^) = ^i + ^2. Силу тока найдем на основании закона Ома: . $ kSQ-\-2klvt -\-vlBQ “/Г = R * Ток в цепи направлен от а к Ь. 596. В обоих случаях равновесие установится, если момент сил, действующих со стороны магнитного поля на индукционный ток в кольце, равен нулю или отсутствует индукционный ток. Это будет в том случае, когда плоскость кольца располагается вдоль силовых линий поля (индукционный ток равен нулю), либо же тогда, когда плоскость кольца строго перпендикулярна силовым линиям (момент сил равен нулю). В соответствии с правилом Ленца в нарастающем магнитном поле устойчиво будет первое положение кольца, а второе будет неустойчивым. В убывающем магнитном поле, напротив, рав- новесие будет устойчивым при наличии прямого угла между плос- костью кольца и силовыми линиями и неустойчивым, когда плоскость кольца параллельна силовым линиям. 597. Так как согласно условию напряженность магнитного поля прямо пропорциональна времени: И — (Nfl) kt, то э.д.с. самоиндук- ции равна Si = ^o kS (S = nr2) 337
и направлена против тока. Напряжение на зажимах соленоида должно быть равно U==iL0(N*/l)kS+ kRt. В этом случае 1 — (U — $i)jR = kt. 598. При /?*=0 э.д.с. самоиндукции остается постоянной, так как напряжение на зажимах соленоида £/= = Из решения задачи 597 следует, что при постоянной ток изменяется пропор- ционально времени: I = kt, причем k — Ul/^SN2. Следовательно, Если сопРотивление конечно, а не равно нулю, ток будет возрастать по такому же закону до тех пор, пока падение на- пряжения IR на сопротивлении R будет пренебрежимо мало по сравнению с 599. Работа батареи за время т будет равна A = <£>Q, где Q — количество электричества, прошедшего за время т по соленоиду. Ток в соленоиде возрастает прямо пропорционально времени: I — -*^3 (см. решение задачи 598). Поэтому Q будет равно произведению средней силы тока (/н + /к)/2 (/н = 0) на время т, или же численно равно площа- ди заштрихованного треугольника рис. 473): 4 2|i0S№ Отсюда работа будет равна 2MoS№' (см. г * Рис. 473. Эта работа идет на увеличение энергии магнитного поля. Мы можем записать, что A = W, где W — энергия магнитного поля. Учитывая, что H = и подставляя выражение для тока, эту энергию мы и № можем представить в виде SI. 600. Так как сопротивление кольца равно нулю, то и суммарная электродвижущая сила в нем всегда должна быть равна нулю. Это может быть только в том случае, если изменение полного магнитного потока, пронизывающего кольцо, равно нулю. Следовательно, изме- нение внешнего магнитного потока Фо равно по величине и проти- воположно по знаку изменению магнитного потока, созданного индук- ционным током: ДФ0 = £ А/. Учитывая, что поток Фо нарастает от 0 до лг2В0, а индукционный ток меняется при этом от 0 до /, полу- чим nr2BQ^LI. Отсюда / = лг2В0/Л. 601. Магнитный поток через кольцо не может изменяться (см. задачу 600). Следовательно, ф == лгаВ. Вначале этот поток создавался внешним магнитным полем, а после его выключения — индуцирован- ным в кольце током. 602. Если омическим сопротивлением кольца можно пренебречь, то полный поток магнитной индукции через кольцо не будет меняться (см. задачу 600). А это значит, что поле индукционных токов в 338
кольце направлено всегда против поля электромагнита. Следовательно, кольцо будет отталкиваться. 603. Если по обмотке соленоида течет ток /, то, по определению коэффициента самоиндукции L, поток магнитной индукции Ф сквозь соленоид рдвен Ф —L/. (1) Поток магнитной индукции есть Ф — BSN, где В — индукция магнитного поля соленоида, <$—сечение каждого витка, // — полное число витков соленоида. Как известно, индукция магнитного поля длинного (d /0) соленоида равна В = |гоп/, где л —число витков на единицу длины соленоида. В условиях нашей задачи л, I I 1 А;=— > л = * 7~- nd nd lQ Поток магнитной индукции Ф, следовательно, равен ф-ц Jutland 4 4nlo (2) Сравнивая (1) и (2), находим г__Но I2 4п10' 604. По определению магнитный поток через соленоид равен Ф — BSN, где В —индукция магнитного поля соленоида, S —сечение каждого витка, W— число витков соленоида. Как известно (см. за- дачу 603), индукция В соленоида, по которому течет ток /, равна В = ц0 (Nil) /, rj^Z — длина соленоида. Итак, Ф = |г0 (№/Z) SI, Посколь- ку в условиях данной задачи / = const, изменение магнитного потока ДФ происходит только из-за изменения длины соленоида (его геомет- рии). Другими словами, поток через соленоид (Ф = £/) изменяется из-за изменения коэффициента самоиндукции L: ДФ/Д/ = Д (LI)/Nt = I &L/M. Сохранить ток постоянным при растяжении соленоида можно, если изменять разность потенциалов на его концах на величину, в любой момент равную и противоположную э.д.с. самоиндукции $ — —ДФ/Д/. Вычислим ДФ/Д/. Для этого достаточно вычислить &LI&t: A/2Q±r_1_____________1 1 Д/“Мо/¥ ^Д/ l/o+ctf /0 + v(/+ДО] • При Д/, стремящемся к нулю, Д£/Д t = W(/o + ^)2’ 339
Соленоид растянется вдвое за время tlt которое легко определить из равенства Z0 + tf/i = 2/0, откуда и в момент На такую величину надо изменить разность потенциалов на концах соленоида. 605. U = BQnr2N(d sin (dt. 606. Z/^5 = BonrWcd sin со/, 7 = и^в//?==(Волг2№д/7?) sin со/. 607. / = BonrWto sin (со/ — <p), где tg ф = 1 /toCT?; UAB = [BonrWto cos (to/ — ф)]/соС = [BQnr2N cos (to/ — <p)]/C; UBc==B0nr2N(i)R sin (to/ —ф). 608. 1) Так как концы А и В разомкнуты/по участку АС ток не течет. Следовательно, падение напряжения на участке АС равно нулю. Поэтому напряжение (72 = £/1. 2) В случае, когда между точками В и С приложена перемен- ная разность потенциалов, ток, текущий по участку ВС, создает переменный магнитный поток,' который наводит на участке АС э.д.с. индукции. Так как то амплитуда наведенной э.д.с. также будет равна (/х. Поэтому амплитуда напряжения U2 между точками А и В будет равна 2{/х (повышающий автотрансформатор). 609. Наиболее неудовлетворителен 3-й способ, так как токи Фуко циркулируют в плоскостях витков обмотки, не встречая в этом слу- чае изолирующего слоя. 1-й способ позволяет избавиться от большей части вихревых токов, однако не от всех, так как в одном слое обмотки автотрансформатора фактически имеется много витков вокруг сердечника и один виток вдоль него. Наилучшим является 2-й спо- соб, который и применяется на практике. 610. Так как /? = 0, то ток / отстает по фазе от напряжения U на л/2. Графики U — UQ sin to/, I — Io sin (to/— л/2) и мгновенной мощности W —IU изображены на рис. 474. Знак W меняется каж- дую четверть периода. Положительному значению W соответствует поступление энергии от источника в катушку. При отрицательном W энергия из катушки поступает обратно в источник. В среднем за пе- риод катушка не потребляет мощности; средняя мощность равна нулю. 611. Эффективной величиной переменного тока называется вели- чина постоянного тока, который выделяет в проводнике такое же 340
количество тепла, что и переменный, за равное с ним время. Под- считаем количество тепла, выделившегося за период: Q = llRT/8 + IqRT/8 = llRT/i. С другой стороны, (? = /?фф/?Г. Отсюда Л,фф=Л)/2. 612. При прохождении по проводнику переменного тока коли- чество выделенного тепла равно W = Ia$$Rt. Выражение же для ко- личества выделенного тепла W = (£/эфф/₽) t справедливо только тог- да, когда имеет место закон Ома в обычной форме: I — U/R. У об- мотки трансформатора велико индуктивное сопротивление. Поэтому закон Ома в обычной форме и, следовательно, выражение W = (Ul$$/R)t не имеют места. Количество выделенного тепла мало, так как мала сила тока и мало омическое сопротивление обмотки. 613. Если La)^> R, то сдвиг фаз между током и напряжением велик и потребляемая сетью мощность не может быть большой. Включение конденсаторов уменьшает этот сдвиг фаз, так как ток через конденсатор опережает напряжение, компенсируя тем самым отставание тока по фазе в приборах с большой индуктивностью. В результате потребляемая сетью мощность увеличивается. 614. W = (U2-Ul-Ul)/2R. 615. Соленоид с числом витков обозначим индексом /, а со- леноид с числом витков Л/2 индексом 2. Индукция магнитного поля соленоида 1 равна Это поле создает сквозь соленоид 2 магнитный поток Ф12, равный Ф12 = В15№2=ц0(№1адВ/, откуда для коэффициента взаимной индукции L12 получается выра- женив 4 = Но (ад/OS. (1) Аналогично получаем выражение для потока магнитной индук- ции Ф21, созданного соленоидом 2 и пронизывающего соленоид /: ф21=но (ад/os/, откуда для коэффициента взаимной индукции Ь21 получается выра- жение Д2х = Но (ад/OS. (2) Из сравнения (1) и (2) находим, что Lia = L21. 616. Скорость изменения магнитного потока постоянна, и, следова- тельно, постоянна э. д. с. индукции в катушке. Если катушка включена в замкнутую цепь, то по ней будет течь постоянный ток. Как и любой постоянный ток, он устанавливается не сразу. Время его установления определяется коэффициентом самоиндукции катушки и ее сопротивлением. 617. Полный поток магнитной индукции Ф, пронизывающий сверхпроводящее кольцо, как показывает решение задачи 600, не может измениться. Вследствие этого при сближении колец увеличе- ние потока за счет взаимной индукции компенсируется его умень- шением в результате уменьшения тока, текущего по кольцу. При 341 *
большом расстоянии между кольцами их взаимной индукцией можно j пренебречь, и тогда <D = L/0, где L — индуктивность кольца. Когда | проводники сблизятся вплотную, то магнитный поток через каждое 1 кольцо будет равен Ф = 2£/, где / — искомый ток. Следовательно, J / = /0/2, т. е. токи в кольцах уменьшатся в два раза. | 618. Допустим, что в начальный момент ток /01 в первом кольце I больше тока /02 во втором кольце. При сближении колец токи в них | будут уменьшаться (см. задачу 617). В некоторый момент времени 1 ток /2 станет равным нулю и, следовательно, станет равной нулю 1 сила взаимодействия колец. Однако по инерции кольца будут про- ] должать сближаться. Во втором кольце возникнет ток в направле- 1 нии, противоположном первоначальному, а, ток в первом кольце $ начнет увеличиваться. При этом потоки магнитной индукции через каждое кольцо останутся неизменными. Между кольцами возникнет отталкивание, их движение вследствие этого начнет тормозиться. Сблизиться вплотную кольца не смогут, так как при этом одно- временно должны были бы выполняться равенства £./qi = /./i LI 2, L/02=£./j ///g» \ что возможно лишь при условии /О1 = /о2- Кольца на мгновение а остановятся на некотором расстоянии друг от друга и затем начнут ‘Ц расходиться. При этом будет уменьшаться ток /2 во втором кольце | до тех пор, пока не станет равным нулю. При /2=0 ток Л приоб- “ ретет то же значение, что и при сближении колец в момент пре- , j кращения тока /2. Далее начнется притяжение колец и т. д. Процесс | будет повторяться периодически. 619. Напряжение на зажимах первичной обмотки UY при пре- небрежении ее омическим сопротивлением может быть представлено как алгебраическая сумма э.д.с. самоиндукции этой обмотки и э.д.с. индукции, возбуждаемой в ней током, текущим во вторичной обмотке: U± = L^ ЬЦЫ — М Д/2/Д/. ; % Знак минус возникает вследствие того, что токи /х и /2 имеют ш противоположные фазы. Если токи меняются по законам — /01 sin о/, Я /2 =/02 sin со/, то Я kl-jM = to/01 cos со/, Д/2/Д/ = со/О2 cos со/. Л Так как напряжение Ur смещено' по фазе относительно тока /х на i л/2, то можно записать Ux = t/10 cos со/. . i Разделив выражение для на cos со/, будем иметь * ^1, М Цш 01“Е?02, j есть ток холостого хода, если омическое сопротивление обмотки не учитывать. Пренебрегая током холостого хода, найдем /oi//o2 = M/L1. Используя выражения для коэффициентов самоиндук- ции и взаимной индукции, получим /1//2 = /01/^02 = ^2/^1 • 620. Индукция суммарного поля внутри соленоидов равна в=Й! ± ± ад. 342
где выбор знака плюс или минус зависит от того, одинаковы вли противоположны направления токов в соленоидах. Энергия W сум- марного поля во всем объеме внутри соленоидов равна ' w=^si=^r К^Л)2 ± 2(ад(ад+(А/2/а)2). Воспользовавшись выражениями для индуктивностей Lx и L2 соленоидов: Lx = р0 (A^i/Z) S, L.2 — р0(Аф/) S и выражением для коэф- фициента взаимной индукции: Ь12 = Р'0(^1#2/0'$, можно магнитную энергию соленоидов представить в виде Первый член этой формулы дает собственную энергию тока вто- рой — определяет собственную энергию тока /2. Наличие третьего слагаемого показывает, что энергия двух токов в контурах, обла- дающих магнитной связью, отли- чается от суммы собственных энер- гий токов на величину №12 = = ± Ь12/1/2. Она называется вза- имной энергией двух токов. 621. Положительные, полувол- ны тока зарядят конденсатор до амплитудного напряжения город- ской сети, равного 127 У^2 В=180 В. Когда диод заперт (не проводит ток), к нему приложено напряже- ние сети (с амплитудой 180 В) плюс такое же напряжение заряженного конденсатора. Изменение потен- циала вдоль цепи в этот момент времени изображено на рис. 475. Если выпрямитель работает без нагрузки, то конденсатор должен рассчитываться на пробивное напряжение, не меньшее 180 В, а диод — на напряжение, не меньшее 360 В. § 24. Электрические машины 622. Если частота переменного тока остается прежней, то это означает, что осталось прежним число оборотов двигателя и генера- тора. Поэтому не изменяется и э.д.с. генератора. При большем внешнем сопротивлении в цепи потечет меньший ток и, значит, будет выделяться меньшая мощность. Следовательно, мощность дви- гателя, вращающего генератор, должна быть уменьшена. 623. При включении конденсатора С в цепь переменного тока будет происходить периодическая перезарядка конденсатора. Заряд, притекающий по проводам к обкладкам конденсатора, будет тем больше, чем больше емкость конденсатора С. Так как за половину периода этот заряд должен смениться противоположным по знаку, то ток, протекающий по цепи, должен быть пропорционален частоте. Итак, при одной и 1ой же амплитуде переменного напряжения на конденсаторе ток в цепи с конденсатором пропорционален частоте и емкости конденсатора (7 ~ ссС). Можно сказать поэтому, что конден- сатор обладает «емкостным сопротивлением», обратно пропорциональ- 343
ным частоте й емкости. При параллельном соединении I = Лсд1Спарал = ka)^ (Cj 4“ С2). • При последовательном соединении Г k,. ^*1^'2 / = кСО2С Послед ~ ^tt)2 • Отсюда ^2 __ (Ql ~Н ^г)2__ । о । ^2 й>1 ^•'1^'2 ^2 624. Работа, совершаемая полем по перемещению проводников с током (обмотки якоря), не равна полной работе поля. Кроме работы по перемещению проводников, магнитное поле совершает работу по торможению электронов в проводнике, что приводит к появлению в' обмотке якоря э.д.с. индукции. Первая часть работы положительна, а вторая —отрицательна. Полная работа магнитного поля равна нулю. Электродвижущая сила источника, создающего в якоре мотора ток, совершает положительную работу, компенси- рующую отрицательную работу магнитного поля по торможению электронов. Двигатель совершает работу, по существу, за счет энер- гии источника, питающего мотор. . 625. Мощность, потребляемая мотором, W = IU; U>= где э.д.с. индукции, возникающая в якоре. Следовательно, W = Igi+PR, Здесь Z2/?—джоулево тепло, выделяемое в обмотках, a мощ- ность против э.д.с. индукции. Она равна механической мощности развиваемой мотором. Эта мощность = —^2)//?, так как I = (U — Данное выражение имеет максимум при ^ = С//2. Следовательно, максимальное значение TF1 = C/2/4Z?= 180 В. Мощ- ность в 200 Вт мотор развить не может. 626. Согласно решению задачи 625 1Гг1 = С/2/47?. Если якорь мотора неподвижен, то протекающий по обмотке ток I = U/R. Коли- чество тепла, выделяемое в обмотке в единицу времени, W = I2R ~ = U2!R. Следовательно, W — ^Wr. 627. Сила тока, текущего по обмотке двигателя, будет опреде- ляться э.д.с. сети ее сопротивлением г и э.д.с. индукции возникающей в якоре мотора: I = — Разность потенциалов U на клеммах двигателя равна в любой момент так как сопро- тивление обмотки равно нулю. Следовательно, мощность W — IU — = (SSi~~ St)/r определяется э.д.с. сети, ее сопротивлением и 628. Для сериесного мотора развиваемая максимальная мощность (см. задачу 625) ^тах~4(^1 + /?2). Потребляемая мотором мощность w=ui = u =-—У2...... t + 2(/?x+R,)’ так как $i = U/2. Следовательно, коэффициент полезного действия 344
rj = 1/2. Для шунтового мотора 1Fmax = £/2/4₽i. Потребляемая мощ- ность w-UI-и ( и~£' I и -^2(2/?1+^) w-ui-uy 2RtR2 * Следовательно, __ 1 1 П “ 2 1 +2RJR, * т. е. меньше 50%. 629. При заданной скорости вращения момент внешних сил М, действующих на якорь, равен моменту сил, действующих на якорь со стороны магнитного поля. Так как этот последний пропорциона- лен произведению силы тока / в якоре на индукцию магнитного поля В, то M = aIB (a = const). (1) Э.д.с. в обмотке якоря пропорциональна индукции магнитного поля и числу оборотов в секунду: = (₽ = dbnst). (2) Напряжение U на клеммах мотора равно I U = IR + Si = IR + $nBt (3) где 7? —сопротивление обмоток. При включении мотора на холостом ходу момент внешних сил определяется только трением, которое при хороших подшипниках очень мало. Поэтому I и В согласно уравнению (1) также малы (В пропорционально / и уменьшается вместе с током). Из уравне- ния (3) следует, что при малом значении I и В возможны только большие п. Поэтому двигатель набирает очень большое число обо- ротов. Рис. 476. 2BS BS Рис. 477. 630. Обозначим через I длину, а через d ширину витка (рис. 476). Сила F, действующая на проводник длины /, равна F—IBI. При этом мощность W = 21 Blv = IBSto. Сила тока / определяется из выражения / = ((/ — $i)/R, где == BSco. Окончательно W запишется в следующей форме: w BSU B*S* 2 w=— 315
W достигает максимального значения 1Гтах = U*{4R при со = = U!2BS. При этом (gi = Ul2 и I — U/2R. В единицу времени бата- рея совершает работу С/2/2/?. Из этого количества половина превра- щается в механическую мощность, другая же половина выделяется в виде тепла. На рис. 477 изображена графически зависимость W от о. .. BSU B*S* 631. Л1=—-5-=—* со. Момент будет равен нулю тогда, когда А А (n = U/BS (см. рис. 478). При этом / = 0, так как <si = U. 632. Характер зависимости IF от В изображен на рис. 479. Мощность достигает максимального значения при B — UI2S^. При этом $>/ = С//2 и TFmax = C/2/4/?. 633. Вращающий момент М достигает своего максимального зна- чения Afmax = ^72/4/?ю при B — U!2S(n. - 634. Мощность шунтового мотора, так же как и сериесного, равна W = (U— R — сопротивление якоря (см. задачу 625). Мощности W = 160 Вт соответствуют два значения (^\ = 80В, .(^2 = 40 В. То или иное значение определяется конструктивными особенностями мотора. По закону Фарадея прямо пропорционально числу оборотов п якоря в секунду и индукции магнитного поля, созданного статором. Для шунтового мотора эта индукция не зависит от нагрузки. Поэтому можно записать: ^ — ап, где а —постоянная вели- чина, определяемая конструкцией двигателя и приложенным напря- жением. По данным задачи получаем ях=8 и а2 = 4. ^г- не может превышать 120 В. Следовательно, максимальное число оборотов п равно либо /1!= 15 об/с, либо и2 = 30 об/с. 635. Если напряжение на статоре поддерживается постоянным, то при заданной скорости вращения якоря э.д.с. индукции в якоре совершенно не зависит от того, вращается ли якорь мотора за счет действия на якорь магнитного поля статора или же якорь приво- дится во вращение с помощью механического привода. Развиваемая мотором мощность W — Ма>. В нашем случае IF = 160 Вт. Э.д.с. индукции определится из уравнения W = = (U£i—$F)/R (см. задачу 625). Отсюда 346
увеличении тока в обмотках имеет два значения: ^ = 80 6, <£2 = 40В. Э.д.с. генератора также будет иметь эти значения. Неоднозначность результата связана с тем, что одна и та же мощность получается при одинаковом значении произведения а данному значению этого произведения соответствуют две пары возможных значений / и То или иное значение а следова- тельно, и тока определяется конструктивными особенностями мотора: числом витков, их конфигурацией и т. д. 636. Механическая мощность, развиваемая мотором, равна (см. решение задачи- 625). Э.д.с. индукции в якоре (§i~kBn, где Z? — коэффициент пропорциональности, определяемый числом витков обмотки якоря и их площадью, а В —индукция магнитного поля статора, прямо пропорциональная то- ку. Исключая из этих уравнений, найдем __ U 2xMR П kB kW • Зависимость п от В изображена на рис. 480. Если В В^ = 2пМ^!ки, то п^О. Физически это означает, что якорь мотора не . будет вращаться. При B — Bm — ^nMRIkU число обо- ротов достигает максимума. Следо- вательно, если Вт > В > Во, то при статора число оборотов растет, а при В > Вт число оборотов падает. В случае, когда мотор работает без нагрузки (М = 0), число оборотов , т- е. обязательно уменьшается с увеличением В. 637. Магнитные поля В1} В2 и В3 можно записать в Следую- щем виде: Bl —Во sin о/, я 13 = Во sin Выберем оси координат х и у, как показано на,рис. 220, и найдем сумму проекций напряженностей полей на эти оси: (-2X2 (i)Q т л ) cos -Q- л + о у о + Во sin \ - 2 Л ) Sin -у л \ 4 л] COS-g-Л, . 4 Sin -5- л. <э Проведя несложные преобразования, будем иметь 3 3 Вх = у Во sin о/, Ву = — Вю cos to/. 347
Эти значения проекций будут иметь место только в том случае, если вектор, изображающий магнитное поле, вращается с постоянной угло- вой скоростью со в направлении часовой стрелки. 638. Поток вектора магнитной индукции через рамку равен Ф = Bab cos (со — Q) t. Согласно закону электромагнитной индукции э.д.с., наводимая в рамке, равна = — ДФ/Д/ = Bab (со — Q) sin (со — Q) /. Следовательно, ток в рамке со временем изменяется по закону _ Bab (со — Q) sin (со — Q) t Отсюда амплитудное значение тока /0 равно r __Bab (со —Q) 0 £ • Зависимость /0 от Q является линейной. 639. Момент М сил, приложенных к рамке, равен М = IBab sin (со — Q) t. Согласно решению задачи 638 .Bab(& — Q) sin (со —Q)/ /- £ . Следовательно, BW2 (со—Q) sin2 (со —Q) t R Амплитудное значение момента _BW2(co-Q) ^о- £ ‘ Подобная зависимость имеет место в асинхронных двигателях. Глава IV. КОЛЕБАНИЯ И ВОЛНЫ § 25. Механические колебания 640. Центр тяжести доски в исходном положении равновесия находится на высоте ус над горизонтальным уровнем, на котором расположена ось цилиндра, причем yc~R-\-h/2. При повороте доски без проскальзывания на малый угол <р центр тяжести доски займет положение на высоте ус,, которую легко определить: Ус = (я+ 4) cos ф+sin ф. 348
Заметим, что при малых углах ф с точностью до членов второго порядка малости sin ф « ф, cos ф « 1 — ф2/2. Действительно, так как cos ф = У I — sin2 ф » У 1 —ф2, то, обозначив cos ф = 1 — х, получим 1—ф2 « 1 — 2х-|-х2, откуда, пренебрегая х2, находим х « ф2/2, и, следовательно, cos ф « 1—ф2/2. Таким образом, Ус- = (*+4)(1-у) + *Ф2- Колебания доски около положения равновесия возникнут при условии ус, ~УС> т. е. при Если R — h/2 < 0, то доска упадет. 641. Уравнение движения груза массы т имеет вид та = — kxt где х —абсолютное удлинение пружины. Из уравнения следует, что период колебаний т может зависеть только от т и k. Размерность коэффициента жесткости [k]~ М/Т2. Следовательно, х = С У m/k, где С — безразмерный коэффициент. Точный расчет приводит к выра- жению т = 2л У m/k. 642. Период малых колебаний математического маятника не зави- сит от амплитуды (т. е. от наибольшего угла отклонения) и равен Т = 2пУ l/g. Независимость периода от величины максимального угла отклонения является следствием того, что сила, направленная к положению равновесия, в каждый момент пропорциональна углу а отклонения маятника. Если бы эта пропорциональность углу откло- нения а сохранялась и при больших амплитудах, период колебаний такого воображаемого маятника в любом случае был бы равен Т = 2л yljg. Однако при больших углах отклонения сила, возвра- щающая маятник, пропорциональна не углу а, а синусу этого угла- Так как sin а < а (а # 0), то при тех же амплитудах возвращающая сила, а значит/ и ускорение реального маятника меньше, чем у того, у которого сохраняется пропорциональность между силой и углом. Поэтому период 7\ колебаний реального маятника больше, чем воображаемого, т. е. Т± > 2п У l/g. Следовательно, с увеличе- нием максимального угла отклонения период колебаний математи- ческого маятника увеличивается. 643. Пусть x=gafl, где х — период колебаний математического маятника, 7 —его длина: [Т] = т, [ g“] [ /₽] = Из последнего соотношения следует, что а + Р = 0, 2а = —1. Таким образом, х — f (ф) УТ/g. Здесь /(ф)—функция максимального угла ф (f (фа) > f (Ф1)> если Фа > Фр см. задачу 642). Расчет показывает, что при ф—*0 /(ф) имеет предельное значение 2л. Итак, период малых колебаний математического маятника равен х — 2пУ l/g. 349
644. Обозначим через I длину недеформированной пружины. Тогда расстояния первого и второго брусков до центра масс опреде- ляются из соотношений /n1Z1 = m2Z2, /1 + /2 = /. Обозначим через х и у перемещения первого и второго брусков в случае, когда пружина сжата. Тогда расстояния брусков до центра масс удовлетворяют соотношению т1 (Zi — х) — т^(12—у)> или т1х^=т2у. Пружина сжата на величину Сила, с которой пружина действует на первый брусок, равна F = krx, где k± — k —. Таким образом, период колебаний первого бруска равен (см. за- дачу 641) т= 2л тут2 Период колебаний второго бруска, очевидно, будет таким же. 645. Пружина, распрямляясь, действует на оба груза. Груз, стоящий у стенки, сначала неподвижен, а второй груз начинает двигаться. Когда пружина полностью распрямится (т. е. придет в недеформированное состояние), второй груз будет иметь скорость v — m2t>2 2 Следовательно, система приобретёт количество движения, равное m2v = km2x. Это количество движения будет сохраняться, так как внешние силы в горизонтальном направленни не действуют. Центр масс системы будет двигаться со скоростью К km2x Vc~~m1 + m2' При этом грузы будут колебаться относительно центра масс с перио- дом колебаний т = 2л V к(т^т2) (см. задачу 644). 646. Период колебаний груза на пружине Т — 2л V^n/k, где k— коэффициент упругости пружины, равный отношению силы, вызвавшей удлинение пружины, к этому удлинению: k = F/x. При 350
последовательном соединении двух одинаковых пружин, растягивае- мых силой F, fe1 = F/x1 = F/2x=:^/2, так как каждая из пружин увеличивает свою длину на х. При параллельном соединении двух одинаковых пружин сила Flt необходимая для увеличения длины каждой из пружин на х, должна быть в 2 раза больше F. Следо- вательно, fe2 = Л/* = • При последовательном соединении 7\ = 2л — 2л У2т/k, а при п араллельном Т2 = 2л У m/2k. Отсюда 7\/Т2 = 2. Период уменьшится в 2 раза. 647.-Отклоним оба маятника от вертикали в одну и ту же сто- рону на одинаковый угол. Пружина при таком отклонении не будет деформирована. Легко сообразить, что, отпущенные из этого поло- жения, маятники будут колебаться в фазе с частотой со = Vgii. При отклонении в разные стороны на одинаковые углы колебания маятников будут происходить в противофазе и пружина будет де- формироваться. Для того чтобы подсчитать частоту этих колебаний, найдем силу, возвращающую маятники в положение равновесия. При отклонении на угол <р сила, действующая со стороны пружины на тело /п, равна 2kl sin ф. Сумма проекций силы тяжести и силы упругости на касательную к окружности, так называемая «восста- навливающая» сила /\, будет рацна == mg sin <р + 2kl sin ф cos ф (рис. 481). Так как при малых углах соэ'ф ~ 1, то P1^(mg-\~2kl) sin ф, или P1 = m(g~^2kl/m) sin ф. Для математического маятника «восстанавливающая» сила равна mg sin ф. При этом частота колебаний при малых углах ф опреде- ляется формулой (d—yg/l. В нашем случае роль g играет величина g 2kl/m. Следовательно, со= У (g+2kl/m)/l. Период колебаний T,.^2nVl/(g+2kl/m). 648. Вертикальная составляющая силы натяжения Т равна F = Т cos а (рис. 482). Для конического маятника F = mg, так как 351
груз не обладает ускорением в вертикальной плоскости. В случае математического маятника при максимальном отклонении его от по- ложения равновесия (на угол а) результирующая сила направлена по касательной к траектории груза. Следовательно, T — mg cos а. При отклонении на угол а натяжение нити конического маятника будет больше. 649. На поверхности земли период колебаний маятника 70 = 2лУl/g, На высоте h над землей период колебаний маятника 7Х = 2n]f l/gv Число колебаний за сутки = 24.60-60-1/71 = &/71. Следовательно, на высоте h над землейг часы отстанут за сутки на время * Д/1 = ^(71~70) = ^(1-70/71). Отношение периодов Tq/1\ = V &/£ = Я/(Я + Л), как вытекает из закона всемирного тяготения. Отсюда kh/R « 2,7 с. Если часы опущены в шахту, то отношение ускорений g2/g = = (R—h)/Rt так как 4л 1 4л 1 g==v —^р__, Следовательно, Tq/T2 — Vg2/g = V (R—h)/R « 1— h/2R. Часы в этом случае отстанут на. время Д/2 = &(1—TQ/T2) = kh/2R « 1,35 с. 650. Каждая половина стержня с шариком на конце представ- ляет собой математический маятник длины d/2, совершающий коле- бания в поле тяготения большого шара. Математический маятник в поле тяготения Земли имеет период малых колебаний 70 = 2л УТ/g. По закону всемирного тяготения mg^ymM^/R2, следовательно, Т„ = 2л где у—гравитационная постоянная, /Из — масса Земли, R — расстояние маятника от центра Земли. Соответственно в поле тяготения большого шара период малых колебаний матема- тического маятника длины / = d/2 будет равен 7 = 2л УdL2/2yM « « 5,4 часа. 651. Период колебаний математического маятника равен 7 = 2л У l/g', где g'—ускорение свободного падения в соответству- ющей системе координат. В нашем случае g’ — Vg2+a2t где g — ускорение свободного падения относительно Земли. Таким образом, Т = 2лУ l/Yg2+a*. ' 652. Т = 2л К l/(g+a). 653. Заменить в предыдущем ответе g+a на |g—а|. 654. Колебания кубика в чаше вполне аналогичны колебаниям математического маятника, только вместо натяжения нити на кубик действует реакция опоры. Следовательно, искомый период колебаний Т=2л УяЦ. 655. При /И^>т ускорение чаши a = F/M—g. Следовательно (см. задачу 654), Т = 2л y>/(g+«) = 2л У RM/F. 352
При F = 0, т. е. при свободном падении чаши, Т = оо — колебания отсутствуют. Если F = Mg, то Т — 2л Rig. 656. Колебания кубика вызовут периодическое смещение чаши в горизонтальной плоскости. Следовательно, период колебаний ку- бика уменьшится, так как в системе координат, связанной с чашей, появится дополнительное переменное ускорение, направленное гори- зонтально (см. задачу 651). 657. Сравним движение центра обруча с движением конца мате- матического маятника длины 7?—г. Обе эти точки описывают дугу окружности радиуса R—г. Предположим, что при угле <р0 обруч и маятник покоятся. На основании закона сохранения энергии для скорости у0 центра обруча и скорости ум конца маятника в зависи- мости от угла ср имеем следующие выражения: г»О — V —г) (COS ф —COS ф0), V„ = У 2g(R —г)(СО5ф—cos Фо). • (Выражение кинетической энергии обруча, катящегося без проскаль- зывания, см. в задаче 215.) Из этих . выражений следует, что у0 = = &м/ К2- Так как центр обруча движется в У~2 раз_медленнее маятника, то период движения центра обруча будет в ]/^2 раз боль- ше, чем период движения математического маятника длины R—г. Таким образом, для искомого периода имеем выражение Т = = 2л У 2(R—r)/g. Заметим, что при г—►О Т = 2л yilR/g, хотя на первый взгляд может показаться, что при г = 0 должно иметь место равенство Т = 2л УRig- Это связано о тем, что при г—>0 энергия вращательного движения обруча не исчезает. 658. Пусть стержень первоначально отклонен от положения равновесия на угол а. В тот момент, когда стержень будет состав- лять с вертикалью угол р, угловая скорость ©j стержня на основа- нии закона сохранения энергии будет равна Рассмотрим теперь математический маятник длины I. В этом случае при тех же углах аир Подберем I так, чтобы ©1 = со2. Для этого 7надо потребовать, чтобы "~m1/1 + zn2Z2 ’ Угловая скорость характеризует изменения угла р с течением вре- мени. Так как co1 = co2, то периоды колебаний двух маятников равны. Для математического маятника Т = 2л Уl/g. Следовательно, иско- мый период равен т/ /иг/14-/и2/| 1 У m1l1 + m2l2 g 12 Б. Б. Буховцев и др. 353
659. Данная задача решается тем же методом, что и задача 658. Пусть полукольцо первоначально отклонено от положения равнове- сия на угол а. При движении все точки полукольца имеют одну и ту же линейную скорость. Кинетическая энергия равна /nrW/2. Изменение потенциальной энергии при повороте полукольца на угол а—ф равно mg(2ln)r (cos ф—cos а), так как центр тяжести находится на расстоянии (2/л)г от точки О (см. задачу 150). Приравнивая изменение кинетической энергии изменению потен- циальной, имеем для со выражение следующего вида: V (c°s<p-cos«). Отсюда вытекает, что математический маятник, имеющий длину лг/2, будет иметь тот же период колебаний, что и полукольцо. Таким образом, искомый период равен Т =2л У яг/2g. 660. В положении равновесия пружина будет растянута на ве- личину /, которая определяется-из соотношения kl — mg. Положим, что в начальный момент времени груз покоится, а длина пружины по сравнению с равновесным "положением изменилась на х0. Если теперь систему предоставить самой себе, то груз будет колебаться около положения равновесия с амплитудой, равной |х0|. В случае невесомого блока (Л4 = 0) период колебаний Т0 = 2л У m/k. Обозна- чим через х смещение груза, отсчитываемое от положения равнове- сия. Скорость груза в зависимости от х может быть найдена из за- кона сохранения энергии fe(X,+ 02 _mv* k(x+l)* 2 mg%o — 2 । 2 mgx. Учитывая, что mg — kl, находим v = yk(x%—x2)/m. Если M 0, то закон сохранения энергии запишется в форме *(*о + 7)2 мх I fe(^+08 tn^x 2 mgxo — 2 2 r mgx. Отсюда следует, что v — ^^(.x2—*2)/(М+m). Таким образом, во втором случае (М 0) груз движется так, как если бы его масса по сравнению с первым случаем увеличилась на.ТИ. Следовательно, искомый период Т = 2л У (m-[- M)/k. 661. При смещении палки относительно положения равнове- сия на величину х действующая на палку сила окажется равной F- — pgSx, где р —плотность воды. Знак минус означает, что сила направлена против смещения х. Согласно второму закону Ньютона колебания палки определяются уравнением та ——pgSx. Это уравнение совершенно аналогично уравнению Для колебания груза на пружине: та ——kx. Так как для груза (д — 2я/Т — Уk/m, то частота колебаний палки v ® » т/р£!«0,25 Гц. 2л 2л г т 354
662. Уравнение движения ртути имеет вид та=— pgS2x, где х—смещение уровня ртути от положения равновесия. Уравнение движения имеет ту же форму, что и в случае колебаний груза на пружине. Поэтому Т = 2л УtnjZpgS ж 1,54 с. 663. Действующая на тело сила F 4/8улр/пг, где г—расстоя- ние от центра оемли. Учитывая, что #==4/8улр/?, можно придать этому выражению следующую форму: F = mg^-. Здесь R — радиус Земли. Уравнение движения тела,имеет вид mg L та—---------------------------тГг===— kr. R Сила пропорциональна смещению от положения равновесия и на- правлена к центру Земли. Следовательно, тело будет совершать гармонические колебания с частотой со = У"к/т = У g/R. Отсюда период Тело достигнет колебаний Т=2я VTig. центра Земли за время т л -1/7F — I/ — » 21 мин. 4 2 V g Интересно, что время т совершенно не зависит от того, на каком расстоянии от центра Земли тело начнет свое движение. (Важно только, чтобы это расстояние было много больше размеров тела.) 664. Сила F, действующая на отклоненный от положения равно- весия груз, равна F = 2f sin <р (рис. 483). Так как угол <р мал, то можно считать, что F — bfx/l, или F = #x, где k — Воспользо- вавшись формулой Т = 2л У:m/k, получаем для искомой величины следующее выражение: Т — 2п У mlfif. 665. Рассмотрим синфазные колебания двух грузов. При малом отклонении х сила F не изменяется,- поскольку изменение длины пружины второго порядка малости и им можно пренебречь. Учиты- вая только первые степени х, уравнение движения каждого груза 12* 855
можно записать в виде та = — Fx/l. Отсюда легко определить период колебаний: Т = 2зт У'ml/F. 666. Если грузы совершают колебания в противофазе, уравне- ние движения каждого груза, с точностью до первых степеней х, принимает вид х D 2х ( I 2\ Отсюда Т = 2л J/ р^^у 667. Можно, постепенно раскачивая дверь с частотой, равной собственной частоте колебаний двери. При резонансе амплитуда ко- лебаний может достигать больших значений. 668. На основании закона сохранения энергии имеем со2 ~ (ml2-]- Mr2) ~ Mgr а—mgl (1— cos а), где со—угловая скорость вращения блока. Отсюда /2(Mgra—2mgl sin2 (а/2)) m/24-Afr2 Необходимым условием возникновения колебательного режима является равенство нулю угловой скорости при некотором значении угла а. При этом Mgra = 2mgl sin2 (а/2), или, введя обозначение a —Mrjinl, получим аа/2 = sin2 (а/2). Каждому значению а соответ- ствует определенное максимальное отклонение от положения равно- весия а, которое определяется данным трансцендентным уравнением. 356
Решение этого уравнения проще всего осуществляется графи- чески. Для этого нужно построить кривую у— sin2 (а/2) (рис. 484). Тогда пересечение этой кривой с прямой у — аа/2 даст точку Д, определяющую значение а при данном а. (Значение а, соответст- •вующее пересечению этой прямой с другой ветвью кривой у — — sin2 (а/2), недостижимо при заданных в задаче начальных усло- виях.) Очевидно, что наше уравнение имеет решение, отличное от нуля, только при а, меньшем некоторого предельного значения а0, которое определяется из условия, что прямая у = аоа/2 касается кривой у = sin2 (а/2) в точке С. Как видно из рис. 484, а0 « 133°. Следовательно, a0 = 2CD/a0 ~ 0,73. Колебания возможны при Mr/ml а0 « 0,73. 669. Ядро водорода состоит из одного протона, масса которого равна т. Ядро дейтерия—дейтрон—состоит из одного протона и одного нейтрона. Масса дейтрона 2т. Ядро трития состоит из одно- го протона и двух нейтронов. Масса трития 3m. Так как силы, действующие во всех трех случаях между ядрами, одинаковые, т. е. одинаков коэффициент жесткости kt то Cdjj • ==—г~—* —/---* —г == • Р^~2. Н’ D’ Т| /т /2/п /Зт 670. Обозначим смещение произвольно выбранного ьго атома от положения равновесия через Xi = A cos (со/ + /ха), где х—произвольная величина. Тогда сила, действующая на и-й атом со стороны ближайших соседей, равна F — kA {cos [<о/ + (п-Ь l)xa] + cos [со/ + (а— 1) ха] —2 cos (со/ + «ха)}. Из последнего выражения с помощью простых тригонометрических преобразований получаем F = — kA cos (co/-f-nxa)-4 sin2 (xa/2). Запишем уравнение движения n-го атома: тап = — kA cos (со/ + nxa) • 4 sin2 (xa/2), где an—ускорение a-го атома, равное ап — — со2Л cos (со/ +nxa). Обозначим со2 = А*/т. Из написанных -выше выражений получаем 02—02.4 sin2 (xa/2), co = 2coo-sin (xa/2). \ Если рассмотреть цепочку из N атомов, то крайние атомы бу- дут иметь по одному соседу. Чтобы не усложнять расчеты, в дан- ном случае не будем учитывать краевые эффекты и воспользуемся условием периодичности для бесконечной цепочки, т. е. будем счи- тать, что условия колебания атомов, имеющих номера п, М + 357
2Я+л, 3# + п и т. д., одинаковы. В этом случае cos (at + nxa) == cos [art + (л + N) па] = cos [со/ + (п+2N) па] и т. д. Отсюда для х получаем N различных значений: 2л Nxa —2лу, x=Tr-v, где v = 0, 1, ...» Я—1. Na § 26. Электрические колебания 671. Без постоянного магнита было бы удвоение частоты коле- баний. В этом случае при прохождении через катушку телефона си- Рис. 485. нусоидального тока мембра- на совершала бы два коле- бания за один период коле- баний тока, так как график напряженности магнитного поля Я, создаваемого этим током, имел бы вид, указан- ный на рис. 485, а, а сила притяжения мембраны не зависит от знака И. При наличии постоян- ного магнита, создающего напряженность магнитного поля, превышающую макси- мальную напряженность поля тока, график резуль- тирующей напряженности имеет вид, изображенный на рис. 485, б. Поэтому одно колебание тока будет соответствовать одному колебанию мембраны, и звук4 будет значительно меньше искажаться. 672. Частота собственных колебаний v=l/2nprLC. Так как то v = 1/ -----^r-ё- и 24-10» Гц. 2 л V 673. Частота собственных колебаний контура определяется фор- мулой Томсона to=l/yrLC. 1) Если в катушке находится медный сердечник, то при перио- дических изменениях магнитного поля катушки в нем возникнут индукционные токи (токи Фуко); магнитное поле которых будет ослаблять магнитное поле катушки; Это приведет к уменьшению индуктивности катушки и, следовательно, к увеличению частоты о. 2) Если внести в катушку сердечник из феррита, то магнитное поле катушки увеличится. Соответственно увеличится индуктивность L катушки, а частота © уменьшится. 674. В системе возникнут незатухающие (если пренебречь незна- чительными потерями энергии на излучение электромагнитных волн) 358
колебания. В момент, когда заряд распределен поровну между конденсаторами, энергия электростатического поля минимальна, но сила тока и Энергия магнитного поля максимальны. Полная энергия не изменяется, но происходит переход энергии из одной формы в другую. 675. Смещение электронного луча под влиянием поданного на- пряжения вдоль вертикали запишется следующим образом: Z£ х= 2^д7*Ую cos = а cos (о/ (см. задачу 559). Вдоль горизонтали (ось у) смещение луча равно у = V20 cos (со/ — ф) = b cos (со/ — ср). Для получения траектории нужно исключить из данных урав- нений время. После несложных преобразований будем иметь х2 . z/2 2ху . 9 ЪГС03(₽=8,Пф- Если ф1 = л/2, то х2/а24-^2/^2= 1. Это — уравнение эллипса. Если- ф2 = л, то х——(а/Ь)у, т. е. коле- бания луча происходят вдоль пря- мой, составляющей с осью х угол а, определяемый равенством tga= =bla (рис. 486). 676. Зависимость напряжения от времени изображена на рис. 487. Напряжение на конденсаторе (кривая Оа) возрастает до тех пор, пока не достигнет V3. В этот момент лампа зажигается, и конден- сатор разряжается через лампу (кривая ab), пока напряжение не падает до Vr. После этого процесс повторяется. Возникают так на- зываемые релаксационные колебания, период которых равен т. Ток зарядки и разрядки конденсатора не является постоян- ным, так как зависит от напряжения на конденсаторе (напри- 359
мер, при зарядке уменьшается с ростом напряжения). Поэтому Оа, ab, be и т. д. не являются отрезками прямых. 677. При увеличении емкости время зарядки конденсатора до потенциала V3 и разрядки до Vr увеличится. Следовательно, воз- растет и период. Увеличение R вызовет уменьшение тока за- рядки конденсатора и, следовательно, также приведет к увеличению периода. 678. Когда заряд на пластинах конденсатора достигнет своего максимального значения, пластины следует раздвинуть. Чтобы пре- одолеть силы притяжения между пластинами, при этом нужно совер- шить работу. Эта работа идет на увеличение энергии контура. Когда заряд равен нулю, пластины следует сдвинуть до прежнего поло- жения. При этом энергия в контуре не изменится. § 27. Волны 679. Скорость распространения волн определяется силой тяжести. Сила тяжести характеризуется величиной g, когорая имеет размер* ность [g\ = L/T2. Чтобы получить величину, имеющую размерность скорости, нам надо ввести характерную величину, имеющую размер- ность длины. Такой величиной является только длина волны, по- скольку глубина сосуда бесконечно велика, а амплитуда колебаний частиц в волне бесконечно мала. Из g и Л можно сконструировать величину, имеющую размер- ность скорости, единственным образом, а именно: c = k У gh, где k— безразмерный коэффициент. Теоретические расчеты показывают, что *=1/}<2л. 680. Рассуждая подобно тому, как в задаче 679, можно заклю- чить, что в данном случае скорость распространения волн может определяться только плотностью среды р, коэффициентом поверх- ностного натяжения о и длиной волны 1. Эти величины имеют следующие размерности: [р] = Л1/£А [о] = МТ-2, [1]=£. Следовательно, с = ^У\у/А,р, где £ —безразмерный коэффициент. (Его числовое значение k — У~ 2л .) 681. sin фх/sin <р2 = с1/с2 = 682. Из закона Гука F/S = ЕМ/1 следует, что размерность модуля Юнга есть [£] = М/Т2£. Размерность плотности [р] = Следовательно, выражение для скорости можно сконструировать в виде c—k УЕ/р, где k — безразмерный коэффициент. 683. Натяжение струны нужно увеличить в четыре раза. 684. v = von, где п= 1, 2, 3, 4, ..., ' 360
685. В трубе должно укладываться целое число полуволн: ^k = l (/г = 1, 2, 3, ...). Собственные частоты Vk=с/к = kc/2l = k • 50 Гц (с — 340 м/с — ско-, рость звука в воздухе). 686. Звучание камертона усиливается в момент, когда частота собственных колебаний воздушного столба в сосуде совпадает с ча- стотой камертона. Собственные частоты колебаний воздушного стол- ба в трубе, закрытой с одного конца, 2А* 4-1 с где Z—длина трубы, а с = 340м/с — скорость звука в воздухе; k принимает значения 0, 1, 2, 3, .Л Следовательно, возможные по- ложения уровня воды в сосуде, определяемые расстоянием от поверх- ности жидкости до верхнего края сосуда, равны При / — 1м возможны два положения уровня воды: /О = 25см и = 75 см. 687. Рассмотрим ряд последовательных положений пули, летящей вдоль КА? К, Г, £, D, В, А (рис. 488). В каждой точке пуля создает перед собой сжатие, распространяющееся во все стороны в виде сферического импульса, причем, так как скорость пули и больше скорости звука с, эти импульсы появляются только позади Рис. 488. пули. В момент, когда пуля находится в точке Л, отдельные импульсы изображены на рис. 488 кругами разных радиусов. Огибающая этих сфер — волновой фронт — представляет собой коническую поверхность. Конус движется вперед со скоростью пули. Угол раствора конуса определяется соотношением sin а = ВН/АВ = ci/vt == c/v. 361
688. Находящегося в точке В человека (рис. 489) достигают, звуковая волна, испущенная в тот момент, когда самолет находился в некоторой точке D (см. решение задачи 687). Расстояние СВ = 6 км. Искомое расстояние АВ — ВС/sin a — BCvlc, где и —скорость самолета, а с —скорость звука. Отсюда ЛВ = 9км, 689. Обычно скорость ветра на высоте больше, чем у Земли. Поэтому волновые поверхности, которые в неподвижном воздухе имеют вид сфер с центром в точке расположения источника звука Ветер Рис. 490. (пунктирные линии на рис. 490), изменяют свою форму. В направ- лении ветра скорость волн больше, чем против ветра. Примерные формы волновых поверхностей изображены на рис. 490 сплошными линиями. Распространение звука происходит в направлении, перпендику- лярном в каждой точке волновым поверхностям. Поэтому звук, рас- пространяющийся против ветра, отклоняется вверх (кривая ДВ)ине 362
достигает наблюдателя у поверхности земли. При распространении по ветру звук отклоняется к земле (кривая АС) и достигает наблюдателя. 690. Телевидение осуществляется на длинах волн, меньших 10 м. Ионосфера для этих волн «прозрачна», и отражение волн от нее не происходит. Распространяются же короткие волны практически по прямой, так как на наземных препятствиях (дома и пр.) они почти не испытывают дифракции. 691. Для того чтобы по положению отраженного импульса на экране электронно-лучевой трубки можно было судить о расстоянии до цели, необходимо, чтобы отраженный импульс пришел не ранее, чем через время т, и не позднее, чем через время Т = 1//, после начала посылки прямого импульса. Следовательно, минимальное расстояние до цели Z = ct/2=120m, максимальное расстояние L—cT/2 « 90 км. 692. Отраженная от крыши волна достигает приемной антенны с запаздыванием т = АВ/с —10_?с. Скорость электронного луча вдоль экрана и=//Д/, где ^~25~625 с“”вРемя» в течение которого луч прочеркивает одну строку. (Временем обратного хода луча пре- небрегаем.) Смещение изображений Д/ = спг « 7,8 см. 693. Емкость С вибратора при его погружении в керосин уве- личивается в е раз. Частота собственных колебаний контура про- порциональна 1/У*С. Следовательно, частота колебаний уменьшится в раз. В пустоте частота собственных колебаний вибратора равна v0 = c/2Z, а в диэлектрике v = cf2l]T г. Этой частоте в вакууме соответствует длина волны k — c/v — 2l е « 1,4 м. Короче этот результат можно получить так. Длина волны в керосине 1 = 2Z. В пустоте она увеличивается в “К е раз. Следовательно, l0 = 2Z Уе. Глава V. ГЕОМЕТРИЧЕСКАЯ ОПТИКА § 28. Фотометрия 694. Наименьшая освещенность стены зала (рис. 491) = = (/cosa)/r2. Наименьшая освещенность пола £2== (^ cos Р)/г2. По условию £i/B2 = cos a/cos 0 = D/2h — 2. Отсюда h = D/4 = 7,5 м. 695. Освещенность середины стола £ = /1/Я1 = /2/^2, где Н2— высота второй лампы над столом. Освещенность края стола в пер- вом и во втором случаях равна Р _ 71Я1 р _ 1- (Hf + D^2 ’ 2 (//22 + Z>2/4)3/2 ’ Отсюда Еъ (Я?4-£>а/4)3/2 Освещенность края стола уменьшится в 3 раза. 363
696. Если нормаль к пластинке составляет угол а с направле- нием то освещенность пластинки Е = [cos a-pcos (90° —a)]=-~ 2 cos 45° cos (a—45°). Следовательно, освещенность пластинки будет максимальна, если она параллельна стороне треугольника Зх52. £тах= V~2l/a*. 697. При использовании вспомогательного источника вместе с эталонным равенство освещенностей имело место при соотношении /0//х = г?/Г2> где /0 — сила света эталонного источника, а 1Г — вспо- могательного. Во втором случае равенство освещенностей было достигнуто при /х/Ц — гз/г^, где Iх — искомая сила света. Отсюда lx = rl rllrl г| = 400/о. 698. Полный световой поток от лампы Ф0 = 4л/. Если лампа закреплена на потолке, то на стены и пол приходится половина этого потока. Следовательно, искомый поток Ф=2л/ = 628 лм. 699. Количество световой энергии, поглощаемое внутренними стенками цилиндра за единицу времени (световой поток), в обоих случаях одно и то же. Однако площадь внутренней поверхности цилиндра изменится в /?х/₽2 раз. Поэтому освещенность изменится (увеличится) в ₽х//?2 раз, т. е. E^E^ — R^/R^ 700. Освещенность на краю стола „ / cos ф 1 . _ £ = —2 cos ф sin2 ф, где I—сила света лампы, R — радиус стола, ф—угол падения лучей (рис. 492). Максимальное значение Е достигается при угле ф, удов- летворяющем уравнению 1 — sin2 ф=~ sin2 ф, т. е. при ф = = arcsin \2/3. Лампа должна быть повешена над столом на высоте Л = ( ^2/2) ₽ «0,71 R. 701. Папиросная бумага рассеивает во все стороны падающие на нее световые лучи. Если бумага находится на некотором рас- 364
стоянии от текста книги, то расходящиеся пучки света, отражен- ного от белых участков страницы (между буквами), перекрываются на стороне папиросной бумаги, обращенной к тексту (рис. 493). В результате бумага окажется освещенной приблизительно равно- мерно, и вследствие рассеяния ею света прочитать текст будет нельзя. Рис. 493. Если бумага непосредственно наложена на текст, то освещенность прилегающей к тексту стороны бумаги не будет равномерной. Соответ- ственно интенсивность рассеянного света будет различной в раз- личных участках листа бумаги. Это и позволяет прочесть текст. § 29. Основные законы оптики 702. Тень будет всюду одинаково отчетливой только от точечного источника. Отдельные участки протяженного источника создают тени, накладывающиеся друг на друга. При этом тень будет иметь тем более резко очерченную границу, чем меньше расстояние от предмета до поверхности, на которой образуется тень, так как при этом расстояния между границами теней от различных участков источника будут наименьшими. Именно поэтому ноги дают более резкую тень, чем голова. 703. Карандаш нужно расположить параллельно лампе и воз- можно ближе к столу. При этом тени, создаваемые отдельными участками лампы, будут почти точно накладываться друг на друга. Если карандаш перпендикулярен лампе, то тени от отдельных участков лампы настолько сдвинуты друг относительно друга, что заметной тени не возникнет. 704. Явление можно наблюдать в том случае, если угловое рас- стояние между ветвями меньше углового диаметра солнечного диска. Предположим для определенности, что нижняя ветвь толще, чем верхняя. Чтобы понять, почему освещенность внутри тени меняется так, как сказано в условии, представим себе, что мы смотрим на Солнце попеременно из различных участков тени. В стороне от тени солнечный диск виден целиком. При поло- жении глаза в участке тени А (рис. 240) глаз находится в полутени от нижней ветки. Перед солнечным диском видна только эта ветка (рис. 494, а). Так как она заслоняет часть солнечного диска, осве- щенность этой точки будет меньше. Перемещая глаз дальше в поло- жение В (рис. 240), мы увидим, что и вторая ветка частично заго- раживает солнечный диск (рис. 494, б). Поэтому освещенность будет еще меньше. При дальнейшем перемещении глаз займет положение С (рис. 240), при котором обе ветки накладываются друг на друга (рис. 494, в). Теперь перекрытая ветками часть солнечного диска стала меньше и, соответственно, освещенность больше. Как виден 365
Рис. 494. 705. Как видно из рис. 495, # = Lsina, b=*DE—диаметр поперечного сечения светового ности земли. При угловом размере солнечного 1 L = 6/Р- Следовательно, Я = —=9 м. a sin а = Ь/а, где конуса у поверх- диска р получим Рис. 495. 706. Высота зеркала должна быть равна .половине роста чело- века. Расстояние нижнего края зеркала от пола должно быть равно половине расстояния от глаз человека до его ступней (рис. 496). 707. Пусть h—высота предмета, а а—угол падения лучей на зеркало (рис. 497). Если экран находится на расстоянии I h tg а от предмета, то на экране будут видны две тени, прямая и переверну- тая, сложенные основаниями. Общая Длина тени будет равна 2Л. Тень освещена Солнцем и выделяется по контрасту с остальными участ- ками экрана, освещенными и прямыми, и отраженными лучами. Если экран расположен ближе,' то Длина тени меньше 2Л, причем на ней будут участки, не освещенные ни прямыми, ни отраженными лучами. 708. Точечный источник света всегда дает «зайчик», форма кото- рого определяется формой зеркала. Солнце имеет конечные размеры. Каждый малый участок светящейся поверхности дает светлое пятно, пе- редающее форму зёркала. Эти пятна от разных участков Солнца на- кладываются друг на друга и дают более или менее размытую картину. Если поверхность, на которой наблюдается «зайчик*, находится далеко 366
от зеркала, то форма светлого пятна не будет зависеть от формы зер- кала. Только на небольшом расстоянии от зеркала пятно будет все же передавать форму зеркала, так как углы, под которыми падают на зеркало лучи от различных участков Солнца, мало отличаются друг от друга. Рис. 497. 709. Отраженный пейзаж мы видим так, как если бы мы смотрели на него из точки, расположенной под поверхностью воды на расстоянии, равном расстоянию объектива фотоаппарата от воды., 710. Изображение отрезка АВ будет видно целиком только при расположении глаза внутри заштрихованной области на рис. 498. 367
711. При движении зеркала AfW по направлению к стене положе- ние «зайчика» АВ на стене, как непосредственно видно из рис. 499, будет неизменно (Sx и S2 — изображения источника § при двух поло- жениях зеркала: MN и Размеры «зайчика» также не будут меняться, оставаясь все время равными удвоенным размерам зеркала. 712. Если пренебречь поте- рями при отражении, то осве- щенность «зайчика» будет всег- да в 4 раза меньше, чем осве- щенность зеркала. В то же вре- мя освещенность зеркала меня- ется ввиду изменения расстоя- ния от лампы до зеркала и из- менения угла падения лучей. Для маленького зеркала наи- большая освещенность будет при расстоянии зеркала от сте- ны /=( j/" 2/2) dt где d—расстоя- ние от источника света до той тс. точки стены, к которой прибли- жается зеркало. 713. При повороте зеркала на угол а отраженный луч окажется повернутым на 2а, так как угол падения возрастает на а и на столько же возрастает угол отражения. Следовательно, угловая скорость вращения отраженного луча <д = 2шг-2. Линейная скорость движения «зайчика» по экрану и = 4лп7?«62,8 м/с. 714. 1) Луч, отраженный от первого зеркала, составляет с лучом падающим угол 2а, где а —угол падения. За время t зеркало по- вернется на угол со/, и новый угол падения станет равным а + со/. Таким же будет и угол отражения. Следовательно, угол между лучом падающим и лучом отраженным увеличится на 2со/, т. е. отраженный луч повернется на угол 2со/. Ввиду этого угол падения на второе зеркало, если бы оно не вращалось, составлял бы 04-2©/, где р —угол падения при неподвижных дисках. Но за время i зер- кало также поворачивается на уголхо/, поэтому угол падения стано- вится равным р + Зсо/. Таков же и угол отражения. Следовательно, после двух отражений луч повернется на угол Зсо/ от своего направ- ления при неподвижных зеркалах. После трёх отражений луч повер- нется на 5со/, после п отражений —на (2п—1)-2со/. Таким образом, его угловая скорость будет Q = (2n—1)-2со. 2) Изображение в зеркале, удаляющемся от источника со ско- ростью и, будет удаляться от источника со скоростью 2у, а от второго зеркала—со скоростью Зу. Поэтому второе изображение движется со скоростью Зи относительно второго зеркала и со ско- ростью 4v относительно источника. Скорость третьего изображе- ния относительно источника будет 6у, а скорость n-го равна 2пи. 715. 1) При повороте первого зеркала на угол со/ отраженный луч повернется на угол 2со/ (см. решение задачи 714). Следовательно, увеличится на 2<о/ угол падения на второе зеркало, и, если бы оно не вращалось, угол отражения также увеличился бы на 2со/. После двух отражений луч повернулся бы по сравнению со случаем неподвижных зеркал на угол 2со/. Однако вследствие вращения 368
второго зеркала угол падения луча на него за время t уменьшается на со/. На столько же уменьшается угол отражения, поэтому отря- женный луч пойдет по тому же направлению, как и в случае не- подвижных дисков. Поскольку такое рассуждение можно провести для любых двух последовательных отражений, угловая скорость вращения луча, испытавшего п отражений, будет Q = 0, если п четное; Й = 2со, если п нечетное. 2) Первое изображение удаляется от источника со скоростью 2у, а от второго зеркала —со скоростью v. Следовательно, второе изо- бражение движется относительно второго зеркала со скоростью — v, т. е. является неподвижным по отношению к источнику. Продолжая рассуждение далее, найдем, что искомая линейная скорость л-го изо- бражения будет равна нулю, если п четное, и равна 2и, если п нечетное. 716. Отраженный от зеркала ON луч образует с падающим лу- чом угол ф (см. рис. 244), не зависящий от угла падения I. Дей- ствительно, как видно из треугольника АВС, ф= 180°—2(i*4-r). В то же время в треугольнике ОАВ а+(90°—/)4-(90° — г)~ 180°. Отсюда ф=180°—2а = 60°. При вращении зеркала направление отраженного луча не меняется. Следовательно, если попавший на зеркало ОМ луч отразится от зеркала ON, то он обязательно попадет в приемник. Как нетрудно видеть из рис. 500, на котором изображены два крайних положения зеркал, при которых луч попадает в приемник (ОМ, ON и ОМ', ON'), это происходит на протяжении 1/6 оборота. Поэтому 1/6 часть всей энергии луча попадет в приемник за время одного оборота, а также , и за достаточно большой про- Рис. 500. Рис. 501. 717. Нет, так как в глаз будут попадать лучи, дающие изобра- жение только малых участков кадра (см. рис. 501, на котором изо- бражен ход лучей от крайних участков кадра А и В). 718. Из равенства треугольников $ОЛ, SOB, S'[OAt S^OB сле- дует (рис. 502), что источник света S, его изображение Si в зер- кале ОВ и изображение Si в зеркале ОА лежат на окружности с центром в точке О. / S^OS — 5О5х=ф. Мнимый источник St отражается в зеркале АО, давая изображение $2, лежащее на той же окружности на расстоянии' 2ф дуговых градусов от источника S. Точно так же образуется изображение мнимого источника S[ в зеркале ОВ. 369
Продолжая построение дальше, получаем третьи изображения S8 и S3, отстоящие от источника на 3<р градусов, четвертые и S4 (отстоящие на 4<р градусов) и т. д. Если число п четное (n = 2k), то изображение Sk совпадает с S& и будет находиться на одном диа- метре с источником. Всего, таким образом, будет 2k — 1=л — 1 изо- бражений. Если же п нечетное (n — 2i-\-\), то, как легко убедиться, t-е изображения лежат на продолжениях зеркал и, следовательно, совпадают с (1*+1)-ми и всеми последующими. Поэтому всего изображений будет 2Z, т. е. по-прежнему п— 1. Рис. 502. Рис. 503. 719. Используя решение задачи 718, построим последовательно первые, вторые, третьи и т. д. изображения источника S в зеркалах (рис. 503). Все они будут лежать на окружности с радиусом OS и центром в точке О. Если а — целое число, то последние i-e изобра- жения либо попадут в точки С и D пересечения окружности с про- должениями зеркал, либо совпадут с точкой F, диаметрально про- тивоположной источнику. В том и в другом случае число изображений будет а—1. Если же а не целое, например а —21 ± £, где £< 1, а г—целое число,.то последние ье изображения будут лежать на дуге CFD, находящейся позади как первого, так и второго зеркала, и, следовательно, дальнейших отражений не будет. Поэтому общее число изображений будет равно 2Л 720. Построим изображение точки В в зеркале bd (рис. 504). Далее строим изображение Вг в зеркале cd, В3 — изображение В2 в зеркале ас, В4—изображение В3 в зеркале ab. Соединим точки А и В4. С—точка пересечения ab с линией ЛВ4. Из В3 проводим линию В3С. Точку D пересечения этой линии с ас соединим с В2. Е соединим с Blt a F—с В. Можно утверждать, что ломаная ли- ния ACDEFB есть искомая траектория луча. Действительно, так как А В3СВ4 равнобедренный, CD есть отражение луча АС. Ана- логично можно убедиться, что DE есть отражение CD и т. д. Данное решение задачи не является единственным, так как пер- воначально луч не обязательно направлять на зеркало ab. 370
721. Коэффициент отражения света от поверхности воды убывает с уменьшением угла падения. Если наблюдатель смотрит то й его глаз попадают лучи, отраженные под малыми. углами. Лучи, отраженные от участков моря у горизонта, попадают в глаз пой большими углами. 722. Согласно' закону преломления sini/sinr = n (рис. 505). По выходе из пластинки sin г/sin • Перемножая эти выражения, получим sin i = sin ilt т. e. вы- ходящий из пластинки луч CD параллелен падающему АВ. Как видно из чертежа, a = i — г. Искомое смещение луча- х~ = ЕС—ВС sin (i — г). Так как BC — dfcQs г, то d sin (i — г) cos г , . . (, cos i \ = d sin i f 1--. \ Kn2 — sin2i/ Максимальное смещение, равное d, будет при i —> 90°. 723. Угол падения луча на грани АС и ВС равен 45°. Для полного отражения необ: п > о» 1,4. , чтобы sin i > 1/п. Отсюда 724. Угол падения луча на грань ВС равен искомому углу а. Для того чтобы луч полностью отразился от грани ВС, необходимо, чтобы угол а был больше предельного. Следовательно, sin a > n2/nlt где п2 —показатель преломления воды. Отсюда a > 62°30'. 725. Это явление —не что иное, как мираж, который часто на- блюдается в пустынях. Нагретый слой воздуха, непосредственно прилегающий к асфальту, имеет показатель преломления меньший, чем у вышележащих слоев. Вследствие этого происходит полное отражение и асфальт кажется столь же хорошо отражающим свет, как и поверхность воды. 371
726. Разобьем пластину^на множество тонких пластинок столь малой толщины, чтобы в пределах каждой пластинки можно было считать показатель преломления постоянной величиной (рис. 506). Предположим, что луч входит в пластину из среды с показателем преломления /г0, а выходит из пластины в среду с показателем пре- ломления п3. Тогда согласно закону преломления sin a «j sin 6 п' sin у n" m w —7—3-=—, -7—^-=—, -—z=— и т. д. sin (3 nQ sin -y sin о n sin <p__ n2 sin g _ n3 * sin g ’ sin x ~ n2 ’ Перемножив эти равенства, получим sin a/sin х = л3/и0. Следова тельно, угол, под которым луч выйдет из пластины, x = arcsin ( — sin а \ лз зависит только от угла падения луча на пластину и от показателей преломления сред по обеим сторонам пластины. В частности, если п3 = п0, то х=а. Вообще угол 0 наклона луча к вертикали связан с показателем преломления п в любой точке пластины соотношением п sin 0 = = const = n0 sin а. Если где-либо внутри пластины показатель пре- ломления достигнет значения n = nosina, то произойдет полное отражение. В этом случае луч выйдет из пластины в среду под тем же углом а, под которым он вошел в пластину (рис. 507). 727. Минимальное количество воды, определяемое уровнем х (рис. 508), можно найти из треугольника MNF. NF =х—6 = xtgr. 372
Из закона преломления sinr = sini/n. Следовательно, b b V~n* — 3\n*i - х =-------= ----------«27 см, 1— tgr У л2 — sin2 i — sin i так как i = 45°, а п — 4/3. Необходимый объем воды V — ха2 « 43,2 л. 728. ф=120°. 729. Ход луча в призме изображен на рис. ,509. Между углами а и Р^существует очевидная связь: 2а-|-р = 180°, а = 2р. Отсюда а = 72°; р = 36°. Рис. 509. Рис. 510. 730. Ход луча в призме изображен на рис. 510. Чтобы не произошло полного отражения на грани BW, необходимо, чтобы sin р 1/п. Как видно из чертежа, р = а—г. Следовательно, чем больше г, тем больше допустимое значение а. Максимальное г определяется условием sinr = l/n (угол падения 90°). Отсюда amax = 2arcsin (2/3) « 83°40'. 373
731. Рассматривая треугольники ЛВС, АМС и ADC (рис. 511), нетрудно убедиться, что г + г1==(р; у = а+[3 —ф. По закону пре- ломления sina/sinr = n, sin rx/sin [3=—. Решая полученную систему уравнений, найдем, что Ф=а+Р—У И 732. По условию задачи луч падающий и луч, прошедший через * призму, взаимно перпендикулярны. Следовательно, Z<P=Za» а также Z y = Z Р (рис. 512). Сумма углов четырехугольника AKMN равна 360°. Поэтому / /(ТИМ = 90° и луч КМ падает на грань ВС под углом 45°. Зная углы треугольника КВМ, нетрудно н^йти, что Р = 2Ю°. По закону преломления sin a/sin f = п. Следовательно, sin a = 0,5n, a = arcsin 0,5n. Так как полное отражение под углом 45° наблюдается только при ]/*2, то угол а лежит в пределах 45° a ^90°. 733. Бумага частично пропускает свет. Однако вследствие ее волокнистого строения и большого числа пор. рассеяние света во все стороны очень велико. Поэтому прочесть текст невозможно. Клей или вода, заполняя поры, уменьшают рассеяние света, так как их показатель преломления близок к показателю преломления бумаги. Свет начинает проходить сквозь бумагу, не испытывая значительных отклонений. Вследствие этого текст легко прочесть. 734. На первой границе раздела пучок, несущий энергию В, разделится на два пучка (рис. 513). В первую среду возвратится пучок с энергией рЕ, внутрь пластины пройдет свет, обладающий энергией (1— р)Е. На второй границе раздела пучок с энергией (1— р)Е снова разделится. Выйдет из пластины во вторую среду 374
(1— р)*Е. Рассуждая далее аналогично, придем к выводу, что через пластину пройдут й выйдут во вторук) среду световые пучки с энер- гиями (1—р)2Е, р2(1—р)2Е, р4(1 — р)*Ё и Т. д. (рис. 513). Суммарная энергия пучков, вышедших во вторую среду, В первую среду возвратится £1 = рЕ+(1 - Р)а ЕР(1 +р’+р4+ .. .) = £ . Общая энергия § 30. Линзы и сферические зеркала 735. Пусть расстояние от светящейся точки до зеркала равно d, расстояние от изображения до зеркала /. Тогда по формуле вогну- того зеркала d' f R По условию | d—f | = 0,757?. Решая полученную систему уравнений, находим следующие решения: = 1,5/?, da — 0,25/?, d3 = 0,75/?. Четвертое решение d4 =—0,5/? непригодно, так как должно быть d >0. 736. / = 2/?. 737. Линза рассеивающая. Ее фокусное расстояние равно 3 м. 738. В первом случае фокусное расстояние определяется по формуле 375
Так как в пустоте фокусное расстояние линзы равно f, то _L+_L=................................L_. Отсюда 1)^=90 см. Во втором случае искомое фокусное расстояние f2 — ——^—=—102 см. Линза будет рассеивающей. я/л2 — * 739. Если линза обращена выпуклой стороной внутрь трубки, то Fx= —П1 — R. Если же внутрь трубки обращена плоская сторона п2 — 1 линзы, ТО F2 =-—---Я» 740. Как показано в решении задачи 738, fDnx o°Aa ”2-fDTT—«1.67. 741. Обозначим: R— радиус кривизны — показатель преломления стекла. Тогда J_____n2_ О fl. fl2— л - 1 D(ni-n^W поверхностей линзы» В то же время, если линза находится на границе раздела сред, а лучи идут из воды, то фокус линзы расположен в воздухе на рас- стоянии F' определяемом из равенства 1 —п , —1 ’Fr~—R 1 R~~' Соответственно, если лучи идут из воздуха, то фокус расположен в воде на расстоянии F", причем п __п±—п , п1 — 1 Tr~~R 1 R~~* И? этих четырех уравнений следует:, 2FXF2 6FxF2 2nFrF2 __ 8FjF2 F2 + nFi 4Fj + 3F2 ’ F2 + nFr +3F2 ’ 742. Очевидно, что одно из изображений будет мнимым. Поэтому, обозначая через и а2 расстояния от источников света до линзы, а через Ь± и Ь2 расстояния от линзы до изображений, имеем J___1 , = f ’ a2'b2 — f* причем по условиям задачи ai4-a2 = ^ bi = b9. Решая данную си- стему уравнений, получим „ I (1 ± V1-2F//) а,=---------5-------. 376
Линза должна помещаться на расстоянии 6 см от одного источника и 18 см от другого. 743. Применяя к обоим случаям формулу линзы, получим -L_L—— Д_— =С1 «1 bl " f ’ b2=x: f ‘ По условию задачи а2 = а1-{-/, bl/a1 = k1=3 (увеличение в первом случае); b2/a2~k2 = 2 (увеличение во втором случае). Отсюда 744. 1) Ход лучей в данном случае изображен на рис. 514, а. Ис- пользуя обратимость световых пучков, можно точку В рассматривать как источник света, а точку А — как изображение. Тогда по формуле линзы 1 1 1 ai b “ f ’ Отсюда f = a1b/(a1 — b) = 2O см. 2) Ход лучей изображен на рис. 514, б. В данном случае и изо- бражение (точка Л) и источник (точка В) являются мнимыми. По формуле линзы 1 1 __ 1 b~ f Отсюда f = a2b/(a2+b) = 12 см. 745. На основании формулы линзы 1+ ’ , а * d—a f где а—расстояние между линзой и лампочкой. Отсюда a2 — ad -pdf = 0. Решая данное уравнение, получим Следовательно, возможны два положения линзы: на расстоянии а1==70 см от лампочки и на расстоянии а2 = 30 см. При /'=26 см отчетливого изображения на экране не будет при любых положениях линзы, так как для получения изображения необходимо, чтобы d 4/. 377
746. В первом случае hjH где at и Ьх — расстояния от предмета и изображения до линзы. Во втором случае соответственно h^H = Ьг1а2. Как вытекает из решения задачи 745, а1 = Ь2 и Ь1 = а2. Следовательно, Я= hfa. 747. F = — ~~П1____см n1(n~l)f1 + /2 748. На основании формулы зеркала ____1___1_ . a b ~~ f Линейное увеличение зеркала Hlh — bja. Угловые размеры изображе- ния в вогнутом зеркале по условию задачи в 1,5 раза больше угловых размеров изображения в плоском зеркале: 0 = 1,5а (рис. 515). Очевидно, что tga=/i/2a, tg0 = 7//(a+^)* При Л<^2а углы а и 0 малы. Для малых углов Н/(а+Ь) « 1,5Л/2а. Исключая из уравнений неизвестные H/h и Ь, найдем f — Zaft. Сле- * довательно, 7? = 2/ = За = 6 м. ’ N 749. Искомое построение и зоб- * ’ ражено на рис. 516. Продолжим , А В до пересечения с фокальной плоскостью линзы MV. Пучок па- раллельных лучей после преломле- ния в линзе идет так, что продол- жения преломленных лучей долж- ны пересекаться в F'. Луч F'O не ' ! А преломляется. Следовательно, до ! линзы луч С А, попадающий в точку N Л, идет параллельно F'O. , Рис. 516. 750. Если А — источник, а В —» * изображение, то линза собираю- щая. Положение оптического центра линзы О и ее фокусов .F нахо- ' дитея построением, изображенным на рис. 517. Если В —источник, 378
751. Центр линзы О—точка пересечения прямых SS' и Фокусы легко находятся построением лучей, параллельных главной оптической оси (рис. 519). 752. Опустив перпендикуляр ВО на прямую находим точку О — оптический центр линзы (рис. 520). Проводим побочную оптическую ось DO, параллельную лучу АВ. Продолжим прямую ВС 1 до пересечения с DO в точке Е, лежащей в фокальной плоскости. 379
Опустив из Е перпендикуляр на найдем точку Г —один из главных фокусов линзы. Используя свойство обратимости луча, л *2 Рис. 520. аналогичным .образом можно найти другой главный фокус F±. 753. Изображение S' может быть действительным или мнимым. В обо- их случаях для нахождения положе- ния источника проведем произволь- ____________ ный луч ADS' и параллельную ему N2 побочную оптическую ось ВОС (рис. 521). Соединив прямыми точки пере- сечения В и С (побочной оси с фо- кальными плоскостями) с точкой О, найдем положения источника (если изображение S' действительное) и S2 (если изображение S' мнимое). 754. Воспользовавшись тем, что луч, падающий на зеркало в его полюсе, отразится симметрично относительно главной оптической оси, построим симметричную с S' точку *SX и проведем луч до пересечения с осью в точке Р (рис. 522). Эта точка и будет полюсом зеркала. Оптический центр С зеркала, очевидно, может быть найден как точка пересечения луча SS' с осью MV'. Фокус найдется обыч- ным построением луча SM, параллельного оси, который, отразив- 380
шись, должен пройти через фокус F (лежащий на оптической оси зер- кала) и через 755. 1) Построив, как и в решении задачи 754, луч ВАС, найдем точку С (оптический центр зеркала) (рис. 523, а). Полюс Р можно найти, построив с помощью симметричной точки Л' ход отраженного в полюсе луча АРА'. Положение фокуса зеркала F найдется с по- мощью обычного построения луча AM, параллельного оси. 2) Аналогичным построением находим центр зеркала С и полюс Р (рис. 523,6). Луч ВМ, отразившись, пойдет параллельно оптической оси зеркала. Поэтому для нахождения фокуса предварительно нахо- дим точку пересечения М прямой AM, параллельной оптической оси, с зеркалом, а затем продолжаем ВМ до пересечения с осью в фокусе F. 756. 1) Лучи, отраженные от плоского зеркала, увеличивают освещенность в центре экрана. Наличие зеркала эквивалентно появ- лению нового источника (с той же силой света), расположенного от экрана на расстоянии в 3 раза большем, чем первый источник. По- этому освещенность должна увеличиться на х/9 той освещенности, которая была раньше: Е1 = 2,5 лк. 2) Вогнутое зеркало расположено так, что источник находится в его фокусе. Лучи после отражения от зеркала идут параллельным пучком. Освещенность по оси пучка параллельных лучей всюду оди- накова и равна освещенности, создаваемой точечным источником в ближайшей к нему точке зеркала. Полная рсвещенность в центре экрана равна сумме освещенностей, созданных самим источником в центре экрана и отраженными лучами: Е2 = 2»2,25 лк = 4,5 лк. 381
3) Мнимое изображение точечного источника в выпуклом зеркале находится на расстоянии 2,5г от экрана (г — расстояние от экрана до источника). Световой поток Ф, посылаемый этим мнимым источ* ником,' равен световому потоку истинного источника, падающему на зеркало: /1<д1 = /2ш2. Так как телеснцй угол ©i потока, падающего на зеркало от источника S (рис. 524), в четыре раза меньше телесного угла <о2, внутри которого распространяются лучи от мнимого источника то сила света /2 мнимого источника в четыре раза меньше силы света источника S. Поэтому мнимый источник создает в центре экрана освещенность в 4-(2,5)а~25 раз меньшую, чем истинный источник. Следовательно, Е3 = 2,34 лк. 757. Каждый участок линзы создает полное изображение неза- висимо от других. Поэтому никаких полос на изображении не по- лучится. Оно просто будет менее ярким. 758. Любой участок линзы дает такое же по форме изображение, как и вся линза. Сложную линзу можно поэтому рассматривать как две линзы* с разными фокусными расстояниями, но с общим оптиче- ским центром. Соответственно эта линза будет давать два изобра- жения: в точке Si и в точке S2 (рис. 525). На экране, расположен- ном перпендикулярно оптической оси в точке Si или S2, изображе- ние источника будет окружено светлым ореолом диаметра ab или, соответственно, cd. 759. Для доказательства равенства видимых размеров солнечного диска у самого горизонта и высоко над ним нужно спроецировать солнечный диск в том и другом случае на лист бумаги о помощью длиннофокусной линзы. При этом линза и лист должны быть пер- пендикулярны солнечным лучам. Линза должна быть длиннофокусной, так как размеры изображения пропорциональны фокусному расстоя- нию. Измеряя размеры изображений, можно убедиться, что они равны. § 31. Оптические системы и оптические приборы 760. Рассеивающую линзу следует расположить на расстоянии 25 см от собирающей. Тогда фокусы рассеивающей и собирающей линз будут совпадать. На рис. 526 изображен ход лучей. 382
761. Возможны два случая. 1) Зеркало находится на расстоянии d=f-}-R от линзы. Ход луча, параллельного оптической оси системы, а также изображение предмета АВ приведены на рис. 527. Изображение А'В' (прямое и действительное) получается в натуральную величину при любом по- ложении предмета. 25см ^^Ясм 2) Зеркало находится на расстоянии d = f=R от линзы (рис. 528). _ Изображение предмета А 'В', также в натуральную величину, будет -обратным и мнимым при любом положении предмета. 762. Ход лучей в данной оптической системе изображен на рис. 529. Первая линза при отсутствии второй дает изображение Л'В', находящееся на расстоянии = 60 см от линзы. Это расстояние находится по формуле линзы bl fl 383
Изображение А'В' является «мнимым» по отношению ко второй линзе. Следовательно, __L+_L=±, аг ^2 /а 763. Из решения предыдущей задачи следует, что в случае двух собирающих линз, находящихся на некотором расстоянии d друг от друга, справедливо равенство ±+±=±+±+_£_. а1 ^2 /1 /г а2^1 В нашем случае рассеивающая линза расположена вплотную к со- бирающей (d = 0), поэтому ±+±=-±+-=-, а1 ^2 fl ft f где /—искомое фокусное расстояние системы. Отсюда/ = Л/2/(/1~/а)- 764. Средняя линза (третья) — собирающая. Если расстояние от предмета до передней линзы системы равно dt то расстояние от зад- ней линзы до изображения / = r2/4d, где F — фокусное расстояние рассеивающей линзы. Увеличение системы k — Fftd. При наименьшем расстоянии между предметом и его изображением (это расстояние равно 3F) k= 1. ’ 765. На вторую линзу падает световой пучок, выходящий из точки, расположенной на расстоянии а2 = 5 см от линзы. Продол- 384
жения световых лучей, преломленных этой линзой, пересекаются, как вытекает из формулы линзы, на расстоянии &2 = 4 см от нее (рис. 530). Эта точка как раз совпадает с фокусом третьей линзы. Поэтому лучи, вышедшие из системы, пойдут параллельным пучком. Данная система является телескопической. 766. Изображение, даваемое второй системой, получается мни- мым. Оно располагается в передней фокальной плоскости первой линзы. Размер изображения 2 мм. 767. Пусть расстояние между линзами равно а. Тогда, если лучи падают сначала на рассеивающую линзу, то уравнение для второй (собирающей) линзы будет иметь вид |_±=± F+a+f, Р В случае, когда первой стоит собирающая линза, уравнение для рас- сеивающей запишется в виде 1 1 1 F ’ f2 Здесь и f2—расстояния от задней линзы до изображения в первом и во втором случаях. По условию /х — f2 = /. Из этих равенств Е = =1/2 — 10 см. 768. Диаметр изображения Луны, полученного с помощью одной линзы, d1 = q)/?x, где <р—угловой диаметр Луны. Диаметр действи- тельного изображения, образованного данной системой, d2 = (pF2/a, где а —расстояние между линзами, F—их фокусные расстояния (см. решение задачи 767). Следовательно, Fх — F2/a — 25 см. 769. Размер изображения, образованного первой линзой, не за- висит от того, рассеивающая эта линза или собирающая. В обоих случаях диаметр изображения равен ф/7, где ф —угловой диаметр Луны. Если вторая линза собирающая, то ее увеличение А’1=/1/(Г+а)= — F/a. Аналогично, если вторая линза рассеивающая, то k2 — Kl(F—а)- ~ ) окончательного изображения Отсюда следует, что kt — k2 и диаметр d — qF*F/a—qF2/a одинаков в обо- их случаях. 770. На рис. 531 изображен ход луча через пластинку от точки S предмета. В результате преломле- ния света пластинкой луч BE ка- •• жется выходящим из точки S'; S' — мнимое изображение S в пла- стинке. Таким образом, расстоя- ние между изображением предмета в пластинке и линзой а' = a —SS'. Смещение SS' = AD — d — DC. Счи- тая углы падения на пластинку малыми, имеем d-r__d_ i ~~~п ’ так как i/г « п. Следовательно, SS'= d(l — 1/и) = 4 см. До помеще- ния пластинки экран находился на расстоянии Ь = а//(а—/) = 120 см, после ее помещения —на расстоянии Ь' — — f) = 180 см. Эк- ран нужно сместить на 60 см. 13 Б. Б. Буховцев и др. 385
771. Без зеркала изображение предмета А'В получится на рас стоянии b~afl(a—f)= 180 см от линзы (рис. 532). После отражения в изображение займет положение А"В" и будет находиться от оптической оси. Слой воды тол« щи ной d сместит изображение нг расстояние Н — Н' = d(l — 1/п), где п—показатель преломления воды. Это вытекает непосредственно из решения задачи 770. Следовательно, 27 = //'4-d(l —-1/п) = 85 см. 772. Возможны два случая: 1) Оптическая ось линзы пер- пендикулярна передней грани клина. Лучи, отраженные от перед- ней грани, пройдя линзу, дадут изображение точечного источника, совпадающее с самим источником. Лучи, отраженные от задней грани, отклонятся на угол <р (рис. 533), определяемый равенством sin cp/sin 2а=л. Ввиду малости углов <р « 2аи. Рис. 534. Второе изображение источника получится на расстоянии d от первого изображения: d = /:-(p = /-2an. Отсюда п = d/Zaf. 386
2) Оптическая ось линзы перпендикулярна задней поверхности клина. Лучи, отраженные от передней поверхности, отклонятся на угол ф = 2а и дадут изображение, отстоящее от источника на рас- стояние с?1 = 2а/:(рис. 534). Лучи, отраженные от задней поверхности, отклонятся на угол 0, определяемый из уравнений sin а/sin 0 = и, sin (а + 0)/sin (2а— 0) = п. При малых углах 0 = 2а(л— 1). Поэтому второе изображение будет находиться от источника на расстоянии d2 = 2a(n—l)f. Полное рас- стояние между изображениями d — + d2 = 2anf. Отсюда n = dl2aft как и в случае (1). 773. Так как изображение, совпадающее с источником, образуется в результате отражения от части зеркала, не покрытой жидкостью, то очевидно, что источник расположен в центре полусферы О. Най- дем положение другого изображения (точка А на рис. 535). По за- кону преломления sin a/sin 0 = и « a/0, sin ф/sin 0 = n « <р/0. Как видно из чертежа, 0 = p~f-2-y, где у »а — 0 — угол падения преломленного луча на зеркало, и (Я —/—h) tg <р « (/? —/г) tg а. Пре- небрегая h по сравнению с R, из полученной системы уравнений найдем и = (2/? — /)/2 (/? — /)= 1,6. 774. Получаемое в системе изображение Л"'В"' показано на рис. 536. Ft и Fg—фокусы линзы и зеркала. Л'В' —изображение, даваемое линзой в случае, если ее поверхность не посеребрена. Изображение Л "В", даваемое вогнутым зеркалом, можно постро- ить, учитывая, что луч ВО после прохождения линзы и отражения от зеркальной поверхности пойдет по пути ОВ", причем / ВОЛ = = / В"ОЛ. Луч ВС выходит из линзы параллельно оптической оси системы и после отражения идет через В2. Отраженные от зеркала лучи преломляются в линзе еще раз и дают изображение Л"'В"'. Точка В'" лежит на пересечении лучей ОВ" и CD. Луч ОВ" проходит через оптический центр линзы после отражения и потому не преломляется. Луч CD строится следующим 13* 387
образом. После первого преломления в линзе и отражения луч ВС пойдет в направлении В2 и преломится в линзе еще раз. Его направ- ление после второго преломления находится способом, изложенным в задаче 753: через оптический центр О проводится до пересечения с фокальной плоскостью линзы луч OD, параллельный CF2. Тогда, соединяя С и D, получим искомый луч. Рис. 536. Так как лучи преломляются в линзе дважды, фокусное расстоя- ние f системы может быть цайдено из соотношения (см. задачу 763) где f8 = /?/2—фокусное расстояние зеркала. Таким образом, f = =fif2/(fi + 2/2) = 2,5 см. Отсюда расстояние b до изображения Л "'В'" находится по формуле 1+1=1 a'b f • Следовательно, b=af/(a — f) = 3 см. 775. Оптическая сила системы равна сумме оптических сил со- ставляющих ее компонентов (если они сложены вплотную). В первом случае Во втором случае 1 / <х 1 I 2 1 / 1 F2 R "J" Я ‘ Отсюда F2 = F1(1 —1/п). 388
776. Фокусное расстояние тонкой линзы , где г— радиус сферической поверхности. Пусть лучи, параллельные оптической оси сферической поверхности, падают на нее из воздуха (рис. 537). Пре- ломившись на поверхности, nynrNK отклоняется на угол а—0 от оптической оси. Как видно из рис. 537, a, OP*tg а=>Г]Р tg(a—0). Рис. 537. По закону преломления sina/sin0 = n. Ввиду малости рассматрива- емых углов отсюда вытекает, что га » Л(а—0) и а = 0п. Следова- тельно, л=я-4тг=п/. Если же параллельные лучи падают из стекла (рис. 537,6), то аналогичное рассмотрение приводит к уравнениям sin a/sin 0= 1/n, г tga = f2 tg (0 —a). Ввиду малости углов 0 = na, ra = f2 (0 — a). Отсюда /2 — r/(n — 0 = /• 777. Возможны два случая: фокус находится вне шара и внутри шара. Рассмотрим сначала первый случай. Ход луча, падающего на 389
шар под углом £, изображен на рис. 538. Учитывая, что углы /иг малы в соответствии с условием задачи, имеем п ; BC = R sin a — R sin (2r— i) « R (2r—i) « (2 —n). Очевидно, что фокус лежит вне шара при п < 2. Если п = 2, то фо- кус лежит на поверхности шара. Расстояние CF — BC-ctgр « ВС/р, р « 2(i-r)« 2i(n-l)/n, как нетрудно определить с помощью рис. 538. Искомое расстояние f=R-[-CF = Rn/2(n — 1). Ход луча при п > 2 изображен на рис. 539. Искомое расстояние f' — OF &CF— R. Как видно из рис. 539, CF = AC ctg (I — г) ® и . Отсюда f' = R/(n — 1). 778. Преломление лучей от источника на передней поверхности шарика описывается формулой 1 , п п — 1 где fi — расстояние от изображения до передней поверхности. От задней (посеребренной) поверхности изображение находится на рас- стоянии 2R— fa. Отражение лучей в образовавшемся сферическом зеркале описывается формулой __!_+±=А. Новое изображение находится на расстоянии fa от зеркальной задней поверхности и на расстоянии 2R— fa от передней. Расчет преломления на передней поверхности производится по формуле 2R — fa*d R ’ где учтено, что окончательноё изображение получается в воздухе на расстоянии d от шарика. 390
Решив систему трех уравнений, получим ^ = 5/?, d2 =—R. Вто- рое значение соответствует расположению источника в центре шари- ка, что противоречит условию. 779. Продолжим луч BF до пересечения с продолжением луча, падающего на шар параллельно оптической оси (рис. 538). Нетрудно видеть, что отрезок DO, соединяющий точку пересечения с центром шара, образует с направлением падающего луча прямой угол. Тре- угольник ODF прямоугольный, так как OF• р » 4- = Я• » 2 и— 1 n (см. задачу 777). Следовательно, главные плоскости шара MN совпа- дают и проходят через его центр. 780. Фокусное расстояние шара е R П /=Т^й=15 см (см. задачи 777 и 779). Используя формулу линзы, что допустимо, так как главные плоскости совпадают, найдем расстояние от центра линзы до изображения: b = af/(a— f) = —15 см. Изображение мнимое и находится перед шаром. 781. Тонкую стенку сферической колбы можно рассматривать как рассеивающую линзу с фокусным расстоянием f = 1 ~ Я2 (п —!)(!//?!—1//?2) ~(/г-1)Д7?- Пройдя две такие линзы, расположенные на расстоянии 27? одна от другой (рис. 540), лучи, параллельные главной оптической оси (диаметру колбы), преломятся так, что их продолжения пересекутся в фокусе F системы на расстоянии b от второй линзы, причем но формуле линзы 1 1 _ 1 fi+27? b Отсюда 6 = /:1(Л + 2^)/2(/1 + ^)- Точка D пересечения отрезка АВ (продолжение падающего луча) и отрезка CF (продолжение луча 391
преломленного) лежит на главной плоскости системы находящейся на расстоянии х от второй линзы. Из подобия треугольников АСВ и F^CO, а также треугольников DCB и FCO следует, что х __ 2R b^2R + fC Главная плоскость лежит от второй линзы на расстоянии 2Rb frR Следовательно, фокусное расстояние системы ' ° х 2(Л + Я)~ 2 2(п— В силу симметрии данной оптической системы положения второго фо- куса и другой главной плоскости очевидны. 782. Как видно из рис. 541, угол преломления г = / ОАВ = £ АВО= £ ОВС—£ ОСВ, а BAD=£ BCD = i-r. В точке А луч поворачивает на угол i—г, в точке В — на угол л — 2г и в точке С — на угол i — г. Следовательно, полный угол отклонения луча от первоначального направления 0 = i — г+л*— 2г+ f—г = л+2/ —4г. Угол г можно найти из соотношения sin Z/sin r — n. 783. При падении на капельку параллельного пучка лучей луч, идущий вдоль диаметра, имеет угол падения 1=0°, а лучи, лежащие выше и ниже его, имеют углы падения, принимающие всевозможные значения от 0° до 90°. 1) Используя результаты предыдущей задачи и закон преломле- ния, можно найти значение 0 для различных i: 392
Таблица II i 0 i 0 0° 180° 55° 138°20' 20° 160°24' 60° 137°56' 40° 144°40' 65° 138°40' 50° 139°40' 70° 140°44' 2) График зависимости 0 от i изображен на рис. 542. 3) Наименьшее значение угла отклонения приближенно рав- но 0min=138°. Лучи, вышедшие из капли, идут приблизительно параллельно именно при 0 = Ornin, так как при этом, как видно из таблицы и графика, 0 меняется наиболее медленно при изменении i. Примерный ход лучей в капле изображен на рис. 543, 393
784. Матовое стекло нужно, во-первых, для того, чтобы фикси- ровать плоскость, в которой получается изображение, и, во-вторых, для увеличения угла зрения. Прозрачное стекло употребляется при рассматривании изображе- ния, даваемого фотообъективом, в микроскоп. Для этого на прозрач- ном стекле наносят черту, которая фиксирует плоскость наводки, и добиваются резкого изображения в микроскопе этой черты и приле- гающего к ней участка картины, даваемой объективом. Матовое стекло в этом случае применено быть не может, так как в микроскоп будут видны все искажения, обусловленные структурой матовой поверхности. 785. 1) Фонари будут казаться одинаково яркими, так как осве- щенность сетчатки глаза E-BS/b2 одинакова для обоих фонарей. (Здесь В—яркость фонаря, S —площадь входного отверстия зрачка, b—расстояние от хрусталика до сетчатки.) 2) Изображение более удаленного предмета лежит ближе к объ- ективу, чем изображение близкого предмета. Поэтому удаленный фо- нарь создаст большую освещенность на фотопластинке и изображение его на фотографии будет более ярким. 786. Освещенность фотопластинки Е S/b2 ~ A (a — f)2la2, где A — d2lf2 — светосила объектива, f—фокусное расстояние, а — расстоя- ние от объектива до фотографируемого предмета (см. задачу 785). Отсюда очевидно, что у короткофокусного аппарата выдержка должна быть меньше. 787. Рассмотрим, от чего зависит освещенность изображения. Так как свет исходит от каждой точки протяженного источника, то све- товой поток Ф, попадающий на линзу, пропорционален площади источника $х. Кроме того, световой поток пропорционален телесному углу о, под которым видна линза из точек источника. Если площадь лин^ы S2, а расстояние от источника до линзы равно d, то to = S2/d2. Таким образом, O = BS1to, где коэффициент пропорциональности В характеризует яркость источника. Световой поток Ф распределяется по площади изображения S3. Поэтому освещенность изображения ~ Ф BS](i)_____BSiS2 ==ST:=^r==:~^d2'* Отношение площадей источника и изображения равно отношению квадратов их расстояний от линзы: S1/S3 = d2//2. Следовательно, осве- щенность изображения E — BS^/f2 зависит при неизменных В и S2 лишь от расстояния f от линзы до изображения. Используя формулу линзы и соотношение k — ffd, легко найти f = F(A’ + l). Поэтому искомое отношение освещенностей Л. E1~ H~\k2+iJ ~ • 788. F = 2a//(a—2/) = 60 см. 789. Сила света изображения данного точечного источника не за- висит от положения экрана. Поэтому п//(/—/)2 = ///2. Это квадратное 394
уравнение дает два значения f: f = Z/(1 ± V п)> Применив формулу линзы ±+2 1 F 9 получим два значения F: Id = 12 см, ^2 = Id 60 см. F 790. п = 0,75+ 0,25 /^/(^ — = 1,25. 791. Если лучи падают сначала на собирающую линзу диаметра D (или на заменяющую эквивалентную линзу), весь световой поток Ф, прошедший через линзу, участвует в образовании изображения. Если же первой стоит рассеивающая линза, то часть света проходит мимо второй (собирающей) линзы. «Действующий диаметр» d рассеивающей р линзы можно найти из подобия треугольников по формуле d=D. Поэтому в образовании изображения участвует световой поток Ф 2 • Ф1 Площадь изображения во всех случаях одинакова. Следовательно, освещенность изображения, если впереди стоит рассеивающая линза, будет в два раза меньше, чем в остальных случаях. 792. Расстояния между Солнцем и Землей и Солнцем и Луной практически равны. Поэтому, если бы Луна и стена имели одинако- вые коэффициенты отражения, то яркость их казалась бы одинаковой. Следовательно, можно считать, что поверхность Луны состоит из темных пород. 793. В воздухе внешняя выпуклая роговая оболочка глаза соби- рает лучи и создает изображение на сетчатке. Хрусталик только помогает ей в этом. Показатель преломления жидкости внутри глаза очень близок к показателю преломления воды. Поэтому в воде рого- вица почти не преломляет свет и глаз становится крайне дальнозор- ким. В маске преломляющие свойства роговицы целиком сохраняются. 794. Рассматривая удаленные предметы через очки, человек видит их так же, как видел бы предметы, находящиеся на расстоянии а2 = 60 см, без очков. Поэтому для человека, вооруженного очками (см. решение задачи 763), _L+J_=_L+_L а + b f + /о ’ где а— оо. Для человека без очков _L+_L=_L. а2 ‘ b f Здесь Ь — глубина глаза, 1/f—наименьшая оптическая сила глаза, 1//о — оптическая сила очков. Предполагается, что очки придвинуты вплотную к глазу. Отсюда /0 = —а2. 395
Определим теперь положение ближней точки аккомодации глаза, вооруженного очками: 1 1 _ 1 J_ । J__J_ i _L Л bi fi fo Отсюда । JL—J________________________________L a3 aY f$ at a2 и, следовательно, a3=15 cm. 795. Надев чужие очки, дальнозоркий видит резко только очень удаленные предметы. Следовательно, расстояние а2 наилучшего зре- ния глаза дальнозоркого определяется из уравнения Л—-=°i. aj а2 где аг — очень большое расстояние (^-*00), Dr — оптическая сила очков близорукого. Оптическую силу D2 очков, исправляющих дефект зрения даль- нозоркого, можно найти по формуле ^0 ^2 где ао = О,25 м—расстояние наилучшего зрения нормального глаза. Расстояние а3 наилучшего зрения близорукого глаза определится из уравнения Если же близорукий наденет очки дальнозоркого, то расстояние наилучшего зрения, т. е. минимальное расстояние а, на котором бли- зорукий сможет без напряжения читать мелкий шрифт, можно опре- делить по формуле Решив эти 4 уравнения, получим а— 12,5 см. 796. При рассматривании предмета высотой I с расстояния D угол зрения <рх определяется выражением <pj = 1/D. Если рассматри- вать тот же предмет в лупу, то ф2 = /7(^+г) = /7^, где /' — высота изображения (рис. 544). Угловое увеличение /V = ф2/ф! = I'DjlL — W/L, где k = Vll = blD = (f + b)lf — линейное увеличение, определяемое по формуле линзы (/ — фокусное расстояние). Следовательно, D b + f D L-r + f f L “ f L ’ 396
1) При L=oo N-—. 797. Увеличение трубы N — fi/fz, где /j —фокусное расстояние объектива, а /2—фокусное расстояние окуляра. Так как у установлен- ной на бесконечность трубы расстояние между объективом и окуляром равно Л+ /2, то D/d — tfi + fzj/b. Здесь b—расстояние от окуляра до изображения диафрагмы. По формуле линзы _L_j__L=_L. h+fi ь ft Исключая из этих уравнений Ь, найдем D/d^fJf^N. 798. Резкие изображения удаленных предметов будут получаться при двух различных положениях собирательной линзы. Ее можно поместить перед рассеивающей линзой или за ней. Для первого расположения расстояние d между линзами можно найти, рассматривая точку К как мнимое изображение точки А в рассеивающей линзе (рис. 545): I ~ fi 397
Луч AMf параллелен оптической оси системы. Отсюда 4 = /2-г4Ц=3.5 см. /1"Г Z Для второго расположения (собирательная линза за рассеиваю- щей) ход лучей изображен на рис. 546. Рассматривая точку А как изображение К в собирательной линзе, применим формулу линзы 1 , 1 __ 1 А+</* l-d~ ft • Отсюда 4fg l+fi ’ Расстояние между линзами может быть d2 = 35 см или d3 = 5 см. 799. Пусть лучи, идущие от одного из концов диаметра видимого диска Луны, направлены вдоль оптической оси системы. Они дадут изображение на оптической оси в точке Д, отстоящей на расстояние Z = 45 см от рассеивающей линзы (рис. 547). Лучи, идущие от дру- гого конца диаметра, составляют с первыми лучами по условию угол <р. Пройдя систему, они дадут изображение (точка В), лежащее в пло- скости, перпендикулярной оптической оси и отстоящей от рассеивающей линзы на то же расстояние /. Для нахождения диаметра изображения D1~AB рассмотрим ход луча, проходящего через оптический центр первой линзы. При пер- вом расположении линз собирательная линза помещена перед рас- 398
свивающей на расстоянии ^ = 3,5 см. В этом случае, рассматривая точку Е кай мнимое изображение точки О, можно написать J______1_=____1_ di Xi fi Используя подобие треугольников АВЕ и OtPE и учитывая, что OiP = ditgq>, получим A _djtg(p dtf l + Xi Xj ~ xt Исключая из данных уравнений, найдем D1 = 0,72 см. Для второго расположения линз (d2 = 35 см) ход лучей изобра- жен на рис. 548. Величину изображения Луны D2 можно найти из уравнений (Xz+dJ—l х9 ~ х2 (рассматривая треугольники EOP, ЕАВ и ОРОГ), _L+_L=J_ ^2 *2 f2 (рассматривая Е как изображение Ох). Отсюда D2« 0,011 см. Для третьего расположения (d3 = 5 см) ход лучей будет не- сколько иным (рис. 549) по сравнению с изображенным на рис. 548. Уравнения для определения D3 запишутся по аналогии с предшест- вующими случаями так: £>з ^d3tgq~d3q J______________1 1 _ (/ — ^з)4~хз хз хз ' d3 х3 f3 Отсюда £>3 = 0,18 см. 399
800. Из формулы линзы следует, что увеличение объектива k1 — bla = FQ^Ka — F0^) = 30. Дей- ствительное обратное увеличенное изображение предмета, даваемое объективом, рассматривается через окуляр, как че^ез лупу, причем в первом случае мнимое изображение, даваемое этой лупой, распо- лагается от глаза на расстоянии D = 25 см. По формуле лупы J______1 _ 1 F) Fок где а± — расстояние изображения, даваемого объективом, до окуляра. Увеличение окуляра k2 = D/al—(D-\-FOii)/FOK = 6. Полное увели- чение микроскопа k = kxk2 = 180 раз. Во втором случае k2 — D/Fок = 5 и k = k±k2 = 150 раз. Глава VI. ФИЗИЧЕСКАЯ ОПТИКА § 32. Интерференция света 801. Нет. Наличие минимумов освещенности в интерференционной картине означает, что световая энергия не поступает в данные участки пространства. 802. В произвольной точке экрана С будет наблюдаться максимум освещенности, если разность хода = где & = 0, 1, 2, ...— целые числа (рис. 550). По теореме Пифагора Отсюда d-l = О2 + (hk + //2)2, dl = D* + (hk - //2)2. d22 - d2 = fd2 + di) (d2 - d2) = 2hkl. В соответствии с условием задачи d2 + ^i~2Z). Следовательно, d2 _dt = kk « Zhkl/ZD. Расстояние k-й светлой полосы от центра экрана hk — k^Dll. Расстояние между полосами A/j = /i^+1—hk — KDIl. 400
XD 803. Расстояние между интерференционными полосами ДЛ = -р (см. задачу 802). В данном случае D — a l — S^— расстояние между изображениями St и <S2 источника S в плоских зеркалах (рис. 551). I можно определить из треугольника //2 = 26а/2, или / = 2да. Следовательно, Д/г = 1 (а + b)!2ba. Рис. 551. 804. Второй когерентный источник получается в опыте Ллойда путем отражения лучей от зеркала АО. При отражении происходит изменение фазы на л (потеря полуволны), поэтому в точке О, где должна была бы наблюдаться светлая полоса, произойдет гашение колебаний — минимум освещенности. По сравнению с задачей 802 вся картина окажется сдвинутой на ширину светлой (или темной) полосы. 805. Усиление освещенности на экране получается, когда раз- ность хода d2 — di = kX. Геометрическое место точек экрана, до кото- рых лучи от обоих источников доходят с такой разностью хода, есть окружность с центров в точке А (рис. 552). Следовательно, интер- ференционные полосы будут представлять собой концентрические окружности. В случае 1 — пк в точке А будет наблюдаться усиление освещен- ности (интерференционный максимум л-го порядка). Ближайшая светлая интерференционная полоса (окружность) (п— 1)-го порядка 401
находится от точки А на расстоянии, определяемом из уравнения <4-dj = |/(пХ4.D)2 + ft2., - Ко2 + ЛЙ-i = (л-1)Х. Приняв во внимание условия задачи 1<^D, 1^/, получим Л„_! » 2D(2-|-nX) = у 2DX ^4.1 ) , 806. Разность хода лучей для k-ro светлого кольца d2 - di = V (2nX)2+^ - Г(«Х)2+^ = а. Отсюда гк=^ У (9л2—Л2) (л2—/г2). 807. Для создания второго когерентного источника, расположен- ного ближе к экрану, чем первый, можно использовать полупрозрач- ную пластину с отверстием. На основании принципа Гюйгенса отвер- стие можно рассматривать как вторичный источник. На экране получается интерференционная картина. Если расстояние между источниками велико, то для получения интерференционной картины необходимо располагать источником, дающим волны, очень близкие ✓ к монохроматическим. 808. Чтобы найти искомое расстояние ДЛ, нужно предварительно вычислить расстояние I между мнимыми источниками и S2, рас- положенными на пересечении продолжений лучей, преломленных гранями призмы-Для этого проще всего рассмотреть ход луча» падаю- щего на грань призмы нормально (рис. 553). Такого луча в дейст- вительности нет, но можно построить его, мысленно продолжив верх- нюю призму вниз. Все преломленные призмой лучи от точечного источника можно считать сходящимися в точке, и такой прием вполне допустим. Так как преломляющий угол призмы мал (призма тонкая), то мнимые изображения и S2 источника можно считать лежащим» на том же расстоянии от призмы, что и источник S. Как видно из рис. 553, 1 = аи 5Д=аа. По закону преломления г « па. Рассмат- ривая треугольник AS^B, можно записать I , — -|-аа « аап. 402
Отсюда Z = 2aa(n—1). Используя решение задачи 802, находим a к (а -|- b) n 1 к Д/1=-7-= о , —-=0,15 см. I 2aa(n —1) 809. N = L/&h, где L—ширина интерференционной картины. Как видно из рис. 260, L = (bla)l. Используя результаты предыдущей задачи, получим 4aba2(n — I)2 _ N= 810. Бипризма, изготовленная из вещества с показателем пре- ломления n2t отклоняет лучи на угол Ф1 = (90° —Р/2) (n2 —nJ, где пх — показатель преломления среды, из которой падают лучи. Для бипризмы, находящейся в воздухе, <р2 = (90°—6/2) (n2 —1). В случае эквивалентности бипризм <pi = <p2. Отсюда 6 = ₽ 180° ^=4. г П2 — 1 1 n2—1 Для заданных в условии величин 6 « 179°37'. 811. Ход лучей в системе изображен на рис. 554. Si и S2 — изо- бражения источника S в половинках линзы. Очевидно, что b = fa!(a—f). Из подобия треугольников SAB и SS^Sz можно найти расстояние I между Sj и S2: l — ad[(a—f). Расстояние между соседними интерфе- ренционными полосами на экране ДЛ==^5^1 = А(ра_£)/_а/)=10-2 СМ (см. задачу 802). Искомое число интерференционных полос Л,. 1<о±г>_25. An a М .403
812. Расстояние между мнимыми источниками и S2 можно чЛ найти методом, изложенным в решении задачи 811 (рис. 555). Рас- стояние между интерференционными полосами K(Df-Da+af) da ' Число полос на экране N = L/Mi, где L = Dl/b — размер участка экрана, на котором наблюдаются интерференционные^полосы. Отсюда NabfX adl + abNK — bfNK 1 СМ’ 5 Максимально возможное число полос найдется из условия adl + NabX — bfN X = О (при этом D —> оо). Следовательно Л,тах bfX—ahX Число полос получается конечным, так как по мере удаления экрана одновременно с увеличением размеров участка экрана, на котором возникает интерференционная картина, растет расстояние между - полосами. 813; Расстояние между интерференционными полосами не будет 1 зависеть от положения экрана только в том случае, если источник расположен в фокальной плоскости линзы. Это непосредственно вы- текает из выражения которое получено при решении задачи 812. Если a — f, то Ah = 'kf/d= = 10"’2см при любом D. 404
Ход лучей для данного случая изображен на рис. 556. Как видно из этого рисунка, число интерференционных полос будет максимально, когда экран займет положение АВ. Расстояние экрана до линзы можно найти из треугольника ОАВ, учитывая, что угол а « d/f, a AB — R: D = Rf!d — 2 м. Рис. 556. 814. Внутри стекла длина световой волны уменьшается в п раз,- так как частота не изменяется, а скорость уменьшается в п раз. Вследствие этого между когерентными волнами в пучках возникает дополнительная разность хода. На расстоянии dr в верхнем пучке уложится kl=d1n/<k цлнн волн, а в нижнем,на том же расстоянии уложится k2 = d2n/‘k-[-(d1—d9)/K длин волн. Световые волны в любой точке экрана окажутся дополнительно сдвинутыми друг относительно друга на kr— k2 длин волн. Вследствие этого вся интерференционная картина сместится вверх на fe2 = --. (п —1)= 100 полос. Про- цесс смещения можно наблюдать в момент введения пластин. После того как пластины введены, интерференционная картина на экране будет иметь прежний вид. 815. Толщина линзы слишком велика. Интерференция имеет место только в случае тонких пленок. Воздушная прослойка вблизи сопри- косновения линзы и стекла является тонкой. 816. Нет, не изменится. Разность хода между волнами, встречаю- щимися на экране от источников S и St или S и S2, велика. В этих условиях спектры различных порядков, соответствующие спектраль- ному интервалу источника, налагаются друг на друга подобно тому, как это имеет место при отражении волн от границ толстой пленки. Если ширму убрать, то это приведет только к наложению на интер- ференционную картину от источников Sj и *$2 монотонно меняющейся освещенности. 817. При наблюдении колец в отраженном свете интенсивность интерферирующих пучков примерно одинакова. В проходящем же свете интенсивность одного пучка, не испытавшего отражений, зна- чительно превышает интенсивность второго пучка, испытавшего два отражения. В результате максимумы и минимумы возникнут на фоне равномерной освещенности, полного гашения света не произойдет, и вся картина будет менее контрастной, чем в отраженном свете. 818. При отсутствии контакта радиус пятого кольца определяется уравнением ri//? + 2d = 5X. Если пыль удалить, то радиус этого кольца определится равенством rl/R — 5k. Отсюда d = (г2 — гi)/2R = — 1,8 • 10—4 см. 405
о.„ 1/ feA, 819. rk- у l/Rt_i/Rt- 820. Для уменьшения коэффициента отражения необходимо, чтобы лучи 1 и 2 (рис. 557), отраженные от внешней и внутренней поверх- ностей пленки, нанесенной на оптическое стекло, гасили друг друга. Гашение будет иметь место при условии 2Лл=(2Л + 1)у. (1) где k = 0, 1, 2, ... Отсюда минимальная толщина пленки ftmln = к/4п. Условие (1) не может быть выполнено для всех длин волн. Поэтому обычно h выбирают так, чтобы гасилась средняя часть спектра. На- носимая пленка имеет толщину, превышающую hmin в нечетное число раз, так как более толстые пленки изготовить проще, чем тонкие (в четверть длины волны). 821. Для наблюдения интерференционной картины необходимо, чтобы максимум fc-ro порядка, соответствующий длине волны 1, не перекрывался с максимумом (&+1)-го порядка, соответствующивг длине волны Х + ДХ, где ДЛ = 100А. Это будет иметь место при уело- вии (XДА<) X (/г + 1). Отсюда/г сД/ДХ. > 406 Ji
I Максимально допустимая толщина прослойки Лтах удовлетворяет уравнению 2/гтах = (Х + ДМ^тах, гДе /?тах = ^/Д^. Если в качестве к выбрать длину волны, соответствующую середине видимого участка , спектра (1 = 5000А), то /imax « 1,3-10-3 см. J Если вместо воздушной прослойки взять тонкую пленку, имею- I щую показатель преломления и, то максимальная толщина должна I быть в п раз меньше, чем у воздушной прослойки. I 822. При интерференции лучей 1 и 2 (рис. 558), отраженных от j различных граней клина, условие минимума запишется следующим I образом: 2Лп = М (£ = 0, 1,2). Так как угол а мал, то h «ха. Сле- | довательно, расстояние между интерференционными полосами на самом клине Дх = 1/2ал. Согласно формуле увеличения линзы кх/Ы — а/Ь, где а—расстоя- |ние от экрана до линзы, а Ь—от линзы до клина. Так как b = d—ai то по формуле линзы а 1 d — a f у Исключая из данных выражений а и &, найдем искомое значение J угла а: I а.., Х d^^-4fd 2пМ d ±y~d2—4fd' Решение этой задачи не однозначно. Это связано с тем, что четкое । изображение на экране при фиксированных d и f можно получить ! при двух положениях линзы. § 33. Дифракция света 823. Радиус первой зоны Френеля можно найти из треугольни- ков ADE и DEB (рис. 559): г? = а2-(а-х)2 = (д + 1/2)2-(^+х)2. Так как длина волны мала, то х = 61/2 (а+ 6). Следовательно, Г1 = 2дх —х2. Пренебрегая малой величиной х2, окончательно получим гх= Vab'kKa-Yb). 407
Аналогичным образом можно найти радиусы последующих зон Френеля. Для зоны номера k rk = abk'k)(a 824. Плоской волне соответствует расстояние от точечного источ- ника до фронта волны а—>оо. Искомые радиусы зон: rk— lim V^abk'kKa-yb)— КkbX а-* <ю (см. решение задачи 823). 825. Для решения задачи необходимо подсчитать число k зон Френеля, укладывающихся в отверстиях диаметров D и Dv Исполь- зуя результаты задачи 823, имеем Уk ab\l(a -f- b) = £)/2. Отсюда легко найти, что k — З (нечетное число). При диаметре отверстия 5,2 мм в нем укладывается приблизительно 4 зоны (четное число). Следо- вательно, увеличение отверстия приведет к уменьшению освещенности в точке В. 826. Темное пятно на оси пучка при открытых четырех зонах Френеля окружено светлыми и темными кольцами. Суммарная осве- щенность экрана при увеличении отверстия возрастает, но распреде- ление световой энергии по экрану меняется таким образом, что в центре будет минимум. 827. Искомая освещенность будет максимальна в том случае, когда в диафрагме укладывается одна зона Френеля. Учитывая решение задачи 824, имеем D = 2-K^ = 0,2 см. 828. Дифракция будет заметна, если в отверстии укладывается небольшое число зон Френеля, т. е. радиус отверстия будет того же порядка (или меньше), что и радиус первой зоны Френеля: V'abk/ia-i-b)^ R, где R — радиус отверстия. При а — Ь имеем dk^?2R2. Рис. 560. 829. На рис. 560 построены зоны Френеля, позволяющие опреде- лить интенсивность света в точке В. Освещенность в точке В создается первой и последующими зонами Френеля. Если размеры экрана не 408
превышают значительно радиуса первой центральной зоны, опреде- ляемого по формуле задачи 823, то в точке В обязательно возникает светлое пятно с освещенностью, мало отличающейся от той освещен- ности, которая имела бы место в отсутствие экрана. 830. Приблизительно 3 м. 831. Зоны Френеля в данном случае удобно выбрать в виде поло* сок, параллельных краям щели. В направлении ф будет наблюдаться минимум в том случае, если в щели АВ (рис. 561) укладывается Рис. 561. четное число зон. (На рис. 561 изображено четыре зоны Френеля.) b = 2kx, где х — ширина зоны Френеля, £=1,2,3, ... АК представ- ляет собой разность хода между крайними лучами, посылаемыми одной зоной: АК = х sin ф = Х/2. Отсюда x = A/2sinq>. Следовательно, в направлении ф будет наблю- даться минимум, если £8тф = £^. 832. Лучи, падающие на отверстие камеры от удаленного точеч- ного источника, идут приблизительно параллельно. Если бы не было дифракции, то размеры светлого пятна были бы равны ЛВ = 2г (рис. 562). Вследствие дифракции размеры пятна увеличатся до DC. Рис 562. Расстояние ОС определяется углом ф, дающим направление на пер- вый минимум (темное кольцо). Согласно указанию, 2r sin фя 1, Сле- довательно, радиус пятна OC = r-[-AC~r-[-d sin ф « r-}-dX/2r( 409
Эта величина достигает минимума при г = kd/Zr. Оптимальные размеры отверстия r = V^kd/2, 833. Углы, определяющие направления на максимумы второго и третьего порядков, удовлетворяют уравнениям d sin ф2 = 21 и d sin ф3==31. Отсюда к — d (sin ф3 — sin <p2) = 2d cos sin (Фз Фа) « d(q>3—(p2) = da » 1,7-10-5 см. 834. Максимальному k соответствует sin ф = 1. Следовательно k = d/k = 4. 835. Чтобы возник спектр первого порядка, необходимо условие d^k. Следовательно, искомый период решетки не может быть мень- ше 0,02 см. 836. Направление на первый максимум определяется выражением d sin <р = Х. Экран расположен в фокальной плоскости линзы. Считая угол ф малым, имеем 1 = Отсюда k — dl/f — ^* 10~5 см. 837. В воде длина всех волн уменьшается в п раз (л — показатель преломления воды). Следовательно, углы ф, определяющие направле- ния на максимумы, и расстояния от центра дифракционной карти- ны до максимумов, соответствующих различным длинам волн, также уменьшатся в п раз, так как по условию углы ф малы и sin ф « ф. 838. Спектры разных порядков будут соприкасаться при условии Н2 = (& +IjXp Отсюда /г = ^/(Хг —1х) = 5. Следовательно, частично перекрываться могут только спектры шестого и седьмого порядков. Но данная решетка (см. задачу 834) может дать для данного интер- вала длин волн спектр только четвертого порядка. Поэтому спектры в нашем случае перекрываться не будут. 839. При наклонном падении лучей на решетку под углом 0 (рис. 563) разность хода между волнами, идущими от краев соседних 410
щелей, b = BD — AC = d sin ф—d sin 0. Эти волны, складываясь, усиливают друг друга при d (sin ф — sin 0) = Л1, где k = 1,2, 3, ... для максимумов, лежащих правее центрального (k = 0), и k — —1, —2, —3, ... для максимумов, лежащих левее центрального. Наибольший порядок спектра будет при ф =—90°. Тогда d(—1 —1/2) = &1. Отсюда k = —6. Может наблюдаться спектр шестого порядка. Знак минус указывает на то, что спектр лежит левее центрального. 840. Как вытекает из формулы d(sin ф — sin 0) = fcl (см. решение задачи 839), минимальное значение периода решетки будет при сколь- зящем падении лучей: 0 = 90°. В этом случае d « 1/2. Следовательно, период решетки должен удовлетворять неравенству d^l/2. 841. В общем случае, как показано в решении задачи 839, иско- мое условие имеет вид d (sin ф — sin 0) = kk. Его можно переписать в форме 2d cos sin Если d^kk, то ф « 0. При этом ф ”Ь ® Л cos « cos 0, Следовательно, условие, определяющее направления на главные максимумы, примет вид (d cos 0) (ф —0) » £1. Постоянная решетки как бы уменьшилась и стала равной d cos 0 вместо d. Углы ф—0 отсчитываются от направления падающего света. § 34. Дисперсия света и цвета тел 842. -Как показано в задаче 730, угол падения а, преломляющий угол призмы ф и показатель преломления п связаны с углом 0, под которым луч выходит из призмы, выражением • а 1 /7 sin а 1 V , . n=sinb I/ .---НС^Ф +!• г Г \ sin р sin ф 1 / 1 411
Отсюда для sin f получаем следующее уравнение: • во/i', х о ч п • о sin а . sin2 а „ Л sin2 6 (1 + ctg2 ®)4- 2 sin 6 -т-—г-s--п2 = О, г 4 1 & ‘ r sin ф tg ф 1 sin2 ф или (ири данных значениях а и ф) 2sin2P+ J<2 sinР+у-»8=0. Решая это уравнение, найдем . о -1 ± sin р =---------------------------т=----- 2 /2 Физический смысл имеет решение со знаком плюс. Для красных лучей sin рк ~ 0,26. Следовательно, рк « 15°6'. Для фиолетовых sin р* « 0,31 и Рф ~ 18°6'. Искомый угол 0 == рф — рк « 3°. 843. Для красных лучей фокусное расстояние линзы /к = = /?/2(лк —1) « 27 см; для фиолетовых /ф = 25 см. По формуле линзы изображение, даваемое красными лучами, будет расположено*на рас- стоянии bK = afK/(a — fK) = 58,7 см; фиолетовыми — на расстоянии дф = 50 см. х На экране (рис. 564) изображение источника будет иметь форму пятна, края которого окрашены в красный цвет. Диаметр пятна d 844. Солнечные лучи, падающие на капли дождя, можно считать параллельными. По выходе из капли после однократного отражения на внутренней поверхности капли лучи расходятся по всем направ- лениям. Лишь лучи, испытавшие наименьшее отклонение, идут при- близительно параллельно. Поэтому именно эти лучи, попадая в глаз, вызовут наибольшее зрительное впечатление. Эти лучи идут, так сказать, с наибольшей «плотностью». Остальные лучи рассеиваются во все стороны. Для параллельных лучей угол отклонения, как показано в задаче 783, равен 138°. Следовательно, угол между па- дающими от Солнца лучами и направлением на радугу составляет 42Q (для красного света) (рис. 565). В глаз попадает свет от тех капель, которые находятся в на- правлении, составляющем угол 42° с линией, проведенной через 412
глаз и Солнце. Для фйолетовых лучей этот угол составляет при мерно 40°. 845. Первая (основная) радуга наблюдается благодаря лучам, испытавшим одно отражение внутри капелек воды. При преломлении наиболее сильно отклоняются от первоначального направления фио- летовые лучи (см. задачу 782) (^ 0 растет с ростом и, так как г уменьшается), поэтому внешняя дуга будет красной, а внутренняя — фиолетовой. Вторая радуга вызвана лучами, испытавшими два отражения внутри капелек. Примерный ход луча изображен на рис. 566. Направ- ление на радугу составляет, как можно показать, 51° с линией, соединяющей глаз и Солнце. Чередование цветов при двух преломле- ниях и двух отражениях получается обратным: внешняя дуга будет фиолетовой, а внутренняя —красной. После двух отражений интен- сивность света оказывается сильно ослабленной, вследствие чего вто- рая радуга бывает гораздо менее интенсивной, чем первая. 846. Географическая широта Москвы, т. е. угол между плоскостью экватора и нормалью к поверхности земного шара, хр = 56°. Солнце в этот момент стоит в зените над северным тропиком (широта а — 23,5°). Следовательно, угол между направлением на Солнце и горизонтом (рис. 567) 0 = 90° —ф + а = 57°30'. 413
Радуга же может быть видна только в том случае, когда высота Солнца над горизонтом не превышает 42° (см. рис. 565). Следовательно, наблюдать радугу в указанное время нельзя. 847. Наш глаз получает ощущение того или иного цвета, когда его чувствительные элементы раздражаются световой волной опреде- ленной частоты. Частота же световых волн не меняется при переходе из одной среды в другую. 848. Необходимо смотреть через зеленое стекло. При этом надпись будет видна черной на зеленом фоне бумаги, так как красный свет надписи «отлично» не пропускается зеленым стеклом. При рассматри- вании через красное стекло красная надпись не будет видна на красном фоне бумаги. 849. Объектив преимущественно отражает крайние части видимого спектра: красную и фиолетовую (см. задачу 820). От смешения этих цветов возникает сиреневый оттенок. 850. Цвета радуги являются чистыми спектральными цветами (см. задачу 844), так как по данному направлению виден лишь луч вполне определенной длины волны. Цвета тонких пленок, наоборот, получаются из-за гашения (полного или частичного) лучей некоторого спектрального интервала в результате интерференции. Цвет пленки будет дополнительным к цвету этого спектрального интервала. 851. Под действием силы тяжести мыльная вода стекает в ниж- нюю часть пленки, которая всегда толще, чем верхняя. Следовательно, полосы, которые указывают геометрическое место точек одинаковой толщины, должны быть расположены горизонтально. Голубой (сине- зеленый) оттенок получается при исключении из полного спектра его длинноволновой (красно-оранжевой) части (см. задачу 850). При гашении средней (зеленой) части спектра оставшиеся лучи придают пленке пурпурный (малиновый) оттенок, а при вычитании из сплош- ного спектра его коротковолновой (сине-фиолетовой) части пленка выглядит желтой. Если разность хода взаимно гасящихся лучей составляет одно и то же число полуволн во всех трех случаях, то вверху должна быть желтая полоса, затем пурпурная и внизу голубая. 414
852. Днем рассеянный небом голубой свет добавляется к желто- ватому свету самой Луны. Это смешение цветов воспринимается глазом как белый цвет. После захода Солнца голубой свет неба ослабевает и Луна принимает желтоватый оттенок. 853. На темном фоне мы видим дым вследствие того, что он рас- сеивает падающие на него сверху солнечные лучи. Частицы дыма рассеивают синий свет гораздо сильнее, чем красный или желтый. Поэтому цвет дыма кажется синим. На фоне светлого неба дым виден в проходящем свете. Дым кажется желтоватым, так как синий свет рассеивается во все сто- роны и только длинноволновый участок спектра белого света дости- гает глаз. 854. Тонкая пленка воды, покрывающая влажный предмет, от- ражает падающий белый свет по одному определенному направлению. Поверхность предмета уже не рассеивает белый свет во все стороны, и господствующим становится его собственный цвет. Рассеянный свет не налагается на отраженный от предмета, и поэтому цвет кажется более насыщенным.
' Буховцев Борис Борисович, Кривченков Владимир Дмитриевич, Мякишев Геннадий Яковлевич, Сараева Ирина Макаровна СБОРНИК ЗАДАЧ ПО ЭЛЕМЕНТАРНОЙ ФИЗИКЕ М., 1974 р., 416 стр. с илл. Редактор Б. Ю. Кован Техн, редактор В, Н. Кондакова Корректоры Т. С. Плетнева, Л, С. Сомова Сдано в набор 10/VI 1974 г. Подписано к печати 30/IX 1974 г. Бумага 84Х1081/з2» тип. № 3. Фиат. печ. л. 13. Условн. печ. л. 21,84. Уч.-изд. л. 23,02. Тираж 400 000 экз. (2-й завод 150 001—400 000). Цена книги 72 коп. Заказ Ха 1472 Издательство «Наука» Главная редакция физико-математической литературы 117071, Москва, В-71, Ленинский проспект, 15 Ордена Трудового Красного Знамени Первая Образцовая типография имени А. А. Жданова Союзполиграфпрома при Государственном комитете Совета Министров СССР по делам издательств, полиграфии и книжной торговли. Москва, М-54, Валовая, 28